You are on page 1of 304

http://t.

me/iitjeehelps
EBD_7167
http://t.me/iitjeehelps

• Corporate Office : 45, 2nd Floor, Maharishi Dayanand Marg, Corner Market,
Malviya Nagar, New Delhi-110017
Tel. : 011-49842349 / 49842350 How to access the ebook(s)?
Educore

INSTRUCTIONS
1. Mail your Order ID at ebooks.support@aiets.co.in
to get the 16 digit Access Code.
2. Go to www.educoreonline.com/register.htm
3. Enter your details along with your 16 digit Access
Code.
4. Click Register & you would be successfully
redirected to the Login Page.
Note: If you are already registered with us, you just
have to login (http://www.educoreonline.com/login.
htm) & enter your new 16 Character Unique Code
under your Account Section.
5. Login with your registered email ID & password.
6. You can now view you e-book(s) under your Library.
7. You can read your e-books either Online or Offline.
For offline, simply download our Educore App once
& download the e-books inside the app. Educore App
is available for Windows Desktop, IOS & Android.
8. Educore works best in Desktop, Laptop, 7” & 10”
tablets.
9. Contact us at support@educoreonline.com for any
further assistance.
Note: This app is not accessible on Mobiles.
DISHA PUBLICATION
ALL RIGHTS RESERVED

© Copyright Publisher

No part of this publication may be reproduced in any form without prior permission of the publisher.
The author and the publisher do not take any legal responsibility for any errors or misrepresentations
that might have crept in. We have tried and made our best efforts to provide accurate up-to-date
information in this book.

For further information about the books from DISHA,


Log on to www.dishapublication.com or email to info@dishapublication.com
http://t.me/iitjeehelps

Contents
SECTION-I : PHYSICS P-1 – P-84

1. Units and Measurements 1-2 16. Electrostatic Potential and

2. Motion in a Straight Line 3-4 Capacitance 47 - 50

3. Motion in a Plane 5-7 17. Current Electricity 51 - 54


18. Moving Charges and Magnetism 55 - 58
4. Laws of Motion 8 - 10
19. Magnetism and Matter 59- 59
5. Work, Energy and Power 11 - 13
20. Electromagnetic Induction 60 - 62
6. System of Particles and
21. Alternating Current 63 - 65
Rotational Motion 14 - 17
22. Electromagnetic Waves 66 - 66
7. Gravitation 18 - 20
23. Ray Optics and Optical
8. Mechanical Properties of Solids 21 - 22 Instruments 67 - 70
9. Mechanical Properties of Fluids 23 - 25 24. Wave Optics 71- 73

10. Thermal Properties of Matter 26 - 28 25. Dual Nature of Radiation


and Matter 74 - 76
11. Thermodynamics 29 - 31
26. Atoms 77 - 78
12. Kinetic Theory 32 - 34
27. Nuclei 79- 81
13. Oscillations 35 - 38
28. Semiconductor Electronics: Materials,
14. Waves 39 - 42
Devices and Simple Circuits 82 - 84
15. Electric Charges and Fields 43 - 46

SECTION-II : CHEMISTRY C-1 – C-76


1. Some Basic Concepts of Chemistry 1–3 10. The s-Block Elements 22 – 23
2. Structure of Atom 4–4 11. The p-Block Elements 24 – 24
3. Classification of Elements 12. Organic Chemistry : Some
and Periodicity in Properties 5 –7 Basic Principles and Techniques 25 – 30
4. Chemical Bonding and Molecular 13. Hydrocarbons 31 – 33
Structure 8 – 10 14. Environmental Chemistry 34 – 35
5. States of Matter 11 – 12 15. The Solid State 36 – 36
6. Thermodynamics 13 – 15 16. Solutions 37 – 39
7. Equilibrium 16 – 18 17. Electrochemistry 40 – 43
8. Redox Reaction 19 – 20 18. Chemical Kinetics 44 – 47
9. Hydrogen 21 – 21 19. Surface Chemistry 48 – 48
EBD_7167
http://t.me/iitjeehelps

20. General Principles and Processes 26. Aldehycles, Ketones and


of Isolation of Elements 49 – 49 Carboxylic acids 63 – 66
21. The p-Block Elements
27. Amines 67 – 69
(Group 15, 16, 17 and 18) 50 – 51
28. Biomolecules 70 – 71
22. The d-and f-Block Elements 52 – 54
29. Polymers 72 – 72
23. Coordination Compounds 55 – 56
24. Haloalkanes and Haloarenes 57 – 58 30. Chemistry in Everyday Life 73 – 73
25. Alcohols, Phenols and Ethers 59 – 62 31. Analytical Chemistry 74 – 76

SECTION-III : MATHEMATICS M-1 – M-104


1. Sets 1–2 14. Statistics 44 – 46
2. Relations and Functions 3–4 15. Probability 47 – 48
3. Trigonometric Functions 5–9 16. Relations and Functions 49 – 51
4. Principle of Mathematical 17. Inverse Trigonometric Functions 52 – 54
Induction 10 – 11 18. Matrices 55 – 56
5. Complex Numbers and 19. Determinants 57 – 61
Quadratic Equations 12 – 16 20. Continuity & Differentiability 62 – 66
6. Linear Inequalities 17 – 17 21. Application of Derivatives 67 – 73
7. Permutations & Combinations 18 – 20 22. Integrals 74 – 79
8. Bionomial Theorem 21 – 23 23. Application of Integrals 80 – 82
9. Sequences & Series 24 – 28 24. Differential Equations 83 – 85
10. Straight lines & Pair of 25. Vector Algebra 86 – 90
straight lines 29 – 32 26. Three Dimensional Geometry 91 – 95
11. Conic Sections 33 – 39 27. Probability 96 – 99
12. Limits and Derivatives 40 – 42 28. Properties of Triangles 100 – 102
13. Mathematical Reasoning 43 – 43 29. Linear Programming 103 – 104

SECTION-IV : ENGLISH PROFICIENCY & LOGICAL REASONING EL-1 – EL-36

Unit - 1 : English Proficiency Unit - 2 : Logical Reasoning

1. Vocabulary & Grammar 1-8 3. Verbal Reasoning 19 - 27

2. Composition & Comprehension 9 - 18 4. Non Verbal Reasoning 28 - 36


http://t.me/iitjeehelps

SECTION I - PHYSICS
Chapter
Units and Measurements
1
1. In the formula X = 3 YZ2, X and Z have dimensions dy
of capacitance and magnetic induction 7. In the relation : = 2w sin (wt + f0) the
dx
respectively. The dimensions of Y in MKSA
dimensional formula for (wt + f0) is : [2012]
system are : [2017]
(a) [M–3L–2T–2A–4] (b) [ML–2] (a) MLT (b) MLT0
(c) [M–3L–2A4T8] (d) [M–3L2A4T4] (c) ML0T0 (d) M0L0T0
2. The frequency of vibration of string is given by
p éFù
½ ML3
v= ê ú 8. If T = 2 p then find the dimensions of q.
2l ë m û 3 Yq
Here p is number of segments in the string and l Where T is the time period of bar of mass M,
is the length. The dimensional formula for m will length L and Young modulus Y. [2011]
be [2016] (a) [L] (b) [L2] (c) [L4] (d) [L3]
(a) [M0LT–1] (b) [ML0T–1] 9. If P represents radiation pressure, c represents
(c) [ML –1T0] (d) [M0L0T0]
3. In a vernier callipers, n divisions of its main scale speed of light and Q represents radiation energy
match with (n + 1) divisions on its vernier scale. striking a unit area per second, the non-zero
Each division of the main scale is a units. Using in tegers x,y and z such th at P x Q y c z is
the vernier principle, calculate its least count. dimensionless, are. [2010]
a a (a) x = 1, y = 1, z = –1 (b) x = 1, y = –1, z = 1
(a) (b) [2014] (c) x = – 1, y = 1, z = 1 (d) x = 1, y = 1, z = 1
n +4 n +2
10. How many significant figures are there in
a a 0.30100? [2010]
(c) (d)
n+3 n +1 (a) 1 (b) 3
4. The expression [ML–1 T–2] does not represent (c) 5 (d) None of these
(a) pressure (b) power [2014]
(c) stress aZ
a - kq
(d) Young’s modulus 11. In the relation : P = e
5. The unit of specific resistance is [2013] b
(a) ohm/m2 (b) ohm m3 P is pressure, Z is distance, k is Boltzmann
(c) ohm m (d) ohm/m constant and q is the temperature. The
6. Wave pulse can travel along a tense string like a dimensional formula of b will be [2009]
violin spring. A series of experiments showed 0 2 0 1 2 1
that the wave velocity V of a pulse depends on (a) [M L T ] (b) [M L T ]
the following quantities, the tension T of the (c) [M1L0T–1] (d) [M0L2T–1]
string, the cross-section area A of the string and 12. Which of the following is most accurate?
then as per unit volume r of the string. Obtain [2009, 2007]
an expression for V in terms of the T, A and r (a) A screw gauge of least count 0.001 mm
using dimensional analysis. [2013] (b) A screw gauge having pitch 1 mm and 50
T T divisions on circular scale
(a) V = k (b) V = k (c) A vernier callipers of least count 0.01 mm
Ar A
(d) Vernier callipers having 20 divisions on the
Ar sliding scale (vernier scale) coinciding 19
(c) V = k (d) None of these divisions on the main millimetre scale.
T
EBD_7167
http://t.me/iitjeehelps

P– 2 BITSAT Topicwise Solved Papers


13. The dimensional formula for strain is same as (b) volt-second/ampere
that of [2008] (c) joule/(ampere)²
(a) thrust (b) angle (d) ohm-meter
(c) modulus of elasticity (d) stress 17. Which of the following group have different
14. What is the unit of magnetic permeability? dimension? [2005]
[2008] (a) Potential difference, EMF, voltage
(a) Wb A–1 m–1 (b) Wb–1 Am (b) Pressure, stress, Young’s modulus
(c) Wb A m–1 (d) Wb A–1 m (c) Heat, energy, work done
1.53 ´ 0.9995 (d) Dipole moment, electric flux, electric field
15. Find the value of with due regard 18. Which of the following quantities has a unit but
1.592 dimensionless? [2005]
for significant figures – [2007] (a) Strain
(a) 0.961 (b) 0.123 (c) 0.921 (d) 0.913 (b) Reynold’s number
16. The SI unit of the inductance, the henry cannot (c) Angular displacement
be written as [2006] (d) Poisson’s ratio
(a) weber/ampere

Hints & Solutions


dimensions of X Comparing power on both sides we have
1. (c) Dimensions of Y = 2 a + c = 0, a + 2b – 3c = 1, –2a = –1
dimensions of Z
M -1 -2 4 2
L T A 1 1 1 T
\a = , c = - Þ b = - \V = k
= = [M–3 L–2 T8 A4] 2 2 2 Ar
( MT -2 A-1 )2
7. (d)
1/ 2
p éFù
2. (c) u= ê ú ML3
2l ë m û 8. (c) T = 2p , writing dimensions of
3 Yq
p F p2F é ML3 ù
1/ 2
u2 = Þ m =
4l 2 m 4l 2 u 2 both the sides, we get [ T ] = ê -1 -2 ú
Now, dimensional formula of R.H.S. êë ML T q úû
or q = [L4]
MLT -2 9. (b) 10. (c) 11. (a)
= 2
= ML–1T0.
æ 1 ö 12. (a)
L2 ç ÷ 13. (b) Both are dimensionless
èTø 14. (a) From Biot Savart’s law
[p will have no dimension as it is an integer]
So, dimensions of m will be ML–1T0. m i dl sin q
B= 0
3. (d) 4p r2
4. (b)
4 p Br2 Wb m-2m2
energy ML2 T -2 m0 = = = Wb A-1 m-1
Power = = = [ ML2 T -3 ] idl sin q Am
time T
1.53 ´ 0.9995 1.529235
R A ohm m 2 15. (a) =
5. (c) r = = = ohm - m 1.592 1.592
l m = 0.9605747 = 0.961
6. (a) Let V = kTa Abrc, (Rounding off to three significant digits)
k = dimensional constant 16. (d)
Writing dimension on both we side 17. (d)
[LT–1] = [MLT–2]a [L2]b [ML–3]c 18. (c) Angular displacement has unit
= [Ma+cLa+2b–3cT–2a] (degree or radian) but it is dimensionless.
http://t.me/iitjeehelps

Chapter
Motion in a Straight Line
2
1. A frictionless wire AB is fixed on a sphere of 4. A ball is released from the top of height h metre.
radius R. A very small spherical ball slips on this It takes T second to reach the ground. Where is
wire. The time taken by this ball to slip from A to the ball at the time T/2 sec : [2013]
B is [2016] (a) At (h/4) m from the ground
(b) At (h/2) m from the ground
2 gR (c) At (3h/4) m from the ground
(a) (d) Depends upon the mass and volume of the
g cos q A
cos q ball
(b) 2 gR
q 5. A particle is moving in a straight line with initial
g velocity and uniform acceleration a. If the sum
O
of the distance travelled in t th and (t + 1)th
R
(c) 2 B R seconds is 100 cm, then its velocity after t
g seconds, in cm/s, is [2011]
gR C (a) 80 (b) 50 (c) 20 (d) 30
(d) 6. The position x of a particle varies with time (t) as
g cos q x = A t2 – B t3. The acceleration at time t of the
2. A 2 m wide truck is moving with a uniform speed particle will be equal to zero. What is the value
v0 = 8 m/s along a straight horizontal road. A of t? [2010, 2006]
pedestrain starts to cross the road with a uniform 2A A A
speed v when the truck is 4 m away from him. (a) (b) (c) (d) zero
3B B 3B
The minimum value of v so that he can cross the 7. The velocity acquired by a body moving with
road safely is [2015] uniform acceleration is 30 ms-1 in 2 seconds and
60 ms-1 in four seconds .The initial velocity is
(a) 4 ms–1 (b) 0 ms–1 [2008]
(c) 2 ms –1 (d) 10 ms–1
2m Truck v0 v
8. A balloon starts rising from the ground with an
acceleration of 1.25 ms–2. After 8 s, a stone is
Man released from the balloon. The stone will (Taking
4m g = 10 m s–2) [2007]
(a) 2.62 m/s (b) 4.6 m/s (a) begin to move down after being released
(c) 3.57 m/s (d) 1.414 m/s (b) reach the ground in 4 s
3. A body is thrown vertically upwards from A, (c) cover a distance of 40 m in reaching the
ground
the top of the tower, reaches the ground in time
(d) will have a displacement of 50 m.
t1. If it is thrown vertically downwards from A 9. A bus starts moving with acceleration 2 m/s2. A
with the same speed, it reaches the ground in cyclist 96 m behind the bus starts simultaneously
time t2. If it is allowed to fall freely from A, then towards the bus at 20 m/s. After what time will
the time it takes to reach the ground is given by he be able to overtake the bus? [2006]
[2015, 2009] (a) 4 sec (b) 8 sec (c) 12 sec (d) 16 sec
t +t t1 - t 2 10. A body, thrown upwards with some velocity
(a) t = 1 2 (b) t = reaches the maximum height of 50m. Another
2 2 body with double the mass thrown up with
t1 double the initial velocity will reach a maximum
(c) t = t1t 2 (d) t = t2
height of [2005]
(a) 100 m (b) 200 m (c) 300 m (d) 400 m
EBD_7167
http://t.me/iitjeehelps

P– 4 BITSAT Topicwise Solved Papers

Hints & Solutions


1. (c) Acceleration of body along AB is g cos q 3. (c)
Distance travelled in time t 2
1 2 1 æ Tö h
1 4. (c) h= gT ; y = g ç ÷ = from top
sec = AB = ( g cos q)t 2 2 2 è 2ø 4
2 3h
From DABC, AB = 2R cos q = from ground
4
1 5. (b)
Thus, 2 R cos q = g cos qt 2 6. (c) Given that x = A t2 – B t3
2
dx
4R R \ velocity = = 2 A t - 3B t 2
Þ t2 = Þt = 2 dt
g g d æ dx ö
and acceleration = ç ÷ = 2 A - 6 Bt
2. (c) Let the man starts crossing the road at an dt è dt ø
angle q as shown in figure. For safe crossing the For acceleration to be zero 2A – 6Bt = 0.
condition is that the man must cross the road by
2A A
the time the truck describes the distance 4 + AC \ t= =
or 4 + 2cot q. 6 B 3B
4 + 2 cot q 2 / sin q 7. (b) 30 = u + 2a
\ =
8 v 60 = u + 4a
8 Solving, u = 0 m/s
or v= … (i)
2 sin q + cos q 8. (b) v = 1.25 × 8 ms–1 = 10 ms–1
dv 1
For minimum v, =0 s = ´ 1.25 ´ 8 ´ 8m = 40m
dq 2
A C 1 2
Now , 40 = -10t + ´ 10 ´ t
2
or 5t2 – 10t – 40 = 0
2m Truck v0 v or t2 – 2t – 8 = 0 or t = 4 s.
9. (b) Let after a time t, the cyclist overtake the
q 1
B
bus. Then 96 + ´ 2 ´ t 2 = 20 ´ t or t2 – 20 t +
2
4m 96 = 0
- 8(2 cos q - sin q)
or =0 20 ± 400 - 4 ´ 96
( 2 sin q + cos q) 2 \t =
or 2 cos q – sin q = 0 2 ´1
or tan q = 2 20 ± 4
From equation (i), = = 8 sec . and 12 sec .
2
8 8 10. (b) Hmax µ u2, if body is projected with double
vmin = = = 3.57 m/s
æ 2 ö 1 5 velocity then maximum height will become four
2ç ÷+
è 5ø 5 times i.e. 200 m.
http://t.me/iitjeehelps

Chapter
Motion in a Plane
3
1. Given that A + B = R and A = B = R. What should (a) 1m (b) 2m (c) 3m (d) 4m
be the angle between A and B ? [2017] 7. A body is projected, making an acute angle with
(a) 0 (b) p/3 (c) 2p/3 (d) p r
the horizontal. If angle between velocity v and
2. Two projectiles A and B thrown with speeds in r
acceleration g is q, then [2013]
the ratio 1 : 2 acquired the same heights. If A
(a) q = 90º (b) q = 0º
is thrown at an angle of 45° with the horizontal,
(c) 90º < q < 0º (d) 0º < q < 180º
the angle of projection of B will be [2017]
8. When a particle is in uniform circular motion it
(a) 0° (b) 60° (c) 30° (d) 45°
does not have [2012]
3. A stone projected with a velocity u at an angle q
(a) radial velocity and radial acceleration
with the horizontal reaches maximum height H1.
(b) radial velocity and tangential acceleration
When it is projected with velocity u at an angle
(c) tangential velocity and radial acceleration
æp ö (d) tangential velocity and transverse
çè - q÷ø with the horizontal, it reaches maximum acceleration
2
height H2. The relation between the horizontal 9. The range of the particle when launched at an
range R of the projectile, heights H1 and H2 is angle of 15º with the horizontal is 1.5 km. What
[2016] is the range of the projectile when launched at
an angle of 45º to the horizontal. [2012]
(a) R = 4 H1H 2 (b) R = 4(H1 – H2)
(a) 1.5 km (b) 3.0 km
H12 (c) 6.0 km
(c) R = 4 (H1 + H2) (d) R = r r(d) 0.75 km
H 22 10. The two vectors A and B are drawn from a
r r r
4. The position of a projectile launched from the common point and C = A + B , then angle
r r r
( )
origin at t = 0 is given by r = 40iˆ + 50 ˆj m at t = between A and B is – [2011]
2s. If the projectile was launched at an angle q 2
(1) 90° if C = A + B2 2
from the horizontal, then q is (take g = 10 ms–2) (2) greater than 90° if C2 < A2 + B2
[2015] (3) greater than 90° if C2 > A2 + B2
-1 2 -1 3 (4) less than 90° if C2 > A2 + B2
(a) tan (b) tan Correct options are –
3 2
7 4 (a) 1, 2 (b) 1, 2, 3, 4
-1 -1
(c) tan (d) tan (c) 2, 3, 4 (d) 1, 2, 4
4 5
5. A projectile is fired with a velocity u making an 11. A passenger in a open car travelling at 30 m/s
angle q with the horizontal. What is the throws a ball out over the bonnet. Relative to
magnitude of change in velocity when it is at the the car the initial velocity of the ball is 20 m/s at
highest point – [2014] 60° to the horizontal. The angle of projection of
(a) u cos q (b) u the ball with respect to the horizontal road will
(c) u sin q (d) u cos q – u be [2011]
6. The velocity and acceleration vectors of a æ 2ö æ 3ö
particle undergoing circular motion are (a) tan –1 çè ÷ø (b) tan –1 ç 4 ÷
r r 3 è ø
v = 2iˆ m / s and a = 2iˆ + 4jˆ m / s 2 respectively
at an instant of time. The radius of the circle is – æ 4 ö æ 3ö
(c) tan –1 çè ÷ø (d) tan –1 çè ÷ø
[2013] 3 4
EBD_7167
http://t.me/iitjeehelps

P– 6 BITSAT Topicwise Solved Papers


r r
12. Two projectiles A and B are thrown with the same 17. A = 4iˆ + 3jˆ r and B = 4iˆ + 2ˆj . Find a vector
speed but angles are 40º and 50º with the parallel to A but has magnitude five times that
horizontal. Then [2010] r
of B . [2007]
(a) A will fall earlier
(b) B will fall earlier (a) ˆ
20 (2i + 3j)ˆ ˆ
(b) 20 (4i + 3j)ˆ
(c) both will fall at the same time (c) 20 (2iˆ + ˆj) (d) 10 (2iˆ + ˆj)
(d) None of these
18. A stone is projected from the ground with
13. A body is travelling in a circle at a constant speed. It
velocity 25 m/s. Two seconds later, it just clears
(a) has a constant velocity [2010] a wall 5m high. The angle of projection of the
(b) is not accelerated stone is (g = 10 m/sec2) [2007]
(c) has an inward radial acceleration (a) 30º (b) 45º (c) 50.2º (d) 60º
(d) has an outward radial acceleration 19. The position vector of a particle is
14. A projectile projected at an angle 30º from the r
r = (a cos wt )iˆ + (a sin wt ) ˆj. The velocity of the
horizontal has a range R. If the angle of projection
at the same initial velocity be 60º, then the range particle is [2006]
will be- [2009] (a) directed towards the origin
(a) R (b) R/2 (c) 2R (d) R2 (b) directed away from the origin
15. Given that A + B = R and A2 + B2 = R2. The angle (c) parallel to the position vector
between A and B is [2009] (d) perpendicular to the position vector
(a) 0 (b) p/4 (c) p/2 (d) p 20. A particle is moving in a circular path. The
16. A cannon ball has the same range R on a acceleration and momentum vectors at an instant
r r
horizontal plane for two angles of projection. If of time are a = 2iˆ + 3jˆ m/s2 and p = 6iˆ - 4jˆ kgm/
h1 and h2 are the greatest heights in the two s. Then the motion of the particle is [2005]
paths for which this is possible, then [2008] (a) uniform circular motion
(a) R = h1 h2 (b) R = 4 h1 h 2 (b) circular motion with tangential acceleration
1 (c) circular motion with tangential retardation
(c) R = 3 h1 h 2 (d) R = ( h1 h 2 ) 4 (d) we cannot say anything about a and p only.

Hints & Solutions


1. (c) R2 = [A2 + B2 + 2AB cos q] 2
æu ö
R2 = R2 + R2 + 2R2 cos q sin q = ç A ÷ sin 2 45°
2

–R2 = 2R2 cos q or cos q = –1/2 è uB ø


or q = 2p/3 2 2
2. (c) For projectile A æ 1 ö æ 1 ö 1
sin 2 q = ç ÷ ç ÷ =
u 2A sin 2 45° è 2ø è 2ø 4
Maximum height, HA = 1 æ1ö
2g sin q = Þ q = sin -1 ç ÷ = 30°
For projectile B 2 è2ø
u 2B sin 2 q u 2 sin 2 q
Maximum height, HB = 3. (a) H1 =
2g 2g
As we know, HA = HB u 2 sin 2 (90° - q) u 2 cos 2 q
and H 2 = =
u 2A sin 2 45° u 2B sin 2 q 2g 2g
=
2g 2g u 2 sin 2 q u 2 cos 2 q (u 2 sin 2q)2 R 2
H1 H 2 = ´ = =
sin 2 q u 2A 2g 2g 16 g 2 16
=
sin 2 45° u 2B \ R = 4 H1 H 2
http://t.me/iitjeehelps

Motion in a Plane P– 7
4. (c) From question, 13. (c) Body moves with constant speed it means
Horizontal velocity (initial), that tangential acceleration a T = 0 & only
40 centripetal acceleration aC exists whose direction
ux = = 20m/s is always towards the centre or inward (along
2
1 2 the radius of the circle).
Vertical velocity (initial), 50 = uy t + gt 14. (a) If sum of angle of projection = 90° for given
2
1 speed then range for that angle of projection is
Þ uy × 2 + (–10) ×4 same.
2
or, 50 = 2uy – 20 R 2 - A 2 - B2 R2 - R2
15. (c) cos q = = =0
70 2AB 2A B
or, uy = = 35m / s
2 \ q = p/2
u 35 7
\ tan q = y = = u 2 sin 2 a
u x 20 4 16. (b) h1 = ,
2g
7
Þ Angle q = tan–1 u 2 sin 2 (90 - a ) u 2 cos 2 a
4 h2 = =
2g 2g
5. (c) Initially u = cos a ˆi + u sin a ˆj .
2 2
At highest point v = u cos a ˆi 1 æ u 2 sin a cos a ö 1 æ Rö
h1h 2 = ç ÷ = çè ÷ø ,
\ difference is u cos a. C is correct 4è g ø 4 2
6. (a) It can be observed that component of
R = 4 h1h 2
acceleration perpendicular to velocity is a
= 4 m/s2 17. (b) Required vector
r r æ ˆ ˆö
v2 (2)2 ˆ = 5 20 4i + 3j = 20 (4iˆ + 3j)
R = 5| B| A ˆ
\ radius = = = 1 metre ç 5 ÷
ac 4 è ø
7. (d) Here velocity is acting upwards when 1
projectile is going upwards and acceleration 18. (a) 5 = 25sin q ´ 2 - g(2) 2 , q = 30°
r r is 2
downwards. The angle q between v and a is r
more than 0º and less than 180º. 19. (d) r = (a cos wt )iˆ + (a sin wt ) ˆj
r
8. (c) r d (r ) d
v= = {(a cos wt )iˆ + (a sin wt ) ˆj}
u 2 sin 90 dt dt
u 2 sin 30
9. (b) 1.5 = ; R= = 3km = (- aw sin wt )iˆ + (aw cos wt ) ˆj
g g
10. (d) = w[(- a sin wt )iˆ + (a cos wt ) ˆj ]
r r r
11. (b) (vbc ) x =(vb )x - (vc ) x a sin wt
r Slope of position vector = = tan wt
20cos 60° = (vb ) x - 30 a cos wt
r r r r and slope of velocity vector
(vb ) x = 40 ; (vbc ) y = (v b ) y - (vc ) y
r - a cos wt -1
20sin 60° = (vb ) y - 0
= =
r a sin wt tan wt
r (v b ) y 10 3 3
(v b ) y = 10 3 ; tan q = r = = \ velocity is perpendicular to the displacement.
(v b )x 40 4 20. (d) The nature of motion can be determined
2 u sin q only if we know velocity and acceleration as
12. (a) T = , lesser is the value of q, lesser function of time.
g
is sinq and hence lesser will be the time taken. Here acceleration at an instant is given and not
Hence A will fall earlier. known at other times.
EBD_7167
http://t.me/iitjeehelps

P– 8 BITSAT Topicwise Solved Papers

Chapter
Laws of Motion
4
10 g and a muzzle velocity of 800 ms–1. The
1. Two equal heavy spheres, each of radius r, are
in equilibrium within a smooth cup of radius 3r. velocity which the rifle man attains after firing
The ratio of reaction between the cup and one 10 shots is [2014]
(a) 8 ms –1 (b) 0.8 ms –1
sphere and that between the two sphere is
(c) 0.08 ms–1 (d) – 0.8 ms–1
5. A person with his hand in his pocket is skating
on ice at the rate of 10m/s and describes a circle
of radius 50 m. What is his inclination to vertical:
[2017] (g = 10 m/sec2) [2014]
(a) tan–1(½) (b) tan–1 (1/5)
(c) tan–1 (3/5) (d) tan –1(1/10)
(a) 1 (b) 2 (c) 3 (d) 4 6. The minimum velocity (in ms-1) with which a car
2. The masses of blocks A and B are m and M driver must traverse a flat curve of radius 150 m
respectively. Between A and B, there is a and coefficient of friction 0.6 to avoid skidding
constant frictional force F and B can slide on a is [2013]
smooth horizontal surface. A is set in motion (a) 60 (b) 30 (c) 15 (d) 25
7. A bob is hanging over car
with velocity while B is at rest. What is the a pulley inside a car
distance moved by A relative to B before they through, a string. The a
a
move with the same velocity? [2015] secon d en d of the
v0 string is in the hand m
A m of a person standing
B M in the car. The car is
moving with constant acceleration 'a' directed
mMv20 mMv20 horizontally as shown in figure. Other end of the
(a) (b) string is pulled with constant acceleration ‘a’
F (m - M) 2F ( m - M )
vertically. The tension in the string is equal to –
mMv02 mMv 02 [2013]
(c) (d)
F (m + M ) 2F ( M + m ) (a) m g2 + a 2 (b) m g 2 + a 2 - ma
3. In figure, two blocks are separated by a uniform
strut attached to each block with frictionless (c) m g 2 + a 2 + ma (d) m (g + a)
pins. Block A weighs 400N, block B weighs 8. A man of mass 100 kg. is standing on a platform
300N, and the strut AB weigh 200N. If µ = 0.25 of mass 200 kg. which is kept on a smooth ice
surface. If the man starts moving on the platform
under B, determine the minimum coefficient of
with a speed 30 m/sec relative to the platform
friction under A to prevent motion. [2015] then calculate with what velocity relative to the
ice the platform will recoil? [2012]
/////

B
(a) 5 m/sec (b) 10 m/sec
//////

(c) 15 m/sec (d) 20 m/sec.


//////

30°
9. An object experiences a net force and accelerates
///////

A 60°
////////////////////////////////// from rest to its final position in 16s. How long
(a) 0.4 (b) 0.2 (c) 0.8 (d) 0.1 would the object take to reach the same final
4. A rifle man, who together with his rifle has a position from rest if the object's mass was four
mass of 100 kg, stands on a smooth surface and times larger ? [2011]
fires 10 shots horizontally. Each bullet has a mass (a) 64 s (b) 32 s (c) 16 s (d) 8s
http://t.me/iitjeehelps

Laws of Motion P– 9
10. Three blocks of masses m1, m2 and m3 are (c) Newton’s laws are applicable in this frame
connected by massless strings, as shown, on a (d) both (a) and (b)
frictionless table. They are pulled with a force 15. The mass of the lift is 100 kg which is hanging
T3 = 40 N. If m1 = 10 kg, m2 = 6 kg and m3 = 4kg, on the string. The tension in the string, when
the tension T2 will be [2011] the lift is moving with constant velocity, is (g =
T1 T2 T3 9.8 m/sec2) [2008]
M1 M2 M3 (a) 100 newton (b) 980 newton
(c) 1000 newton (d) None of these
(a) 20 N (b) 40 N (c) 10 N (d) 32 N 16. A shell explodes and many pieces fly off in
11. Two blocks are different directions. The following is conserved:
connected over a (a) Kinetic energy [2007]
massless pulley as A (b) Momentum
shown in fig. The mass (c) Neither momentum nor KE
of block A is 10 kg and 30º B (d) Momentum and KE.
the coefficient of kinetic friction is 0.2. Block A 17. In the figure shown the velocity of lift is 2 m/s
slides down the incline at constant speed. The mass while string is winding on the motor shaft with
velocity 2 m/s and block A is moving downwards
of block B in kg is: [2010]
with a velocity of 2 m/s, then find out the velocity
(a) 3.5 (b) 3.3 (c) 3.0 (d) 2.5
of block B – (2006)
12. A spring is compressed between two toy carts
of mass m1 and m2 . When the toy carts are 2m/s

released, the springs exert equal and opposite


average forces for the same time on each toy
cart. If v1 and v2 are the velocities of the toy
carts and there is no friction between the toy B A

carts and the ground, then : [2010] 2m/s

(a) v1/v2 = m1/m2 (b) v1/v2 = m2/m1 (a) 2 m/s ­ (b) 2 m/s ¯
(c) v1/v2 = –m2/m1 (d) v1/v2 = –m1/m2 (c) 4 m/s ­ (d) None of these
13. A block of mass M is pulled along a horizontal 18. Inside a horizontally moving box, an
frictionless surface by a rope of mass M/2. If a experimenter finds that when an object is placed
force 2Mg is applied at one end of the rope, the on a smooth horizontal table and is released, it
force which the rope exerts on the block is – moves with an acceleration of 10 m/s2. In this
[2009] box if 1 kg body is suspended with a light string,
(a) 2Mg/3 (b) 2Mg (c) 4Mg/3 (d) zero the tension in the string in equilibrium position
14. A reference frame attached to earth cannot be (w.r.t. experimenter) will be (Take g = 10 m/s2)
an inertial frame because [2008]
(a) earth is revolving around the sun (a) 10 N (b) 10 2 N [2005]
(b) earth is rotating about its axis (c) 20 N (d) zero

Hints & Solutions


mg mg N
1 q N1 F
1. (b) sin q =
2 A
Thus, N1 sin q = N2 N2 B
F N N¢
N1 1
\ N = = 2. Equations of motion
2 sin q
2. (d) For the blocks A and B FBD as shown F
a A = ( in - x direction )
below M
F
a B = ( in + x direction )
M
EBD_7167
http://t.me/iitjeehelps

P – 10 BITSAT Topicwise Solved Papers


Relative acceleration, of A w.r.t. B, T3
a=
F F m1 + m 2 + m3
a A,B = a A - a B = - -
m M For equilibrium of m1 & m2
æM+mö T2 = (m1 + m2) × a
= -F ç ÷ (along – x direction) (m1 + m 2 )T3
è Mm ø or, T2 =
Initial relative velocity of A w.r.t. B, uAB = v0 m1 + m 2 + m3
using equation v2 = u2 + 2as Given m1 = 10 kg, m2 = 6 kg, m3 = 4 kg,
2F ( m + M ) S Mmv 20 T3 = 40 N
0 = v 20 - Þ S= (10 + 6).40
Mm 2F ( m + M ) \ T2 = = 32N
10 + 6 + 4
i.e., Distance moved by A relative to B 11. (b) Considering the equilibrium of A, we get
Mmv 02 10a = 10g sin 30º – T – mN
SAB =
2F ( m + M ) where N = 10g cos 30°
N a
3. (a) T
4. (b) According to law of conservation of
mN T a
momentum, 30 º
A
si n
10 10g B
100v = - ´ 10 ´ 800 10g cos30º
1000 mBg
i.e., v = 0.8 ms–1. 10g
5. (b) Since surface (ice) is frictionless, so the 10
\ 10 a = g - T - m ´ 10 g cos 30 º
centripetal force required for skating will be 2
provided by inclination of boy with the vertical but a = 0, T = mBg
and that angle is given as
0 .2 3
v2 0 = 5g - m B g - × 10 ×g
tan θ = where v is speed of skating & r is 2
rg Þ mB = 3.268 » 3.3 kg
radius of circle in which he moves. 12. (c) Applying law of conservation of linear
6. (b) The condition to avoid skidding, momentum
m1v1 + m2v2 = 0,
v= mrg = 0.6 ´150 ´10 = 30 m/s. m1 v v m
7. (c) Applying Newton’s = - 2 or 1 = - 2
m2 v1 v2 m1
law along string 13. (c)
2 2
T 14. (c) If earth is revolving around the sun or earth
Þ T - m g + a = ma is rotating about its axis, then forces are acting
ma m a
on the earth and hence there will be acceleration
or T = m g 2 + a 2 + ma a of earth due to these forces. That is why earth
8. (b) mg can not be an inertial frame of reference.
9. (b) When the mass increases by a factor of 4 the 15. (b) T = m (g + a) = 100 (9.8 + 0) = 980 N
acceleration must decrease by a factor of four if 16. (b)
the same force is applied. The question asks about r
17. (d) v B,l = 4 m / s ­
position so we need to relate acceleration and time r r r
vB,l = vB,g - vl,g
to position. We can do this by the equation : xf – xi r
4m / s = vB,g - 2m / s v
= vxi t + ½ ax t2 r B A
We want the change in position to stay the same. Þ v B,g = 6 m / s ­ 4m/s

The initial velocity is zero so in order for the 18. (b) Acceleration of
change in position to remain constant the term box = 10 m/s²
Inside the box ma
(1/2) at2 must remain the same. If the acceleration
forces acting on (Pseudo
is reduced by a factor of 4 you can see that the
time must be increased by a factor of 2 in order bob are shown in force)
for the term to remain the same. the figure.
10. (d) For equilibrium of all 3 masses,
T = (mg) 2 + (ma) 2 = 10 2 N
http://t.me/iitjeehelps

Chapter
Work, Energy and Power
5
1. A particle of mass m moving in the x direction 7. Work done by a conservative force is positive if
with speed 2v is hit by another particle of mass (a) P.E. of the body increases [2013]
2m moving in the y direction with speed v. If the (b) P.E. of the body decreases
collision is perfectly inelastic, the percentage (c) K.E. of the body increases
loss in the energy during the collision is close to (d) K.E. of the body decreases
[2017] 8. If the unit of force and length be each increased
(a) 56% (b) 62% (c) 44% (d) 50% by four times, then the unit of energy is increased
2. Consider elastic collision of a particle of mass m by [2012]
moving with a velocity u with another particle of (a) 16 times (b) 8 times
the same mass at rest. After the collision the (c) 2 times (d) 4 times
projectile and the struck particle move in 9. Which of the following must be known in order
directions making angles q1 and q2 respectively to determine the power output of an automobile?
with the initial direction of motion. The sum of (a) Final velocity and height [2012, 2006]
the angles q1 + q2, is : [2016] (b) Mass and amount of work performed
(a) 45° (b) 90° (c) 135° (d) 180° (c) Force exerted and distance of motion
3. A neutron moving with speed v makes a head (d) Work performed and elapsed time of work
on collision with a hydrogen atom in ground 10. A massless platform is kept on a light elastic
state kept at rest. The minimum kinetic energy spring as shown in fig. When a sand particle of
of the neutron for which inelastic collision takes mass 0.1 kg is dropped on the pan from a height
place is [2015] of 0.24 m, the particle strikes the pan and spring
(a) 10.2 eV (b) 20.4 eV is compressed by 0.01 m. [2011]
(c) 12.1 eV (d) 16.8 eV From what height should the particle be dropped
4. An elastic string of unstretched length L and to cause a compression of 0.04 m.
force constant k is stretched by a small length x. (a) 3.96 m (b) 0.396 m (c) 4 m (d) 0.4 m
It is further stretched by another small length y.
The work done in the second stretching is
[2015]
(a) 1/2 Ky2 (b) 1/2 Ky(2x + y)
(c) 1/2 K(x2 + y2) (d) 1/2 k (x + y)2
5. A small block of mass m is kept on a rough
inclined surface of inclination q fixed in a elevator.
The elevator goes up with a uniform velocity v r
and the block does not slide on the wedge. The 11. The potential energy for a force field F is given
work done by the force of friction on the block by U (x,y) = cos (x + y). The force acting on a
in time t will be : [2014] particle at position given by coordinates
(a) zero (b) mgvt cos2q (0, p / 4) is – [2010]
(c) mgvt sin2q (d) mgvt sin 2q
6. A 3.628 kg freight car moving along a horizontal 1 ˆ ˆ 1 ˆ ˆ
(a) - (i + j) (b) (i + j)
rail road spur track at 7.2 km/hour strikes a 2 2
bumper whose coil springs experiences a
æ1 3 ˆö æ1 3 ˆö
maximum compression of 30 cm in stopping the ˆ ˆ
(c) ç 2 i + 2 j÷ (d) ç 2 i - 2 j÷
car. The elastic potential energy of the springs è ø è ø
at the instant when they are compressed 15 cm 12. A long string is stretched by 2 cm and the
is [2013] potential energy is V. If the spring is stretched
(a) 12.1 × 104 J (b) 121 × 104 J by 10 cm, its potential energy will be [2010]
(c) 1.21 × 104 J (d) 1.21 × 106 J (a) V / 25 (b) V/5 (c) 5 V (d) 25 V
EBD_7167
http://t.me/iitjeehelps

P – 12 BITSAT Topicwise Solved Papers


13. A chain of mass M is placed on a smooth table 18. Which one of the following statements does not
with 1/n of its length L hanging over the edge. hold good when two balls of masses m1 and m2
The work done in pulling the hanging portion of undergo elastic collision [2006]
the chain back to the surface of the table is (a) When m1 < m2 and m2 at rest, there will be
(a) MgL/n (b) MgL/2n [2009] maximum transfer of momentum.
(c) MgL/n2 (d) MgL/2n2 (b) When m1 > m2 and m2 at rest, after collision
14. A particle of mass 10 kg moving eastwards with the ball of mass m2 moves with four times
a speed 5 ms–1 collides with another particle of the velocity of m1.
the same mass moving north-wards with the (c) When m1 = m2 and m2 at rest, there will be
same speed 5 ms–1. The two particles coalesce maximum transfer of K. E.
on collision. The new particle of mass 20 kg will (d) When collision is oblique and m2 at rest
move in the north-east direction with velocity with m1 = m2 after collision the ball moves
(a) 10 ms–1 (b) 5 ms–1 [2009] in opposite directions.
(c) (5 / 2)ms -1 (d) none of these 19. In the figure shown the potential energy U of a
particle is plotted against its position ‘x’ from
15. A ball is allowed to fall from a height of 10 m. If origin. Then which of the following statement is
there is 40% loss of energy due to impact, then correct. A particle at – [2005]
after one impact ball will go up to [2008] (a) x1 is in stable equilibrium
(a) 10 m (b) 8 m (c) 4 m (d) 6 m (b) x2 is in stable equilibrium
16. When two bodies stick together after collision,
the collision is said to be [2007] (c) x3 is in stable equilibrium
(d) None of these
(a) partially elastic
U
(b) elastic
(c) inelastic
(d) perfectly inelastic
17. A sphere of mass m, moving with velocity V,
enters a hanging bag of sand and stops. If the x1 x2 x3 x
mass of the bag is M and it is raised by height h, O
then the velocity of the sphere was – [2007] 20. A mass m1 moves with a great velocity. It strikes
M+m M another mass m2 at rest in a head on collision. It
(a) 2gh (b) 2gh comes back along its path with low speed, after
m m
collision. Then [2005]
m m (a) m1 > m2 (b) m1 < m2
(c) 2gh (d) 2gh
M+m M (c) m1 = m2 (d) m1 ³ m2

Hints & Solutions


Y
2 2mv = 3mV
pf = 3 m V
2 2v
m 2v Þ = Vcombined
1. (a) pi 45°
X
3
Loss in energy
1 1 1
v DE = m1V12 + m2V22 - (m1 + m2 )Vcombined
2
2m 2 2 2
4 5
Initial momentum of the system DE = 3mv2 - mv2 = mv2 = 55.55%
3 3
pi = m ´ 2 viˆ + 2m ´ vjˆ 2. (b) u1
y axis
A
v1
u =0 2

= (m ´ 2v)2 + (2m ´ v )2 (magnitude) q


m1
x axis
m1 m2 f
= 2 2 mv A
B
Final momentum of the system = 3mV B m2
v2

By the law of conservation of momentum


Before collision After collision
http://t.me/iitjeehelps

Work, Energy and Power P – 13


Given m1 = m2 1
We will apply the principle of conservation of Fx = sin(x + y) ](0,p / 4) = ,
2
momentum in the mutually perpendicular dirn. 1
Along x-axis, m1u1 = m1v1 cosq + m2 v2 cosf Fy = sin(x + y)](0, p / 4) = \ F=
1 ˆ ˆ
[i + j]
or u1 = v1 cosq + v2 cosf ...(i) 2 2
Along y-axis, 0 = m1v1 sinq - m2 v2 sinf 2V V
1 1
or 0 = v1 sinq – v2 cosf ...(ii) 12. (d) V = k ( x ) 2 = k (2) 2 or k = =
Again for elastic collision, kinetic energy is 2 2 4 2
conserved 1 2 1 æVö 2
V¢ = k (10) = ´ ç ÷ (10) = 25V
1 1 1 2 2 è2ø
Þ mu 12 = mv12 + mv 2 2 13. (d) W = change in PE of COM of hanging part
2 2 2
M L MgL
or u12 =v12 + v22 ...(iii) = g =
n 2n 2n 2
Squaring and adding (i) & (ii), we get r
u12 = v12 (cos2q + sin2q) + v22 (cos2f + sin2f) 14. (c) Here ˆimv + ˆjmv = 2mV
+ 2v1v2 cosq cosf – 2v1v2 sinq sinf r v
or u12 = v12 + v22 + 2v1v2 cos (q + f) ...(iv) That is V = (iˆ + ˆj)
2
Using (iii) & (iv), we get v v
Hence V = ´ 2 = . [Here v = 5 ms–1]
p p 2 2
cos (q + f) = 0 = cos Þ q+f =
2 2 5
Note : This is a standard case of oblique collision. So, V = ms –1
3. (b) 2
4. (b) In the string elastic force is conservative 15. (d) Kinetic energy of ball when reaching the
in nature. ground
\ W = – DU = mgh = mg × 10
Kinetic energy after the impact
Work done by elastic force of string,
60
W = – (UF – Ui) = Ui – UF. = ´ mg ´ 10 = 6mg
100
1 2 k
W = kx - ( x + y )
2 If the ball rises to a height h, then mgh = 6 mg.
2 2 Hence, h = 6 m.
1 2 1
2 2
( 2
= kx - k x + y + 2xy
2
) 16. (d) When the two bodies stick together after
collision, then it is perfectly inelastic collision
1 2 1 2 1 2 1 and in this case, the coefficient of restitution e is
= kx - ky - kx - k ( 2xy ) equal to zero.
2 2 2 2 17. (a) By the conservation of linear momentum
1 2 Initial momentum of sphere = Final momentum
= -kxy - ky
2 of system
Therefore, the work done against elastic force
mV = (m + M)vsys. ...... (1)
ky
Wexternal = - W = ( 2x + y ) If the system rises up to height h then by the
2 conservation of energy
5. (c) 6. (c) 7. (b) 1 2
8. (a) Since unit of energy = (unit of force).(unit (m + M)vsys. = (m + M) gh Þ v sys. = 2gh
of length) so if we increase unit of length and 2
force, each by four times, then unit of energy Substituting this value in equation (1)
will increase by sixteen times. æ m+ Mö
V =ç 2gh
9. (d) Power is defined as the rate of doing work. è m ÷ø
For the automobile, the power output is the 18. (d)
amount of work done (overcoming friction)
divided by the length of time in which the work 19. (d) x = x1 and x = x3 are not equilibrium positions
was done. dU
10. (b) because ¹ 0 at these points.
dx
¶U x = x2 is unstable, as U is maximum at this point.
11. (b) Fx = - = sin(x + y) ,
¶x (m1 - m 2 ) u1
¶U 20. (b) v1 = As v1 is negative and
Fy = - = sin(x + y) m1 + m 2
¶x less than u1, therefore, m1 < m2.
EBD_7167
http://t.me/iitjeehelps

P – 14 BITSAT Topicwise Solved Papers

Chapter
System of Particles and
RotationalMotion 6
1. A thin but rigid semicircular wire frame of radius F
r is hinged at O and can rotate in its own vertical
plane. A smooth peg P starts from O and moves a a
horizontally with constant speed v0, lifting the
frame upward as shown in figure. [2017]
a
If the coefficient of friction is sufficiently high
so that the prism does not slide before toppling,
then the minimum force required to topple the
prism is – [2014]
mg mg mmg mmg
Find the angular velocity w of the frame when its (a) (b) (c) (d)
diameter makes an angle of 60° with the vertical : 3 4 3 4
5. A particle of mass ‘m’ is projected with a velocity
(a) v0/r (b) v0/2r (c) 2v0/r (d) v0r v making an angle of 30° with the horizontal. The
R magnitude of angular momentum of the projectile
2. A circular disc of radius R and thickness has
6 about the point of projection when the particle
moment inertia I about an axis passing through is at its maximum height ‘h’ is [2013]
its centre perpendicular to its plane. It is melted
3 mv 2
and recasted into a solid sphere. The moment of (a) (b) zero
inertia of the sphere about its diameter is[2016] 2 g
2I I I mv 3 3 mv3
(a) I (b) (c) (d) (c) (d)
8 5 10 2g 16 g
3. A thin rod of length 4l and mass 4m is bent at 6. A disc is performing pure rolling on a smooth
the points as shown in figure. What is the stationary surface with constant angular velocity
moment of inertia of the rod about the axis as shown in figure. At any instant, for the lower
passes through point O and perpendicular to most point of the disc – [2013]
the plane of paper? [2015] v/R
Ml 2
(a) O
3
l R v
10Ml 2
(b)
3 90° 90° (a) velocity is v, acceleration is zero
l l
Ml 2 (b) velocity is zero, acceleration is zero
(c) (c) velocity is v, acceleration is v2/R
12 l (d) velocity is zero, acceleration is v2/R
Ml 2 7. What is the moment of inertia of a solid sphere
(d) of density r and radius R about its diameter?
24
4. An equilateral prism of mass m rests on a rough [2012, 2009]
horizontal surface with coefficient of friction µ. 105 5 105 2
A horizontal force F is applied on the prism as (a) R r (b) R r
176 176
shown in the figure.
176 5 176 2
(c) R r (d) R r
105 105
http://t.me/iitjeehelps

System of Particles and Rotational Motion P – 15


8. A wheel of radius R rolls on the ground with a 13. A thin uniform circular disc of mass M and radius
uniform velocity v. The relative acceleration of R is rotating in a horizontal plane about an axis
topmost point of the wheel with respect to the passing through its centre and perpendicular to
bottom most point is [2012] the plane with angular velocity w. Another disc
A of same mass but half the radius is gently placed
v² over it coaxially. The angular speed of the
R composite disc will be : [2008]
(a) (5/4) w (b) (4/5) w
(c) (2/5) w (d) (5/2) w
v² 14. One solid sphere and disc of same radius are
R falling along an inclined plane without slipping.
B One reaches earlier than the other due to
2
v 2v 2 v2 4v 2 (a) different radius of gyration [2008]
(a) (b) (c) (d)
R R 2R R (b) different sizes
9. A constant torque of 31.4 N-m is exerted on a (c) different friction
pivoted wheel. If angular acceleration of wheel is (d) different moment of inertia
4 p rad/s2 , then the moment of inertia of the wheel 15. Two uniform solid spheres having unequal
is [2011] masses and unequal radii are released from rest
(a) 2.5 kg m2 (b) 3.5 kg m2 from the same height on a rough incline. If the
(c) 4.5 kg m2 (d) 5.5 kg m2 spheres roll without slipping, [2007]
10. The ratio of the accelerations for a solid sphere (a) the heavier sphere reaches the bottom
(mass ‘m’ and radius ‘R’) rolling down an incline first
of angle ‘q’ without slipping and slipping down (b) the bigger sphere reaches the bottom first
the incline without rolling is : [2010] (c) the two spheres reach the bottom together
(a) 5 : 7 (b) 2 : 3 (d) the information given is not sufficient to
(c) 2 : 5 (d) 7 : 5 tell which sphere will reach the bottom first.
11. A system consists of three particles, each of mass 16. A thin wire of length L and uniform linear mass
m and located at (1, 1), (2, 2) and (3, 3). The co- density r is bent into a circular loop with centre
ordinates of the centre of mass are [2010] at O as shown in the figure. The moment of inertia
(a) (1, 1) (b) (2, 2) of the loop about the axis XX' is : [2006]
(c) (3, 3) (d) (6, 6) X X'
12. A uniform cube of side a and mass m rests on a
rough horizontal table. A horizontal force F is
applied normal to one of the faces at a point that O
is directly above the centre of the face, at a height
3a/4 above the base. The minimum value of F for
3rL3 rL3
which the cube begins to topple an edge is (a) (b)
(assume that cube does not slide) [2009] 16p 2 16p 2
5rL3 3rL3
(c) 2 (d)
F 16p 8p 2
3a/4 17. A smooth sphere A is moving on a frictionless
horizontal plane with angular speed w and centre
of mass velocity v. It collides elastically and head
on with an identical sphere B at rest. Neglect
mg mg friction everywhere. After the collision, their
(a) (b)
3 2 angular speeds are wA and wB, respectively. Then
2mg 3mg (a) wA< wB (b) wA= wB [2005]
(c) (d) (c) wA= w (d) wB= w
3 4
EBD_7167
http://t.me/iitjeehelps

P – 16 BITSAT Topicwise Solved Papers

Hints & Solutions


x r Putting all values we get
1. (a) = æ Ml 2 ö
sin 2q sin(90 - q)
Moment of inertia, I = 10 ç ÷
Þ x = 2r sin q ç 3 ÷
è ø
dx dq 4. (a) The tendency of rotation will be about the
\ = 2r cos q ´ point C.
dt dt
F A
dq dx / dt v0 v
= = = 0
dt 2r cos q 2r cos 60° r a a
2. (c) According to problem disc is melted and a 3
2
recasted into a solid sphere so their volume will
be same.
B a C
2 4 3 mg
VDisc = VSphere Þ pRDisc t = pRSphere
3 For minimum force, the torque of F about C has
to be equal to the torque of mg about C.
2 æ RDisc ö 4 3 é RDisc ù
Þ pRDisc ç 6 ÷ = 3 pRSphere êt = 6 , given ú æ 3ö æaö mg
è ø ë û \ F ç a 2 ÷ = mg çè 2 ÷ø Þ F =
è ø 3
3 3 RDisc 5. (d)
Þ RDisc = 8RSphere Þ RSphere =
2
Moment of inertia of disc
1 v
2
I Disc = MRDisc = I (given) v cos q
2 H
\ M (RDisc)2 = 2I q

1 2 Angular momentum
Moment of inertia of sphere ISphere = MRSphere
2 L0 = pr^
2 ( Q linear momentum p = mv cos q and r^ = H)
2 æ RDisc ö M 2I I Þ L = mv cos qH
=M = ( RDisc )2 = =
5 çè 2 ÷ø 10 10 5 0
3 v 2 sin 2 30° 3 mv3
3. (b) Total moment of inertia = mv · =
2 2g 16 g
= I1 + I2 + I3 + I4 = 2I1 + 2I2
= 2(l1 + l2) [I3 = I1, I1 = I4] 6. (d) As the disc is in combined rotation and
translation, each point has a tangential velocity
MI2 and a linear velocity in the forward direction.
Now, I2 = I3 =
3 From figure
Using parallel axes theorem, we have
v/R
2 l2 2
I = ICM + Mx and x = l +
4
2
é 2ù
Ml 2 æl ö
I1 = I4 = + M ê l2 + ç ÷ ú v=R
12 ê è2ø ú
ë û
http://t.me/iitjeehelps

System of Particles and Rotational Motion P – 17

vnet (for lowest point)


y=
( y1 + y 2 + y3 ) m = 2
= v – Rw = v – v = 0 m+m+m
v2 v2 so coordinates of C.M. of three particle are (2,2)
and acceleration = +0=
R R V = 300 m/s
(since linear speed is constant) 12. (c) For toppling about edge xx'
7. (c) For solid sphere At the moment of toppling the normal force pass
through axis xx'.
2 2æ4 ö
I= M R 2 = ç p R 3 r÷ R 2 3a a 2mg
5 5è3 ø Fmin = mg or Fmin =
4 2 3
176 5 13. (b) Apply conservation of angular momentum.
r= R r
105 14. (a) (i) For solid sphere, the moment of inertia
8. (b) As a CM = 0 [vCM = constant], Tangential 2 2
about the diameter is I s = MR
r 2v 2 5
acceleration of each point | a AB | = Now I = MK2 for any body, where K is radius of
R
gyration of that body.
τ 31.4 31.4
9. (a) Ι= = = = 2.5 kg m 2 . 2
so MK 2 = MR 2 Þ K = R 2 / 5 ...(i)
α 4 π 4 ´ 3.14 5
10. (a) For solid sphere rolling without slipping on
(ii) The moment of inertia of disc about an axis
inclined plane, acceleration
passing through its centre & perpendicular to
g sin q plane is
a1 =
K2 MR 2
1+ 2 Id = = MK 2 Þ K = R ½ ...(ii)
R 2
Now acceleration of any body which is rolling
For solid sphere slipping on inclined plane
on an inclined plane is
without rolling, acceleration
a2 = g sin q g sin q ...(iii)
a=
a 1+ K2 / R 2
Therefore required ratio = 1
a2 For same R, the acceleration of the body depends
1 1 5 only on radius of gyration K, [see eq(iii)] so solid
= = =
K2 1 + 2 7 sphere will reach earlier to bottom of an inclined
1+ 2 5 plane than disc.
R
11. (b) The coordinates of C.M of three particle 15. (c)
are 3
16. (d) I = MR 2 ; M = rL, L = 2pR
m x + m 2x 2 + m3x3 m y + m 2 y 2 + m 3 y3 2
x= 1 1 &y = 1 1
m1 + m 2 + m 3 m1 + m 2 + m 3 17. (c) Since the spheres are smooth, there will be
no transfer of angular momentum from the sphere
here m1 = m2 = m3= m
A to sphere B. The sphere A only transfers its
so x =
( x1 + x 2 + x 3 ) m = 2 , linear velocity v to the sphere B and will continue
m+m+m to rotate with the same angular speed w.
EBD_7167
http://t.me/iitjeehelps

P – 18 BITSAT Topicwise Solved Papers

Chapter
Gravitation
7
1. A body moves in a circular orbit of radius R under where r0 is a constant. A test mass can undergo
the action of a central force. Potential due to the circular motion under the influence of the
central force is given by V(r) = kr (k is a positive gravitational field of particles. Its speed V as a
constant). Period of revolution of the body is function of distance r (0 < r < ¥) from the centre
proportional to : [2017] of the system is represented by [2014]
(a) R 1/2 (b) R –1/2 (c) R –3/2 (d) R –5/2 V V
2. What is the minimum energy required to launch
a satellite of mass m from the surface of a planet
of mass M and radius R in a circular orbit at an (a) (b)
altitude of 2R? [2017]
5GmM 2GmM GmM GmM R r R r
(a) (b) (c) (d) V V
6R 3R 2R 2R
3. Kepler's third law states that square of period of
revolution (T) of a planet around the sun, is (c) (d)
proportional to third power of average distance
r between sun and planet i.e. T2 = Kr3 here K is
constant. If the masses of sun and planet are M R r R r
and m respectively then as per Newton's law of 7. If the radius of the earth were to shrink by one
per cent, its mass remaining the same, the value
gravitation force of attraction between them is F
of g on the earth’s surface would [2012]
GMm , (a) increase by 0.5% (b) increase by 2%
= here G is gravitational constant. The (c) decrease by 0.5% (d) decrease by 2%.
r2
relation between G and K is described as [2016] 8. A man of mass m starts falling towards a planet
(a) GMK = 4p2 (b) K = G of mass M and radius R. As he reaches near to
the surface, he realizes that he will pass through
1
(c) K = (d) GK = 4p2 a small hole in the planet. As he enters the hole,
G he sees that the planet is really made of two
4. What should be the velocity of rotation of earth pieces a spherical shell of negligible thickness
due to rotation about its own axis so that the of mass 2M/3 and a point mass M/3 at the centre.
3 Change in the force of gravity experienced by
weight of a person becomes of the present
5 the man is [2011]
weight at the equator. Equatorial radius of the
earth is 6400 km. [2016] 2 GMm
(a) (b) 0
(a) 8.7 × 10–7 rad/s (b) 7.8 × 10–4 rad/s 3 R2
(c) 6.7 × 10–4 rad/s (d) 7.4 × 10–3 rad/s
1 GMm 4 GMm
5. An artificial satellite is moving in a circular orbit (c) (d)
around the earth with a speed equal to half the 3 R2 3 R2
magnitude of the escape velocity from the earth. 9. Geo-stationary satellite is one which [2011]
The height (h) of the satellite above the earth’s (a) remains stationary at a fixed height from
surface is (Take radius of earth as Re) [2015] the earth’s surface
(b) revolves like other satellites but in the
(a) h = Re2 (b) h = Re (c) h = 2Re (d) h = 4Re opposite direction of earth’s rotation
6. A spherically symmetric gravitational system of (c) revolves round the earth at a suitable height
ìr for r £ R with same angular velocity and in the same
0
particles has a mass density r = í0 for r > R direction as earth does about its own axis
î
(d) None of these
http://t.me/iitjeehelps

Gravitation P – 19
10. Suppose the gravitational force varies inversely (a) 71 J (b) 13 58J (c) – 71 J (d) 1 J
as the nth power of distance. Then the time 14. A particle released at a large distance from a planet
period of a planet in circular orbit of radius ‘R’ reaches the planet only under gravitational
around the sun will be proportional to attraction and passes through a smooth tunnel
[2010, 2007] through its centre. If ve is the escape velocity of
æ n -1 ö æ n +1 ö æ n -2 ö a body at centre of the planet, then the particle’s
ç ÷ ç ÷ ç ÷
(a) Rn (b) Rè 2 ø (c) Rè 2 ø (d) Rè 2 ø speed at the centre of the planet is : [2007]
11. Two planets A and B have the same material
(a) v e (b) 1.5 ve (c) 1.5 ve (d) 2ve
density. If the radius of A is twice that of B, then
the ratio of the escape velocity vA/vB is[2010] 15. A geostationary satellite is orbiting the earth at
a height of 5R above that surface of the earth, R
(a) 2 (b) 2 (c) 1/ 2 (d) 1/2 being the radius of the earth. The time period of
12. The rotation of the earth having radius R about
another satellite in hours at a height of 2R from
its axis speeds upto a value such that a man at
the surface of the earth is [2006]
latitude angle 600 feels weightless. The duration
of the day in such case will be : [2009] (a) 5 (b) 10 (c) 6 2 (d) 6 / 2
R g R g 16. The potential energy of a satellite of mass m and
(a) 8p (b) 8p (c) p (d) 4p revolving at a height Re above the surface of
g R g R
earth where Re = radius of earth, is [2005]
13. The gravitational field in a region is given by
® -m g R e
(a) – m g Re (b)
g = 5N / kgiˆ + 12N / kgjˆ . The change in the 2
gravitational potential energy of a particle of -m g R e -m g R e
mass 1 kg when it is taken from the origin to a (c) (d)
3 4
point (7 m, – 3 m) is: [2008]

Hints & Solutions


1. (a) U = mV = kmr. Therefore minimum required energy,
dU 5GMm
K=
Force, F = - = - km 6R
dr
mv 2 1/2 GM
Now , = km Þ v µ r 3. (a) As we know, orbital speed, Vorb =
r r
2pr 2pr 2 pr 2 pr
\T= = 1/2 Þ T µ r1/2 Time period T = = r
v cr vorb GM
2. (a) As we know, Squarring both sides,
- GMm 2
Gravitational potential energy = æ 2 pr r ö 4p 2 3
r T2 = ç ÷ = GM . r
è GM ø
and orbital velocity, v0 = GM / (R + h)
T 2 4p 2
GM GM Þ = =K Þ GMK = 4p2.
= = r 3 GM
(R + 2R) 3R 4. (b)
1 2 GMm -GMm 5. (b)
E f = mv 0 - =
2 3R 6R 6. (c)
-GMm GM dg dR
Ei = +K 7. (b) g = 2
Þ = -2
R R g R
Ei = Ef dR dg
= -1% Þ = 2%
R g
EBD_7167
http://t.me/iitjeehelps

P – 20 BITSAT Topicwise Solved Papers


8. (a) Change in force of gravity vA
as ve µ R for same density, =2
M vB
G m
GMm
= - 32 12. (c)
R2 R dv
(only due to mass M/3 due to shell gravitational 13. (d) I=-
field is zero (inside the shell)) dr

2GMm éx y ù
= v = - ê ò I x dx + ò I y dy ú
3R 2 êë 0 úû
0
9. (c) Geo-stationary satellites are also called
synchronous satellite. They always remain about = – éë I x .x + I y .y ùû
the same path on equater, i.e., it has a period of
exactly one day (86400 sec) = - éë35 + ( -36 )ùû = 1 J / kg
æ ö i.e., change in gravitational potential 1 J/kg.
r3 ÷
So orbit radius ç T = 2p comes out to be Hence change in gravitational potential energy 1J.
çç GM ÷÷
è ø 14. (a)
42400 km, which is nearly equal to the
15. (c) According to Kepler’s law of period T2 µ R3
circumference of earth. So height of
Geostationary satellite from the earth surface is T12 R13 (6 R )3
42,400 – 6400 = 36,000 km. = = = 8 Þ T2 = 6 2
T22 R23
(3 R )3
10. (c) F = KR - n = MRw 2 Þ w 2 = KR - ( n +1)
16. (b) At a height h above the surface of earth the
-( n +1) gravitational potential energy of the particle of
or w = K' R 2 mass m is
[where K' = K1/2, a constant] GM e m
Uh = -
-(n +1) (n +1) Re + h
2p
a R 2 \T a R 2 Where Me & Re are the mass & radius of earth
T
respectively.
4
2Gd pR 3 In this question, since h = Re
2GM 3
11. (a) ve = 2gR = R= R GM e m - mgRe
R2 R2 So U h = Re = - =
2 Re 2
4
= R 2Gd p
3
http://t.me/iitjeehelps

Chapter
Mechanical Properties
ofSolids 8
1. Two wires are made of the same material and (a) 18° (b) 0.18° (c) 36° (d) 0.36°
have the same volume. However wire 1 has 7. A metallic rod breaks when strain produced is
cross-sectional area A and wire 2 has cross- 0.2%. The Young’s modulus of the material of
sectional area 3A. If the length of wire 1 increases the rod is 7 × 109 N/m2. What should be its area
by Dx on applying force F, how much force is of cross-section to support a load of 104 N ?
needed to stretch wire 2 by the same amount? [2009]
[2016] (a) 7.1 × 10–8 m2 (b)7.1 × 10–6 m2
(a) 4 F (b) 6 F (c) 9 F (d) F (c) 7.1 × 10–4 m2 (d)7.1 × 10–2 m2
2. A steel wire is stretched by 1 kg wt. If the radius 8. The length of a metal is l1 when the tension in it
of the wire is doubled, its Young’s modulus will is T1 and is l2 when the tension is T2. The original
[2014] length of the wire is [2007]
(a) remain unchanged (b) become half l1 + l 2 l T
1 2 + l 2 T1
(c) become double (d) become four times (a) (b)
2 T1 + T2
3. A cube is subjected to a uniform volume l 1T2 - l 2 T1
compression. If the side of the cube decreases (c) (d) T1T2 l 1l 2
by 2% the bulk strain is [2013] T2 - T1
(a) 0.02 (b) 0.03 (c) 0.04 (d) 0.06 9. A thick rope of density r and length L is hung
4. The Young’s modulus of a perfectly rigid body from a rigid support. The Young’s modulus of
is [2012] the material of rope is Y. The increase in length
(a) unity of the rope due to its own weight is [2006]
(b) zero (a) (1/4) r g L2/Y (b) (1/2) r g L2/Y
(c) infinity (c) r g L2/Y (d) r g L/Y
(d) some finite non-zero constant 10. If the ratio of radii of two wires of same material
5. An iron rod of length 2m and cross-sectional is 2 : 1 and ratio of their lengths is 4 : 1, then the
area of 50 mm2 stretched by 0.5 mm, when a mass ratio of the normal forces that will produce the
of 250 kg is hung from its lower end. Young’s same extension in the length of two wires is
modulus of iron rod is [2011] [2006]
(a) 19.6 × 1020 N/m2 (a) 2 : 1 (b) 4 : 1 (c) 1 : 4 (d) 1 : 1
(b) 19.6 × 1018 N/m2 11. The elastic limit of brass is 3.5 × 1010 N/m2. Find
(c) 19.6 × 1010 N/m2 the maximum load that can be applied to a brass
(d) 19.6 × 1015 N/m2 wire of 0.75 mm diameter without exceeding the
6. The upper end of a wire of diameter 12mm and elastic limit. [2005]
length 1m is clamped and its other end is twisted (a) 4.12 × 104 N (b) 5.15 × 104 N
through an angle of 30°. The angle of shear is (c) 0.55 × 104 N (d) 1.55 × 104 N
[2010]
EBD_7167
http://t.me/iitjeehelps

P – 22 BITSAT Topicwise Solved Papers

Hints & Solutions


1. (c) l 7. (c) Maximum possible strain = 0.2/100
F
A Y \A=
Y ´ strain
Wire (1)
10 4 ´ 100
= = 7 .1 ´ 10 - 4 m2
(7 ´ 10 9 ) ´ 0 .2
3A Y 8. (c) If l is the original length of wire, then
change in length of first wire, Dl1 = (l1 – l)
change in length of second wire, Dl2 = (l2 – l)
l/3
Wire (2) T l T l
Now, Y = 1 ´ = 2´
As shown in the figure, the wires will have the A Dl 1 A Dl 2
same Young’s modulus (same material) and the T1 T T1 T2
length of the wire of area of cross-section 3A will or = 2 or =
Dl 1 Dl 2 l1 - l l 2 - l
be l/3 (same volume as wire 1).
F/A or T1 l 2 – T1 l = T2 l 1 – l T2
For wire 1, Y = ...(i)
D x/l T2 l 1 - T1l 2
or l=
F '/ 3 A T2 - T1
For wire 2 , Y = ...(ii)
Dx /( l / 3)
F (L / 2) (A L r g) (L / 2)
F l F' l 9. (b) Dl = =
From (i) and (ii) , ´ = ´ Þ F ' = 9F AY AY
A Dx 3 A 3Dx
2. (a) Young modulus is defined for a solid æ1ö 2
material, when we apply normal stress & it is a = ç ÷rg L / Y
è2ø
property of material. Hence it is constant and
Fl YADl
does not change by hanging a weight. 10. (d) Y = ;Þ F=
ADl l
3. (d)
F1 A1 l 2 r12 l 2 4 1
4. (c) For a perfectly rigid body strain produced = ´ = ´ = ´ =1:1
is zero for the given force applied, so F2 A 2 l1 r 2 l1 1 4
2
Y = stress/strain = ¥ F
11. (d) Stress = ; For elastic limit,
250 ´ 9.8 A
F/A -6 stress = 3.5 × 1010 N/m2 (given).
5. (c) Y = = 50 ´ 10
Dl / l 0.5 ´ 10-3 Thus, F = (pr2) × stress
2
250 ´ 9.8 2 2 æ 0.75 ö
= -6
´ -3 Þ 19.6 ´ 10 N / m
10 2 = 3.14 × ç ´10-3 ÷ × 3.5 × 1010
50 ´ 10 0.5 ´ 10 è 2 ø
rq 6mm ´ 30° (0.75)2
6. (b) rq = lf Þ f = = = 0.18° = 3.14 × × 3.5 × 104 = 1.55 × 104 N
l 1m 4
http://t.me/iitjeehelps

Chapter
Mechanical Properties
of Fluids 9
1. A mercury drop of radius 1 cm is sprayed into 7. A square gate of size 1 m × 1m is hinged at its
106 drops of equal size. The energy expressed in mid-point. A fluid of density r fills the space to
joule is (surface tension of Mercury is 460 × 10 –3 the left of the gate. The force F required to hold
N/m) [2017] the gate stationary is [2015]
(a) 0.057 (b) 5.7 rg
(c) 5.7 × 10–4 (d) 5.7 × 10–6 (a)
3
2. A capillary tube of radius R is immersed in water rg
and water rises in it to a height H. Mass of water (b)
2 Hinge
in the capillary tube is M. If the radius of the rg
tube is doubled, mass of water that will rise in (c)
the capillary tube will now be : [2017] 6
rg F
(a) M (b) 2 M (c) M/2 (d) 4 M (d)
3. A capillary tube of radius r is immersed vertically 8
in a liquid such that liquid rises in it to height h 8. Two capillary tubes are of the same diameters.
(less than the length of the tube). Mass of liquid One is dipped in a liquid of relative density 0.8
in the capillary tube is m. If radius of the capillary while the other in a liquid of relative density 0.6.
tube is increased by 50%, then mass of liquid If surface tensions of these liquids are 60 and 50
that will rise in the tube, is [2016] milli N/m, respectively, and the angles of the
contact are equal, compare the rise of liquids in
2 3 9
(a) m (b) m (c) m (d) m the capillary tubes. [2014]
3 2 4 (a) 9 : 10 (b) 7 : 10 (c) 3 : 10 (d) 1 : 10
4. The approximate depth of an ocean is 2700 m. 9. Two liquids of densities d1 and d2 are flowing in
The compressibility of water is 45.4 × 10–11 Pa–1 identical capillary tubes uder the same pressure
and density of water is 10 3 kg/m 3 .What difference. If t1 and t2 are time taken for the flow of
fractional compression of water will be obtained equal quantities (mass) of liquids, then the ratio of
at the bottom of the ocean ? [2016] coefficient of viscosity of liquids must be [2014]
(a) 1.0 × 10–2 (b) 1.2 × 10–2
(c) 1.4 × 10–2 (d) 0.8 × 10–2 d1 t 1 t d t d1 t 1
(a) (b) 1 (c) 2 2 (d)
5. A glass capillary tube of internal radius r = 0.25 d2t2 t2 d1 t 1 d 2t 2
mm is immersed in water. The top end of the
10. An ice block floats in a liquid whose density is
tube projected by 2 cm above the surface of the
less than water. A part of block is outside the
water. At what angle does the liquid meet the
liquid. When whole of ice has melted, the liquid
tube? Surface tension of water = 0.7 N/m. [2015]
level will [2012]
(a) 90° (b) 70° (c) 45° (d) 35°
(a) rise
6. Water is flowing on a horizontal fixed surface, (b) go down
such that its flow velocity varies with y (vertical (c) remain same
æ 2 y2 y3 ö (d) first rise then go down
direction) as v = k çç 2 - 3 ÷÷ . If coefficient 11. A large drop of oil (density 0.8 g/cm 3 and
è a a ø viscosity h0 ) floats up through a column of
of viscosity for water is h, what will be shear another liquid (density 1.2 g/cm3 and viscosity
stress between layers of water at y =a. [2015] hL ). Assuming that the two liquids do not mix,
hk h the velocity with which the oil drop rises will
(a) (b) depend on : [2012]
a ka
(a) h0 only (b) hLonly
ha (c) both on h0 and hL (d) neither h0 nor hL
(c) (d) None of these
k
EBD_7167
http://t.me/iitjeehelps

P – 24 BITSAT Topicwise Solved Papers


12. Viscosity is the property of a liquid due to which 17. A cylindrical vessel
it : [2011] filled with water is
(a) occupies minimum surface area released on an inclined
(b) opposes relative motion between its surface of angle q as Fixed
adjacent layers shown in figure.
(c) becomes spherical in shape The friction coefficient
of surface with vessel is µ (< tan q).
(d) tends to regain its deformed position
Then the contact angle made by the surface of
13. A spherical ball is dropped in a long column of water with the incline will be – [2008]
a viscous liquid. The speed (n) of the ball as a (a) tan–1 µ (b) q – tan–1 µ
function of time (t) may be best represented by (c) q + tan–1 µ (d) cot–1 µ
[2010] 18. The lift of an air plane is based on [2007]
v v (a) Torricelli's theorem
(b) Bernoulli's theorem
(a) (b) (c) Law of gravitation
(d) conservation of linear momentum
t t 19. A vertical glass capillary tube of radius r open at
v v both ends contains some water (surface tension
T and density r). If L be the length of the water
(c) (d) column, then : [2007]
4T
t t (a) L =
rrg
14. Two mercury drops (each of radius r) merge to
2T
form a bigger drop. The surface energy of the (b) L =
rrg L
bigger drop, if T is the surface tension, is
(a) 25/3 pr2T (b) 4 pr2T [2010] T
(c) 2 pr2T (d) 28/3 pr2T (c) L =
4rrg
15. Two circular plates of radius 5 cm each, have a T
0.01 mm thick water film between them. Then what (d) L =
will be the force required to separate these plate 2rrg
(S.T. of water = 73 dyne/cm) ? [2010] 20. Two pieces of metals are suspended from the
(a) 125 N (b) 95 N (c) 115 N (d) 105 N arms of a balance and are found to be in
16. A liquid is flowing through a non-sectional tube equilibrium when kept immersed in water. The
with its axis horizontally. If two points X and Y mass of one piece is 32 g and its density 8 g
on the axis of tube has a sectional area 2.0 cm3 cm–3. The density of the other is 5 g per cm3.
Then the mass of the other is [2006]
and 25 mm2 respectively then find the flow (a) 28 g (b) 35 g (c) 21 g (d) 33.6 g
velocity at Y when the flow velocity at X is 10m/s. 21. Due to capillary action, a liquid will rise in a
[2009] tube if angle of contact is [2005]
(a) 20 m/s (b) 40 m/s (c) 80 m/s (d) 60 m/s (a) acute (b) obtuse (c) 90º (d) zero

Hints & Solutions


1. (a) W = TDA = 4pR2T(n1/3 – 1)
= 0.057 New mass m2 = pr22h2r =
2
(
3 2
) 3
pr1 h1 r = m
2
æ 2T cos q ö 1 ( DV / V)
2. (b) M = ( pr 2 h )r = pr 2 ç r
è rrg ÷ø 4. (b) K = = (P = Pgh)
B P
2 æ 2T cos q ö So,(DV/V) = KPgh = 1.2258 × 10–2
and M¢ = p (2r ) çè r ´ 2 r ´ g ÷ø r = 2M.
2T
2T cos q 1 h r 2 5. (b) h = [Q water wets glass, q =0°]
3. (c) h= Þhµ Þ 2 = 1 = rrg
rrg r h1 r2 3
http://t.me/iitjeehelps

Mechanical Properties of Fluids P – 25


2 ´ 0.07 11. (b) 12. (b) 13. (b)
= -3
0.25 ´10 ´1000 ´ 9.8 4 3 4
As the tube is only 2 cm above the water and 14. (d) pR = 2 ´ pr 3 Þ R = 21/ 3 r
3 3
so, water will rise by 2 cm and meet the tube at Surface energy of bigger drop,
an angle such that,
E = 4pR 2T = 4 ´ 2 2 / 3 pr 2T = 28 / 3 pr 2T
h¢r rg
Þ cos q = 2 ´ p ´ (0.05)2 ´ 73 ´ 10 -3
2T 2AT
15. (c) F = =
2 ´10 -2 ´ 0.25 ´ 10-3 ´1000 ´ 9.8 d 0.01 ´ 10-3
= = 36.5 p » 115 newton
2 ´ 0.07
The liquid will meet the tube at an angle, q @ 70° 16. (c) According to principle of continuity
dv vx Ax
6. (a) Newton’s law of viscosity, F = hA vy =
dy Ay
F æ dv ö æ 4 y 3y2 ö 10(m/s) ´ 2(cm 2 )
Stress = = h ç ÷ = hk ç 2 - 3 ÷ = = 80 m/s
A è dy ø èa a ø 25 ´ 10-2 (cm 2 )
æ 4 3 ö hk 17. (a) Figure shows forces acting on a particle on
At y = a, stress = hk ç - ÷ = the surface, with respect to vessel.
è a aø a
7. (c) The net force acting on the gate element of mgsin
width dy at a depth y from the surface of the mgcos
fluid, is y q µmgcos

( )
dy = p0 + rg y - p0 ´1dy
pA mg
µmgcos resultant
dy
= rgydy tan q = µ \ q = tan–1 µ
Torque about the hinge is 18. (b) Apply Bernoulli’s theorem.
æ1 ö F 19. (a)
dt = pgydy ´ ç - y ÷ 32
è 2 ø 20. (b) Volume of first piece of metal = = 4 cm3
Net torque experienced by the gate is 8
Upthrust = 4 gf
1 Effective weight = (32 – 4) gf = 28 gf
tnet = ò dt + F ´ = 0
2 If m be the mass of second body, volume of
rg m
Þ F= second body is
6 5
h1 T1 d2 9 m
8. (a) = × d = Now, 28 = m - Þ m = 35 g
h2 T2 1 10 5
9. (a) 2 T cos q
21. (a) h = ; The liquid will rise i.e., h is
10. (b) Ice is lighter than water. When ice melts, rr g
the volume occupied by water is less than that positive if cosq is +ve; It is so if q < 90º or q is
of ice. Due to which the level of water goes down. acute.
EBD_7167
http://t.me/iitjeehelps

P – 26 BITSAT Topicwise Solved Papers

Chapter
ThermalProperties
ofMatter 10
1. A steel wire of length ‘L’ at 40°C is suspended 6. If the radius of a star is R and it acts as a black
from the ceiling and then a mass ‘m’ is hung body, what would be the temperature of the star,
from its free end. The wire is cooled down from in which the rate of energy production is Q ?
40°C to 30°C to regain its original length ‘L’. The [2013]
coefficient of linear thermal expansion of the steel (a) Q/4pR2 s (b) (Q/4pR2s)–1/2
is 10–5 /° C, Young’s modulus of steel is 1011 N/ (c) (4pR2Q/s)1/4 (d) (Q/4pR2s)1/4
m2 and radius of the wire is 1 mm. Assume that L 7. A solid body of constant heat capacity 1 J/°C is
>>diameter of the wire. Then the value of ‘m’ in being heated by keeping it in contact with
reservoirs in two ways :
kg is nearly [2017]
(i) Sequentially keeping in contact with 2
(a) 1 (b) 2 (c) 3 (d) 5
reservoirs such that each reservoir supplies
2. During vapourisation [2017] same amount of heat.
(a) change of state from liquid to vapour state (ii) Sequentially keeping in contact with 8
occurs. reservoirs such that each reservoir supplies
(b) temperature remains constant. same amount of heat.
(c) both liquid and vapour states coexist in In both the cases body is brought from initial
equilibrium. temperature 100°C to final temperature 200°C.
(d) All of the above Entropy change of the body in the two cases
3. Two spheres of different materials one with respectively is : [2012]
double the radius and one-fourth wall thickness (a) ln2, 2ln2 (b) 2ln2, 8ln2
of the other are filled with ice. If the time taken (c) ln2, 4ln2 (d) ln2, ln2
for complete melting of ice in the larger sphere is 8. The radiation emitted by a perfectly black body
25 minute and for smaller one is 16 minute, the is proportional to [2011]
ratio of thermal conductivities of the materials (a) temperature on ideal gas scale
of larger spheres to that of smaller sphere is (b) fourth root of temperature on ideal gas scale
[2016] (c) fourth power of temperature on ideal gas
(a) 4 : 5 (b) 5 : 4 (c) 25 : 8 (d) 8 : 25 scale
4. The wavelength of radiation emitted by a body (d) square of temperature on ideal gas scale
9. A copper sphere cools from 62°C to 50°C in 10
depends upon [2014]
minutes and to 42°C in the next 10 minutes.
(a) the nature of its surface Calculate the temperature of the surroundings.
(b) the area of its surface (a) 18.01ºC (b) 26ºC [2011]
(c) the temperature of its surface (c) 10.6ºC (d) 20ºC
(d) All of the above 10. One kilogram of ice at 0°C is mixed with one
5. Figure shows a copper rod joined to a steel rod. kilogram of water at 80°C. The final temperature
The rods have equal length and equal cross- of the mixture is (Take specific heat of water =
sectional area. The free end of the copper rod is 4200 kJ/kg-°C, Latent heat of ice = 336 kJ/kg)
kept at 0ºC and that of steel rod is kept at 100ºC. [2010]
Find the temperature of the junction of the rod. (a) 0°C (b) 40°C (c) 50°C (d) 60°C
Conductivity of copper = 390 W/mºC. 11. A body of length 1m having cross-sectional area
Conductivity of steel = 46 W/m ºC [2013] 0.75m2 has heat flow through it at the rate of
6000 Joule/sec. Then find the temperature
0ºC Copper Steel 100ºC difference if K = 200 Jm–1K–1. [2009]
(a) 18.01ºC (b) 26ºC (c) 10.6ºC (d) 20ºC (a) 20°C (b) 40°C (c) 80°C (d) 100°C
http://t.me/iitjeehelps

Thermal Properties of Matter P – 27


12. Which of the following combinations of 16. Two straight metallic strips each of thickness t
properties would be most desirable for a cooking and length l are rivetted together. Their
pot? [2009] coefficients of linear expansions are a1 and a2 .
(a) High specific heat and low conductivity If they are heated through temperature DT, the
(b) Low specific heat and high conductivity bimetallic strip will bend to form an arc of radius
(c) High specific heat and high conductivity [2006]
(d) Low specific heat and low conductivity (a) t/{a1 + a2)DT} (b) t/{(a2 – a1)DT}
13. Two bodies A and B are placed in an evacuated (c) t(a1 – a2)DT (d) t(a2 – a1)DT
vessel maintained at a temperature of 27ºC. The 17. A steel rod of length 50 cm has a cross-sectional
temperature of A is 327ºC and that of B is 227ºC. area of 0.4 cm2. What force would be required to
The ratio of heat loss from A and B is about stretch this rod by the same amount as the
[2008] expansion produced by heating it through 10ºC.
(a) 2 : 1 (b) 1 : 2 (c) 4 : 1 (d) 1 : 4 (a = 10–5 K–1 and Y = 2 × 1011 N/m2) [2006]
14. 250 gm of water and an equal volume of alcohol (a) 600 N (b) 800 N (c) 500 N (d) 400 N
of mass 200 gm are placed successively in the 18. Which of the following circular rods, (given
same calorimeter and cools from 60ºC to 55ºC in radius r and length l) each made of the same
130 sec and 67 sec respectively. If the water material and whose ends are maintained at the
equivalent of the calorimeter is 10 gm., then the same temperature will conduct most heat?
specific heat of alcohol in cal/gm ºC is : [2008] [2005]
(a) 1.30 (b) 0.67 (c) 0.62 (d) 0.985 (a) r = 2r0; l = 2l0 (b) r = 2r0; l = l0
15. A heat flux of 4000 J/s is to be passed through a (c) r = r0; l = 2l0 (d) r = r0; l = l0
copper rod of length 10 cm and area of cross- 19. Find the quantity of heat required to convert 40
section 100 sq. cm. The thermal conductivity of gm of ice at –20°C into water at 20°C.Given Lice
copper is 400 W/mC. The two ends of this rod = 0.336 × 106 J/kg.
must be kept at a temperature difference of – Sp heat of ice = 2100 J/kg-K sp heat of water=
(a) 1ºC (b) 10ºC [2007] 4200 J/kg-K [2005]
(a) 12480 J (b) 10420 J
(c) 100ºC (d) 1000ºC (c) 16450 J (d) 18480 J

Hints & Solutions


1. (c) We know that 3. (d) Radius of small sphere = r
mg / A mg l Thickness of small sphere = t
Y= = ....(1) Radius of bigger sphere = 2r
Dl / l ADl
Also Dl = l aDT ...(2) Thickness of bigger sphere = t/4
Mass of ice melted = (volume of sphere) ×
From (1) and (2)
(density of ice)
mg l mg Let K1 and K2 be the thermal conductivities
Y= =
Al a DT A a DT of larger and smaller sphere.
For bigger sphere,
YA a DT 4
\ m= g K1 4p (2r)2 ´ 100 3
p(2r)3 rL
=
1011 ´ p(10-3 )2 ´10-5 ´10 t/4 25 ´ 60
= =p»3 For smaller sphere,
10
2. (d) The change of state from liquid to vapour 4 3
pr rL
(for gas) is called vapourisation. It is observed K 2 ´ 4pr 2 ´ 100 3
=
that when liquid is heated, the temperature t 16 ´ 60
remains constant untill the entire amount of the K1 8
liquid is converted into vapour. \ K = 25
The temperature at which the liquid and the 2
vapour states of the substance coexists is called 4. (d)
its boiling point.
EBD_7167
http://t.me/iitjeehelps

P – 28 BITSAT Topicwise Solved Papers


5. (c) Heat current in first rod (copper) = 14. (c)
390 ´ A(0 - q) DQ DT
15. (c) = KA
l Dt Dx
Here q is temperature of the junction and A & l DT
4000 = 400 × 100 × 10–4
are area and length of copper rod. 10 ´ 10-2
Heat current in second rod (steel) = -2
40000 ´ 10
46 ´ A(q - 100) or DT = = 102 °C.
40000 ´ 10-4
l 16. (b) Let the angle subtended by the arc formed
In series combination, heat current remains be q. Then
same. So, l Dl l 2 - l 1
q = or q = =
390 ´ A(0 - q) 46 ´ A(q - 100) r Dr r1 - r2
=
l l l ( a 2 - a1 ) D T
\ q=
– 390 q = = 46 q – 4600 t
436 q = 4600 Þ q = 10.6ºC l l (a 2 - a1 ) DT
6. (d) Stefan’s law for black body radiation or =
r t
Q = se AT4 t
é Q ù
1/ 4
So, r = (
T=ê a 2 - a 1 ) DT
2 ú (b) We have, Force = YA a.Dq
ëê s(4pR ) ûú 17.
Here e=1 = 2 × 1011 × 0.4 × 10–4 × 10–5 × 10 = 0.8 × 103
A = 4pR2 = 800 N
7. (d) The entropy change of the body in the two TH - TL
18. (b) We know that Q =
cases is same as entropy is a state function. R
8. (c) l l
9. (b) By Newton's law of cooling, Also Thermal resistance, R = KA = 2
Kpr
q1 - q 2 éq + q ù Heat flow will be maximum when thermal
= - k ê 1 2 - q0 ú ....(1) resistance is minimum. From given option
t ë 2 û 2l l
0 0
A sphere cools from 62°C to 50°C in 10 min. (i) r = 2r0, l = 2l0 \ R = Kp ( 2 r ) 2 = 2 Kpr 2
0 0
62 - 50 é 62 + 50 ù l0 l0
= -k ê - q0 ú ....(2) (ii) r = 2r 0, l = l0 \ R= 2
= 2
10 ë 2 û Kp ( 2 r0 ) 4 Kpr0
2l 0 2l 0
Now, sphere cools from 50°C to 42°C in next 10 \ R= =
(iii) r = r0, l = 2l0 Kp r02 Kpr0
2
min.
l0 l0
50 - 42 é 50 + 42 ù (iv) r = r0, l = l0 \ R= =
= -k ê - q0 ú ....(3) Kp r02 Kpr0
2
10 ë 2 û
It is clear that for option (b) resistance is
56 - q0 1.2 minimum, hence heat flow will be maximum.
Dividing eqn. (2) by (3) we get, =
46 - q 0 0.8 40
or 0.4q0 = 10.4 Hence q0 = 26°C 19. (d) Mass of ice = = 0.04 Kg
1000
10. (a) Heat required to raise the temperature of ice from
Q KADq 200 ´ 0.75 ´ Dq –20°C to 0°C = 0.04 × 2100 × 20 = 1680 J
11. (b) = Þ 6000 =
t l 1 Heat required to convert the ice into water at
6000 ´ 1 0°C = mL
\ Dq = = 40°C
200 ´ 0.75 = .04 × 0.336 × 106 = 13440 J
12. (b) Heat required to heat water from 0°C to 20°C
E1 s (T14 - T04 ) (600 ) 4 - (300 ) 4 = .04 × 4200 × 20 = 3360 J
13. (a) E = = Total heat required = 1680 + 13440 + 3360=18480 J
2 s( T24 - T04 ) (500 ) 4 - (300 ) 4
http://t.me/iitjeehelps

Chapter
Thermodynamics
11
1. A cyclic process ABCD is shown in the figure P- 4. 0.5 mole of an ideal gas at constant temperature
V diagram. Which of the following curves 27°C kept inside a cylinder of length L and cross-
represent the same process? [2017] section area A closed by a massless piston.
P A B

C L
D
V
L
P
A B
P
A B The cylinder is attached with a conducting rod
of length L, cross-section area (1/9) m2 and
(a) C (b) C thermal conductivity k, whose other end is
D D maintained at 0°C. If piston is moved such that
T T rate of heat flow through the conducing rod is
P P constant then velocity of piston when it is at
B
A A B height L/2 from the bottom of cylinder is :
(c) (d) [Neglect any kind of heat loss from system]
D C D C [2015]
T T
2. One mole of an ideal gas is P æ kö æ k ö
(a) çè ÷ø m / sec (b) çè ÷ m / sec
taken from state A to state B B R 10R ø
by three different processes, æ k ö æ k ö
(i) ACB (ii) ADB (iii) AEB as C D E (c) çè ÷ø m / sec (d) çè ÷ m / sec
1000R ø
100R
shown in the P-V diagram. The
heat absorbed by the gas is
A
V 5. One mole of O2 gas having a volume equal to
(a) greater in process (ii) than in (i) [2017] 22.4 Litres at 0°C and 1 atmospheric pressure in
(b) the least in process (ii) compressed isothermally so that its volume
(c) the same in (i) and (iii) reduces to 11.2 litres. The work done in this
(d) less in (iii) than in (ii) process is- [2014, 2009]
3. Figure below shows two paths that may be taken (a) 1672.5 J (b) 1728 J
by a gas to go from a state A to a state (c) (c) –1728 J (d) –1572.5 J
6. A Carnot’s heat engine works between the
P B C
6×104 Pa temperatures 427°C and 27°C. What amount of
heat should it consume per second to deliver
mechanical work at the rate of 1.0 kW ? [2013[
2×104 Pa (a) 0.417 kcal/s (b) 4.17 kcal/s
A [2016]
(c) 41.7 kcal/s (d) 0.212 kcal/s
2 × 10–3 m3 4 × 10–3 m3 7. Which of the following process is possible
V according to the first law of thermodynamics –
In process AB, 400 J of heat is added to the (a) W > 0, Q < 0 and dU = 0 [2012]
system and in process BC, 100 J of heat is added (b) W > 0, Q < 0 and dU > 0
to the system. The heat absorbed by the system (c) W > 0, Q < 0 and dU < 0
in the process AC will be (d) W < 0, Q > 0 and dU < 0
(a) 500 J (b) 460 J (c) 300 J (d) 380 J
EBD_7167
http://t.me/iitjeehelps

P – 30 BITSAT Topicwise Solved Papers


8. For an isothermal expansion of a perfect gas, the
DP C B C B
value of is equal to
P
(a) p (b) p
1/ 2 DV DV
(a) – g (b) – A A
V V
V V
DV 2 DV
(c) – g (d) – g
V V C B
9. Figure shows the variation of internal energy
(U) with the pressure (P) of 2.0 mole gas in cyclic (c) p (d) p
process abcda. The temperature of gas at c and A A C
B
d are 300 K and 500 K. Calculate the heat V
V
absorbed by the gas during the process.
U
14. The slopes of isothermal and adiabatic curves
are related as [2008]
a d (a) isothermal curve slope = adiabatic curve
slope
(b) isothermal curve slope = g × adiabatic curve
slope
b c (c) adiabatic curve slope = g × isothermal curve
slope
P 1
P0 2P0 (d) adiabatic curve slope = ´ isothermal
(a) 400 R ln 2 (b) 200 R ln 2 2
curve slope
(c) 100 R ln 2 (d) 300 R ln 2 15. A Carnot engine works first between 200°C and
10. In the equation PVg = constant, the value of g is 0°C and then between 0°C and –200°C. The ratio
unity. Then the process is [2010] of its efficiency in the two cases is [2007]
(a) isothermal (b) adiabatic (a) 1.0 (b) 0.577 (c) 0.34 (d) 0.68
(c) isobaric (d) irreversible 16. An ideal gas with adiabatic exponent (g = 1.5)
11. An ideal refrigerator has a freezer at a temperature undergoes a process in which work done by the
of 13ºC. The coefficient of performance of the gas is same as increase in internal energy of the
engine is 5. The temperature of the air (to which gas. The molar heat capacity of gas for the
heat is rejected) is [2010] process is – [2007]
(a) 320ºC (b) 39ºC (c) 325 K (d) 325ºC (a) C = 4R (b) C = 0 (c) C = 2R (d) C = R
12. A sample of gas expands from volume V1 to V2. 17. A cylinder with a movable piston contains 3
The amount of work done by the gas is greatest moles of hydrogen at standard temperature and
when the expansion is [2009] pressure. The walls of the cylinder are made of a
(a) isothermal (b) isobaric heat insulator, and the piston is insulated by
(c) adiabatic (d) equal in all cases having a pile of sand on it. By what factor does
13. A cyclic process is the pressure of the gas increases if the gas is
shown in the p-T compressed to half its original volume ? [2006]
C B (a) (2)7/5 (b) (2)1/5 (c) (5)7/5 (d) (2)1/5
diagram. Which of the 18. The first operation involved in a carnot cycle is
curves show the same p (a) isothermal expansion [2005]
process on a P-V (b) adiabatic expansion
A (c) isothermal compression
diagram ? [2009]
O T (d) adiabatic compression

Hints & Solutions


1. (a) Process AB is isobasic and BC is QACB = DU + WACB,
isothermal, CD isochoric and DA isothermic QADB = DU,
compression. QAEB = DU + WAEB
2. (d) The change in internal energy DU is same Here WACB is positive and WAEB is negative.
in all process. Hence QACB > QADB > QAEB.
http://t.me/iitjeehelps

Thermodynamics P – 31
3. (b) In cyclic process ABCA T2 260
Qcycle = Wcycle K= T -T ; 5 =
1 2 T1 - 260
QAB + QBC + QCA = ar. of DABC
or T1 – 260 = 52; T1 = 312 K,
1 T2 = 312 – 273 = 39°C
+ 400 + 100 + QC®A = (2 × 10–3) (4 × 104)
2 12. (a) 13. (b)
Þ QC ® A = – 460 J 14. (c)
Þ QA ® C = + 460 J Slope of adiabatic curve (dP / dV ) adi
= = +g
DQ DW k aq Slope of isothermal curve ( dP / dV ) iso
4. (c) = =
Dt Dt L æ C ö
So slope to adiabatic curve is g ç = P ÷ times
dW dv aq nRT è CV ø
=P =k , P=
dt dt L V of isothermal curve, as clear also from figure.
ka æ 27 ö k P
Þ v= ç ÷=
R è 300 ø 100 R Isothermal
5. (d) curve
6. (a) The efficiency of the heat engine is
T2 æ 273 + 27K ö 4
h = 1- = 1- ç =
T1 è 273 + 427K ÷ø 7
V
W Adiabatic curve
But h =
Q1 15. (a)

\ Q1 = =
W 1.0kW
=1.75kW =0.417 kcal / s
16. (a) ò PdV =ò nC dt
v
h 4/7 Þ dQ = 2 dU Þ nCdT = 2nCvdt Þ C = 2Cv
Thus, the engine would require 417 cal of heat
per second, to deliver the requisite amount of 2R
Þ C= = 4R
work. 1.5 - 1
7. (c) 17. (a) The process is adiabatic
8. (b) Differentiate PV = constant w.r.t V
DP DV \ P1V1g = P2 V2g
Þ PDV + VDP = 0 Þ =–
P V V1 2 7
9. (a) Given, V = 1 and g = (for diatomic gas)
2 5
10. (a) PV = constant represents isothermal
process. P2 7/5
\ P = (2) i.e., P2 = (2)7/5P1
11. (b) T2 = 273 – 13 = 260, 1
18. (a)
EBD_7167
http://t.me/iitjeehelps

P – 32 BITSAT Topicwise Solved Papers

Chapter
Kinetic Theory
12
1. Five gas molecules chosen at random are found 1 2
to have speeds of 500, 600, 700, 800 and 900 m/s. (c) 1 + (d) 1 +
n n
Then which of the following statements is
6. Two gases occupy two containers A and B the
correct? [2017]
gas in A, of volume 0.10m3, exerts a pressure of
(a) The root mean square speed and the 1.40 MPa and that in B of volume 0.15m3 exerts a
average speed are the same. pressure 0.7 MPa. The two containers are united
(b) The root mean square speed is 14 m/s by a tube of negligible volume and the gases are
higher than the average speed. allowed to intermingle. Then if the temperature
(c) The root mean square speed is 14 m/s lower remains constant, the final pressure in the
than the average speed. container will be (in MPa) [2013]
(d) The root mean square speed is Ö14 m/s (a) 0.70 (b) 0.98 (c) 1.40 (d) 210
higher than the average speed. 7. The average translational kinetic energy of O2
2. The molecules of a given mass of a gas have (molar mass 32) molecules at a particular
r.m.s. velocity of 200 ms–1 at 27°C and 1.0 × 105 temperature is 0.048 eV. The translational kinetic
Nm –2 pressure. When the temperature and energy of N2 (molar mass 28) molecules in eV at
pressure of the gas are respectively, 127°C and the same temperature is [2012]
0.05 × 105 Nm –2 , the r.m.s. velocity of its (a) 0.0015 (b) 0.003 (c) 0.048 (d) 0.768
molecules in ms–1 is : [2016] 8. For a gas if ratio of specific heats at constant
pressure and volume is g then value of degrees
400 100 2 100 of freedom is [2012]
(a) 100 2 (b) (c) (d)
3 3 3 3g –1 2
3. A vessel of volume 20L contains a mixture of (a) (b)
2g –1 g –1
hydrogen and helium at temperature of 27°C and 9 25
pressure 2 atm. The mass of mixture is 5g. (c) ( g – 1) (d) (g –1)
Assuming the gases to be ideal, the ratio of 2 2
mass of hydrogen to that of helium in the given 9. An air bubble of volume v0 is released by a fish
mixture will be [2015] at a depth h in a lake. The bubble rises to the
(a) 1 : 2 (b) 2 : 3 (c) 2 : 1 (d) 2 : 5 surface. Assume constant temperature and
standard atmospheric pressure above the lake.
4. In the kinetic theory of gases, which of these
The volume of the bubble just before touching
statements is/are true ? [2014] the surface will be (density) of water is r [2011]
(i) The pressure of a gas is proportional to the (a) v 0 (b) v0 (rgh/p)
mean speed of the molecules.
(ii) The root mean square speed of the v0
molecules is proportional to the pressure. (c) (d) v0 æ1 + rgh ö
æ rgh ö çè p ÷ø
(iii) The rate of diffusion is proportional to the çè1 + p ÷ø
mean speed of the molecules.
(iv) The mean translational kinetic energy of a 10. The molecules of a given mass of gas have a
gas is proportional to its kelvin temperature. root mean square velocity of 200m s–1 at 27°C
(a) (ii) and (iii) only and 1.0 × 105 N m –2 pressure. When the
(b) (i),(ii)and (iv) only temperature is 127°C and the pressure 0.5 × 105
(c) (i) and (iii) only Nm–2, the root mean square velocity in ms–1, is
(d) (iii) and (iv) only [2011, 2005]
5. If a gas has ‘n’ degrees of freedom, the ratio of 400
the specific heats g of the gas is [2013] (a) (b) 100 2
3
1+ n n 100 2 100
(a) (b) 1 + (c) (d)
2 2 3 3
http://t.me/iitjeehelps

Kinetic Theory P – 33
11. 3 moles of an ideal gas at a temperature of 27°C 3 3
are mixed with 2 moles of an ideal gas at a (c) must be ³ PV (d) must be £ PV
2 2
temperature 227°C, determine the equilibrium 15. One mole of a gas occupies 22.4 lit at N.T.P.
temperature of the mixture, assuming no loss of
energy. [2010] Calculate the difference between two molar
(a) 327°C (b) 107°C (c) 318°C (d) 410°C specific heats of the gas. [2006]
12. Which one the following graphs represents the J = 4200 J/kcal.
behaviour of an ideal gas [2009] (a) 1.979 k cal/kmol K
PV PV (b) 2.378 k cal/kmol K
(c) 4.569 kcal/kmol K
(a) (b) (d) 3.028 k cal/ kmol K
V V 16. Four mole of hydrogen, two mole of helium and
PV one mole of water vapour form an ideal gas
PV
mixture. What is the molar specific heat at
(c) (d) constant pressure of mixture? [2006]
V 16 7 23
V (a) R (b) R (c) R (d) R
13. The ratio of mean kinetic energy of hydrogen 7 16 7
and oxygen at a given temperature is- [2008] 17. A real gas behaves as an ideal gas [2005]
(a) 1 : 16 (b) 1 : 8 (c) 1 : 4 (d) 1 : 1 (a) at very low pressure and high temperature
14. An ideal gas has volume V and pressure P. The (b) high pressure and low temperature
total translational kinetic energy of the gas (c) high temperature and high pressure
[2007] (d) low pressure and low temperature
3 3
(a) must be PV (b) may be PV
2 2

Hints & Solutions


é 500 + 600 + 700 + 800 + 900 ù 3. (d) Let there are n 1 moles of hydrogen and n 2
1. (b) vav = ê
ë 5 úû = 700 m/s moles of helium in the given mixture. As Pv =
nRT
and
Then the pressure of the mixture
5002 + 6002 + 700 2 + 800 2 + 900 2 n RT n 2 RT RT
vrms = = 714 m/s P= 1 + = ( n1 + n 2 )
5 V V V
Thus vrms is greater than average speed by Þ
14 m/s. (8.3 ´ 300 )
2 ´101.3 ´103 = ( n1 + n 2 ) ´
2. (b) Here v1 = 200 m/s; 20 ´ 10-3
temperature T1 = 27°C = 27 + 273 = 300 k or,
temperature T2 = 127° C = 127 + 273 = 400 k, 2 ´ 101.3 ´ 103 ´ 20 ´10 -3
V= ? ( n1 + n 2 ) =
(8.3)( 300 )
R.M.S. Velocity, V µ T or, n1 + n2 = 1.62 ... (1)
v 400 The mass of the mixture is (in grams)
Þ = n1 × 2 + n2 × 4 = 5
200 300
Þ (n1 + 2n2) = 2.5 ... (2)
200 ´ 2
Þ v= m/s Solving the eqns. (1) and (2), we get
3 n1 = 0.74 and n 2 = 0.88
400
Þ v= m/s m H 0.74 ´ 2 1.48 2
3 Hence, = = =
m He 0.88 ´ 4 3.52 5
EBD_7167
http://t.me/iitjeehelps

P – 34 BITSAT Topicwise Solved Papers


4. (d) 11. (b) Energy possessed by the ideal gas at 27°C is
5. (d) Let us consider 1 mole of an ideal gas as a
æ3 ö 2700 R
Kelvin temperature T. It has NA molecules E1 = 3 ç R ´ 300÷ =
è2 ø 2
(Avogadro’s number). The internal energy of an
ideal gas is entirely kinetic. The average K.E. per Energy possessed by the ideal gas at 227°C is
1 æ 3R ö
molecule of an ideal gas = n k T, where n is E2 = 2 ç ´ 500÷ = 1500 R
2 è 2 ø
degrees of freedom. Therefore the internal
energy of one mole of an ideal gas would be If T be the equilibrium temperature, of the
æ1 ö 1 æ 3RT ö
U = N ç n k T ÷ = n R T (Q k = R / N A ) mixture, then its energy will be E m = 5 ç
è2 ø 2 è 2 ÷ø
dU n Since, energy remains conserved,
Now C v = = R and
dT 2 E m = E1 + E 2
n æn ö
C P = R + R = ç + 1÷ R
or 5 æç
3RT ö 2700R
2 è2 ø = + 1500R or T = 380 K
è 2 ÷ø 2
æn ö or 107°C
+1 R
CP çè 2 ÷ø æ 2ö 12. (b) For an ideal gas PV = constant i.e., PV does
= = ç1 + ÷ = γ
Cv n è nø not vary with V.
R
2 13. (d)
14. (a)
6. (b) We know that
15. (a) V = 22.4 litre = 22.4 × 10–3 m3, J = 4200 J/kcal
PAVA = nART, PBVB = nBRT
by ideal gas equation for one mole of a gas,
and Pf (VA + VB) = (nA + nB) RT
PV 1.013 ´ 105 ´ 22.4 ´ 10 -3
Pf (VA + VB) = PAVA + PBVB R= =
T 273
æ PA VA + PB VB ö R 1.013 ´ 105 ´ 22.4
\ Pf = ç V + V Cp - C v = = = 1.979
è ÷ø J 273 ´ 4200
A B
kcal/kmol K
1.4 ´ 0.1 + 0.7 ´ 0.15
= MPa = 0.98 MPa 5
0.1 + 0.15 16. (d) Cv for hydrogen = R
2
7. (c)
8. (b) 3R
Cv for helium =
2 2 f 1 2 2
g = 1+ , Þ g –1 = Þ = Þ f =
f f 2 g –1 g –1 6R
Cv for water vapour = = 3R
9. (d) As the bubble rises the pressure gets reduced 2
for constant temperature, if P is the standard \ (Cv)mix
atmospheric pressure, then ( P +rgh ) V0 = PV 5 3
4 ´ R + 2 ´ R + 1 ´ 3R
æ rgh ö 2 2 16
= R
or V = V0 çè1 + ÷ =
4 + 2 +1
P ø 7
\ Cp = Cv + R
c2 400 2
10. (a) = = 16 23
c1 300 3 Cp = R + R or Cp = R
7 7
2 400
Þ c2 = ´ 200 = ms -1 17. (a)
3 3
http://t.me/iitjeehelps

Chapter
Oscillations
13
1. The following figure y

///////////////
depict a circular motion. T = 4s P(t = 0)
3m/s
The radius of the circle, a 45° x 0.5kg 1 kg
the period of revolution, ////////////////////////////////////////////////////
the initial position and p p
(a) 5cm, s (b) 5cm, s
the sense of revolution are indicated on the 10 5
figure. 2p p
(c) 4cm, s (d) 4cm, s
The simple harmonic motion of the x-projection 5 3
of the radius vector of the rotating particle P can 4. Vertical displacement of a Planck with a body of
be shown as: [2017] mass m on it is varying according to law y = sin
æ 2p t p ö wt + 3 cos wt . The minimum value of w for
(a) x(t ) = a cos ç + ÷ which the mass just breaks off the Planck and
è 4 4ø
the moment it occurs first after t = 0, are given
æ pt p ö by [2015]
(b) x(t ) = a cos ç + ÷
è 4 4ø
2 p g 2
æ 2p t p ö (a) g / 2, (b) , p/g
(c) x(t ) = a sin ç + ÷ 6 g 2 3
è 4 4ø
p
æ pt p ö (c) g / 2, 2/g (d) 2g, 2p / 3g
(d) x(t ) = a cos ç + ÷ 3
è 3 2ø 5. A load of mass m falls from a height h on to the
2. A point particle of mass 0.1 kg is executing S.H.M. scale pan hung from the spring as shown in the
of amplitude of 0.1 m. When the particle passes figure. If the spring constant is k and mass of
through the mean position, its kinetic energy is the scale pan is zero and the mass m does not
8 × 10–3 Joule. Obtain the equation of motion of
this particle if this initial phase of oscillation is bounce relative to the pan, then the amplitude
45º. [2016] of vibration is [2015]
(a) mg/d
æ pö
(a) y = 0.1sin çè ±4t + ÷ø mg æ 1 + 2hk ö
4 K
k çè mg ÷ø
(b)
æ pö
(b) y = 0.2sin çè ±4t + ÷ø
4
æ p ö mg mg æ 1 + 2hk ö
+
(c) y = 0.1sin çè ±2t + ÷ø (c) k k çè mg ÷ø
4 m
æ pö mg æ 1 + 2hk mg ö h
(d) y = 0.2sin çè ±2t + ÷ø (d) k çè mg - k ÷ø
4
3. A 1 kg mass is attached to a spring of force 6. The displacement of a particle is given at time t,
constant 600 N/m and rests on a smooth
horizontal surface with other end of the spring by:
tied to wall as shown in figure. A second mass of x = A sin( -2w t ) + B sin 2 w t Then, [2015]
0.5 kg slides along the surface towards the first (a) the motion of the particle is SHM with an
at 3m/s. If the masses make a perfectly inelastic
collision, then find amplitude and time period of B2
amplitude of A2 +
oscillation of combined mass. [2016] 4
EBD_7167
http://t.me/iitjeehelps

P – 36 BITSAT Topicwise Solved Papers


(b) the motion of the particle is not SHM, but
oscillatory with a time period of T = p/w m
(a) 2p
(c) the motion of the particle is oscillatory with k
a time period of T = p/2w
(d) the motion of the particle is a periodic. m m
(b) p +p
7. Two particles P and Q describe S.H.M. of same k k/2
k
amplitude a, same frequency f along the same
straight line. The maximum distance between the m m
(c) p
3k / 2
two particles is a 2 . [2014] k
The initial phase difference between the particle m m
is – (d) p + p .
k 2k
(a) zero (b) p/2 13. A simple pendulum has time period 't'. Its time
(c) p/6 (d) p/3 period in a lift which is moving upwards with
8. A tunnel has been dug through the centre of the acceleration 3 ms–2 is [2010]
earth and a ball is released in it. It executes
S.H.M. with time period [2014] 9.8 12.8
(a) t (b) t
(a) 42 minutes (b) 1 day 12.8 9. 8
(c) 1 hour (d) 84.6 minutes 9.8 6. 8
9. An instantaneous displacement of a simple (c) t (d) t
6. 8 9.8
harmonic oscillator is x = A cos (wt + p/4). Its
speed will be maximum at time [2013] 14. In case of a forced vibration, the resonance wave
becomes very sharp when the [2009]
(a) p/4 w (b) p/2 w
(a) restoring force is small
(c) p/w (d) 2 p/w (b) applied periodic force is small
10. One end of a long metallic wire of length L tied (c) quality factor is small
to the ceiling. The other end is tied with a (d) damping force is small
massless spring of spring constant K. A mass 15. A pendulum bob carries a +ve charge +q. A
hangs freely from the free end of the spring. The positive charge +q is held at the point of support.
area of cross section and the young’s modulus
Then the time period of the bob is – [2009]
of the wire are A and Y respectively. If the mass
slightly pulled down and released, it will oscillate (a) greater than 2p L / g
with a time period T equal to : [2012]
(b) less than 2p L / g
(a) 2p (m / K)
(c) equal to 2p L / g
(b) 2p m(YA + KL) / (YAK)
(d) equal to 2p 2L / g
(c) 2p (m YA / KL)
16. Two oscillating systems; a simple pendulum and
(d) 2p (mL / YA) a vertical spring-mass-system have same time
11. Which of the following expressions corresponds period of motion on the surface of the Earth. If
to simple harmonic motion along a straight line, both are taken to the moon, then– [2008]
where x is the displacement and a, b, c are positive (a) Time period of the simple pendulum will be
constants? [2011] more than that of the spring-mass system.
(a) a + bx – cx 2 (b) bx 2 (b) Time period of the simple pendulum will be
(c) a – bx + cx2 (d) – bx equal is that is of the spring-mass system.
12. A mass m is suspended from a spring of force (c) Time period of the simple pendulum will be
less than of the spring-mass system.
constant k and just touches another identical
(d) Nothing can be said definitely without
spring fixed to the floor as shown in the figure.
observation.
The time period of small oscillations is [2011]
http://t.me/iitjeehelps

Oscillations P – 37
17. One end of a spring of force constant k is fixed k
to a vertical wall and other to a body of mass m (a) Ux = (x - a) 2
2
resting on a smooth horizontal surface there is (b) U x = k1 x + k 2 x 2 + k 3 x 3
another wall at a distance x0 from the body. The
spring is then compressed by 2x0 and released. (c) U x = A e - bx
The time taken to strike the wall is : [2007] (d) Ux = a constant
p k 19. A child swinging on swing in sitting position
(a) stands up. The time period of the swing will
6 m
x0 (a) increase [2006]
k (b) decrease
(b) (c) remain same
m
A B C (d) increase if the child is tall and decrease if
2p m 2x0 the child is short.
(c)
3 k 20. The height of liquid column in a U tube is 0.3
meter. If the liquid in one of the limbs is
p k depressed and then released, then the time
(d)
4 m period of liquid column will be- [2005]
18. The potential energy of a particle (Ux) executing (a) 0.11 sec (b) 19 sec
S.H.M. is given by [2007] (c) 1.1 sec (d) 2 sec

Hints & Solutions


1. (a) t=t Solving we getw = ± 4
wt t=0
Substituting the values of a, w and f in the
45° equation of S.H.M., we get
x
y = 0.1 sin (± 4t + p/4) metre.
3. (a)
p 4. (a) From, figure,
x = a cos æç wt + ö÷
è 4ø
( 3)
2 2
AR = + (1) = 2
æ 2pt p ö
or x = a cos ç + ÷ æ 3ö p
è 4 4ø q = tan -1 ç =
2. (a) The displacement of a particle in S.H.M. is ç 1 ÷÷ 3
è ø
given by : 3 2
y = a sin (wt + f) æ pö
\ y = 2sin ç wt + ÷
dy è 3ø f
velocity = = wa cos (wt + f)
dt 2
d y æ pö 1
The velocity is maximum when the particle = a = -2w2 sin ç wt + ÷
2 è 3ø
passes through the mean position i.e. dt
a max = -2w2 = g
æ dy ö
çè ÷ø For which mass just breaks off the plank
dt max = w a
w= g/2
The kinetic energy at this instant is given This will be happen for the first time when
by
p p p
2 wt + + or wt =
1 æ dy ö 1 3 2 6
mç ÷ = mw2 a2 = 8 × 10–3 joule
2 è dt ø max 2 p p 2
1 \ t= =
or × (0.1) w2 × (0.1)2 = 8 × 10–3 6w 6 g
2
EBD_7167
http://t.me/iitjeehelps

P – 38 BITSAT Topicwise Solved Papers


5. (b) According to energy conservation 22 8 ´ 10 3
principle, = 2´ ´
7 7´ 2
If, x1 is maximum elongation in the spring
when the particle is in its lowest extreme 2 ´ 22 ´ 8 ´ 1000
= min = 84.6 min
position. Then, 49 ´ 60
1 dx
mgh = kx12 - mgx1 9. (a) Velocity, v = = - A w sin (wt + p / 4)
2 dt
1 2 Velocity will be maximum, when
Þ kx1 - mgx1 - mgh = 0 wt + p/4 = p/2 or wt = p/2 – p/4 = p/4 or t
2 = p/4w
2 2mg 2mg 10. (b)
or, x1 - x1 - .h = 0
k k 11. (d) In linear S.H.M., the restoring force acting
éæ 2mg ö 2 on particle should always be proportional to the
2mg 2mg ù
± êç ÷ + 4´ hú displacement of the particle and directed towards
k êëè k ø k úû the equilibrium position.
\ x1 = i.e., F µ x
2
Amplitude A = X1 – X0 (elongation in spring or F = –bx where b is a positive constant.
for equilibrium position) 12. (d) When the spring undergoes displacement in
the downward direction it completes one half
mg æ 2hk ö oscillation while it completes another half
A= ç1 + ÷ oscillation in the upward direction. The total time
k è mg ø
6. (a) The displacement of the particle is given period is:
by: m m
T =p +p
x = Asin(–2wt) + Bsin2 wt k 2k
B
1 1
= - A sin 2wt + (1 - cos 2wt ) 13. (a) t µ , t' µ
2 9.8 12.8
B B (Q g ' = 9.8 + 3 = 12.8)
= -( A sin 2wt + cos 2wt ) +
2 2 t' 9.8 9.8
\ = Þ t' = t
This motion represents SHM with an t 12.8 12.8
B2 14. (d)
amplitude: A2 + , and mean position kq 2 1
4 15. (a) Effective g' = g - , Tµ
B d2m g eff .
.
2 l
7. (b) x1 = a sin(wt + f1 ), x 2 = a sin(wt + f 2 ) 16. (a) For simple pendulum : T = 2p
g
æ f + f2 ö æ f - f2 ö As g will decrease on moon, time period will
Þ | x1 - x 2 | = 2a sin ç wt + 1 cos ç 1 increase
è 2 ÷ø è 2 ÷ø
m
æ f1 + f 2 ö For spring mass system : T = 2p
To maximize | x1 - x 2 | : sin çè wt + ÷ =1 k
2 ø It will not change and remains the same
æ f1 - f 2 ö 17. (c)
Þ a 2 = 2a ´ 1 ´ cos çè 2 ÷ø 18. (a) P.E. of body in S.H.M. at an instant,
1 1
æ f - f2 ö U = m w 2 y 2 = ky 2
1 p f1 - f2 2 2
= cos ç 1 =
Þ
2 è 2 ÷ø Þ 4 2 If the displacement, y = (a – x) then
p 1 1
U = k(a - x)2 = k(x - a) 2
Þ f1 - f 2 = 2 2
2
19. (b)
R 64 ´ 106
8. (d) T = 2p = 2p h 0.3
g 9.8 20. (c) T = 2p or T = 2 × 3.14 × = 1.1 sec
g 9.8
http://t.me/iitjeehelps

Chapter
Waves
14
1. A tuning fork of frequency 392 Hz, resonates 6. Two tuning forks with natural frequencies 340
with 50 cm length of a string under tension (T). If Hz each move relative to a stationary observer.
length of the string is decreased by 2%, keeping One fork moves away from the observer, while
the tension constant, the number of beats heard the other moves towards the observer at the
when the string and the tuning fork made to same speed. The observer hears beats of
vibrate simultaneously is : [2017] frequency 3 Hz. Find the speed of the tuning
(a) 4 (b) 6 (c) 8 (d) 12 forks. [2015]
2. A sonometer wire resonates with a given tuning (a) 1.5 m/s(b) 2 m/s (c) 1 m/s (d) 2.5 m/s
fork forming standing waves with five antinodes 7. A sound source, emitting sound of constant
between the two bridges when a mass of 9 kg is frequency, moves with a constant speed and
suspended from the wire. When this mass is crosses a stationary observer. The frequency
replaced by a mass M, the wire resonates with (n) of sound heard by the observer is plotted
the same tuning fork forming three antinodes against time (t). Which of the following graphs
for the same positions of the bridges. The value represents the correct variation ? [2014]
of M is [2017] n n
(a) 25 kg (b) 5 kg (c) 12.5 kg(d) 1/25 kg
3. A source of sound S emitting waves of frequency
100 Hz and an observor O are located at some (a) (b)
distance from each other. The source is moving
with a speed of 19.4 ms–1 at an angle of 60° with t t
the source observer line as shown in the figure. n n
The observor is at rest. The apparent frequency
observed by the observer is (velocity of sound
in air 330 ms–1) [2016] (c) (d)
t t
8. When a string is divided into three segments of
length l1, l2, and l3 the fundamental frequencies
60° of these three segments are v 1, v 2 and v 3
S O
respectively. The original fundamental frequency
(a) 103 Hz (b) 106 Hz (v) of the string is [2014]
(c) 97 Hz (d) 100 Hz
4. A string of length l is fixed at both ends. It is (a) v = v1 + v2 + v3
vibrating in its 3rd overtone with maximum (b) v = v1 + v2 + v3
amplitude 'a'. The amplitude at a distance l/3 1 1 1 1
from one end is [2016] (c) = + + `
v v1 v2 v3
3a a
(a) a (b) 0 (c) (d) 1 1 1 1
2 2 (d) = + +
5. The frequency of a sonometer wire is 100 Hz. v v1 v2 v3
When the weights producing the tensions are 9. Two waves of wavelengths 99 cm and 100 cm
completely immersed in water, the frequency both travelling with velocity 396 m/s are made to
becomes 80 Hz and on immersing the weights in interfere. The number of beats produced by them
a certain liquid, the frequency becomes 60 Hz. per second is [2013]
The specific gravity of the liquid is [2015]
(a) 1 (b) 2 (c) 4 (d) 8
(a) 1.42 (b) 1.77 (c) 1.82 (d) 1.21
EBD_7167
http://t.me/iitjeehelps

P – 40 BITSAT Topicwise Solved Papers


10 If equation of transverse wave is y = x0 cos nance when the two air columns are 24 cm and
25 cm respectively. Calculate the frequencies of
æ xö forks. [2009]
2p çè nt - ÷ø . Maximum velocity of particle is
l (a) 120 Hz, 124 Hz (b) 110 Hz, 114 Hz
twice of wave velocity, if l is- [2013] (c) 150 Hz, 144 Hz (d) 170 Hz, 118 Hz
(a) p/2x0 (b) 2px0 (c) px (d) px0 16. When a source of sound crosses a stationary
11. The transverse displacement y(x, t) of a wave on observer then the change in apparent frequency
2 + bt 2 + 2 ab xt )
of sound observed by the observer, when Vs < <
a string is given by y ( x, t ) = e –( ax V, will be - [2008]
This represents a [2012] 2nVs 2nV
(a) Dn = (b) Dn = V
b V s
(a) wave moving in – x direction, speed
a 2nV0
(b) standing wave of frequency b (c) Dn = 2nVsV (d) Dn =
V
1 17. The equation of a wave is represented by
(c) standing wave of frequency
b æ xö
a y = 10-4 sin çè 100t - ÷ø m, then the velocity of
10
(d) wave moving in + x direction, speed
b wave will be [2007]
12. The fundamental frequency of an open organ (a) 100 m/s (b) 4 m/s
pipe is 300 Hz. The first overtone of this pipe (c) 1000 m/s (d) 10 m/s
has same frequency as first overtone of a closed 18. A cylindrical tube open at both ends has a
organ pipe. If speed of sound is 330 m/s, then fundamental frequency n in air. The tube is
the length of closed organ pipe is [2011] dipped vertically in water so that half of it is
(a) 41 cm (b) 37 cm (c) 31 cm (d) 80 cm immersed in water. The fundamental frequency
13. A wave y = a sin (wt – kx) on a string meets with of air column is [2006]
another wave producing a node at x = 0. Then (a) n/2 (b) n (c) 2 n (d) 4 n
the equation of the unknown wave is [2010] 19. A source of sound of frequency 256Hz is moving
(a) y = a sin (wt + kx) rapidly towards a wall with a velocity of 5m/s.
(b) y = –a sin (wt + kx) How many beats per second will be heard if
(c) y = a sin (wt – kx) sound travels at a speed of 330 m/s by an
(d) y = –a sin (wt – kx) observer behind the source. [2006]
14. A source of sound produces waves of (a) 1.7 (b) 5.2 (c) 3.5 (d) 7.7
wavelength 60 cm when it is stationary. If the 20. The loudness and pitch of a sound depends on
speed of sound in air is 320 m s–1 and source (a) intensity and velocity [2005]
moves with speed 20 m s–1, the wavelength of (b) frequency and velocity
sound in the forward direction will be nearest to (c) intensity and frequency
[2010] (d) frequency and number of harmonics
(a) 56 cm (b) 60 cm (c) 64 cm (d) 68 cm 21. A progressive wave of frequency 500 Hz is trav-
15. Two tuning forks A and B sounded together elling with a velocity of 360 m/s. How far apart
give 6 beats per second. With an air resonance are two points 60o out of phase. [2005]
tube closed at one end, the two forks give reso- (a) 0.12 m (b) 0.24 m (c) 2.12 m (d) 1.2 m

Hints & Solutions


1. (c) The frequency of tuning fork, f = 392 Hz. From above equations,
1 f ¢ = 400 Hz.
Also 392 = F /m ...(i)
2 ´ 50
\ Beats frequency = 8 Hz.
After decreasing the length by 2%, we have
5 F 5 9g
1 2. (a) f = 2l m = 2l .........(i)
f ¢ = 2 ( 49 ) F / m ..........(ii) m
http://t.me/iitjeehelps

Waves P – 41
5. (b) As we know, frequency
5l / 2 f µ mg or f µ g
3 Mg In water, f w = 0.8fair
and f = 2l m ............(ii)

From above equations, we get M = 25 kg. ( 0.8)2 = 0.64
g
3. (a) Here, original frequency of sound, f0 = 100 Hz
rw
Speed of source Vs = 19.4 cos 60° = 9.7 Þ 1 - r = 0.64
19.4 m
rw
Þ = 0.36 ...(1)
rm
g¢ 2
In liquid, = ( 0.6 ) = 0.36
g
r1 r
60° 1- = 0.36 l = 0.64 ...(2)
S O rm rm
19.4 cos 60° = 9.7 From eq. (1) and (2)
From Doppler's formula rl 0.64
= \ rl = 1.77
æ V - V0 ö rn 0.36
f1 = f0 ç ÷
è V - Vs ø 6. (a) Let v = speed of sound and vS = speed of
tuning forks. Apparent frequency of fork
æ V-0 ö
f1 = 100 ç ÷ moving towards the observer is
è V - ( +9.7) ø æ v ö
V n1 = ç ÷n
f1 = 100 è v - vs ø
æ 9.7 ö Apparent frequency of the fork moving
V ç1 - ÷
è V ø away from the observer is
æ 9.7 ö æ v ö
f1 = 100 ç 1 + ÷ = 103Hz n2 = ç ÷n
è 330 ø è v + vs ø
Apparent frequency f1 = 103 Hz If f is the number of beats heard per second.
4. (c) For a string vibrating in its nth overtone (n then f = n1 – n 2
+ 1)th harmonic)
æ v ö æ v ö
æ (n + 1) px ö Þ f = ç v - v ÷n -ç v + v ÷n
y = 2A sin çè ÷ø cos wt è sø è sø
L
v ( v + vs ) - v ( v - vs )
Þf= (n)
///////////////////
///////////////////

v 2 - vs2
2vvs n æv ö
? Þ = f Þ 2 ç s ÷ = n = f{if vs << v}
v2 - vs2 è vø
l fv
For x = ]2A = a & n = 3; Þ vs =
3 2n
é æ 4p l ö ù putting v = 340 m/s, f = 3, n = 340 Hz we get,
y = êa sin ç . ÷ ú cos wt
ë è l 3ø û 340 ´ 3
vs = = 1.5m / s
æ 3ö 3 ´ 340
4p
= a sin cos wt = -a ç ÷ cos wt 7. (d)
3 è 2 ø 8. (c) Fundamental frequency is given by
l 3a 1 T 1 1
i.e. at x = , the amplitude is v= Þ vµ Þ P µ
3 2 2l m l v
EBD_7167
http://t.me/iitjeehelps

P – 42 BITSAT Topicwise Solved Papers


Since, P divided into l1 , l2 and l3 segments 15. (c) Let the frequency of the first fork be f1 and
Here l = l1 + l2 + l3 that of second be f2.
1 1 1 1 v
So = + + We then have, f1 = and
v v1 v2 v3 4 ´ 24
9. (c) Velocity of wave v = nl v
f2 =
v 4 ´ 25
where n = frequency of wave Þ n = We also see that f1 > f2
l
v 396 \ f1 – f2= 6 …(i)
n2 = 2 = = 396 Hz f1 24
l 2 100 ´ 10-2 and f = 25 …(ii)
no. of beats = n 1 – n2 = 4 2
æ xö Solving (i) and (ii), we get
10. (d) y = x0 cos2p ç nt - ÷ f1 = 150 Hz and f2 = 144 Hz
è lø
2p 16. (a) 17. (c)
y = x0 cos (vt – x) [Qv = nl] v
l 18. (b) When tube is open, n = , where n is
æ dy ö 2p 2l
çè ÷ø
dt max = x0 × l v = 2v(given) \ l = px0 fundamental frequency of open organ pipe
2 2 2 in air
11. (a) y ( x, t ) = e –( ax + bt + 2 ab xt ) = e – ( ax + bt ) When half of tube is dipped vertically in
It is a function of type water, it behaves as a closed pipe of length
y = f(wt + kx) l
\ y (x, t) represents wave travelling along , so fundamental frequency n in this case
2
–x direction. is
w b b v v
Speed of wave = = = . \ n¢ = = =n
k a a 4 (l / 2 ) 2 l
v 19. (d) When the source S is between the wall (W)
12. (a) For open pipe, n = , where n 0 is the and the observer (O).
2l
For direct sound the source is moving away
fundamental frequency of open pipe. from the observer, therefore the apparent
v 330 11
\ l= = = frequency
2 n 2 ´ 300 20
As freq. of 1st overtone of open pipe = freq. v 330
of 1st overtone of closed pipe n¢¢ = v + v n = × 256
s 330 + 5
v v
\ 2 =3 and frequency of reflected sound
2l 4 l¢
3l 3 11 v 330
Þ l' = = ´ = 41 .25 cm n¢ = v - v n = × 256 = 259.9
4 4 20 s 330 - 3
13. (b) Equation of a wave Number of beats/sec = n¢ – n¢¢ = 259 – 252.2 = 7.7
y1 = a sin (wt – kx) ....(i) 20. (c)
Let equations of another wave may be, 21. (a) We known that for a wave v = f l
y2 = a sin (wt + kx) ....(ii)
v 360
y3 = –a sin (wt + kx) ....(iii) So l = = = 0.72 m
If Eq. (i) propagate with Eq. (ii), we get f 500
y = 2a cos kx sin wt Now as in a wave path difference is related to
If Eq. (i), propagate with Eq. (iii), we get phase difference by the relation
y = –2a sin kx cos wt Phase difference Df = 60o = (p/180) x 60 = (p/3)
At x = 0, y = 0, wave produce node rad
So, Eq.(iii) is the equation of unknown wave l 0.72 p
14. (a) Path difference Dx = (Df) = = 0.12 m
2p 2p 3
http://t.me/iitjeehelps

Chapter
Electric Charges and Fields
15
1. A disk of radius a/4 having a uniformly constant rate. As a result, the spheres approach
distributed charge 6 C is placed in the x - y plane each other with a velocity v. Then v varies as a
with its centre at (–a/2, 0, 0). A rod of length a function of the distance x between the spheres,
carrying a uniformly distributed charge 8 C is as : [2016]
placed on the x-axis from x = a /4 to x = 5a/4. Two 1
point charges – 7 C and 3 C are placed at (a/4, – (a) v µ x2 (b) v µ x
a/4, 0) and (– 3a/4, 3a/4, 0), respectively. -
1
-1
Consider a cubical surface formed by six surfaces (c) v µ x 2 (d) v µ x
x = ± a/2, y = ± a/2, z = ± a/2. The electric flux 4. The surface charge density of a thin charged
through this cubical surface is [2017] disc of radius R is s. The value of the electric
y s
field at the centre of the disc is . With
2 Î0
respect to the field at the centre, the electric field
along the axis at a distance R from the centre of
x the disc [2016]
(a) reduces by 70.7%
(b) reduces by 29.3%
(c) reduces by 9.7%
–2C 2C 10C 12C (d) reduces by 14.6%
(a) (b) e0 (c) (d) 5. At the corners of an equilateral triangle of side
e0 e0 e0
a (1 metre), three point charges are placed (each
2. Two concentric conducting thin spherical shells
of 0.1 C). If this system is supplied energy at the
A, and B having radii rA and rB ((rB > rA) are
rate of 1 kw, then calculate the time required to
charged to QA and –QB (|QB| > |QA|). The electric
move one of the mid-point of the line joining
field along a line passing through the centre is
the other two. [2015]
[2017] (a) 50 h (b) 60 h (c) 48 h (d) 54 h
A

(a) (b)
a a

D
(c) (d) B a C
6. A solid sphere of radius R has a charge Q
distributed in its volume with a charge density r
= kra, where k and a are constants and r is the
3. Two identical charged spheres suspended from distance from its centre. If the electric field at r =
a common point by two massless strings of
R 1
lengths l, are initially at a distance d (d << l) is times that at r = R, the value of a is
2 8
apart because of their mutual repulsion. The [2015]
charges begin to leak from both the spheres at a (a) 3 (b) 5 (c) 2 (d) 7
EBD_7167
http://t.me/iitjeehelps

P – 44 BITSAT Topicwise Solved Papers

p (d) will increase if Q3 is of the same sign as Q1


7. Two point dipoles pkˆ and k̂ are located at and will decrease if Q3 is of opposite sign.
2
(0, 0, 0) and (1m, 0, 2m) respectively. The result- 13. A charge +q is at a distance L/2 above a square
ant electric field due to the two dipoles at the of side L. Then what is the flux linked with the
point (1m, 0, 0) is [2014] surface? [2010]
9p -7p q 2q q 6q
(a) 32p Î k̂ (b) 32p Î k̂ (a) 4e 0 (b) 3e 0 (c) 6e 0 (d) e0
0 0
7p 14. Two metallic spheres of radii 1 cm and 3 cm
(c) 32p Î k̂ (d) None of these
0
are given charges of –1 × 10–2 C and 5×10–2 C,
8. Three equal charges (q) are placed at corners of respectively. If these are connected by a
an equilateral triangle. The force on any charge conducting wire, the final charge on the bigger
is [2013] sphere is [2010]
(a) 2 × 10–2 C (b) 3 × 10–2 C
Kq 2
(a) zero (b) 3 (c) 4 × 10–2 C (d) 1 × 10–2 C
a2 15. If an electron has an initial velocity in a direction
2
Kq Kq 2 different from that of an electric field, the path of
(c) (d) 3 3 2 the electron is [2009]
3a 2 a
9. In a region of space having a uniform electric (a) a straight line (b) a circle
(c) an ellipse (d) a parabola
field E, a hemispherical bowl of radius r is placed.
16. The ratio of magnitudes of electric field due to
The electric flux f through the bowl is [2012] an electric dipole on the axis and on the equatorial
(a) 2pRE (b) 4pR2E (c) 2pR2E (d) pR2E line at equal distances will be [2008]
10. The electric field intensity just sufficient to (a) 4 : 1 (b) 1 : 2 (c) 2 : 1 (d) 1 : 1
balance the earth’s gravitational attraction on 17. A square surface of side L E
an electron will be: (given mass and charge of an meter in the plane of the
electron respectively are 9.1 × 10–31 kg and 1.6 × paper is placed in a q
10–19 C.) [2012, 2006] uniform electric field E
(a) –5.6 × 10–11 N/C (b) –4.8 × 10–15 N / C (volt/m) acting along the
(c) –1.6 × 10–19 N/C (d) –3.2 × 10–19 N / C same plane at an angle q
11. In an uniformly charged sphere of total charge with the horizontal side of
Q and radius R, the electric field E is plotted as the square as shown in Figure. The electric flux
function of distance from the centre. The graph linked to the surface, in units of volt m, is [2007]
which would correspond to the above will be (a) EL2 (b) EL2 cos q
[2011] (c) EL sin q
2 (d) zero
E E 18. Charge Q is distributed to two different metallic
spheres having radii R and 2R such that both
spheres have equal surface charge density, then
(a) (b) charge on large sphere is [2007]
R r 4Q Q 3Q 5Q
R r (a) (b) (c) (d)
E 5 5 5 4
E 19. A charge q is placed at the centre of the open
end of a cylindrical vessel. The flux of the electric
(c) (d) field through the surface of the vessel is [2006]
(a) zero (b) (q/e0) (c) (q/2e0) (d) (2q/e0)
R r 20. A solid spherical conductor of radius R has a
R r
12. A charge Q1 exerts some force on a second spherical cavity of radius a (a < R) at its centre.
charge Q2. If a 3rd charge Q3 is brought near, A charge + Q is kept at the centre. The cahrge at
then the force of Q1 exerted on Q2 – [2011] the inner surface, outer and at a position r (a < r
(a) will increase < R) are respectively [2005]
(b) will decrease (a) + Q, – Q, 0 (b) – Q, + Q, 0
(c) will remain unchanged (c) 0, – Q, 0 (d) + Q , 0, 0
http://t.me/iitjeehelps

Electric Charges and Fields P – 45

Hints & Solutions


1. (a) Total flux through the cubical surface, s
E= (given)
qin 2 Î0
f = Electric field along the axis at any distance
Î0
x from the centre of the disc
é 3 + 2 + (-7) ù 2C

Î0 úC = - æ
s x ö
ë û Î0
E' = ç1 - ÷
ç
2 Î0 ÷
2. (a) For, r < rA , E = 0 è x2 + R 2 ø
1 QA From question, x = R (radius of disc)
r = rA , E= .
4 p Î0 rA2
æ ö 4
1 QA \ E ' = s ç1 - R
÷ = E
rA < r < rB , E= 2 Î0 çè ÷
R2 + R2 ø
14
4 p Î0 r 2
\ % reduction in the value of electric
1 æ QA - QB ö
r = rB , E= . field
4p Î0 çè rB2 ÷ø

1 æ QB - QA ö æ 4 ö
= - 4p Î ç ÷ ç E - E ÷ ´ 100 1000
0è rB2 ø 14
=è ø = % = 70.7%
These values are correctly represent in E 14
option (a). 5. (a) Initial potential energy of the system
–QB é q2 q2 q2 ù
1 1 æ 3q 2 ö
= 4pe ê + + ú= ç ÷
QA 0 ëê a a a ûú 4pe0 èç a ø÷
rA
æ ( 0.1)2 ö
9ç ÷ = 27 ´107 J
= 9 ´ 10 3´
rB ç 1 ÷
è ø
Let charge at A is moved to mid-point O,
O
Then final potential energy of thhe system
Fe
3. (c) From figure tan q = ;q 1 é 2q 2 q2 ù
mg Uf = ê + ú
l
q 4pe0 ëê ( a / 2 ) a ûú
kq 2 x
=
x 2 mg 2l 1 æ q2 ö
= 5´ ç ÷ = 45 ´ 107 J
q q 4pe0 çè a 2 ÷ø
or x3 µ q2 …(1)
or x µ q …(2)
3/2 x Work done = Uf – Ui = 18 × 107 J
Differentiating eq. (1) w.r.t. time Also, energy supplied per sec = 1000 J
(given)
dx dq dq Time required to move one of the mid-point
3x2 µ 2q but is constant
dt dt dt of the line joining the other two
so x2(v) µ q Replace q from eq. (2)
18 ´ 107
x2(v) µ x3/2 or v µ x–1/2 t= = 18 ´ 10 4 s = 50 h
4. (a) Electric field intensity at the centre of the 1000
6. (c)
disc.
EBD_7167
http://t.me/iitjeehelps

P – 46 BITSAT Topicwise Solved Papers


7. (b) The given x 14. (b) At equilibrium potential of both sphere
point is at axis of becomes same if charge of sphere one x
r and other sphere Q – x then
P
dipole and where Q = 4 × 10–2 C
2 (1,0,2)
at equatorial line (1,0,0) v1 = v2
r P/2 kx k (Q - x )
of P dipole so z =
P 1cm 3cm
that field at given
3x = Q – x Þ 4x = Q
point.
Q 4 ´ 10 -2
x= = C = 1 ´ 10 -2
8. (b) Fnet = F2 + F2 + 2F2 cos 60° = 3F 4 4
Q¢ = Q – x = 3 × 10– 2C
9. (c) f = E(ds) cos q = E(2pr2) cos 0° = 2pr2 E.
15. (d) The path is a parabola, because initial
10. (a) – eE = mg velocity can be resolved into r two rectangular
uur components,
r one along E and other ^ to
9.1 ´ 10 -31 ´ 10 E . The former decreases at a constant rate
E =- = –5.6 × 10–11 N/C
1.6 ´ 10 -19 and latter is unaffected. The resultant path is
11. (c) Electric field inside the uniformly charged therefore a parabola.
16. (c)
kQ
sphere varies linearly, E = .r , (r £ R), 17. (d) Electric flux, f = EA cos q, where q
R3 = angle between E and normal to the
while outside the sphere, it varies as inverse surface.
kQ p
square of distance, E = ; (r ³ R ) which Here q = Þ f = 0
r2 2
is correctly represented in option (c). 18. (a) Let q and q' be the charges on spheres of
12. (c) radii R and 2R respectively.
Given q + q' = Q …(i)
q Surface charge densities are
L/2 q q'
s= s=
q 2 and 4p (2R) 2
4pR
L L/2 Given s = s'
13. (c)
L q q'
L \ = or, q' = 4q
2
L 4 pR 4 p (2R) 2
From eq. (i), q' = Q – q or, 4q = Q – q
The given square of side L may be or, Q = 5q …(ii)
considered as one of the faces of a cube Q 4Q
with edge L. Then given charge q will be \ q' = Q – q = Q - = .
5 5
considered to be placed at the centre of the 19. (c)
cube. Then according to Gauss's theorem, 20. (b) The charge at the inner surface, outer
the magnitude of the electric flux through surface and inside the conductor at P =
the faces (six) of the cube is given by (– Q, + Q, 0) as shown in the figure
f = q/e0 +Q +
Hence, electric flux through one face of the + r +
cube for the given square will be –Q P
1 q + +Q +
f' = f =
6 6e 0
+ +
+
http://t.me/iitjeehelps

Chapter
ElectrostaticPotential
and Capacitance 16
1. What is equivalent capacitance of circuit A/2 A/2
between points A and B? [2017]
3m F 9m F 27m F
K d
d
Infinite section K
B 2
A
3m F 9m F 27m F 3K e 0 A 4 K e0 A
(a) (b)
2
mF
4 4d 3d
(a) (b) mF
3 3
( K + 1) e 0 A K ( K + 3) e 0 A
(c) Infinite (
(d) 1 + 3 mF) (c)
2d
(d)
2 ( K + 1) d
2. A long, hollow conducting cylinder is kept 5. In the figure below, what is the potential
coaxially inside another long, hollow conducting difference between the point A and B and between
cylinder of larger radius. Both the cylinders are B and C respectively in steady state [2016]
initially electrically neutral. Then [2017]
(a) a potential difference appears between the
two cylinders when a charge density is
given to the inner cylinder.
(b) a potential difference appears between two
cylinders when a charge density is given
to the outer cylinder.
(c) no potential difference appears between the
two cylinders when a uniform line charge is
(a) VAB = VBC = 100V
kept along the axis of the cylinders.
(d) no potential difference appears between the (b) VAB = 75V, VBC = 25V
two cylinders when same charge density is (c) VAB = 25V, VBC = 75V
given to both the cylinders. (d) VAB = VBC = 50V
3. Two conducting shells of radius a and b are
6. A parallel plate capacitor of capacitance C is
connected by conducting wire as shown in connected to a battery and is charged to a
figure. The capacity of system is : [2017] potential difference V. Another capacitor of
a b capacitance 2C is similarly charge to a potential
difference 2V. The charging battery is now
disconnected and the capacitors are connected
in parallel to each other in such a way that the
positive terminal of one is connected to the
ab negative terminal of the other. The final energy
(a) 4 pe0 (b) 4 pe0 (a + b)
b-a of the configuration is [2015]
(c) zero (d) infinite
3
4. A parallel plate capacitor of area ‘A’ plate (a) Zero (b) CV 2
separation ‘d’ is filled with two dielectrics as 2
shown. What is the capacitance of the 25 9
(c) CV 2 (d) CV 2
arrangement ? [2016] 6 2
EBD_7167
http://t.me/iitjeehelps

P – 48 BITSAT Topicwise Solved Papers


7. Four point charges –Q, –q, 2q and 2Q are placed, 14. Calculate the area of the plates of a one farad
one at each corner of the square. The relation parallel plate capacitor if separation between
between Q and q for which the potential at the plates is 1 mm and plates are in vacuum [2009]
centre of the square is zero is [2014, 2006] (a) 18 × 108 m2 (b) 0.3 × 108 m2
(c) 1.3 × 108 m2 (d) 1.13 × 108 m2
1 15. A capacitor is charged to store an energy U. The
(a) Q = – q (b) Q = –
q charging battery is disconnected. An identical
1 capacitor is now connected to the first capacitor
(c) Q = q (d) Q = in parallel. The energy in each of the capacitors
q is [2008]
8. Two identical capacitors, have the same capaci- (a) 3 U/2 (b) U (c) U/4 (d) U/2
tance C. One of them is charged to potential V1 16. A parallel plate capacitor is charged and then
and the other to V2. The negative ends of the isolated. What is the effect of increasing the plate
capacitors are connected together. When the separation on charge, potential, capacitance,
positive ends are also connected, the decrease respectively? [2007]
in energy of the combined system is – [2013] (a) Constant, decreases, decreases
1 1 (b) Increases, decreases, decreases
(a) C (V12 - V22 ) (b) C (V1 + V2 )
2 2
(c) Constant, decreases, increases
4 4
1 1 (d) Constant, increases, decreases
(c) C (V1 - V2 )2 (d) C (V1 + V2 )
2
17. Figure shows a solid hemisphere with a charge
4 4 of 5nC distributed uniformly through its volume.
9. Two capacitors C1 and C2 are charged to 120 V
The hemisphere lies on a plane and point P is
and 200 V respectively. It is found that by
located on the plane, along a radial line from the
connecting them together the potential on each centre of curvature at distance 15 cm. The elec-
one can be made zero. Then [2012] tric potential at point P due to the hemisphere, is
(a) 5C1 = 3C2 (b) 3C1 = 5C2
(c) 3C1 + 5C2 = 0 (d) 9C1 = 4C2
10. A hollow metal sphere of radius 5 cm is charged
P
such that the potential on its surface is 10 V. The
potential at a distance of 2 cm from the centre of 15cm [2007]
the sphere is [2011] (a) 150V (b) 300V (c) 450V (d) 600V
(a) zero (b) 10 V (c) 4 V (d) 10/3 V 18. Two parallel metal plates having charges + Q
11. If the potential of a capacitor having capacity 6 and –Q face each other ata certain distance
mF is increased from 10 V to 20 V, then increase in between them. If the plaves are now dipped in
its energy will be [2011] kerosene oil tank, the electric field between the
(a) 4 × 10–4 J (b) 4 × 10–4 J plates will [2006]
(c) 9 × 10–4 J (d) 12 × 10–6 J (a) remain same (b) become zero
12. In a region, the potential is represented by (c) increases (d) decrease
V(x, y, z) = 6x – 8xy – 8y + 6yz, where V is in volts 19. Two capacitors C1 and C2 = Q C1
and x, y, z are in metres. The electric force 2C1 are connected in a circuit
experienced by a charge of 2 coulomb situated at with a switch between them
point (1, 1, 1) is : [2010] as shown in the figure. R

(a) 6 5 N (b) 30 N Initially the switch is open


and C1 holds charge Q. The
(c) 24 N (d) 4 35 N
switch is closed. At steady C2= 2C1
13. If on combining two charged bodies, the current
state, the charge on each capacitor will be
does not flow then [2009]
(a) charge is equal on both Q 2Q
(a) Q, 2Q (b) , [2005]
(b) capacitance is equal on both 3 3
(c) potential is equal on both 3Q 2Q 4Q
(c) ,3Q (d) ,
(d) resistance is equal on both 2 3 3
http://t.me/iitjeehelps

Electrostatic Potential and Capacitance P – 49

Hints & Solutions


1. (b) The effective circuit is shown in figure. 3µF 1µF
B
1mF 3mF 9mF
3µF 1µF

A B V2 V1

10 W
A C
1mF 3mF 9mF 20 W 100 V
The capacitance of upper series, On solving above equations, we get
1 1 1 1 1 V1 = 75V, V2 = 25V
= + + + +----¥ 6. (b) From the figure.
C 1 3 9 27
The net charge shared between the two
2 capacitors
\C= µC
3 Q¢ = Q2 – Q2 = 4CV – CV = 3CV
4 – +
Now CAB = 2C = mF – + Q1 = C1V1 = CV
3 – +
– +
2. (a) When charge is given to inner cylinder, an
electric field will be in between the cylinders.
+ –
So there is potential difference between the + –
+ –
cylinders. + –
q Q2 = C2V2 = (2C)(2V) = 4CV
3. (d) V = 0, and so C = ® ¥. The two capacitors will have some
V
potential, say V¢.
( A / 2) e0 Ae 0 Ae Ae The net capacitance of the parallel
4. (d) c1 = = , c2 = K 0 , c3 = K 0
d /2 d d 2d combination of the two capacitors
C¢ = C1 + C2 = C + 2C + 3C
c1 The potential of the capacitors
c3 Q¢ 3CV
K d V¢ = = =V
d K c2 C¢ 3C
2 The electrostatic energy of the capacitors
1 1 3
E ¢ = C¢V¢2 = ( 3C ) V 2 = CV 2
\
c eq. =
c1 ´ c 2
+ c3 =
( 3 + K ) K Ae 0 2 2 2
c1 + c 2 2 d ( K + 1) 7. (a) Let the side length of square be 'a' then
potential at centre O is
(Q C1 and C2 are in series and resultant
of these two in parallel with C3) –Q –q
5. (c) The equivalent circuit is shown in figure.
V1 + V2 = 100 O
and 2V1 = 6V2
2Q 2q
EBD_7167
http://t.me/iitjeehelps

P – 50 BITSAT Topicwise Solved Papers


r
k (-Q) k (-q) k (2q ) k (2Q) At (1, 1, 1), E = 2i$ + 10$j - 6k$
V= + + + =0 r
æ a ö a a a
çè ÷ Þ (E) = 22 + 102 + 62 = 140 = 2 35
2ø 2 2 2 r
\ F = qE = 2 ´ 2 35 = 4 35
(Given)
13. (c)
= – Q – q + 2q + 2Q = 0 = Q + q = 0
e0 A
=Q=–q 14. (d) For a parallel plate capacitor C =
d
8. (a) Initial energy of combined system
1 1
Cd 1 ´ 10 -3
U1 = CV12 + CV22 \ A= e =
2 2 0 8.85 ´ 10 -12
= 1.13 × 10 m2
8
V + V2
Final common potential, V = 1 , This corresponds to area of square of side
2 10.6 km which shows that one farad is very
Final energy of system,
2 large unit of capacitance.
1 æ V + V2 ö 15. (c) As battery is disconnected, total charge Q
U2 = 2 ´ C ç 1
2 è 2 ÷ø is shared equally by two capacitors.
Hence loss of energy = U 1 – U 2 = Energy of each capacitor
1 ( Q / 2) 2 1 Q 2 1
C ( V1 - V2 )
2
4 = = = U.
2C 4 2C 4
C C 16. (d) As the capacitor is isolated after charging,
+ 1– – 2+
9. (b) charge on it remains constant. Plate
120 V 200 V separation increases d, decreases C = Î0
For potential to be made zero, after A/d and hence increases potential V = Q/C.
connection 17. (b) By argument of symmetry (it will be half of
the potential produced by the full sphere)
é qù
120 C1 = 200 C2
êëQ C = v úû Þ
1 K(2Q) KQ
=
2 R R
Þ 3C1 = 5C2
10. (b) Potential at any point inside the sphere = KQ 9 ´ 109 ´ 5 ´ 10-9
Þ V= = = 300V
potential at the surface of the sphere = 10V. R 15 ´ 10 -2
11. (c) Capacitance of capacitor (C) = 6 mF = 6 × s Q
10–6 F; Initial potential (V1) = 10 V and final 18. (d) Electric field E = =
e Ae
potential e of kerosine oil is more than that of air.
(V2) = 20 V. The increase in energy (DU) As e increases, E decreases.
1 1 -6 2 2 19. (b) In steady state, both the capacitors are at
= C( V22 - V12 ) = ´ (6 ´ 10 ) ´ [( 20) - (10) ] the same potential,
2 2
Q1 Q 2 Q Q
= (3 ´10 -6 ) ´ 300 = 9 ´10 - 4 J . i.e., = or 1 = 2 or Q 2 = 2Q1
C1 C 2 C 2C
r ¶V $ ¶V $ ¶V $
12. (d) E=- i- j- k Also Q1 + Q2 = Q
¶x ¶y ¶z
Q 2Q
\ Q1 = , Q2 =
= -[(6 - 8y)iˆ + (-8x - 8 + 6z)ˆj + (6y)k]
ˆ
3 3
http://t.me/iitjeehelps

Chapter
Current Electricity
17
1. A wire is connected to a battery between the
point M and N as shown in the figure (1). The
same wire is bent in the form of a square and
then connected to the battery between the points
M and N as shown in the figure (2). Which of the
following quantities increases? [2017] (a) 10.2 W (b) 10.6 W (c) 10.8 W (d) 11.1 W
M N M N
4. Two resistances at 0° C with temperature
coefficient of resistance a1 and a2 joined in series
act as a single resistance in a circuit. The
() () temperature coefficient of their single resistance
(1) (2)
will be [2016]
a1a 2
(a) Heat produced in the wire and resistance (a) a1 + a 2 (b)
a1 + a 2
offered by the wire.
a1 - a 2 a1 + a 2
(b) Resistance offered by the wire and current (c) (d)
through the wire. 2 2
(c) Heat produced in the wire, resistance 5. The drift velocity of electrons in silver wire with
offered by the wire and current through the cross-sectional area 3.14 × 10–6 m2 carrying a
wire. current of 20 A is. Given atomic weight of Ag =
(d) Heat produced in the wire and current 108, density of silver = 10.5 × 103 kg/m3. [2016]
through the wire. (a) 2.798 × 10–4 m/sec.
2. In the circuit shown in figure the current through (b) 67.98 × 10–4 m/sec.
[2017] (c) 0.67 × 10–4 m/sec.
3W 2W 2W (d) 6.798 × 10–4 m/sec.
6. In the circuit shown in the figure, find the current
in 45 W. [2016]

180V 90W 45W


9V 8W 8W 4W 90W

100W 100W
50W 50W

2W 2W 2W 100W 50W
(a) 4 A (b) 2.5A (c) 2 A (d) 3.5 A
(a) the 3 W resistor is 0.50 A
7. A constant voltage is applied between the two
(b) the 3 W resistor is 0.25 A
ends of a uniform metallic wire. Some heat is
(c) the 4 W resistor is 0.50 A
developed in it. The heat developed is doubled
(d) the 4 W resistor is 0.25 A
if [2015]
3. A meter bridge is set up as shown, to determine
(a) both the length and the radius of the wire
an unknown resistance ‘X’ using a standard 10
are halved.
ohm resistor. The galvanometer shows null point
(b) both the length and the radius of the wire
when tapping-key is at 52 cm mark. The end-
are doubled.
corrections are 1 cm and 2 cm respectively for
(c) the radius of the wire is doubled.
the ends A and B. The determined value of ‘X’
(d) the length of the wire is doubled.
is [2017]
EBD_7167
http://t.me/iitjeehelps

P – 52 BITSAT Topicwise Solved Papers


8. The resistance of a wire is R. It is bent at the (a) 5 W (b) 10 W (c) 20 W (d) 30 W
middle by 180° and both the ends are twisted 15. The power dissipated in the circuit shown in the
together to make a shorter wire. The resistance figure is 30 Watts. The value of R is [2010]
of the new wire is [2015]
(a) 2 R (b) R/2 (c) R/4 (d) R/8 R
9. Two long coaxial and conducting cylinders of
radius a and b are separated by a material of
conductivity s and a constant potential differ- 5W
ence V is maintained between them, by a bat-
tery. Then the current, per unit length of the cyl-
inder flowing from one cylinder to the other is – 10V
4ps 4ps (a) 20 W (b) 15 W (c) 10 W (d) 30 W
(a) ln (b / a) V (b) V [2014] 16. Which of the following quantities do not change
(b + a) when a resistor connected to a battery is heated
2ps 2ps due to the current? [2010]
(c) ln (b / a) V (d) V
(b + a) (a) Drift speed
10. A wire X is half the diameter and half the length (b) Resistivity
of a wire Y of similar material. The ratio of (c) Resistance
resistance of X to that of Y is [2014]
(a) 8 : 1 (b) 4 : 1 (c) 2 : 1 (d) 1 : 1 (d) Number of free electrons
11. What should be the characteristic of fuse wire? 17. The length of a potentiometer wire is l. A cell of
[2013] emf E is balanced at a length l/3 from the posi-
(a) High melting point, high specific resistance tive end of the wire. If the length of the wire is
(b) Low melting point, low specific resistance increased by l/2. At what distance will be the
(c) High melting point, low specific resistance same cell give a balance point. [2009]
(d) Low melting point, high specific resistance (a) 2l/3 (b) l/2 (c) l/6 (d) 4l/3
12. In the circuit shown in figure potential differ- 18. In a Wheatstone bridge all the four arms have
ence between points A and B is 16 V. the current equal resistance R. If the resistance of
passing through 2W resistance will be [2013]
galvanometer arm is also R, the equivalent
9V 3V resistance of combination is [2008]
A B (a) 2R (b) R/4 (c) R/2 (d) R
4W 1W 4W
19. Potentiometer measures potential more
2W accurately because [2008]
(a) 2.5 A (b) 3.5 A (c) 4.0 A (d) zero (a) it measures potential in open circuit
13. Three voltmeters A, B and C having resistances (b) it uses sensitive galvanometer for null
R, 1.5 R and 3R, respectively, are connected as deflection
shown. When some potential difference is ap- (c) it uses high resistance potentiometer wire
plied between X and Y, the voltmeter readings (d) it measures potential in closed circuit
are VA, VB and VC respectively. Then – [2012] 20. A cell of internal resistance r is connected across
B an external resistance nr. Then the ratio of the
X A Y
terminal voltage to the emf of the cell is [2007]
1 1 n n -1
C (a) (b) (c) (d)
(a) VA ¹ VB = VC (b) VA = VB ¹ VC n n +1 n +1 n
21. In the circuit shown, when the switch is closed,
(c) VA ¹ VB ¹ VC (d) VA = VB = VC
the capacitor charges with a time constant
14. Calculate the effective resistance between A and
[2007]
B in following network. [2011]
10W 15W (a) RC C R
5W
(b) 2RC R
A 10W 10W B (c) (1/2) RC
+ –
(d) RC ln 2 B
10W 20W 30W
http://t.me/iitjeehelps

Current Electricity P – 53
22. In the electric network shown, when no 24. The effective resistance between points P and
current flows through the 4W resistor in the Q of the electrical circuit shown in the figure is –
arm EB, the potential difference between the 2R 2R
points A and D will be : [2006]
2W 2R
F E D

2V P Q
4W r r
R
2W [2005]
4V 2R

A 9V B 3V C
2R 2R
(a) 6 V (b) 3 V (c) 5 V (d) 4 V
23. When a current I is set up in a wire of radius r, the 2Rr 8R (R + r)
drift velocity is vd. If the same current is set up (a) (b)
R+r 3R + r
through a wire of radius 2 r, the drift velocity will
be [2005] 5R
(a) 4 vd (b) 2 vd (c) vd/2 (d) vd/4 (c) 2r + 4R (d)
2 + 2R

Hints & Solutions


1. (d) When the wire is bent in the form of a square 1ampere 3W 0.5A
and connected between M and N as shown 0.5A
in fig. (2), the effective resistance between 9V 9W 9V
8W 8W

M and N decreases to one fourth of the


2W
value in fig. (1). The current increases four
times the initial value according to the 1A 2W 2W 0.25A 3W 2W 0.25A
relation V = IR. Since H = I2 Rt, the decrease 3W 0.25A
in the value of resistance is more than 9V
8W 8W 4W
9V
8W 8W 8W

compensated by the increases in the value


2W 2W 2W 2W 2W
of current. Hence heat produced increases.
Percentage loss in energy during the The current divides into two equal parts if passes
collision ; 56% through two equal resistances in parallel.
2. (d) The net resistance of the circuit is 9W as Thus current through 4W resistor is 0.25 A.
shown in the following figures. X 10
3W 2W 2W 3W 2W
3. (b) At Null point l = l Þ
1 2
9V 8W 8W 4W
9V
8W 8W 8W X 10 53
= \X = = 10.6W
2W 2W 2W 2W 2W
53 50 5
4. (d) R1 = R0(1 + a1t) + R0(1 + a2t)
3W 3W 2W
a + a2
a= 1
9V
8W 8W
9V
8W 4W 2
5. (d) Number of electrons per unit volume of
2W 2W 2W silver
3W 6.023 ´ 1026
n= ´ 10.5 ´ 103
4W 9W
108
9V 9V
I
vd = = 6.798 × 10–4 m/sec.
2W neA
V 9V i
I= = = 1.0 A
R 9W 6. (c) 180V 90W 45W
The flow of current in the circuit is as follows. i

45W
EBD_7167
http://t.me/iitjeehelps

P – 54 BITSAT Topicwise Solved Papers


180 (10) 2
i= = 2A 30 = Þ R = 10 W
90 æ 5R ö
7. (b) The heat produced is given by çè ÷ø
5+ R
V2 rl æ pV 2 ö r 2 16. (d) Only number of free electrons is constant,
H= and R = 2 H =ç
R pr ç r ÷÷ l other factors are temperature dependent.
è ø E
Thus heat (H) is doubled if both length (l) 17. (b) Potential gradient in the first case = 0
l
and radius (r) are doubled. æ l ö æ E0 ö E0
l \ E = ç ÷ .ç ÷ = …(i)
8. (c) Resistance of wire (R) = r è 3ø è l ø 3
A
If wire is bent in the middle then Potential gradient in second case
l E 2E
l¢ = , A¢ = 2 A = 0 (x) 0 …(ii)
2 3l / 2 3l
R From equations (i) and (ii),
\ New resistance, R¢ = .
4
b E 0 æ 2E 0 ö l
l æ bö =ç ÷ x Þ x=
9. (c) V = ò E.dl = ln ..... (1) è 3l ø
2pe 0 r çè a ÷ø
3 2
a 18. (d)
r uuur l sl R R
Now, I = sò E.dA = sò .2pdr = R
2pe0 r e0 R R
2spe 0 2ps
From (1) : I = = v
e 0 ln (b / a) ln (b / a)
l Rx 2
10. (c) R µ Þ = Hint Þ since bridge is balanced hence
D 2 Ry 1
1
11. (d) Fuse wire should be such that it melts equivalent resistance is = (½R + ½R)
immediatley when strong current flows R¢
Þ R' = R
through the circuit. The same is possible if
its melting point is low and resistivity is high. 19. (a)
Er
12. (b) \ 4i1 + 2(i1 + i2) – 3 + 4i1 = 16V ...(i) 20. (c) V = E - Ir Þ V = E -
Using Kirchhoff’s second law in the closed loop (n + 1)r
we have nE V n
V= Þ =
9 – i2 – 2(i1 + i2) = 0 …(ii) n +1 E n +1
Solving equations (i) and (ii), we get 21. (a) The resistance in the middle plays no part
i1 = 1.5 A and i2 = 2 A in the charging process of C, as it does not
\ current through 2W resistor = 2 + 1.5 = 3.5 A. alter either the potential difference across
13. (d) VA = IR the RCcombination or the current through it.
22. (c) As no current flows through arm EB then
æ 2I ö VD = 0V; VE = 0V; VB = –4V; VA = 5V
VB = ç ÷ 1.5 R = IR
è 3ø So, potential difference between the points
æ Iö A and D
VC = ç ÷ 3R = IR \ VA = VB = VC VA – VD = 5V
è 3ø 23. (d) I = n A e vd or vd µ 1/pr2
14. (c) Equivalent resistance = (5 + 10 + 15) | | (10 + 24. (a) From symmetry the equivalent circuit
20 + 30) reduces to as shown in the figure.
30 ´ 60 2Rr
So, R eq = = 20 W R eq =
30 + 60 R+r
V2 2R 2R
15. (c) P = ...(i)
Req
1 1 1 5+ R
= + =
Req R 5 5R P Q
r
æ 5 R ö r
Req = çè ÷ P = 30 W
5 + Rø
Substituting the values in equation (i)
2R 2R
http://t.me/iitjeehelps

Chapter
Moving Charges
and Magnetism 18
1. Two very long, straight, parallel wires carry ur m0 I
steady currents I and -I respectively. The distance (c) B=
4p R
(
p$i – 2k$)
between the wires is d. At a certain instant of
ur m0 I
time, a point charge q is at a point equidistant
from the two wires, in the plane of the wires. Its (d) B =
4p R
(
p$i + 2k$)
instantaneous velocity v is perpendicular to this 4. A deuteron of kinetic energy 50 keV is describing
plane. The magnitude of the force due to the a circular orbit of radius 0.5 metre in a plane
magnetic field acting on the charge at this instant perpendicular to the magnetic field B. The kinetic
is [2017] energy of the proton that describes a circular
m 0 Iqv m 0 Iqv orbit of radius 0.5 metre in the same plane with
(a) (b) the same B is [2016]
2 pd pd
(a) 25 keV (b) 50 keV
2m0 Iqv
(c) (d) 0 (c) 200 keV (d) 100 keV
pd 5. A long straight wire along the Z-axis carries a
2. A charged particle moves through a magnetic current I in the negative Z-direction. The
field perpendicular to its direction. Then[2017] r
magnetic vector field B at a point having
(a) kinetic energy changes but the momentum
is constant coordinates (x, y) in the Z = 0 plane is [2015]
(b) the momentum changes but the kinetic
(a)
(
m 0 I yiˆ - xjˆ ) (b)
(
m 0 I xiˆ + yjˆ )
( )
energy is constant
(c) both momentum and kinetic energy of the
particle are not constant
(
2p x 2 + y 2 ) 2p x 2 + y 2

(d) both momentum and kinetic energy of the


(c)
(
m 0 I xjˆ - yiˆ ) (d)
(
m 0 I xiˆ - yjˆ )
3.
particle are constant
A wire carrying current I has the shape as shown (
2p x + y 2 2
) (
2p x 2 + y 2 )
in adjoining figure. Linear parts of the wire are 6. A charged particle with velocity 2 × 103
m/s
very long and parallel to X-axis while semicircular passes undeflected through electric and
portion of radius R is lying in Y-Z plane. Magnetic magnetic field. Magnetic field is 1.5 tesla. The
field at point O is : [2016] electric field intensity would be [2014, 2007]
Z (a) 2 × 103 N/C (b) 1.5 × 103 N/C
3
(c) 3 × 10 N/C (d) 4/3 × 10–3 N/C
7. A small current element of length dl and carrying
current is placed at (1, 1, 0) and is carrying current
I r
R in ‘+z’ direction. If magnetic field at origin be B1
O
Y r
and at point (2, 2, 0) be B2 then [2014]
r r r r
(a) B1 = B2 (b) | B1 | = | 2B2 |
I r r r r
(c) B1 = -B2 (d) B1 = -2B2
X
8. Field inside a solenoid is [2013]
ur
(a)
m I $
B=– 0
4p R
(
mi ´ 2k$ ) (a) directly proportional to its length
(b) directly proportional to current
ur
(b)
m I $
B=– 0
4p R
(
pi + 2k$ ) (c) inversely proportional to number of turns
(d) inversely proportional to current
EBD_7167
http://t.me/iitjeehelps

P – 56 BITSAT Topicwise Solved Papers


9. A uniform electric field and uniform magnetic m0I ˆ ˆ m0I ˆ ˆ
field are acting along the same direction in a (a) - (i + k) (b) (i + k)
8pa 2 pa
certain region. If an electron is projected in the m0 I ˆ ˆ m0 I ˆ ˆ
region such that its velocity is pointed along (c) (- i + k) (d) (i - k)
the direction of fields, then the electron [2013] 8pa 4pa 2
(a) will turn towards right of direction of motion 14. A conducting circular loop of radius r carries a
constant current i. It is placed in a uniform
(b) speed will decrease r r
(c) speed will increase magnetic field B0 such that B0 is perpendicular
(d) will turn towards left direction of motion to the plane of the loop. The magnetic force
10. A horizontal overhead powerline is at height of acting on the loop is [2009]
4m from the ground and carries a current of 100A (a) ir B0 (b) 2p ir B0
from east to west. The magnetic field directly (c) zero (d) p ir B0
below it on the ground is (m0 = 4p × 10–7 Tm A–1) 15. An ammeter reads upto 1 ampere. Its internal
(a) 2.5×10–7 T southward [2012, 2006] resistance is 0.81ohm. To increase the range to
10 A the value of the required shunt is [2009]
(b) 5 × 10–6 T northward (a) 0.03W (b) 0.3W (c) 0.9W (d) 0.09W
(c) 5 × 10–6 T southward 16. Two parallel wires carrying currents in the same
(d) 2.5 × 10–7 T northward direction attract each other because of [2008]
11. If m is magnetic moment and B is the magnetic (a) mutual inductance between them
field, then the torque is given by [2012, 2006] (b) potential difference between them
r (c) electric forces between them
r r |m|
(a) m.B (b) r (d) magnetic forces between them
|B| 17. Calculate the magnetic field at the centre of a
r r r r
(c) m ´ B (d) | m | . | B | coil in the form of a square of side 2a carrying a
12. A steady current is set up in a cubic network current I. [2007]
composed of wires of equal resistance and length 2 m0I 2 m0I
d as shown in figure. What is the magnetic field (a) (b)
pa pa
at the centre P due to the cubic network [2011] 5 m0I 3 m0 I
m 0 2I
B C (c) (d)
pa pa
(a) 18. The work done by a magnetic field, on a moving
4p d A D
E charge is [2006]
m 0 2I P r r
(b) (a) zero because F acts r parallel to v r
4p 2d F (b) positive because F acts perpendicular to v
(c) 0 r r
H G (c) zero because F acts perpendicular to v
r r
m 0 qpI (d) negative because F acts parallel to v
(d)
4p d ( ) 19. A circular loop of radius 0.3 cm lies parallel to a
13. The magnetic field at the origin due to the current much bigger circular loop of radius 20 cm. The
flowing in the wire is – [2010] centre of the smaller loop is on the axis of the
bigger loop. The distance between their centres
Y
is 15 cm. If a current of 2.0 A flows through the
smaller loop, then the flux linked with bigger loop
[2005]
(a) 9.1 × 10–11 weber (b) 6 × 10–11 weber
(c) 3.3 × 10–11 weber (d) 6.6 × 10–9 weber
I
upto
r
X 20. A particle is moving with velocity v = iˆ + 3jˆ and
a it produces an electric field at a point given by
r
a E = 2kˆ . It will produce magnetic field at that point
equal to (all quantities are in SI units) [2005]
Z I
6iˆ - 2ˆj 6iˆ + 2ˆj
parallel to y axis (a) (b)
c2 c2
(c) zero
upto (d) cannot be determined from the given data
http://t.me/iitjeehelps

Moving Charges and Magnetism P – 57

Hints & Solutions


1. (d) Net magnetic field due to the wires will be 5. (a) The wire carries a current I in the negative
downward as shown below uin r the figure. z-direction. We have to consider the
r ur
Since angle between v and B is 180°, magnetic vector field B at (x, y) in the z = 0
v Z plane.
Y
y
P(x, y)
I -I r q
X
q x
B Wires are in X-Y plane O×
d and velocity in Z-direction
ur r ur ur
Therefore, magnetic force F m = q(v ´ B) = 0 Magnetic field B is perpendicular to OP.
ur
2. (b) When a charged particle enters a magnetic \ B = B sin qiˆ - B cos qˆj
field at a direction perpendicular to the y x m I
direction of motion, the path of the motion sin q = , cos q = B= 0
r r 2 pr
is circular. In circular motion the direction ur m0 I
of velocity changes at every point (the \ B=
2pr 2
(
yiˆ - xjˆ )
magnitude remains constant).
Therefore, the tangential momentum will
or B=
(
ur m 0 I yiˆ - xjˆ
.
)
change at every point. But kinetic energy
1
(
2p x 2 + y 2 )
will remain constant as it is given by mv 2
2 6. (c) E = vB = 2 ´ 10 ´ 1.5 = 3 ´ 103 N C
3

and v2 is the square of the magnitude of 7. (c)


velocity which does not change. 8. (b)
r r
3. (b) 9. (b) v and B are in same direction so that
4. (d) For a charged particle orbiting in a circular magnetic force on electron becomes zero,
path in a magnetic field only electric force acts. But force on
mv2 Bqr electron due to electric field is opposite to
= Bqv Þ v = the direction of velocity.
r m 10. (c) The magnetic field is
2
or, mv = Bqvr
m 2I 2 ´ 100
Also, B= 0 = 10 -7 ´ = 5 × 10–6 T
4p r 4
W N
1 1 r Bqr B 2 q 2 r 2
EK = mv 2 = Bqvr = Bq . =
2 2 2 m 2m 100A
2 2 2
B q r
For deuteron, E1 =
2 ´ 2m 4m
2 2 2 S E
B q r
For proton, E2 = Ground
2m
B
E1 1 50keV 1 According to right hand palm rule, the
= Þ = Þ E2 = 100keV
E2 2 E2 2 magnetic field is directed towards south.
EBD_7167
http://t.me/iitjeehelps

P – 58 BITSAT Topicwise Solved Papers


r r r
11. (c) t = m ´ B 2m 0 I
12. (c) By symmetry, the magnetic field at the or B =
4pa
centre P is zero.
B I
13. (c) BOD = 0, BOB = 0 C
m0 I ˆ = m 0 I ( -iˆ + k)
BAB = [cos 45°( - ˆi) + cos 45°k] ˆ I
4 pa 2 8pa 45º
O I
14. (c) The magnetic field is perpendicular to the a 45º
plane of the paper. Let us consider two
diametrically opposite elements. By
Fleming's Left hand rule on element AB the A D
I
direction of force will be Leftwards and the
magnitude will be 2 m0I
Total magnetic field at O, B' = 4B = 4 ×
dF = Idl B sin 90° = IdlB 4 pa
x x x x x x x =
2 m0I
pa
x xB x x Cxdl x x 18. (c) Force on moving charge while moving in
dl dF r r r
magnetic field is; F < q (vr ´ B) where F is
dF
A D
x x x x Ix x x r
perpendicular to v .
x x x x x x x rr r
Work done/sec = F.v = Fv cos 90º = 0.
On element CD, the direction of force will 19. (a) As we know, Magnetic flux, f = BA
be towards right on the plane of the papper
and the magnitude will be dF = IdlB. m0 (2)(20 ´ 10 -2 ) 2
´ p(0.3 ´ 10 -2 ) 2
15. (d) 2 2
2[(0.2) + (0.15) ]
16. (d) on solving
17. (b) The current carrying coil ABCD may be = 9.216 × 10–11
assumed to be made of four current-carrying
» 9.2 × 10–11 weber
conductors AB, BC, CD and DA.Magnetic r
field at O due to current-carrying conductor r m 0 vr ´ rr r 1 qr
20. (a) B = q and E =
AB is : 4p r3 4p Î0 r3
r r
m0I r r v´E (iˆ + 3j)
ˆ ´ 2 kˆ
B= [sin45° + sin45°] \ m 0 Î0 (v ´ E) = 2 =
4 pa c c2
m0I –2ˆj + 6iˆ 6iˆ – 2ˆj
= 2sin45° = =
4pa c2 c2
m0I 1 é 1 1ù
or B = ×2× êc = Þ m0 Î0 = ú
4 pa 2 ë m Î
0 0 c2 û
http://t.me/iitjeehelps

Chapter
Magnetism and Matter
19
1. Current I is flowing in a coil of area A and number (c) M
of turns is N, then magnetic moment of the coil
is M equal to [2008]
(a) NIA (b) NI/A
(c) NI/ A (d) N2 AI
2. A curve between magnetic moment and O T
temperature of magnet is [2006] (d) M
(a) M

T
O
O T
3. A bar magnet of magnetic moment M, is placed in
(b) M a magnetic field of induction B. The torque exerted
on it is [2005]
r r r r
(a) M . B (b) - M . B
r r r r
(c) M ´ B (d) - B. M
O T

Hints & Solutions


1. (a) Current, I = NIA
2. (c)
r r
3. (c) t = M B sin q = M ´ B
EBD_7167
http://t.me/iitjeehelps

P – 60 BITSAT Topicwise Solved Papers

Chapter
Electromagnetic Induction
20
1. A coil is suspended in a uniform magnetic field, The self - inductance of the coil is : [2013]
with the plane of the coil parallel to the magnetic (a) 6 H (b) 0.67 H
lines of force. When a current is passed through (c) 3 H (d) 1.67 H
the coil it starts oscillating; It is very difficult to
6. Eddy currents are produced when [2013]
stop. But if an aluminium plate is placed near to
the coil, it stops. This is due to : [2017] (a) a metal is kept in varying magnetic field
(a) developement of air current when the plate (b) a metal is kept in steady magnetic field
is placed (c) a circular coil is placed in a magnetic field
(b) induction of electrical charge on the plate
(d) through a circular coil, current is passed
(c) shielding of magnetic lines of force as
aluminium is a paramagnetic material. 7. Two coaxial solenoids are made by winding thin
(d) electromagnetic induction in the aluminium insulated wire over a pipe of cross-sectional area
plate giving rise to electromagnetic A = 10 cm2 and length = 20 cm. If one of the
damping. solenoid has 300 turns and the other 400 turns,
2. A conducting circular loop is placed in a uniform their mutual inductance is [2013]
magnetic field of 0.04 T with its plane (m0 = 4p × 10 –7 Tm A–1)
perpendicular to the magnetic field. The radius (a) 2.4p × 10–5 H (b) 4.8p × 10–4 H
of the loop starts shrinking at 2 mm/s. The (c) 4.8p × 10 H–5 (d) 2.4p × 10–4 H
induced emf in the loop when the radius is 2 cm 8. The laws of electromagnetic induction have been
is [2016] used in the construction of a [2012]
(a) 4.8 p mV (b) 0.8 p mV (a) galvanometer (b) voltmeter
(c) 1.6 p mV (d) 3.2 p mV (c) electric motor (d) generator
3. A conducting square loop is placed in a magnetic 9. A metal rod of length 1 m is rotated about one of
field B with its plane perpendicular to the field. its ends in a plane right angles to a field of in-
The sides of the loop start shrinking at a ductance 2.5 × 10–3 Wb/m². If it makes 1800 revo-
constant rate a. The induced emf in the loop at lutions/min. Calculate induced e.m.f. between its
an instant when its side is ‘a’ is [2015] ends. [2011]
(a) 2aaB (b) a2aB (a) 2.471 V (b) 3.171 V
(c) 2a2aB (d) aaB (c) 0.471 V (d) 1.771 V
4. A coil 10 turns and a resistance of 20W is con- 10. The back emf induced in a coil, when current
nected in series with B.G. of resistance 30W. The changes from 1 ampere to zero in one milli-
coil is placed with its plane perpendicular to the second, is 4 volts, the self inductance of the coil
direction of a uniform magnetic field of induc- is [2010]
tion 10–2 T. If it is now turned through an angle (a) 1 henry (b) 4 henry
of 60° about an axis in its plane. Find the charge (c) 10–3 henry (d) 4 × 10–3 henry
induced in the coil. (Area of a coil = 10–2 m²) 11. Two solenoids of same cross-sectional area have
[2014] their lengths and number of turns in ratio of 1 : 2.
(a) 2 × 10–5 C (b) 3.2 × 10–5 C The ratio of self-inductance of two solenoids is
(c) 1 × 10–5 C (d) 5.5 × 10–5 C [2010]
5. When current in a coil changes from 5 A to 2 A (a) 1 : 1 (b) 1 : 2
in 0.1 s, average voltage of 50 V is produced. (c) 2 : 1 (d) 1 : 4
http://t.me/iitjeehelps

Electromagnetic Induction P – 61
12. Lenz’s law is a consequence of the law of NBA
conservation of [2009] (c) (d) NBAw 2
w2
(a) charge (b) mass 16. A coil of resistance 400W is placed in a magnetic
(c) energy (d) momentum field. If the magnetic flux f (wb) linked with the
13. A 100 millihenry coil carries a current of 1 ampere. coil varies with time t (sec) as f = 50t2 + 4. The
Energy stored in its magnetic field is [2008] current in the coil at t = 2 sec is [2006]
(a) 0.5 J (b) 1 J (a) 0.5 A (b) 0.1 A
(c) 0.05 J (d) 0.1 J (c) 2 A (d) 1 A
14. A generator has an e.m.f. of 440 Volt and internal 17. The loop shown moves L
resistance of 4000 hm. Its terminals are connected with a velocity v in a
to a load of 4000 ohm. The voltage across the uniform magnetic field
of magnitude B, di- v
load is [2008] P
rected into the paper.
(a) 220 volt (b) 440 volt L/2
The potential differ- L
(c) 200 volt (d) 400 volt ence between P and Q x B Q
15. A coil having number of turns N and cross-sec- is e.Then :
tional area A is rotated in a uniform magnetic 1
field B with an angular velocity w. The maximum (a) e = Blv [2005]
2
value of the emf induced in it is – [2007]
(b) e = Blv
NBA (c) P is negative with respect to Q
(a) (b) NBAw
w (d) Q is positive with respect to P.

Hints & Solutions


1. (d) Because of the Lenz's law of conservation df
of energy. \ emf induced E = –
dt
2. (d) Induced emf in the loop is given by Þ E = – B.2 (a0 – a t) (0 – a) = +2aaB
dA 4. (c) Given : n = 10 turns, Rcoil = 20W, RG = 30W,
e= – B. where A is the area of the loop. Total resistance in the circuit = 20 + 30 = 50W.
dt
A = 10–2 m², B = 10–2 T, f1 = 0°, f2 = 60°
d dr f - f 2 BnA cos q1 - BnA cos q 2
e = – B. (p r2) = – B p 2r q= 1 =
dt dt R R
–2
r = 2cm = 2 × 10 m
dr = 2 mm = 2 × 10–3 m BnA(cos 0 - cos 60) BnA (1 - 0.5)
= =
dt = 1s R R
0.5 ´ 10 -2 ´ 10 ´ 10 -2 50 ´ 10-5
2×10–3 = = = 1 × 10–5 C
e = – 0.04 × 3.14 × 2 × 2 ×10–2 × V 50 50
1
(Charge induced in a coil)
= 0.32 p × 10–5V 5. (d) According to Faraday’s law of
= 3.2 p ×10–6V electromagnetic induction,
= 3.2 p mV Ldi
3. (a) At any time t, the side of the square a = (a0 Induced emf, e =
dt
– a t), where a0 = side at t = 0.
At this instant, flux through the square : æ 5–2 ö
50 = L ç ÷
f = BA cos 0° = B (a0 – a t)2 è 0.1sec ø
EBD_7167
http://t.me/iitjeehelps

P – 62 BITSAT Topicwise Solved Papers

50 ´ 0.1 5 m0 N 2A N2
Þ L= = = 1.67 H (b) From L =
3 3 11. a
l l
6. (a)
L1 (1 / 2)
2
m N N A 4p ´10 -7 ´ 300 ´ 400 ´ 100 ´ 10-4 1
7. (d) M= 0 1 2 = we get, = =
l 0.2 L2 1/ 2 2
= 2.4p ´ 10 -4 H
12. (c)
13. (c) Energy stored U is given by
8. (b) Current will be induced,
1 1
when e– comes closer the induced current U = L i 2 = ´ (100 ´ 10 -3 ) (1) 2 = 0.05 J.
will be anticlockwise 2 2
when e– comes farther induced current will 14. (d) Total resistance of the circuit = 4000 + 400 =
4400 W
be clockwise
V 440
Current flowing i = = = 0.1 amp.
R 4400
Voltage across load = R i = 4000 × 0.1 = 400
e– e– volt.
15. (b) The flux linking with the coil at any instant
9. (c) Given : l = 1m, B = 5 × 10–3 Wb/m² t is given as
1800 F = NBA cos wt
f= = 30 rotations/sec
60 dF
\ e=- = NBA sin wt
In one rotation, the moving rod of the metal dt
traces a circle of radius r = l Therefore, the maximum value of emf is emax =
\ Area swept in one rotation = pr2 NBAw
16. (a) According, to Faraday’s law of induction
df d dA Bpr 2 df
= ( BA) = B. = Induced e.m.f. e = - = - (100t )
dt dt dt T dt
= B f p r = (5 × 10 ) × 3.14 × 30 × 1 = 0.471 V
2 –3
Induced current i at t = 2 sec.
di e 100 ´ 2
10. (d) e = – L =+ = + 0.5Amp
dt =
di 0 -1 R 400
But e = 4V and = -3 = – 1/10–3 1
dt 10
-1 17. (a) Potential difference, e = Blv
2
\ (–L) = 4 Þ L = 4 × 10–3 henry
10-3
http://t.me/iitjeehelps

Alternating Current P – 63

Chapter
Alternating Current
21
1. An inductor of inductance L = 400 mH and 5. An inductor 20 × 10–3 henry, a capacitor 100µF
resistors of resistance R1 = 2W and R2 = 2W are and a resistor 50W are connected in series across
connected to a battery of emf 12 V as shown in a source of EMF V = 10 sin 314t. If resistance is
the figure. The internal resistance of the battery removed from the circuit and the value of induc-
is negligible. The switch S is closed at t = 0. The tance is doubled, then the variation of current
potential drop across L as a function of time is with time in the new circuit is – [2014]
[2016] (a) 0.52 cos 314 t (b) 0.52 sin 314 t
E (c) 0.52 sin (314 t + p/3) (d) None of these
12 -3t 6. A sinusoidal voltage of peak value 283 V and fre-
(a) e V L
t R1 quency 50 Hz is applied to a series LCR circuit in
which R = 3W, L = 25.48 mH, and C = 796 µF. Find
(b) (
6 1- e -t / 0.2
)V R2
the impedance of the circuit. [2013]
(a) 5 W (b) 12 W (c) 18 W (d) 22 W
(c) 12e–5t V 7. In an LCR circuit, the voltages across the
S
(d) 6e–5t V components are VL, VC and VR respectively. The
2. A r esistor of resistance R, capacitor of voltage of source will be [2012]
capacitance C and inductor of inductance L are (a) [VR + VL + VC]
connected in parallel to AC power source of (b) [VR2 + VL2 + VC2]1/2
voltage e0 sin wt. The maximum current through
the resistance is half of the maximum current (c) [VR2 + (VL + VC)2]1/2
through the power source. Then value of R is (d) [VR2 + (VL – VC)2]1/2
[2016] 8. The r.m.s. value of potential difference V shown
in the figure is [2012]
3 1 V
(a) (b) 3 – wL
1 wC V0
wC –
wL
1 O t
(c) 5 – wL (d) None of these T/2 T
wC (a) V0 (b) V0 / 2
3. A direct current of 5A is superposed on an
alternating current I = 10 sin wt flowing through (c) V0/2 (d) V0 / 3
the wire. The effective value of the resulting 9. Which one of the following curves represents
current will be [2015] the variation of impedance (Z) with frequency f
(a) (15/2)A (b) 5 3A in series LCR circuit? [2011]
Z Z
(c) 5 5A (d) 15 A
4. In the circuit shown below, the ac source has (a) (b)
voltage V = 20 cos(wt) volt with w = 2000 rad/s.
The amplitude of the current will be nearest to
[2015] f f
(a) 2A Z Z
(b) 3.3A (c) (d)
(c) 2 / 5A
(d) 5A f f
EBD_7167
http://t.me/iitjeehelps

P – 64 BITSAT Topicwise Solved Papers


10. An alternating voltage V = V0 sin wt is applied 15. A resistance of 400
across a circuit. As a result, a current I = I0 sin W is connected in
(wt – p/2) flows in it. The power consumed per series with an
cycle is [2010] inductor of (3/p)
(a) zero (b) 0.5 V0I0 H and this
combination is
(c) 0.707 V0I0 (d) 1.414 V0I0
connected across
11. A resistance R and inductance L and a capacitor an A.C. source as shown in figure. [2007]
C all are connected in series with an AC supply. 5
The resistance of R is 16 ohm and for a given (a) In the circuit, peak value of current is A
4
frequency, the inductive reactance of L is 24 ohm (b) In the circuit, potential difference across
and capacitive reactance of C is 12 ohm. If the resistance is in same phase with the current
current in the circuit is 5 amp., find the potential (c) In the circuit, potential difference across
difference across R, L and C. [2010] inductor is 53º leading with current
(a) 30, 20, 50 volt (b) 40, 100, 60 volt (d) In the circuit, potential difference across
(c) 70, 110, 60 volt (d) 80, 120, 60 volt inductor is 37º leading with current
12. The instantaneous current from an a.c. source is 16. Which of the following curves correctly repre-
I = 6 sin 314 t. What is the rms value of the sents the variation of capacitive reactance (Xc)
with frequency n – [2006]
current ? [2009]
(a) 3 2 amp (b) 2 2 amp XC XC

(c) 2 amp (d) 2 amp (a) (b)


13. A coil has resistance 30 ohm and inductive n n
reactance 20 ohm at 50 Hz frequency. If an ac
source, of 200 volt, 100 Hz, is connected across XC
the coil, the current in the coil will be [2009, 2006] (c) (d)
XC

(a) 4.0 A (b) 8.0 A (c) 7.2 A (d) 2.0 A


14. The primary winding of a transformer has 100 n
n
turns and its secondary winding has 200 turns.
17. If I = I0 sin (wt – p/2), and E = E0 sinwt then the
The primary is connected to an A.C. supply of
power loss is [2005]
120 V and the current flowing in it is 10 A. The
EI E 0 I0
voltage and the current in the secondary are (a) (b)
[2008] 2 2
(a) 240 V, 5 A (b) 240 V, 10 A E0 I 0
(c) 60 V, 20 A (d) 120 V, 20 A (c) (d) Zero
2

Hints & Solutions


E 3
1. (c) i= [1 - e - R2t / L ] ÞR=
R2 æ 1 ö
E R2 - R2t / L E - L2
R t çè wC – ÷
di wL ø
Þ = . .e = e
dt R2 L L 3. (b) Total carrent, l = (5 + 10 sin wt)
Hence, potential drop across 1/2
æ E -R t / L ö é T I2 dt ù
L = ç e 2 ÷ L = 12e–5tV êò ú
èL ø Þ Ieff =ê 0 ú
T
iR0 1 êë ò0 dt úû
2. (a) =
2
(i ) + (i – i ) 2 2
R0 c0 L0
http://t.me/iitjeehelps

Alternating Current P – 65

1/2
é1 T 2 ù 2
= ê ò ( 5 + 10sin wt ) dt ú æ 1 ö
ë T 0 û 9. (c) Z = R 2 + ç 2pfL - ÷
è 2pfC ø
1 T 2 1
T ò0
sin wt.dt = 10. (a) The phase angle between voltage V and
2
current I is p/2.
1/2
é 1 ù 11. (d) VR = i R = 5 ´ 16 = 80 Volt
So, Ieff = ê 25 + ´100ú = 5 3A
ë 2 û
VL = i ´ (wL) = 5 ´ 24 = 120 Volt
2
æ 1 ö VC = i ´ (l / wC) = 5 ´ 12 = 60 Volt
4. (a) Z = R 2 + ç wL - ÷ = 10 W
è wC ø I0 6
12. (a) Irms = = = 3 2 amp.
V0 20 2 2
i= = = 2A.
Z 10 13. (a) If w = 50 × 2p then wL = 20W
5. (a) XC – XL = 31.85 – 2(6.28) = 19.29 If w¢ = 100 × 2p then w¢L = 40W
10 Current flowing in the coil is
Im = = 0.52
19.29 200 200
I= = = 4A
Hence I = 0.52 sin (314 t + p/2) = 0.52 cos 314 t Z R 2 + (w¢L) 2
6. (a) XL = 2pfL
æn ö
= 2 ´ 3.14 ´ 50 ´ 25.48 ´ 10-3 W = 8W 14. (a) Es = E p ´ ç s ÷ = 240V
ç np ÷
1 1 è ø
XC = = = 4W
2p fC 2 ´ 3.14 ´ 50 ´ 796 ´ 10-6 æ np ö
Therefore, Ι s = Ι p ç ÷ = 5 amp
è ns ø
Z = R 2 + (X L - X C )2 = 32 + (8 - 4) 2 = 5W 15. (b)
7. (d) 1 1 1
16. (c) X C = = or XC µ
(T / 2)V0 2 + 0 V wC 2pnC n
8. (b) Vrms = = 0 . 17. (d)
T 2
EBD_7167
http://t.me/iitjeehelps

Chapter
Electromagnetic Waves
22
1. The electric and the magnetic field associated 4. The magnetic field in a travelling electromagnetic
with an E.M. wave, propagating along the +z- wave has a peak value of 20 nT. The peak value
axis, can be represented by [2015] of electric field strength is [2009]
r r (a) 3 V/m (b) 6 V/m (c) 9 V/m (d) 12V/m
ˆ B = B ˆjù
(a) éë E = E 0 i, 0 û 5. A plane electromagnetic wave is incident on a
r r r material surface. If the wave delivers momentum
(b) éë E = E 0 k, B = B0ˆi ùû p and energy E, then [2008]
r r (a) p = 0, E = 0 (b) p ¹ 0, E ¹ 0
(c) éë E = E 0 ˆj, B = B0 iˆ ùû (c) p ¹ 0, E = 0 (d) p = 0, E ¹ 0
r r 6. If vs, vx and vm are the speed of soft gamma rays,
(d) éë E = E 0 ˆj,B = B0 kˆ ùû X-rays and microwaves respectively in vacuum,
2. Which of the following electromagnetic then [2007]
radiations has the smallest wavelength?[2013] (a) vs > vx > vm (b) vs < vx < vm
(a) Ultraviolet rays (b) X-rays (c) vs > vx < vm (d) vs = vx = vm
(c) g-rays (d) Microwaves 7. The electromagnetic radiation used in food
3. An electromagnetic wave passes through space processing sterilizing agent is [2006]
(a) microwaves (b) UV rays
and its equation is given by E = E0 sin (wt – kx)
(c) gamma rays (d) radio waves
where E is electric field. Energy density of 8. A plane electromagnetic wave travels in free
electromagnetic wave in space is [2011] space along x-axis. At a particular point in space,
1 1 the electric field along y-axis is 9.3V m–1. The
(a) e 0 E 20 (b) e 0 E 02 magnetic induction (B) along z-axis is [2005]
2 4
(a) 3.1 × 10–8 T (b) 3 × 10–5 T
(c) e 0 E 20 (d) 2e 0 E 02 (c) 3 × 10 T –6 (d) 9.3 × 10–6 T

Hints & Solutions


1. (a) E.M. wave always propagates in a direction r r r
4. (b) E0 = B0 ´ C
perpendicular to both electric and magnetic
fields. So, electric and magnetic fields r r
| E 0|=| B0 | × | C |= 20 ´ 10-9 ´ 3 ´108 = 6V/m.
should be along + X and + Y- directions
respectively. Therefore, option (a) is the 5. (b) An electromagnetic wave has both energy
correct option. and momentum.
2. (c) 6. (d)
3. (a) Energy density 7. (b)
2 E E 9.3
2 æ E0 ö 1 8. (a) C= ÞB= = = 3.1 ´ 10 -8 T
= e0 E rms = e 0 ç ÷ = e0 E02 B C 3 ´ 108
è 2ø 2
http://t.me/iitjeehelps

Ray Optics and Optical Instruments P – 67

Chapter
Ray Optics and Optical
Instruments 23
1. Two plano-concave lenses (1 and 2) of glass of (a) lies between 2 and 1
refractive index 1.5 have radii of curvature 25 cm
and 20 cm. They are placed in contact with their (b) lies between 2 and 2
curved surface towards each other and the space (c) is less than 1
between them is filled with liquid of refractive (d) is greater than 2
index 4/3. Then the combination is [2017] 5. A lens having focal length f and aperture of
diameter d forms an image of intensity I. Aperture
d
of diameter in central region of lens is covered
2
1 2 by a black paper. Focal length of lens and
(a) convex lens of focal length 70 cm intensity of image now will be respectively:
(b) concave lens of focal length 70 cm [2016]
(c) concave lens of focal length 66.6 cm I 3f I
(d) convex lens of focal length 66.6 cm (a) f and (b) and
4 4 2
2. On a hypotenuse of a right angle prism (30° – 60°
– 90°) of refractive index 1.50, a drop of liquid is 3I f I
(c) f and (d) and
placed as shown in figure. Light is allowed to 4 2 2
fall normally on the short face of the prism. In 6. A ray parallel to principal axis is incident at 30°
order that the ray of light may get totally from normal on concave mirror having radius of
reflected, the maximum value of refractive index curvature R. The point on principal axis where
is : [2017] rays are focussed is Q such that PQ is [2015]
R
(a)
2 M
30°
R
(b)
3
2 R-R P Q C
(c)
(a) 1.30 (b) 1.47 2
(c) 1.20 (d) 1.25 (d) æ 1 ö
R ç1 - ÷
3. A biconvex lens has a radius of curvature of è 3ø
magnitude 20 cm. Which one of the following 7. A planoconvex lens fits exactly into a
options best describe the image formed of an planoconcave lens. Their plane surface are
object of height 2 cm placed 30 cm from the lens? parallel to each other. If the lenses are made of
(a) Virtual, upright, height = 1 cm [2016] different materials of refractive indices µ1 & µ2
(b) Virtual, upright, height = 0.5 cm and R is the radius of curvature of the curved
surface of the lenses, then focal length of
(c) Real, inverted, height = 4 cm
combination is [2015]
(d) Real, inverted, height = 1cm
R 2R
4. For the angle of minimum deviation of a prism to (a) (b)
be equal to its refracting angle, the prism must µ1 – µ2 µ1 – µ2
be made of a material whose refractive index : R R
(c) (d)
[2016] 2 ( µ1 – µ2 ) 2 – ( µ1 + µ2 )
EBD_7167
http://t.me/iitjeehelps

P – 68 BITSAT Topicwise Solved Papers


8. The magnifying power of a telescope is 9. When
æ mm ö æ 1 1ö
it is adjusted for parallel rays, the distance (a) dç 1 2 ÷ (b) d ç + ÷
between the objective and the eye piece is found è m1 + m 2 ø m
è 1 m 2ø
to be 20 cm. The focal length of lenses are
æ 1 1ö æ 1 ö
[2014, 2010] (c) 2d ç + ÷ (d) 2d ç
(a) 18 cm, 2 cm (b) 11 cm, 9 cm è m1 m 2 ø è m1m 2 ÷ø
(c) 10 cm, 10 cm (d) 15 cm, 5 cm 14. A plano-convex lens of focal length 30 cm has
9. When light is refracted, which of the following its plane surface silvered. An object is placed 40
does not change ? [2013] cm from the lens on the convex side. The
(a) Wavelength (b) Frequency distance of the image from the lens is [2009]
(c) Velocity (d) Amplitude (a) 18 cm (b) 24 cm (c) 30 cm (d) 40 cm
10. A Convex lens of crown glass (µ = 1.525) will 15. Time taken to cross a 4 mm window glass of
behave as a divergent lens if immersed in [2013] refractive index 1.5 will be- [2008]
(a) 2 × 10–8 sec (b) 2 × 108 sec
(a) water (µ = 1.33)
(c) 2 × 10–11 sec (d) 2 × 1011 sec
(b) in a medium of µ = 1.525
16. A fish looking up through the water sees the
(c) carbon disulphide (µ = 1.66)
outside world contained in a circular horizon. If
(d) it cannot act as a divergent lens the refractive index of water is 4/3 and the fish is
11. Wavelength of light used in an optical instrument 12 cm. below the surface, the radius of this circle
are l1 = 4000Å and l2 = 5000 Å, then ratio of in cm is – [2007]
their respective resolving power s (a) 36 5 (b) 4 5
(corresponding to l1 and l2) is [2012]
(c) 36 7 (d) 36 / 7
(a) 16 : 25 (b) 9 : 1 (c) 4 : 5 (d) 5 : 4
17. An air bubble under water shines brightly
12. A thin convergent glass lens (mg = 1.5) has a because of the phenomenon of : [2006]
power of + 5.0 D. When this lens is immersed in (a) diffraction
a liquid of refractive index m, it acts as a divergent (b) dispersion
lens of focal length 100 cm. The value of m must (c) interference
be [2011, 2008] (d) total internal reflection
(a) 4/3 (b) 5/3 (c) 5/4 (d) 6/5 18. A vessel is half filled with a liquid of refractive
13. A vessel of depth 2d cm. is half filled with a index m. The other half of the vessel is filled with
liquid of refractive index µ1 and the upper half an immiscible liquid of refrative index 1.5 m. The
with a liquid of refractive index µ2. The apparent apparent depth of the vessel is 50% of the actual
depth of the vessel seen perpendicularly is – depth. Then m is [2005]
[2011] (a) 1.4 (b) 1.5 (c) 1.6 (d) 1.67
http://t.me/iitjeehelps

Ray Optics and Optical Instruments P – 69

Hints & Solutions


1 æ 3 öæ 1 1 ö 1 i = A = 90°
1. (c) -1 - ÷ =-
f1 = çè 2 ÷çøè ¥ 25 ø 50
, from smell’s law
1 sin i = n sin r1
1 æ 4 öæ 1 1 ö 3 A
-1 + ÷=
f 2 = çè 3 ÷çøè 25 20 ø 100 sin A = n sin
2
1 A A A
æ 3 öæ 1 1ö 1 2 sin cos = n sin
and -1 - ÷ =-
f3 = çè 2 ÷çøè -20 ¥ ø 40 2 2 2
A
1 1 1 1 2 cos = n
Now + + 2
f = f1 f 2 f3 when A = 90° = imin
1 3 1 then nmin = 2
=- + -
i = A = 0 nmax = 2
50 100 40
\ f = – 66.6 cm 5. (c) By covering aperture, focal length does not
1
2. (a) Cmax = 60° change. But intensity is reduced by
4
1 d
\ r md = times, as aperture diameter is covered.
sin 60°
2
mg 2 I 3I
or
ml
= \ I' = I - =
3 4 4
3 3 3I
\ µl = mg = ´ 1.5 \ New focal length = f and intensity = .
2 2 4
6. (d) From similar triangles,
= 1.3
3. (c) M
30°
3 0°
4. (b) A A Prism angle R
angle of 30°
Smin P Q C
minimum
deviation
Incident C e angle of
r1 r2
angle emergence R

QC R
=
sin 30° sin120°
B C
sin 30° R
The angle of minimum deviation is given as or QC = R ´ =
sin120° 3
d min = i + e–A Thus PQ = PC – QC
for minimum deviation R 1 ö
= R- = R æç1 -
d min = A then 3 è 3 ÷ø
2A = i + e 7. (a) If F be the equivalent focal length, then
in case of d min i = e 1 1 1
= +
A F f1 f 2
2A = 2i r1 = r2 =
2
EBD_7167
http://t.me/iitjeehelps

P – 70 BITSAT Topicwise Solved Papers

1 æ1 1ö
= ( µ1 –1) ç + ÷ + ( µ2 –1) æ m g ö mg
F è¥ Rø - 5 çç - 1÷÷ = -1
æ 1 1 ö µ – µ2 è m1 ø m a
ç – ÷= 1 1 .5 -1
è –R ¥ ø R -1 = (1 .5 - 1) = -0 .1 ;
R m1 5
F=
µ1 – µ2 1 .5 5
m1 = =
f0 0.9 3
8. (a) = 9 , \ f0 = 9 fe
fe d1 d 2 æ 1 1 ö
13. (b) h ¢ = + =dç + ÷
Also f0 + fe = 20 (Q final image is at infinity) m1 m 2 è m1 m 2 ø
9 fe + fe = 20, fe = 2 cm, \ f0 = 18 cm 14. (b) 15. (c)
9. (b) Frequency does not change on refraction.
1 r
10. (c) 16. (d) sin i c = =
11. (d) Resolving power of an optical instrument m r2 + h 2
1 r
µ
l
Resolving power at λ1 l ic
= 2 h
Resolving power at l 2 l1 ic
æ 1 ö
çç Limit of resolution µ ÷÷ Fish
è resolving power ø Using h = 12 cm, µ = 4/3
5000 36
\ Ratio of resolving power = We get cm.
4000 7
5 17. (d)
= =5:4
4 18. (d) Let d be the depth of two liquids.
æ mg ö Then apparant depth
ç - 1÷÷
Pa è a ç m +5 ( d / 2) ( d / 2) d
12. (b) = ø= = -5 + = or 1 + 2 = 1
P1 æ m g ö - 100 / 100 m 1. 5 m 2 m 3m
ç ÷
ç m - 1÷ Solving we get m = 1.671
è 1 ø
http://t.me/iitjeehelps

Chapter
Wave Optics
24
1. A telescope has an objective lens of 10 cm interference pattern is observed is 1m. Distance
diameter and is situated at a distance of one between the images of the virtual source when
kilometer from two objects. The minimum a convex lens of focal length 16 cm is used at a
distance between these two objects, which can distance of 80 cm from the eyepiece is 0.8 cm.
be resolved by the telescope, when the mean [2015]
wavelength of light is 5000 Å, is of the order of (a) 0.0006 Å (b) 0.0006 m
(a) 5 cm (b) 0.5 m [2017] (c) 600 cm (d) 6000 Å
(c) 5 m (d) 5mm 6. In a YDSE, the light of wavelength l = 5000 Å is
2. There are two sources kept at distances 2 l. A used, which emerges in phase from two slits a
large screen is perpendicular to line joining the distance d = 3 × 10–7m apart. A transparent sheet
sources. Number of maximas on the screen in of thickness t = 1.5 × 10–7m refractive index m =
this case is (l = wavelength of light) [2017] 1.17 is placed over one of the slits. what is the
new angular position of the central maxima of
¥ the interference pattern, from the centre of the
S1 S2 screen? Find the value of y. [2015]
2l D ( m - 1) t
(a) 4.9°and
¥ 2d p

(a) 1 (b) 3 (c) 5 (d) 7 D ( m - 1) t


(b) 4.9°and y
3. In the Young’s double-slit experiment, the d S1
q
intensity of light at a point on the screen where
D ( m + 1) t d q d sinq
the path difference is l is K, (l being the wave (c) 3.9°and S2
length of light used). The intensity at a point d
where the path difference is l/4, will be :[2016]
(a) K (b) K/4 (c) K/2 (d)Zero 2D ( m + 1) t
4. A ray of light of intensity I is incident on a parallel (d) 2.9°and
d
glass slab at point A as shown in diagram. It 7. A parallel beam of monochromatic light is incident
undergoes partial reflection and refraction. At on a narrow rectangular slit of width 1mm. When
each reflection, 25% of incident energy is the diffraction pattern is seen on a screen placed
reflected. The rays AB and A'B' undergo at a distance of 2m. the width of principal maxima
interference. The ratio of Imax and Imin is : is found to be 2.5 mm. The wave length of light is-
B (a) 6250 Å (b) 6200 Å [2014]
B'
(c) 5890 Å (d) 6000 Å
A'
A
8. In Young's double slit experiment 10th order
maximum is obtained at the point of observation
in the interference pattern for l = 7000 Å. If the
[2016] source is replaced by another one of wavelength
C C' 5000 Å then the order of maximum at the same
point will be – [2013, 2007]
(a) 49 : 1 (b) 7 : 1 (c) 4 : 1 (d) 8 : 1 (a) 12 th (b) 14 th (c) 16 th (d) 18 th
5. Calculate the wavelength of light used in an 9. Young’s double slit experiment is carried out by
interference experiment from the following data using green, red and blue light, one color at a
: Fringe width = 0.03 cm. Distance between the time. The fringe widths recorded are bG, bR and
slits and eyepiece through which th e bB, respectively. Then, [2013]
(a) bG > bB > bR (b) bB > bG > bR
EBD_7167
http://t.me/iitjeehelps

P – 72 BITSAT Topicwise Solved Papers


(c) bR > bB > bG (d) bR > bG > bB 15. In Young's double slit experiment, if the slit widths
10. A single slit diffraction pattern is obtained using are in the ratio 1 : 2, the ratio of the intensities at
a beam of red light. If the red light is replaced by minima and maxima will be [2009]
the blue light, then the diffraction pattern (a) 1 : 2 (b) 1 : 3 (c) 1 : 4 (d) 1 : 9
(a) remains unchanged [2012] 16. When the angle of incidence is 60° on the surface
(b) becomes narrower of a glass slab, it is found that the reflected ray
(c) becomes broader is completely polarised. The velocity of light in
(d) will disappear glass is [2008]
11. In a Young’s double slit experiment the angular (a) 8 -1 (b) 3 ´ 10 ms -1
8
width of a fringe formed on a distant screen is 2 ´ 10 ms
1°. The wavelength fo the light used is 6280 Å. (c) 2 ´ 108 ms -1 (d) 3 ´ 108 ms -1
What is the distance between the two coherent 17. In a Young's experiment, the separation between
sources ? [2012] the slits is 0.10mm, the wavelength of light used
(a) 0.036 mm (b) 0.12 mm is 600nm and the interference pattern is observed
(c) 6 mm (d) 4mm on a screen 1.0 m away. Find the separation
12. If the distance between the first maxima and fifth between the successive bright fringes. [2008]
minima of a double slit pattern is 7mm and the (a) 6.6 mm (b) 6.0 mm
slits are separated by 0.15 mm with the screen 50 (c) 6 m (d) 6 cm.
cm. from the slits, then find the wavelength of 18. The two coherent sources of intensity that ratio
the light used. [2011] 2 : 8 produce an interference pattern. The values
(a) 200 nm (b) 100 nm of maximum and minimum intensities will be
(c) 800 nm (d) 600 nm respectively [2007]
13. The diameter of the objective of a telescope is a, (a) I1 and 9 I1 (b) 9I1 and I1
its magnifying power is m and wavelength of (c) 2I1 and 8I1 (d) 8I1 and 2I1
light is l, The resolving power of the telescope Where I1 is the intensity of first source
is: [2010] 19. The first diffraction minima due to a single slit
(a) (1.22 l)/ a (b) (1.22 a)/ l diffraction is at q =30° for a light of wavelength
(c) lm / (1.22 a) (d) a /(1.22 l) 5000 Å. The width of the slit is – [2006]
14. When a mica sheet of thickness 7 microns and m (a) 5 × 10–5 cm. (b) 10 × 10–5 cm.
= 1.6 is placed in the path of one of interfering (c) 2.5 × 10–5 cm. (d) 1.25 × 105 cm.
beams in the biprism experiment then the central 20. The numerical aperture of a microscope is 0.12,
fringe gets at the position of seventh bright and the wavelength of light used is 600 nm. Then
fringe. The wavelength of light used will be its limit of resolution will be nearly – [2005]
(a) 4000 Å (b) 5000 Å [2009] (a) 0.3 µm (b) 1.2 µm (c) 2.3 µm (d) 3.0 µm
(c) 6000 Å (d) 7000 Å

Hints & Solutions


x 1.22l As K = 4I0 so intensity at given point where
1. (d) Here = or
1000 D l
path difference is
1.22 ´ 5 ´10 3 ´ 10 -10 ´ 10 3 4
x=
10 ´ 10 - 2 2 æ pö æ p ö K
K¢ = 4I0 cos ç ÷ ç cos = cos 45º÷ = 2I0 =
or x = 1.22 × 5 × 10–3 m = 6.1 m è 4ø è 4 ø 2
x is of the order of 5 mm. 4. (a) B B¢
2. (b) Dxmax = 0 and Dxmax = 2l I 9
I/4 I
Theortical maximas are = 2n + 1 = 2 × 2 + 1 = 5 64
But on the screen there will be three maximas. A A¢ 3
3. (c) For path difference l, phase difference = 2p I
3 64
rad. I 3
4 I
l p 16
For path difference , phase difference = rad.
4 2
http://t.me/iitjeehelps

Wave Optics P – 73

I 9I Þ I2 = 9 6280 ´ 10 -10
From figure I1 = and I2 = Thus d = ´ 180
4 64 I1 16 3.14
2 = 3.6 × 10–5 m = 0.036 mm
2
æ I2 ö æ 9 ö 12. (d) There are three and a half fringes from first
ç I + 1÷ ç 16 + 1÷ maxima to fifth minima as shown.
Imax 49
By using =ç 1 ÷ =ç ÷ =
Imin ç I2 ÷ ç 9 ÷ 1 5D
ç - 1÷ çè - 1÷ 4D
4B
è I1 ø 16 ø 7 mm 3B
3D
5. (d) 2B
6. (b) The path difference when transparent 2D
1B
sheet is introduced Dx = (m – 1)t 1D
Central bright
If the central maxima occupies position of nth 1D
fringe, then (m – 1)t = n l = d sin q 1B
( m - 1) t 7 mm bD
Þsin q = Þ b= = 2 mm Þ l = = 600 nm
d 3.5 d
(1.17 - 1) ´1.5 ´10-7 13. (d)
= = 0.085 (m - 1)t
3 ´ 10-7 14. (c) l = .....(1)
Therefore, angular position of central maxima n
q = sin–1 (0.085) = 4.88° » 4.9 According to question
For small angles, sin q » q » tan q n = 7 . m = 1.6, t = 7 × 10–6 meter .....(2)
y From eqs. (1) and (2), l = 6 × 10–7 meter
Þ tan q = 15. (d)
D 16. (b) aµg = tan qP where qP = polarising angle.
y (m - 1) t D ( m - 1) t c
\ = Þ y= or, aµg = tan 60° or, = 3
D d d vg
7. (a) Here the width of principal maxima is 2.5
mm, therefore its half width is c 3 ´ 108
or, vg = = = 3 ´ 108 ms -1
b 2.5
= = 1.25 ´ 10-3 m 3 3
2 2 17. (b) The separation between the successive bright
b / 2 1.25 ´ 10 -3 Dl 1 ´ 600 ´ 10 -9
Diffraction angle q = = fringes is b = = Þb = 6.0 mm
D 2 d .1 ´ 10 –3
1.25 ´ 10-3 18. (b) Imax = I1 + I2 + 2 I1 I2 .........(1)
\aq=l \ q = l/a =
2 According to question
1.25 ´ 10 -3 1.25 ´ 10 -3 ´ 10-3 I1 2 1
l= ´a = = =
2 –7 2 I2 8 4 \ I2 = 4I1 .........(2)
l = 6.25 × 10 m = 6250 Å. From eqs. (1) and (2)
8. (b) n1 l1 = n2 l2
10 × 7000 = n2 × 5000 Þ n2 = 14 Imax = I1 + 4I1 + 2 4 I 12 = 5I1 + 4I1
lD Imax = 9I1 .........(3)
9. (d) We know that b =
d Imin = I1 + I2 – 2 4 I1 I2 .........(4)
Now, lR > lG > lB \ bR > bG > bB From eqs. (2) and (4)
10. (b)
l Imin = I1 + 4I1 – 2 4 I 12 Imin = I1
11. (a) The angular fringe width is given by a = 19. (b)
d
where l is wavelength and d is the distance 20. (d) The limit of resolution of a microscope is
l 0.61 l
between two coherent sources. Thus d = given by x =
a m sin q
p
It is given that l = 6 × 10–7 m, and the numerical
Given, l = 6280 Å, a = 1° = radian . aperture m sin q = 0.12. Therefore,
180 0.61 ´ 6 ´ 10-7
x= = 3.05 ´ 10-6 m » 3 µm
0.12
EBD_7167
http://t.me/iitjeehelps

P – 74 BITSAT Topicwise Solved Papers

Chapter
Dual Nature of Radiation and Matter
25
1. When a metal surface is illuminated by light of 6. de-Broglie wavelength of an electron accelerated
wavelengths 400 nm and 250 nm, the maximum by a voltage of 50 V is close to (|e| = 1.6 × 10 –19
velocities of the photoelectrons ejected are v C, me = 9.1 × 10–31 kg, h = 6.6 × 10–34 Js) :[2014]
and 2v respectively. The work function of the (a) 2.4 Å (b) 0.5 Å (c) 1.7 Å (d) 1.2 Å
metal is (h - Planck's constant, c = velocity of
light in air) [2017] 7. The variation of maximum kinetic energy of
(a) 2 hc ×106 J (b) 1.5 hc × 106 J photoelectrons with applied frequency (u) is
(c) hc × 106 J (d) 0.5 hc × 106 J [2013]
2. Find the number of photons emitted per second
by a 25 watt source of monochromatic light of (a) (b)

K.E.max .
wavelength 6600 Å. What is the photoelectric

K.E.max .
current assuming 3% efficiency for photoelectric
effect ? [2016]
25 25
(a) ´ 1019 J, 0.4amp (b) ´ 1019 J, 6.2 amp
3 4
25 (c) (d)
(c) ´ 1019 J, 0.8amp (d) None of these
2

K.E.max .
K .E.max.

3. The beam of light has three wavelengths 4144Å,


4972Å and 6216 Å with a total intensity of 3.6 ×
10–3 Wm2 equally distributed amongst the three
wavelengths. The beam falls normally on the
area 1 cm2 of a clean metallic surface of work 8. A light having wavelength 300 nm fall on a metal
function 2.3 eV. Assume that there is no loss of surface. The work function of metal is 2.54 eV,
light by reflection and that each energetically what is stopping potential ? [2012]
capable photon ejects one electron. Calculate
(a) 2.3 V (b) 2.59 V (c) 1.59 V (d) 1.29 V
the number of photoelectrons liberated in 2s.
[2015] 9. If the energy of a photon is 10 eV, then its
(a) 2 × 109 (b) 1.075 × 1012 momentum is [2011]
–23 –25
(a) 5.33 × 10 kg m/s (b) 5.33 × 10 kg m/s
(c) 9 × 108 (d) 3.75 × 106
4. When 0.50 Å X-rays strike a material, the (c) 5.33 × 10–29 kg m/s (d) 5.33 × 10–27 kg m/s
photoelectrons from the k shell are observed to 10. The photoelectric threshold of Tungsten is 2300Å.
move in a circle of radius 23 mm in a magnetic The energy of the electrons ejected from the
field of 2 × 10–2 tesla acting perpendicularly to
surface by ultraviolet light of wavelength 1800Å
the direction of emission of photoelectrons.
is [2010]
What is the binding energy of k-shell electrons?
(a) 3.5 keV (b) 6.2 keV [2015] (a) 0.15 eV(b) 1.5 eV (c) 15 eV (d) 150 eV
(c) 2.9 keV (d) 5.5 keV 11. A material particle with a rest mass m0 is moving
5. The threshold wavelength of the tungsten is with a velocity of light c. Then the wavelength
2300 Å. If ultraviolet light of wavelength 1800 Å of the de Broglie wave associated with it is :
is incident on it, then the maximum kinetic energy (a) (h/m0c) (b) zero [2010]
of photoelectrons would be about – [2014] (c) ¥ (d) (m0c/h)
(a) 1.49 eV (b) 2.2 eV (c) 3.0 eV (d) 5.0 eV
http://t.me/iitjeehelps

Dual Nature of Radiation and Matter P – 75

12. In a photoelectric experiment, with light of wave- 14. The wavelength of Ka-line characteristic X-rays
length l, the fastest electron has speed v. If the emitted by an element is 0.32 Å. The wavelength
exciting wavelength is changed to 3l/4, the of Kb-line emitted by the same element will be
speed of the fastest emitted electron will become (a) 0.32 Å (b) 0.39 Å [2007]
3 4 (c) 0.49 Å (d) 0.27 Å
(a) v (b) v [2009] 15. An electron of mass ‘m’, when accelerated
4 3
through a potential V has de-Broglie wavelength
4 4 l. The de-Broglie wavelength associated with a
(c) less than v (d) greater than v
3 3 proton of mass M accelerated through the same
13. An electron of mass m and charge e initially at potential difference will be – [2006]
rest gets accelerated by a constant electric field
E. The rate of change of de-Broglie wavelength M m æ Mö æ mö
(a) l (b) l (c) l ç ÷ (d) l ç ÷
of this electron at time t ignoring relativistic effects m M è mø è Mø
is [2008] 16. If the momentum of electron is changed by P,
-h -eht then the de Broglie wavelength associated with
(a) (b) it changes by 0.5%. The initial momentum of
e Et 2 E
electron will be [2005]
- mh -h P
(c) (d) (a) 200 P (b) 400 P (c) (d) 100 P
e Et 2 eE 200

Hints & Solutions


1. (a) We have, E = W0 + K
hc
hc 1 2 3. (b) ( l0 ) = = 5404 Å
or = W0 + mv … (i) f
400 ´ 10-9 2
For each wavelength energy incident on the
hc 1 2
surface per unit time
and = W0 + m(2v ) … (ii)
250 ´ 10-9 2 = intensity of each × area of the surface
On simplifying above equations, we get wavelength = 1.2 × 10–7 joule
E = (1.2 × 10–7) × 2 = 2.4 × 10–7 J
W0 = 2hc × 106 J.
Number of photons n1 due to wavelength 4144 Å

2. (a) n =
P Pl
= =
25 ´ 6600 ´ 10 -10
hc hc 6.64 ´ 10-34 ´ 3 ´ 108 n1 =
( 2.4 ´10 )( 4144 ´10 ) = 0.5 ´ 10
-7 -10
12

l (6.63´ 10 )(3 ´10 )


-34 8

25 Number of photons n 2 due to the wavelength


´ 1019
= 8.28 × 1019 = 4972 Å
3
3% of emitted photons are producing current
n2 =
( 2.4 ´10-7 )( 4972 ´10-10 ) = 0.572 ´1012
\ I=
3
´ ne =
3 25
´ × 1019 × 1.6 × 10–19 = 0.4A ( 6.63 ´10-34 )(3 ´108 )
100 100 3
N = n1 + n2 = 1.075 × 1012
EBD_7167
http://t.me/iitjeehelps

P – 76 BITSAT Topicwise Solved Papers

4. (b) As we know,
1 hc
2 12. (d) mv2 = -f
v q 2 l
F = qvB = m Þ v = BR
R m
1 hc 4hc
m¢ = -f = -f
1 2 1e B R 2 2 2 2 (3l / 4) 3l
K.E. = mv = = 18.36 keV
2 2 m
4
Clearly, v ¢ > v
hc 3
Energy of photon = = 24.8 keV
l
h h h
Binding energy = 24.8 – 18.6 = 6.2 keV 13. (a) l= = =
mv m(eEt / m ) eEt
æ1 1 ö Rate of change of de-Broglie wavelength
5. (a) K max = hn - hn0 = hc ç - ÷
è l l0 ø
dl h æ 1 ö -h
= çç - ÷÷ =
æ 108 108 ö dt eE è t ø e E t 2
2
hc = 1.24 ´ 10-6 ç - ÷ = 1.49 eV
è 18 23 ø
1 é1 1 ù 3R
14. (d) = R ( Z - a) 2 ê - ú= ( Z - a) 2
6. (c) de-Broglie wavelength, la ë12 2 2 û 4

h h h
l= = = = 1.7 Å 1 é1 1ù 8
P mv 2mqV = R ( Z - a ) 2 ê - ú = R ( Z - a) 2
lb ë12
32 û 9
7. (b) As per Einstein’s photoelectric equation :
E = hu = WF + KFmax lb 27
\ = Þ lb = 0.27 Å
i.e. till a certain value of u, KE remains 0, it la 32
only starts increasing once the Work
function (WF) of the metal surface is h l2 M
achieved. 15. (b) l = Þ =
2mK l1 m
8. (c)
K = qV is same for both proton and electron.
E
9. (d) Momentum of a photon µ dl dp
c 16. (a) =-
l P
= 5.33 × 10–27 kg ms–1
0.5 P
hc æ 1 1 ö = Þ P¢ = 200 P
10. (a) Ek = ç - ÷ 100 P '
c è l l0 ø = 0.15 eV
11. (b)
http://t.me/iitjeehelps

Atoms P – 77

Chapter
Atoms
26
1. Hydrogen (H), deuterium (D), singly ionized (a) 910 Å, 1213 Å (b) 5463 Å, 7858 Å
helium (He+) and doubly ionized lithium (Li++) (c) 1315 Å, 1530 Å (d) None of these
all have one electron around the nucleus. 7. If the Ka radiation of Mo (Z = 42) has a
Consider n = 2 to n = 1 transition. The wavelength of 0.71 Å, calculate wavelength of
wavelengths of emitted radiations are l1, l2, l3 the corresponding radiation of Cu, i.e., Ka for
and l4 respectively. Then approximately: [2017] Cu (Z = 29) assuming s = 1. [2013]
(a) l1 = l2 = 4 l3 = 9 l4 (a) 1.52 Å (b) 5.14 Å (c) 3.02 Å (d) 0.52 Å
8. The energies of energy levels A, B and C for a
(b) 4 l1 = 2 l2 = 2 l3 = l4 given atom are in the sequence E A < EB < EC. If
(c) l1 = 2 l2 = 2Ö 2 l3 = 3Ö 2 l4 the radiations of wavelengths l1, l2 and l3 are
(d) l1 = l2 = 2 l3 = 3Ö 2 l4 emitted due to the atomic transitions C to B, B to
A and C to A respectively then which of the
2. If the series limit wavelength of Lyman series for
following relations is correct ? [2011, 2007]
the hydrogen atom is 912 Å, then the series limit
(a) l1 + l2 + l3 = 0 (b) l3 = l12 + l2
wavelength for Balmer series of hydrogen atoms
is [2016] l 1l 2
(c) l3 = l1 + l2 (d) l3 = l + l .
(a) 912 Å (b) 912 × 2 Å 1 2

912 9. The third line of Balmer series of an ion equivalnet


(c) 912 × 4 Å (d) Å to hydrogen atom has wavelength of 108.5 nm.
2
The ground state energy of an electron of this
3. One of the lines in the emission spectrum of Li2+
ion will be [2010]
has the same wavelength as that of the 2nd line
(a) 3.4 eV (b) 13.6 eV
of Balmer series in hydrogen spectrum. The
(c) 54.4 eV (d) 122.4 eV
electronic transition corresponding to this line
10. Hydrogen atom in ground state is excited by a
is n = 12 ® n = x. Find the value of x.
monochromatic radiation of l = 975 Å. Number
[2015, 2008]
of spectral lines in the resulting spectrum emitted
(a) 8 (b) 6 (c) 7 (d) 5 will be [2010]
4. Energy required for the electron excitation in Li++ (a) 3 (b) 2 (c) 6 (d) 10
from the first to the third Bohr orbit is [2014] 11. Taking Rydberg’s constant RH = 1.097 × 107m,
(a) 36.3 eV (b) 108.8 eV first and second wavelength of Balmer series in
(c) 122.4 eV (d) 12.1 eV hydrogen spectrum is [2009]
5. The angular momentum of electron in nth orbit is (a) 2000 Å, 3000 Å (b) 1575 Å, 2960 Å
given by [2014] (c) 6529 Å, 4280 Å (d) 6552 Å, 4863 Å
h 12. In which of the following series, does the 121.5
(a) nh (b) nm line of the spectrum of the hydrogen atom
2pn
lie? [2008]
h 2 h (a) Lyman series (b) Balmer series
(c) n (d) n (c) Paschen series (d) Brackett series.
2p 2p
6. The energy of electron in the nth orbit of 13. The ionisation potential of H-atom is 13.6 V.
When it is excited from ground state by
-13.6 monochromatic radiations of 970.6 Å, the number
hydrogen atom is expressed as E n = 2 eV.
n of emission lines will be (according to Bohr’s
The shortest and longest wavelength of Lyman theory) [2006]
series will be [2013] (a) 10 (b) 8 (c) 6 (d) 4
EBD_7167
http://t.me/iitjeehelps

P – 78 BITSAT Topicwise Solved Papers


14. The angular momentum of an electron in first (a) equal to the diameter of the first orbit
orbit of Li++ ion is – [2007] (b) equal to the circumference of the first orbit
3h 9h h h (c) equal to half the circumference of the first
(a) (b) (c) (d) orbit
2p 2p 2p 6p
15. The de Broglie wavelength of the electron in the (d) independent of the size of the first orbit
first Bohr orbit of the hydrogen atom is [2005]

Hints & Solutions


1 2æ 1 1 ö 4 (ZM0 - 1) 2 l Cu
1. (a) = RZ ç 2 - 2 ÷ Þ l = Þ =
l è1 2 ø 3RZ 2 (ZCu - 1) 2 l Mo
lZ2 = constant
(Z M0 - 1)2
So l1(1)2 = l2(1)2 = l3(2)2 = la(32) Þ l Cu = l Mo = 1.52Å
or l1 = l2 = 4l3 = 9l4. (ZCu - 1)2
8. (d)
1 é 1 1 ù
2. (c) = Rê 2 - 2 ú 9. (c) For third line of Balmer series n1 = 2, n2 = 5
l ëê n1 n2 ûú 1 é1 1 ù n 2n 2
\ = RZ2 ê 2 - 2 ú gives Z2 = 2 1 22
For limiting wavelength of Lyman series l ë n1 n 2 û (n 2 - n1 )lR
1
n1 = 1, n2 = ¥ =R 13.6Z2 -13.6(2)2
lL E=- = = -54.4 eV
For limiting wavelength of Balmer series n2 (1)2
n1 = 2, n2 = ¥ n(n - 1)
10. (c) Number of spectral lines = =6
1 æ 1ö 4 2
= Rç ÷ Þ lB =
lB è 4ø R 1 é 1 1 ù
11. (d) = R ê 2 - 2 ú . For first wavelength, n1
\ lB = 4lL = 4 × 912 Å. l ë n1 n 2 û
3. (b) For 2nd line of Balmer series in hydrogen = 2, n2 = 3
spectrum
Þ l1 = 6563 Å. For second wavelength, n1
1 æ 1 1ö 3 = 2, n2 = 4 Þ l2 = 4861 Å
= R (1) ç 2 - 2 ÷ = R
l è2 4 ø 16 12. (a) Since 121.5 nm line of spectrum of hydrogen
é1 æ 1 1 ö 3R ù atom lies in ultraviolet region, therefore it is
For Li2+ ê l = R ´ 9 çè 2 - 2 ÷ø = 16 ú Lyman series.
ë x 12 û
which is satisfied by n = 12 ® n = 6. 1 é1 1 ù
13. (c) =Rê 2 - 2ú
4. (b) Energy of excitation, l ë n1 n 2 û
æ1 1 ö
D E = 13.6 p2 ç n - n ÷ eV = 108.8 eV Þ n2 = 4
è 1 2ø
5. (c) According to Bohr's second postulate. \ Number of emission line
1 é 1 1 ù n(n - 1) 4 ´ 3
= Rê 2 - 2 ú Þ l N= = =6
6. (a)
l max max = 1213Å 2 2
ë (1) (2) û
14. (c) Angular momentum (mvr)
1 é 1 1ù 1
= R ê 2 - ú Þ l min = » 910Å. h h
l min ¥û R = n. = (n = 1)
ë (1) 2p 2p
7. (a) v = a(Z - 1) 15. (b)
http://t.me/iitjeehelps

Nuclei P – 79

Chapter
Nuclei
27
1. After two hours, one-sixteenth of the starting 8. A mixture consists of two radioactive materials
amount of a certain radioactive isotope remained A1 and A2 with half lives of 20 s and 10 s
undecayed. The half life of the isotope is [2017] respectively. Initially the mixture has 40 g of A1
(a) 15 minutes (b) 30 minutes and 160 g of A2. The amount of the two in the
(c) 45 minutes (d) 4 hours mixture will become equal after [2013]
2. When 92U235 undergoes fission, 0.1% of its (a) 60 s (b) 80 s
original mass is changed into energy. How much (c) 20 s (d) 40 s
energy is released if 1 kg of 92U235 undergoes 9. If the total binding energies of
fission? [2017] 2 4 56 235
(a) 9 × 1010 J (b) 9 × 1011 J 1 H, 2 He, 26 Fe & 92 U nuclei are 2.22, 28.3, 492
(c) 9 × 1012 J (d) 9 × 1013 J and 1786 MeV respectively, identify the most
3. A radioactive element X converts into another stable nucleus of the following. [2012]
stable element Y. Half life of X is 2 hrs. Initially
(a) 56
26 Fe (b) 12 H
only X is present. After time t, the ratio of atoms
of X and Y is found to be 1 : 4, then t in hours is (c) 92235 (d) 42 He
U
(a) 2 (b) 4 [2016]
(c) between 4 and 6 (d) 6 10. Which one is correct about fission? [2011]
4. The mass of 7N15 is 15.00011 amu, mass of 8O16 (a) Approx. 0.1% mass converts into energy
is 15.99492 amu and mP = 1.00783 amu. Determine (b) Most of energy of fission is in the form of
binding energy of last proton of 8O16. [2016] heat
(a) 2.13 MeV (b) 0.13 MeV (c) In a fission of U235 about 200 eV energy is
(c) 10 MeV (d) 12.13 MeV released
5. In an ore containing uranium, the ratio of U238 (d) On an average, one neutron is released per
to Pb206 is 3. Calculate the age of the ore, fission of U235
assuming that all the lead present in the ore is 11. Which of the following is best nuclear fuel
the final stable product of U238. Take the half- [2010]
life of U238 to be 4.5 × 109 yr. [2015] (a) thorium 236 (b) plutonium 239
(a) 1.6 × 193 yr (b) 1.5 × 104 yr (c) uranium 236 (d) neptunium 239
(c) 1.867 × 109 yr (d) 2 × 105 yr 12. The activity of a radioactive sample is measured
6. A nucleus of uranium decays at rest into nuclei as N0 counts per minute at t = 0 and N0/e counts
of thorium and helium. Then : [2014] per minute at t = 5 minutes. The time (in minutes)
(a) the helium nucleus has less momentum than at which the activity reduces to half its value is
the thorium nucleus
5
(b) the helium nucleus has more momentum (a) log e 2 / 5 (b) [2010]
than the thorium nucleus log e 2
(c) the helium nucleus has less kinetic energy (c) 5 log102 (d) 5 loge 2
than the thorium nucleus 13. Nuclear energy is released in fission since
(d) the helium nucleus has more kinetic energy binding energy per nucleon is [2009]
than the thorium nucleus (a) sometimes larger and sometimes smaller
7. Boron rods in a nuclear reactor are used to (b) larger for fission fragments than for parent
(a) absorb excess neutrons [2013] nucleus
(b) absorb alpha particle (c) same for fission fragments and nucleus
(c) slow down the reaction
(d) smaller for fission fragments than for parent
(d) speed up the reaction
nucleus
EBD_7167
http://t.me/iitjeehelps

P – 80 BITSAT Topicwise Solved Papers


14. The binding energy of the innermost electron 16. Half life of a radioactive substance is 20 minute.
tungsten is 40 keV. To produce characteristic X- Difference between points of time when it is 33%
rays using a tungsten target in an X-ray tube disintegrated and 67% disintegrated is
the potential difference V between the cathode approximately [2006]
and the anticathode should be- [2008] (a) 40 minute (b) 10 minute
(a) V < 40 kV (b) V £ 4 kV (c) 15 minute (d) 20 minute
(c) V > 40 kV (d) V > / < 40 kV 17. The Binding energy per nucleon of 73 Li and
15. A radioactive nucleus undergoes a series of 4
decay according to the scheme 2 He nuclei are 5.60 MeV and 7.06 MeV,,
respectively.
a b a g
A ¾¾
® A1 ¾¾
® A 2 ¾¾
® A3 ¾¾
® A4
If the mass number and atomic number of ‘A’ In the nuclear reaction 73 Li + 11H ® 42 He + Q ,
are 180 and 72 respectively, then what are these the value of energy Q released is : [2005]
numbers for A4 [2007] (a) 19.6 MeV (b) – 2.4 MeV
(a) 172 and 69 (b) 174 and 70 (c) 8.4 MeV (d) 17.3 MeV
(c) 176 and 69 (d) 176 and 70

Hints & Solutions


n
æ1ö 1 1 1
1. (b) N = N0 ç ÷ The given ratio lies between and .
è2ø 4 3 7
n Therefore, t lies between 4 hrs and 6 hrs.
N0 æ1ö
or = N0 ç ÷ 4. (d) M(8O16) = M (7N15) + 1mP
16 è2ø binding energy of last proton
or n =4 = M (N15) + mP – M (1O16)
t 2 1 = 15.00011 + 1.00783 – 15.99492
Half life t 1/2 = = = h = 0.01302 amu = 12.13 MeV
n 4 2
2. (d) Mass of uranium changed into energy 5. (c) Let the initial mass of uranium be M0
0.1 Final mass of uranium after time t,
= ´1 3
100 M = M0
= 10–3 kg. 4
The energy released = mC2 According to the law of radioactive
= 10–3 × (3 × 108)2 disintegration.
= 9 × 1013 J. M æ1ö
t /T
M t /T
3. (c) Let N0 be the number of atoms of X at time =ç ÷ Þ 0 = ( 2)
M0 è 2 ø M
t = 0.
Then at t = 4 hrs (two half lives) æ M ö t
\ log10 ç 0 ÷ = log10 ( 2 )
è M ø T
N0 3N0
Nx = and N y = æM ö
4 4 æ4ö
log10 ç 0 ÷ T log10 ç ÷
\ Nx/Ny = 1/3 t=T è M ø= è 3ø
and at t = 6 hrs (three half lives) log10 ( 2 ) log10 ( 2 )

Nx =
N0 7N 0 T log10 (1.333) æ 0.1249 ö
and N y = = = 4.5 ´109 ç ÷
8 8 log10 ( 2 ) è 0.3010 ø
Nx 1
or = Þ t = 1.867 × 109 yr.
Ny 7
http://t.me/iitjeehelps

Nuclei P – 81
6. (d) In an explosion a body breaks up into two 28.3
pieces of unequal masses both part will B.E He = = 7.08
4
have numerically equal momentum and 492
lighter part will have more velocity. B.E Fe = = 8.78 = maximum
56
U ® Th + He
1786
B.E U = = 7.6
P2 P2 235
KETh = , KEHe =
2mTh 2m He 56 is most stable as it has maximum
26 Fe
sinc mHe is less so KEHe will be more. binding energy per nucleon.
7. (a) Boron rods absorb excess neutrons. 10. (a)
8. (d) Let, the amount of the two in the mixture 11. (b) Plutonium 239 is processed by breeder
will become equal after t years. mechanism to be used as nuclear feul.
The amount of A1, which remains after t
12. (d) N = N0e–lt
years
Here, t = 5 minutes
N 01
N1 = N0
(2)t/20 = N 0 × e -5l
e
The amount of A2, which remains, after t
years 1
Þ 5l = 1 , l = ,
5
N 02
N2 = l n2
(2)t/10 Now, T1/2 = = 5 l n2
l
According to the problem
13. (b) Nuclear energy is relased in fission because
N1 = N2
BE/nucleon is larger for fission fragments
40 160 than for parent nucleus.
t/20
=
(2) (2)t/10 14. (c)
æ t ö 180 a b
çè - 2÷ø 15. (a) 72 A ® 70 A1176 ¾¾
¾¾ ® 71 A 2176
t/20 10
2 =2
a g
t t
= -2 ®69 A3172 ¾¾®69 A 4172
¾¾
20 10 16. (d)
t t 17. (d) BE of 2He4 = 4 × 7.06 = 28.24 MeV
- =2
20 10
t BE of 73 Li = 7 × 5.60 = 39.20 MeV
=2 7 1
20 3 Li +1 H ® 2 He 4 + 2He 4 + Q
t = 40 s 39.20 28.24 ´ 2(= 56.48 MeV)
2.22 Therefore, Q = 56.48 – 39.20 = 17.28 MeV.
9. (a) B.E H = = 1.11
2
EBD_7167
http://t.me/iitjeehelps

P – 82 BITSAT Topicwise Solved Papers

Chapter
Semiconductor Electronics: Materials,
Devices and Simple Circuits 28
1. In the circuit given below, V(t) is the sinusoidal base current has to be 10 times the signal
voltage source, voltage drop VAB(t) across the current? [2014]
resistance R is [2017] (a) 14 kW (b) 18 kW (c) 10 kW (d) 5 kW
6. The combination of gates shown below yields

X
[2014]
(a) is half wave rectified B
(b) is full wave rectified
(c) has the same peak value in the positive and (a) OR gate (b) NOT gate
negative half cycles (c) XOR gate (d) NAND gate
(d) has different peak values during positive 7. Transfer characteristics [output voltage (V0) vs
and negative half cycle input voltage (V1)] for a base biased transistor
2. In CE transistor amplifier, the audio signal in CE configuration is as shown in the figure.
voltage across the collector resistance of 2 kW For using transistor as a switch, it is used [2013]
is 2 V. If the base resistance is 1kW and the current (a) in region (III) V0 I II
amplification of the transistor is 100, the input III
(b) both in region
signal voltage is [2015]
(a) 2mV (b) 3mV (c) 10mV (d) 0.1mV (I) and (III)
3. An oscillator is nothing but an amplifer with (c) in region (II)
(a) positive feedback [2012] (d) in region (I) Vi
(b) negative feedback 8. If in a p-n junction diode, a square input signal
(c) large gain of 10 V is applied as shown [2013]
(d) no feedback
4. Assuming the diodes to be of silicon with 5V
forward resistance zero, the current I in the RL
following circuit is [2009]
-5V
2kW
Then the output signal across RL will be
10 V
+5V
E = 20 V (a) (b)

(a) 0 (b) 9.65 mA (c) 10 mA (d) 10.35 mA (c) (d)


-5V
5. For a CE transistor amplifier, the audio signal -10 V
voltage across the collector resistance of 2.0 kW 9. The output of an OR gate is connected to both
is 2.0 V. Suppose the current amplification factor the inputs of a NAND gate. The combination
will serve as a: [2011]
of the transistor is 100, What should be the value
(a) NOT gate (b) NOR gate
of RB in series with VBB supply of 2.0V if the dc
(c) AND gate (d) OR gate
http://t.me/iitjeehelps

Semiconductor Electronics: Materials, Devices and Simple Circuits P – 83


10. In a semiconductor diode, the barrier potential A B Y A B Y
offers opposition to [2011] 0 0 1 0 0 1
(a) holes in P-region only
(c) 0 1 1 (d) 0 1 0
(b) free electrons in N-region only
1 0 0 1 0 1
(c) majority carriers in both regions
(d) majority as well as minority carriers in both 1 1 0 1 1 1
regions 14. In a common base amplifier the phase difference
11. A transistor has a base current of 1 mA and between the input signal voltage and the output
emitter current 90 mA. The collector current will voltage is [2007]
be [2010] (a) 0 (b) p/4 (c) p/2 (d) p
(a) 90 mA (b) 1 mA (c) 89 mA (d) 91 mA 15. A zener diode, having breakdown voltage equal
12. A d.c. battery of V volt is connected to a series to 15V, is used in a voltage regulator circuit shown
combination of a resistor R and an ideal diode D
in figure. The current through the diode is
as shown in the figure below. The potential
difference across R will be [2010] [2006]
R D
250W

V 20V 15V 1kW


(a) 2V when diode is forward biased
(b) Zero when diode is forward biased
(c) V when diode is reverse biased (a) 10 mA (b) 15 mA (c) 20 mA (d) 5 mA
(d) V when diode is forward biased 16. In an unbiased n-p junction electrons diffuse
13. Truth table for system of four NAND gates as from n-region to p-region because : [2006]
shown in figure is : [2008] (a) holes in p-region attract them
A
(b) electrons travel across the junction due to
potential difference
Y (c) only electrons move from n to p region and
not the vice-versa
(d) electron concentration in n-region is more
B
compared to that in p-region
A B Y A B Y 17. In common emitter amplifier, the current gain is
0 0 0 0 0 0 62. The collector resistance and input resistance
(a) 0 1 1 (b) 0 1 0 are 5 kW an 500W respectively. If the input
1 0 1 1 0 1 voltage is 0.01 V, the output voltage is [2005]
1 1 0 1 1 1 (a) 0.62 V (b) 6.2 V (c) 62 V (d) 620 V

Hints & Solutions


1. (d) During the operation, either of D1 and D2 V 20
be in forward bias. Also R1 and R2 are 4. (c) I= = = 10 ´ 10 -3 A = 10 mA
R 2 ´ 103
different, so output across R will have 5. (a) The output ac voltage is 2.0 V. So, the ac
different peaks. collector current iC = 2.0/2000 = 1.0 mA.
2. (c) The signal current through the base is, therefore
3. (a) A positive feedback from output to input in given by
an amplifier provides oscillations of constant iB = iC /b = 1.0 mA/100 = 0.010 mA.
amplitude.
EBD_7167
http://t.me/iitjeehelps

P – 84 BITSAT Topicwise Solved Papers


The dc base current has to be 10 × 0.010 = 0.10 By expanding this Boolean expression
mA. Y = A.B + B.A
RB = (VBB – VBE ) /IB. Thus the truth table for this expression should
Assuming VBE = 0.6 V, RB = (2.0 . 0.6 )/0.10 be (1).
= 14 kW. 14. (a) The phase difference between output
6. (a) The final boolean expression is, voltage and input signal voltage in common
base transistor circuit is zero.
( )
X = A . B = A + B = A + B Þ OR gate
15. (d) Voltage across zener diode is constant.
7. (b) I ® ON
250W i i1kW
II ® OFF
In IInd state it is used as a amplifier it is 5V i–i1kW
active region. 20v 1kW
8. (a) The current will flow through RL when the 15V
diode is forward biased. 15V

9. ( )
(b) A + B = NOR gate Current in 1kW resistor,
When both inputs of NAND gate are connected, 15volt
it behaves as NOT gate (i)1kW = = 15 mA
1kW
OR + NOT = NOR.
Current in 250W resistor,
10. (c)
(20 - 15)V 5V
11. (c) IC = IE –IB = 90 – 1 = 89 m A (i)250W = =
12. (b) In forward biasing, the diode conducts. For 250W 250W
ideal junction diode, the forward resistance is 20
= A = 20 mA
zero; therefore, entire applied voltage occurs 1000
across external resistance R i.e., there occurs no
\ (i) zener diode = (20 - 15) = 5mA.
potential drop, so potential across R is V in
forward biased. 16. (d) Electrons in an unbiased p-n junction,
13. (a) diffuse from n-region i.e., higher electron
A concentration to p-region i.e., low electron
Y = A.AB B.AB

concentration region.
Y2 = A.AB
Vo R 5 ´ 103 ´ 62
17. (b) = o ´b = = 10 ´ 62 = 620
Y1 = AB Vin R in 500

B Vo = 620 × Vin= 620 × 0.01 = 6.2 V


Y3 = B.AB \ Vo = 6.2 volt.
http://t.me/iitjeehelps

SECTION II - CHEMISTRY

Chapter
Some Basic Concepts of
Chemistry 1
1. If a 25.0 mL sample of sulfuric acid is titrated 9. How many grams of concentrated nitric acid
with 50.0 mL of 0.025 M sodium hydroxide to a solution should be used to prepare 250 mL of
phenolphthalein endpoint, what is the molarity 2.0M HNO3? The concentrated acid is 70%
of the acid? [2017] HNO3 [2013]
(a) 0.020 M (b) 0.100 M (a) 90.0 g conc. HNO3
(c) 0.025 M (d) 0.050 M (b) 70.0 g conc. HNO3
2. Find which of the following compound can have (c) 54.0 g conc. HNO3
mass ratios of C:H:O as 6:1:24 [2017] (d) 45.0 g conc. HNO3
(a) HO-(C=O)-OH (b) HO-(C=O)-H 10. Number of atoms of He in 100 amu of He (atomic
(c) H-(C=O)-H (d) H3CO-(C=O)-H wt. of He is 4) are : [2012]
3. The normality of 26% (wt/vol) solution of (a) 25 (b) 100
ammonia (density = 0.855 ) is approximately : (c) 50 (d) 100 × 6 × 10–23
[2016] 11. The product of atomic weight and specific heat
(a) 1.5 (b) 0.4 of any element is a constant, approximately 6.4.
(c) 15.3 (d) 4 This is known as [2011]
4. 1.25 g of a sample of Na2CO3 and Na2SO4 is
(a) Dalton’s law (b) Avogadro’s law
dissolved in 250 ml solution. 25 ml of this
(c) Newton’s law (d) Dulong Pettit law
solution neutralises 20 ml of 0.1N H2SO4.The %
12. 1.520 g of hydroxide of a metal on ignition gave
of Na2CO3 in this sample is [2016]
0.995g of oxide. The equivalent weight of metal
(a) 84.8% (b) 8.48%
is : [2011]
(c) 15.2% (d) 42.4%
(a) 1.52 (b) 0.995
5. A compound of Xe and F is found to have 53.5% (c) 190 (d) 9
of Xe. What is oxidation number of Xe in this
13. The vapour density of ozone is [2010]
compound ? [2016]
(a) 16 (b) 32
(a) – 4 (b) 0
(c) 24 (d) 48
(c) + 4 (d) + 6
14. In redox reaction 1 gm-eq of reducing agent
6. The prefix 1018 is [2015, 2006]
requires P gm-eq. of oxidising agent. The value
(a) giga (b) kilo
of P is [2010]
(c) exa (d) nano
7. The formation of CO and CO2 illustrates the law (a) 1 (b) 2
of [2014] (c) 3
(a) reciprocal proportion (d) Depends on reaction
(b) conservation of mass 15. Given the numbers : 161 cm, 0.161 cm, 0.0161 cm.
(c) multiple proportion The number of significant figures for the three
(d) constant composition numbers are [2009]
8. The number of atoms in 0.004 gm of magnesium (a) 3, 4 and 5 respectively
is close to [2013] (b) 3, 3 and 4 respectively
(a) 24 (b) 2 × 1020 (c) 3, 3 and 3 respectively
(c) 1020 (d) 6.02 × 1023 (d) 3, 4 and 4 respectively
EBD_7167
http://t.me/iitjeehelps

C–2 BITSAT Topicwise Solved Papers


16. In which of the following number all zeros are 19. The equivalent weight of H2O2 in
significant? [2008] 2MnO -4 + 5H 2 O 2 + 6H+ ® 2Mn2+ + 5O2 + 8H2O
(a) 0.0005 (b) 0.0500 is [2007]
(c) 50.000 (d) 0.0050 (a) 17 (b) 34
17. The number of electrons in a mole of hydrogen (c) 68 (d) 85
molecule is [2008] 20. A gas is found to have a formula [CO]x. Its vapour
density is 70, the x is : [2006]
(a) 6.023 × 1023 (b) 12.046 × 1023
(a) 3.0 (b) 3.5
(c) 3.0115 × 1023 (d) indefinite (c) 5.0 (d) 6.5
18. When 30 liters of H2 and 30 liters of N2 are reacted 21. The volume of Cl2 at STP obtained on reacting
NH3 is formed and the yield is only 50%.The 4.35 g MnO2 with conc. HCl (At. wt. of Mn = 55)
composition of the gaseous mixture will be [2006]
[2007] (a) 4.48 litre (b) 2.24 litres
(a) 5L of N2, 5L of H2 and 5 L of NH3. (c) 1.12 litre (d) 0.56 litre
22. The number of sodium atoms in 2 moles of
(b) 5L of N2, 10L of H2 and 10 L of NH3.
sodium ferrocyanide is : [2005]
(c) 10L of N2, 15L of H2 and 5 L of NH3. (a) 12 × 1023 (b) 26 × 1023
(d) 5L of N2, 15L of H2 and 10 L of NH3. (c) 34 × 1023 (d) 48 × 1023

Hints & Solutions


1. (c) M1V1 = M2V2 \ Percentage of Na2CO3
(0.025 M) (0.050 L) = (M2) (0.025 L) 1.06 ´ 100
= = 84.8%
M2 = 0.05 M 1.25
but, there are 2 H’s per H2SO4 so [H2SO4] 5. (d) Given Xe = 53.5 % \ F = 46.5%
= 0.025 M Relative number of atoms Xe
2. (a) Given, mass ratio is C:H:O (6:1:24) so, molar
53.5 46.5
ratio will be 6/12:1/1:24/16 = 1:2:3 = = 0.4 and F = = 2.4
131.2 19
therefore, HO-(C=O)-OH has molar ratio
1:2:3 Simple ratio Xe = 1 and F = 6 ; Molecular
formula is XeF6
26
3. (c) Wt. of NH3 = 26 g = g eq = 1.53 g eq O.N.of Xe is + 6
17
6. (c) Exa = 1018
Vol. of soln. = 100 mc = 0.1 L 7. (c) Formation of CO and CO2 illustrates the
1.53 law of multiple proportion that is constant
\ Normality = = 15.3 N
0.1 mass of C reacts with different masses of
4. (a) Let the amount of Na2CO3 present in the oxygen. These masses here bears simple
mixture be x g . Na2SO4 will not react with ratio of 1 : 2.
H2SO4. Then 8. (c) No. of Mg atoms = g atom × 6.02 ´ 1023
x 20 ´ 0.1 ´ 10 0.004
= \ x = 1.06 g ´ 6.02 ´ 10 23 » 1.0 ´10 20 .
53 1000 =
24
http://t.me/iitjeehelps

Some Basic Concepts of Chemistry C–3


wt ´ 1000 50.000 all zero are significant.
9. (d) Molarity (M) =
mol. wt. ´ vol (ml) 17. (b) No. of electron
wt. 1000 = No. of electrons per molecule × No. of
2= × molecules
63 250
= 2 × 6.023 × 1023 = 12.046 × 1023.
63
wt. = gm 18. (d) N 2(g) + 3H 2(g) ¾¾ ® 2NH3(g) ;
2
N 2 is the limiting reagent in this
100
wt. of 70% acid = ´ 31.5 = 45 gm reaction.10L N2 will react with 30L H2 to
70
produce 20L of NH3.The composition of
100
10. (a) 100 amu of He = atoms of He resultant mixture will be 5L of N2, 15L of H2
4
= 25 atoms. and 10 L of NH3.
[1 a.m.u. = mass of one proton (approx.)] 19. (a) 2MnO-4 1 + 5H2O2 + 6H+ ¾¾ ®
11. (d) According to Dulong and Pettit’s law 2Mn2+ + 5O2 + 8H2O
Atomic weight × Specific heat = 6.4 (approx) H2O2–1 ¾ ¾® 2H2O–2 + 2H+
This law is applicable only to solid elements Total change in oxidation number
but it fails to explain very high specific heat
= – 4 – (– 2) = – 2
of diamond.
The equivalent of H2O2
12. (d) Let E be the equivalent weight of the metal.
M.wt of H 2 O 2
E + 17 1.52 =
So, = Change in oxidation number
E +8 0.995 34
= = 17
[17 is equivalent weight of OH and 8 is 2
equivalent weight of oxygen] 20. (c) (12 + 16) × x = 70 × 2
Þ 0.995 E + 17 × 0.995 = E × 1.52 + 8 × 1.52
[Q 2 × V.D. = M. weight]
Þ 0.525 E = 16. 915 – 12.16 = 4.755
70 ´ 2
4.755 x= = 5 Þ x = 5.
\ E= =9 28
0.525
13. (c) We know that, 21. (c) MnO2 + 4HCl ® MnCl2 + 2H 2O + Cl2
87gm 22.4litres
Molecular weight of compound or at STP
molecules = 2 × V. D. 22.4
1gm MnO 2 gives = litres of Cl 2
Vapour density (V. D.) of ozone molecules 87
M. wt. of O 3 48 22.4 ´ 4.35
= = = 24. 4.35 gm MnO2 gives =
2 2 87
Hence, V. D. of O 3 is 24. = 1.12 lit. of Cl2.
14. (a) In redox reaction, 22. (d) Sodium ferrocyanide
gm equivalent of reducing agent = gm. ˆˆ† 4Na + + Fe(CN) -6 4
Na 4 Fe(CN) 6 ‡ˆˆ
equivalent of oxidising agent 1 mol. of potassium ferrocyanide gives 4
Hence 1gm equ. of reducing agent = P gm ions of Na.
equ. of oxidising agent. 2 × 6.02 × 1023 molecules of potassium
15. (c) Each has three significant figures. When ferrocyanide gives 4 × 2 × 6.02 × 1023 ions
zero is used to locate the decimal point, it of Na+
is not considered as significant figure. i.e. 2 moles of potassium ferrocyanide ions
16. (c) If zero is used to locate the decimal point it of Na+
is considered as a significant figure. In = 48 × 1023
EBD_7167
http://t.me/iitjeehelps

C–4 BITSAT Topicwise Solved Papers

Chapter
Atomic Structure
2
1. The energy of an electron in second Bohr orbit (a) 2 (b) 6 (c) 0 (d) 14.
of hydrogen atom is : [2017] 6. The Bohr orbit radius for the hydrogen atom
(a) –5.44 × 10–19 eV (b) –5.44 × 10–19 cal (n = 1) is approximately 0.530 Å. The radius for
(c) –5.44 × 10–19 kJ (d) –5.44 × 10–19 J the first excited state (n = 2) orbit is (in Å) [2013]
2. The number of radial nodes of 3s and 2p orbitals (a) 0.13 (b) 1.06 (c) 4.77 (d) 2.12
are respectively [2017]
7. Which of the following is not permissible
(a) 2, 0 (b) 0, 2 (c) 1, 2 (d) 2, 2
3. The first emission line in the atomic spectrum of arrangement of electrons in an atom? [2012]
hydrogen in the Balmer series appears at [2016] (a) n = 5, l = 3, m = 0, s = + 1/2
(b) n = 3, l = 2, m = – 3, s = – 1/2
9R 7R
(a) cm -1 (b) cm -1 (c) n = 3, l = 2, m = – 2, s = – 1/2
400 144 (d) n = 4, l = 0, m = 0, s = – 1/2
3R 5R 8. The measurement of the electron position if
(c) cm -1 (d) cm -1 associated with an uncertainty in momentum,
4 36
4. An e– has magnetic quantum number as –3, what which is equal to 1 × 10–18 g cm s– 1 . The
is its principal quantum number? [2016] uncertainty in electron velocity is, [2007]
(a) 1 (b) 2 (c) 3 (d) 4 (mass of an electron is 9 × 10– 28 g)
5. For azimuthal quantum number l = 3, the (a) 1 × 109 cm s–1 (b) 1 × 106 cm s–1
maximum number of electrons will be [2014] 5
(c) 1 × 10 cm s –1 (d) 1 × 1011 cm s–1

Hints & Solutions


4. (d) When m = – 3, l = 3, \ n = 4.
13.6Z 2
1. (d) For H atom, En = - eV 5. (d) l = 3 means f-subshell. Maximum no. of
n2 electrons = 4l + 2 = 4 × 3 + 2 = 14
For 2nd orbit, n = 2, Z = 1 (For H) 6. (d) Radius of hydrogen atom = 0.530 Å,
13.6 ´ (1) 2 -13.6 Number of excited state (n) = 2 and atomic
\ E2 = - = eV number of hydrogen atom (Z) = 1. We know
(2) 2 4
that the Bohr radius.
-13.6 ´1.6 ´ 10-19 -19
= J = -5.44 ´ 10 J n2 (2) 2
4 (r ) = ´ Radius of atom = ´ 0.530
2. (a) For a given orbital with principal quantum Z 1
number (n) and azimuthal quantum number = 4 × 0.530 = 2.12 Å
(l), number of radial nodes = (n - l - 1) 7. (b) m = 2, l +1, thus for l = 2, m = 5, hence
for 3s orbital: n = 3 and l = 0 values of m will be – 2, –1, 0, + 1, + 2.
\ number of radial nodes = 3 - 0 - 1 = 2 \ for l = 2, m cannot have the value – 3.
for 2p orbital: n = 2 and l = 1 8. (a) Dp = mDv
\ number of radial nodes = 2 - 1 - 1 = 0 Substituting the given values of Dx and m,
3. (d) For Balmer n 1 = 2 and n2 = 3; 1 × 10–18 g cm s–1 = 9 × 10–28 g × Dv

æ 1 1 ö 5R 1 ´ 10-18
n = R çç - ÷÷ = cm -1 Dv =
è2 2
32 ø 36 9 ´ 10-28
= 1.1 × 109 cm s–1 ; 1 × 109 cm s–1
http://t.me/iitjeehelps

Classification of Elements and Periodic Properties C–5

Chapter
Classification of Elements and
Periodic Properties 3
1. Which of the following order is wrong? 8. The ions O2– , F– , Na +, Mg2+ and Al 3+ are
[2017, 2005] isoelectronic. Their ionic radii show [2013]
2– –
(a) NH 3 < PH 3 < AsH 3 — Acidic (a) A decrease from O to F and then increase
from Na+ to Al3+
(b) Li < Be < B < C — IE1
(b) A significant increase from O2– to Al3+
(c) Al 2 O3 < MgO < Na 2 O < K 2 O — Basic (c) A significant decrease from O2– to Al3+
(d) An increase from O2– to F– and then
(d) Li + < Na + < K + < Cs + — Ionic radius
decrease from Na+ to Al3+
2. The decreasing order of the ionization potential 9. Which of the following does not have valence
of the following elements is [2016] electron in 3d-subshell? [2012]
(a) Ne > Cl > P > S > Al > Mg (a) Fe (III) (b) Mn (II)
(b) Ne > Cl > P > S > Mg > Al (c) Cr (I) (d) P (0)
(c) Ne > Cl > S > P > Mg > Al 10. Which property is same in elements of same
(d) Ne > Cl > S > P > Al > Mg group of periodic table : [2012]
3. The element, with atomic number 118, will be (a) Ionisation potential
[2015] (b) Electronegativity
(a) alkali (b) noble gas (c) Electron affinity
(c) lanthanide (d) transition element
(d) Number of valency electrons
4. The valency shell of element A contains 3
electrons while the valency shell of element B 11. The correct order of radii is [2011]
contains 6 electrons. If A combines with B, the (a) N < Be < B (b) F - < O 2 - < N 3-
probable formula of the compound formed will be
(c) Na < Li < K (d) Fe3+ < Fe2+ < Fe4+
[2014]
12. Beyllium and aluminium exhibit many properties
(a) AB2 (b) A2B
which are similar. But, the two elements differ in
(c) A2B3 (d) A3B2
[2011]
5. Which has smallest size ? [2014]
(a) forming covalent halides
(a) Na+ (b) Mg2+
(c) Ne (d) O2– (b) forming polymeric hydrides
6. The screening effect of d-electrons is [2013] (c) exhibiting maximum covalency in
(a) Equal to p-electrons compounds
(b) Much more than p-electrons (d) exhibiting amphoteric nature in their oxides
(c) Same as f-electrons 13. Among Al2O3, SiO2, P2O3 and SO2 the correct
(d) Less than p-electrons. order of acid strength is [2011]
7. The long form of the periodic table consists of
(a) Al2O3 < SiO2< SO2 < P2O3
[2013]
(b) SiO2< SO2 < Al2O3 < P2O3
(a) 8 Horizontal and 7 vertical series
(b) 7 Horizontal and 18 vertical series (c) SO2< P2O3 < SiO2 < Al2O3
(c) 7 Horizontal and 7 vertical series (d) Al2O3 < SiO2< P2O3 < SO2
(d) 8 Horizontal and 8 vertical series
EBD_7167
http://t.me/iitjeehelps

C–6 BITSAT Topicwise Solved Papers


14. Chloride ion and potassium ion are isoelectronic. 21. The order in which the following oxides are
Then [2010] arranged according to decreasing basic nature :
(a) Their sizes are same [2008]
(b) Cl– ion is bigger than K+ ion
(a) Na 2 O, MgO, Al 2O3 , CuO
(c) K+ ion is relatively bigger
(d) Their sizes depend on other cation and (b) CuO, Al 2O3 , MgO, Na 2O
anion
15. Which of the following pairs has both members (c) Al2 O3 , CuO, MgO, Na 2O
from the same period of periodic table : [2010] (d) CuO, MgO, Na 2O, Al 2O3
(a) Na, Ca (b) Na, Cl
22. The element with the outer electronic
(c) Ca, Cl (d) Cl, Br
16. Which of the following have same number of configuration 3d64s2 is a : [2007]
electrons in the outermost orbit? [2010] (a) Metalloid (b) Non-metal
(a) Pb, Sb (b) N, O (c) Transition metal (d) Noble gas
(c) As, Bi (d) P, Ca 23. Which of the following elements represents
17. In the periodic table, with the increase in atomic highly electropositive as well as highly
number, the metallic character of an element electronegative character in its period? [2007]
[2010] (a) Hydrogen (b) Nitrogen
(a) Decreases in a period and increases in a group (c) Fluorine (d) None
(b) Increases in a period and decreases in a group 24. The cause of diagonal relationship is [2006]
(c) Increases both in a period and the group (a) Similar electronegativities
(d) Decreases in a period and the group (b) Similar ionic or atomic radii
18. Beryllium resembles much with : [2009] (c) Similar polarizing power of ions
(a) Zn (b) Al (d) All the three
(c) Li (d) Ra 25. The radii of F, F–, O, O2– are in the order of :
19. Which one of the following ions has the highest [2005]
value of ionic radius ? [2009] 2– – 2– –
(a) O > F > O > F (b) O > F > F > O
(a) O2– (b) B3+
(c) F– > O2– > F > O (d) O2– > O > F– > F
(c) Li+ (d) F–
26. The electronegativity of the following elements
20. The first ionization potential of Al, as compared
to that of Mg is : [2008] increases in the order [2005]
(a) Greater (b) Lower (a) C, N, Si, P (b) N, Si, C, P
(c) The same (d) Twice as much (c) Si, P, C, N (d) P, Si, N, C

Hints & Solutions


1. (b) The right sequence of I.E1 of that of inert or noble gases, therefore it
Li < B < Be < C. will be noble gas element.
2. (b) Closed shell (Ne), half filled (P) and 2 1 2 4
completely filled configuration (Mg) are the 4. (c) The element A is ns p and B is ns p .
cause of higher value of I.E. They can form compound of the type
3. (b) Electronic configuration of element with A 2 B3 .
atomic number 118 will be [Rn]5f146d10
5. (b) An isoelectronic species having highest
7s27p6. Since its elctronic configuration in
positive charge among given species has
the outer most orbit (ns2np6) resemble with
smallest size.
http://t.me/iitjeehelps

Classification of Elements and Periodic Properties C–7


6. (d) The screening effect of inner electron of group in second period with second
the nucleus causes the decrease in element of the next group is termed as
ionization potential, therefore the order of diagonal relationship.
the screening effect is 19. (a) The ionic radii follows the order
f< d<p<s
Hence, the screening effect of d-electron O 22 - > F > Li + > B3+
is less than p-electron. 20. (b) The first ionization potential of Al is lower
7. (b) 7 Horizontal and 18 vertical series. than that of Mg because valence shell
8. (c) Amongst isoelectronic species, ionic radii configuration of Mg is 3s2 and of Al is 3s2,
of anion is more than that of cations. 3p1. As s-orbital is close to nucleus than
Further size of anion increase with increase p-orbital so removal of Ist e– from Mg is
in –ve charge and size of cation decrease difficult and require more energy as
with increase in + ve charge. Hence ionic compare to Al.
radii decreases from O– to Al3+. 21. (a) Basic nature of oxide µ metallic nature of
9. (a) P (At no. 15) has electronic configuration element. Metallic nature decrease in the
order.
1s 2 , 2s 2 p6 , 3s 2 p3 , hence no electron in Na > Mg > Al > Cu.
d - subshell 22. (c) Element with the outer electronic
10. (d) All the elements of same group in a periodic configuration 3d64s2 is d-block element. It
table have same number of valency is a transition metal.
electrons. 23. (a) First period has H and He only, out of which
11. (b) Effective nuclear charge (i,e. Z/e ratio) He is inert, hence H behaves as a highly
decreases from F– to N3– , hence the radii electropositive as well as electronegative
follows the order: F– < O2– < N3–. Z/e for element.
F– = 9/10 = 0.9, for O2– = 8/10 = .8, for 24. (d)
N3 – = 0.7 25. (a) We know that of F = 0.72Å , F– = 1.36 Å
12. (c) The valency of Beryllium is +2 while that –
and O 2 = 1.40 Å, O = 0.73 Å
of aluminium is +3 Thus, Radii of atoms and ions depends
13. (d) SO 2 > P2 O3 > SiO 2 > Al2 O3 upon relative nuclear charge therefore the
Acidic Weak acidic Amphoteric radii are in the order
14. (b) O–2 > F– > O > F
26. (c) Si and P are in the 3rd period while C and N
15. (b) 11 Na 2, 8, 1; 17 Cl 2, 8, 7
are in the 2nd period. Elements in 2nd period
These have same number (3) shells. Hence, have higher electronegativities than those
they are the elements of the same period. in the 3rd period. Since N has smaller size
16. (c) and has higher nuclear charge than C, its
17. (a) electronegativity is higher than that of C.
18. (b) Beryllium resembles with aluminium due to Similarly, the electronegativity of P is higher
similarity in the size of ions and similarity than that of Si. Thus, the overall order is :
in electropositive character. This type of Si, P, C, N.
resemblance between first element of a
EBD_7167
http://t.me/iitjeehelps

C–8 BITSAT Topicwise Solved Papers

Chapter
Chemical Bonding and
Molecular Structure 4
1. Which of the following shows correct order of
bond length? [2017] (b) é NH3 and NO3- ù
ë û
(a) O2+ > O2 > O2– > O22–
(b) O2+ < O2– > O2 < O22– (c) [ NF3 and BF3 ]
(c) O2+ > O2 < O2– > O22–
(d) O2+ > O2 < O2– > O22– (d) é BF4- and NH +4 ù
ë û
2. Which of these compounds are directional?
[2017] 9. N2 and O2 are converted into mono anions,
(a) NaCl (b) CO2 N2– and O2– respectively. Which of the following
statements is wrong ? [2014]
(c) BaO (d) CsCl2
(a) In N2, the N—N bond weakens
3. In PO 34– , the formal charge on each oxygen (b) In O2, the O—O bond order increases
atom and the P - O bond order respectively are (c) In O2, bond length decreases
[2016] (d) N2– becomes diamagnetic
(a) –0.75, 0.6 (b) –0.75, 1.0 10. Number of bonds in SO2 are [2013]
(c) –0.75, 1.25 (d) –3, 1.25 (a) Two s and Two p
4. Which one of the following does not follow (b) Two s and One p
octate rule? [2015] (c) Two s , two p and one lone pair
(a) PF3 (b) BF3 (c) CO2 (d) CCl4 (d) None of these
5. The P—P—P angle in P 4 molecule and 11. The vapour pressure of [2012]
S—S—S angle in S8 molecule is (in degree) OH OH
respectively [2015]
is higher than
(a) 60°, 107° (b) 107°, 60°
(c) 40°, 60° (d) 60°, 40° NO 2 O2 N
6. Which of the following molecule/ion has all the due to
three types of bonds, electrovalent, covalent (a) Dipole moment
and co-ordinate : [2015] (b) Dipole- dipole interaction
(a) HCl (b) NH 4+ (c) Cl - (d) H2O2 (c) H – bonding
(d) Lattice structure
7. The molecules BF3 and NF3 are both covalent
compounds, but BF3 is non polar whereas NF3 12. A s bonded molecule MX 3 is T-shaped. The
is polar. The reason for this is [2014] number of non bonded pair of electrons is [2011]
(a) atomic size of Boron is larger than nitrogen (a) 0 (b) 2
(b) Boron is metal while nitrogen is gas (c) 1
(c) B – F bonds are non-polar while N – F (d) can be predicted only if atomic number is
bonds are polar known
(d) BF3 is planar but NF3 is pyramidal 13. The correct bond order in the following species
8. Which one of the following pairs is isostructural is: [2011]
(i.e., having the same shape and hybridization)?
[2014] (a) O 22+ < O2– < O 2+ (b) O +2 < O 2– < O 22+
-
(a) éë BCl3 and BrCl3 ùû (c) O 2– < O +2 < O 22+ (d) O 22+ < O 2+ < O 2–
http://t.me/iitjeehelps

Chemical Bonding and Molecular Structure C–9


14. Which of the following statements is/are true (a) H3C CH 3
[2010]
1. PH5 and BiCl5 do not exist C
2. pp – dp bond is present in SO2 H H
3. I 3+ has bent geometry (b) CH 3C º CCH 3
4. SeF4 and CH4 have same shape (c) CH 3CH 2 C º CH
(a) 1, 2, 3 (b) 1, 3
(c) 1, 3, 4 (d) 1, 2, 4 (d) CH 2 = CH – C º CH
15. Which of the following two are isostructural ? 18. In the dichromate dianion, [2006]
[2009] (a) 4 Cr – O bonds are equivalent
(b) 6 Cr – O bonds are equivalent
(a) XeF2, IF2- (b) NH3, BF3 (c) all Cr – O bonds are equivalent
(d) all Cr – O bonds are nonequivalent
(c) CO32 - ,SO32 - (d) PCI5, ICI5
16. Two ice cubes are pressed over each other until 19. In piperidine , the hybrid state assumed
they unite to form one block. Which one of the N
by N is [2006]
following forces dominates for holding them H
together? [2008]
(a) sp (b) sp 2 (c) sp 3 (d) dsp 2
(a) Dipole-dipole interaction
20. Which of the following is the correct electron
(b) Van der waals’ forces
dot structure of N2O molecule? [2005]
(c) Hydrogen bond formation .. + ..
(b) : N º N = O -
(d) Covalent attraction (a) :N =N =O: .. :
17. Which of the following hydrocarbons has the .. .. .. ..
(c) (d) : N = N = O :
lowest dipole moment ? [2007] .. = N = O:
N ..

Hints & Solutions


1. (b) Bond length decreases with an increase in Three unit negative charge is being shared
bond order. Therefore, the order of bond by four O atoms. Formal charge
length in these species is O2+ < O2-> O2 < = –3/4 = – 0.75.
O22- (bond order - O2+ = 2.5, O2 =2, O2- =1.5, 4. (b) BF3 does not follow octate rule because
O22- =1) central atom, boron lacks an electron pair.
2. (b) Ionic bonding is non directional, whereas Thus, it also acts as Lewis acid.
covalent bonding is directional. So, CO2
is directional. ××
×F ×
××
× ×× B ×
×F
×
3- 3- ××
é O ù é O ù
ê | ú ê | ú
×

3. (c) êO - P - O ú « êO - P = Oú « × ×
|| | × F×
ê ú ê ú × ×
ë O û ë O û
5. (a) In P4 molecule, the four sp3-hybridised
3- 3-
é O ù é O ù phosphorous atoms lie at the corners of a
ê || ú ê | ú
ê O - P - O ú « êO = P - O ú
regular tetrahedron with ÐPPP = 60°.
ê | ú ê
O
| ú In S8 molecule S-S-S angle is 107° rings.
ë O û ë û
S 2m
No. of bonds 21 S S
Bond order = S S
No. of Resonating structures 107°
S
5 S
= = 1.25 S
4
EBD_7167
http://t.me/iitjeehelps

C – 10 BITSAT Topicwise Solved Papers


6. (b) Bond structure of molecules are : E.C.: σ1s2 < σ*1s 2 < σ2s 2 < σ* 2s 2 < σ2p2z
HCl = H – Cl
H + < π2p2y = π2p 2px < π* 2p1y
NH +4 = H — N — H N b - N a 10 - 5 5 1
Bond order = = = =2
H 2 2 2 2
×× – O 2– (Super oxide ion): Total no. of e– s
Cl– = ××Cl ××
××
(16 + 1) = 17 .
H2O2 = H – O – O – H
+ E.C.: σ1s 2 < σ*1s2 < σ2s2 < σ* 2s2 < σ2p2z
hence, clearly NH 4 ion contains all three
types of bonds. < π2p2y = π2px2 < π* 2p 2x = π* 2p1x
7. (d) The shape of BF3 is trigonal planar
d-
(Nb - Na ) 10 - 7 3 1
Bond order = = = =1
F d + d- 2 2 2 2
B – F and m = 0 hence it is non polar..
d- F
O+22 ion: Total no. of e–s = (16 – 2) = 14
The shape of NF 3 is pyramidal
+ E.C.: s1s2 < s*1s2 < s2s2 < s*2s2 < s2pz2 <
:

Nd p2py2 = p2px2
d- F F d- and m ¹ 0 hence it is polar..
F d- (N b – Na ) 10 – 4 6
Bond order = = = =3
8. (d) BF4- hybridisation sp 3 ,
tetrahedral 2 2 2
– + 2+
str ucture. NH +4 h ybridisation sp 3 , So bond order: O2 < O2 < O2
tetrahedral structure. 14. (a) All the three statements are correct.
9. (b) We know that in O2 bond, the order is 2 15. (a) XeF2 and IF -2 both are linear and have
hybridisation sp3d
and in O2– bond, the order is 1.5. Therefore
16. (c) It is due to H - bonding.
the wrong statements is (b).
17. (b) H 3C . C º C . CH 3 , symmetrical and linear..
10. (c) O = S = O (bent structure) 2s, 2p and one
Hence dipole moment = 0.
lone pair.
11. (c) Ortho-nitrophenol has intramolecular 18. (b) The structure of Cr2 O 72 -
.
. . OH O O
2–

H-bonding | |
. O and para- Cr Cr
||

. N | | | | | |
O OOO OOO
nitrophenol has intermolecular H-bonding.
There are six normal Cr - O bonds and two
NO 2 NO 2 NO 2 bridged Cr - O bonds.
The six normal Cr - O bonds are expected to
, be equivalent and different from those of
| | | the bridged Cr - O bonds.
O– H O– H O– H 19. (c) Hybridisation of N = ½ [5 + 3 + 0 – 0] = 4
Hence former is more volatile than latter. hence sp3.
12. (b) For T-shape geometry the molecule must + ..
-
have 3 bonded pair and 2 lone pair of 20. (b) :NºN= O . . : octet of each atom is
electrons. complete.
13. (c) O +2 ion - Total no. of e– s (16 – 1) = 15.
http://t.me/iitjeehelps

Chapter
StatesofMatter
5
1. Which of the following represents Gay Lussac's 7. When the temperature is raised, the viscosity of
law ? [2017] liquid decreases this is because [2010]
P (a) decreased volume of the solution
I. = constant II. P1T2 = P2T1
T (b) increased attraction between molecules
III. P1V1 = P2V2 (c) decreased covalent and hydrogen bond
Choose the correct option. forces
(a) I, II and III (b) II and III (d) increase in temperature increase the
(c) I and III (d) I and II average kinetic energy of molecules which
2. At what temperature, the rate of effusion of N2 overcomes the attractive force between
would be 1.625 times than that of SO2 at 50o C ? them
[2016] 8. At a constant volume the specific heat of a gas
(a) 110 K (b) 173 K is 0.075 and its molecular weight is 40. The gas
(c) 373 K (d) 273 K is: [2010]
3. The average kinetic energy of an ideal gas per (a) Monoatomic
molecule in SI unit at 25° C will be [2016] (b) Diatomic
(a) 6.17 × 10-21 kJ (b) 6.17 × 10–21 J (c) Triatomic
(c) 6.17 × 10–20 J (d) 7.16 × 10–20 J (d) None of above
4. In Van der Waal’s equation the constant b is the
9. The cooking time in a pressure cooker is less
measure of [2014]
(a) Repulsive forces between molecules of gas because : [2009]
(b) Attractive forces between molecules of gas (a) More heat is used
(c) Volume occupied by gas molecules (b) High pressure cooks the food
(d) All of these (c) The boiling point of water increases in the
5. Increasing order of rms velocities of H2, O2, N2 cooker
and HBr is [2013] (d) Heat is uniformly distributed
(a) H 2 > O 2 > N 2 > HBr 10. Density ratio of O2 and H2 is 16:1. The ratio of
(b) HBr < O 2 < N 2 < H 2 their r.m.s. velocities will be [2008]
(c) H 2 > N 2 < O 2 > HBr (a) 4 : 1 (b) 1 : 16
(c) 1 : 4 (d) 16 : 1
(d) HBr > N 2 < O 2 < H 2
11. The inversion temperature (Ti) for a gas is given
6. An ideal gas can’t be liquefied because [2012]
by [2005]
(a) its critical temperature is always above 0°C
(b) its molecules are relatively smaller in size (a) a / Rb (b) 2a / Rb
(c) it solidifies before becoming a liquid Rb 2 Rb
(d) forces operated between its molecules are (c) (d)
a a
negligible
EBD_7167
http://t.me/iitjeehelps

C – 12 BITSAT Topicwise Solved Papers

Hints & Solutions


P 1
1. (d) = constant (Gay Lussac's law) rms µ
T m.wt
P1 P2 The order of increasing m. wt. is
Þ = Þ P1T2 = P2T1
T1 T2 H 2 < N 2 < O 2 < HBr
PV = constant Order of Vrms of molecules.
P1V1 = P2V2 [Boyle's law] HBr < O 2 < N 2 < H 2 .
6. (d) In an ideal gas, the intermolecular forces of
3RT attraction are negligible and hence it cannot
2. (c) r µ U and U =
M be liquefied.
7. (b)
r1 TM
\ = 1 2 or 8. (a) We know that
r2 T2 M1
Cp = Cv × m = 0.075 × 40 = 3.0 cal
rN T1 ´ 64 Q Cp – Cv = R
2
= = 1.625 or Cp = R + Cv = 2 + 3 = 5
rSO2 323 ´ 28
Cp 5
or T1 = 373 K Now, = g; \ g = = 1.66
Cv 3
3 3 8.313 This value shows that the gas is
3. (b) KE = kT = ´ ´ 298
2 2 6.023 ´ 1023 monoatomic.
9. (c) In pressure cooker, pressure is high thus,
= 6.17 ´ 10 -21 J . the boiling point of water increases,
(Average Kinetic energy resulting cooking time is less than other
3 3 R open pots.
KE = kT = T
2 2 N u1 d2 1
10. (c) = = = 1: 4
4. (c) u2 d1 16
5. (b) RMS velocity of molecules depends on
2a
mass. If Mol .wt. increases, rms velocity of 11. (b) Ti =
melocules decreases. Rb
http://t.me/iitjeehelps

Chapter
Thermodynamics
6
1 6. A spontaneous reaction is impossible if[2015]
1. For the reaction CO (g ) + O 2 (g ) ® CO 2 (g ) (a) both DH and DS are negative
2
(b) both DH and DS are positive
Which one of the statement is correct at
(c) DH is negative and DS is positive
constant T and P ? [2017]
(d) DH is positive and DS is negative
(a) DH = DE (b) DH < DE 7. If the enthalpy of vaporization of water is 186.5
(c) DH > DE kJmol–1, the entropy if its vaporization will be :
[2014]
(d) DH is independent of physical state of the –1 –1
(a) 0.5 JK mol (b) 1.0 JK mol–1
–1
reactants
2. For a given reaction, DH = 35.5 kJ mol-1 and (c) 1.5 JK–1mol–1 (d) 2.0 JK–1mol–1
DS = 83.6 JK-1 mol-1. The reaction is spontaneous 8. The heats of neutralisation of CH3COOH,
at : (Assume that DH and DS do not vary with HCOOH, HCN and H2S are – 13.2, – 13.4, – 2.9
tempearature) [2017] and – 3.8 kCal per equivalent respectively.
(a) T > 425 K (b) All temperatures Arrange the acids in increasing order of acidic
(c) T > 298 K (d) T < 425 K strength. [2014]
3. For a particular reversible reaction at (a) HCOOH > CH 3COOH > H 2S > HCN
temperature T, DH and DS were found to be both (b) CH 3COOH > HCOOH > H 2S > HCN
+ve. If Te is the temperature at equilibrium, the
(c) H 2S > HCOOH > CH 3COOH > HCN
reaction would be spontaneous when [2016]
(a) Te > T (b) T > Te (d) HCOOH > H 2S > CH 3COOH > HCN
(c) Te is 5 times T (d) T = Te 9. For determining the spontaneity of a process
4. Given which of the following is considered ? [2013]
Reaction Energy Change (in kJ) (a) DS system
(b) DS surroundings
Li(s) ® Li(g) 161
(c) DS system + DS surroundings
Li(g) ® Li+(g) 520 (d) DS system – DS surroundings
1 10. The enthalpy of combustion of 2 moles of
F (g) ® F(g) 77 benzene at 27°C differs from the value
2 2
F(g) + e– ® F–(g) (Electron gain enthalpy) determined in bomb calorimeter by [2013]
(a) – 2.494 kJ (b) 2.494 kJ
Li+ (g) + F–(g) ® Li F(s) – 1047
(c) – 7.483 kJ (d) 7.483 kJ
1 11. In which of the following reactions, standard
Li(s) + F2(g) ® Li F(s) – 617
2 entropy change (DS°) is positive and standard
Based on data provided, the value of electron Gibb’s energy change (DG°) decreases sharply
gain enthalpy of fluorine would be : [2016] with increasing temperature ? [2012]
(a) – 300 kJ mol–1 (b) – 350 kJ mol–1 1
(a) C graphite + O2(g) ® CO(g)
(c) – 328 kJ mol–1 (d) – 228 kJ mol–1 2
5. Which law of the thermodynamics helps in 1
(b) CO(g) + O2(g) ® CO2(g)
calculating the absolute entropies of various 2
substances at different temperatures? [2015] 1
(c) Mg(s) + O2(g) ® MgO(s)
(a) First law (b) Second law 2
(c) Third law (d) Zeroth law 1 1 1
(d) C graphite + O2(g) ® CO2(g)
2 2 2
EBD_7167
http://t.me/iitjeehelps

C – 14 BITSAT Topicwise Solved Papers

-1 -1
12. Bond enthalpies of H 2 , X2 and HX are in the (a) – 1.55 kJmol -1 , – 96.44 Jmol K
ratio 2 : 1 : 2. If enthalpy of formation of HX is – -1 -1 -1
50 kJ mol–1, the bond enthalpy of X2 is [2012] (b) – 2.55 kJmol , – 106.44 Jmol K
(a) 100 kJ mol–1 (b) 300 kJ mol–1 -1 -1 -1
(c) 200 kJ mol–1 (d) 400 kJ mol–1 (c) 1.55 kJmol , 96.44 Jmol K
13. What is the free energy change, DG , when 1.0 (d) – 1.55 kJmol -1 , 96.44 Jmol -1K -1
mole of water at 100º C and 1 atm pressure is 19. Temperature of 5 moles of a gas is decreased by
converted into steam at 100°C and 1 atm. 2K at constant pressure. Indicate the correct
pressure? [2011]) statement [2007]
(a) 540 cal (b) –9800 cal (a) Work done by gas is = 5 R
(c) 9800 cal (d) 0 cal (b) Work done over the gas is = 10 R
14. For the reaction : N2 + 3H2 ‡ˆˆ ˆˆ† 2NH [2009]
3 (c) Work done by the gas = 10 R
Which one of the following is correct regarding
DH : (d) Work done = 0
(a) DH = DE + 2RT (b) DH = DE – 2RT 20. Compounds with high heat of formation are less
(c) DH = DE + RT (d) DH = DE – RT stable because [2006]
15. One mole of an ideal gas at 300 K is expanded (a) it is difficult to synthesize them
isothermally from an initial volume of 1L to 10 L. (b) energy rich state leads to instability
The DE for this process is (c) high temperature is required to synthesize
(R = 2 cal mol–1 K–1) [2009] them
(a) 163.7 cal (b) zero (d) molecules of such compunds are distorted.
(c) 1381.1 cal (d) 9 lit atom 21. The energy that opposes dissolution of a
16. At 25°C and 1 bar which of the following has a solvent is [2006]
non-zero DH°f ? [2009] (a) hydration energy (b) lattice energy
(a) Br2(l) (b) C (graphite) (c) internal energy (d) bond energy
(c) O3(g) (d) I2(s) 22. The energy that opposes dissolution of a
17. For which of the process, DS is negative? solvent is [2006]
[2008] (a) hydration energy (b) lattice energy
(a) H 2 (g) ® 2 H (g) (c) internal energy (d) bond energy
(b) N 2 (g )(1atm) ® N 2 (g)(8atm) 23. In the reaction
(c) 2SO 3 (g) ® 2SO 2 (g) + O 2 ( g) CO 2 (g) + H 2 (g) ¾¾
® CO(g) + H 2 O(g)
(d) C( diamond) ® C(graphite) DH = 40 kJ; DH represents : [2005]
18. Ka for CH 3COOH at 25°C is 1.754 × 10–5 . At (a) Heat of formation
(b) Heat of combustion
50°C Ka is 1.633 × 10–5. What are DH° and DS° (c) Heat of neutralisation
for the ionisation of CH 3COOH ? [2008] (d) Heat of reaction

Hints & Solutions


1 1 DH
1. (b) Dn = - ; D H = DE - RT ; Þ D E > D H \ DH = TeDS or Te =
2 2 DS
2. (a) DG = DH – TDS For a spontaneous reaction
For a reaction to be spontaneous, DG must be negative which is possible
DG = –ve i.e., DH < TDS only if DH – TDS < 0
DH
\ DH < TDS or T > ; Te < T
DH 35.5 ´ 103 Jmol -1 DS
\T> = 4. (c) Applying Hess’s Law;
DS 83.6 JK -1
So, the given reaction will be spontaneous 1
D f H° = D sub H + Ddiss H +
at T > 425 K 2
3. (b) At equilibrium DG = 0 I.E. + E.A + D lattice H
Hence, DG = DH – TeDS = 0 – 617 = 161 + 520 + 77 + E.A. + (– 1047)
http://t.me/iitjeehelps

Thermodynamics C – 15
E.A. = –617 + 289 = –328 kJ mol–1 Where DH = enthalpy of reaction at
\ electron affinity of fluorine constant pressure
= –328 kJ mol–1 DE = Heat of reaction at constant volume
5. (c) The third law helps to calculate the R = molar gas constant
absolute entropies of pure substances at T = temperature of the reaction
different temperature. n = (no. of moles of product) – (no. of moles
6. (d) DG = DH – TDS; DG is positive for a reaction of reactant.)
to be non-spontaneous when DH is From reaction, n = n P - n R = 2 – 4 = – 2
positive and DS is negative.
Hence, DH = DE - 2RT .
7. (a) Given: DH = 186.5 kJ mol -1 15. (b) For isothermal process, DE = 0
B. pt of water 16. (c) Ozone is allotropic form of oxygen and is
= 100°C = 100 + 273 = 373K of higher energy (by 68 K Cal mol –1) than
Entropy change, O2. Hence it can not be taken as the
DH 186.5 KJ mol -1 reference in standard state.
DS = =
T 373K 17. (b) High pressur e reduces volume and
= 0.5 kJ mol -1K -1 decreases entropy hence DS negative.
8. (a) The greater the negative value of heat of 18. (a) DG° = – 2.303 RT log K
neutralisation, more is the strength of the DG°(323) = – 2.303 × 8.314 × 323 log
acid. Hence, 1.633 × 10–5 = 29605 J
DG°(298) = 2.303 × 8.314× 298 log
HCOOH > CH 3COOH > H 2S > HCN
1.754 × 10–5 = 27194 J
9. (c) The criteria for the spontaneity, DS system DG° = DH° – TDS°
+ DS surroundings, must be + ve. 29605 = DH° – 323DS° ...(i)
10. (c) By bomb calorimeter we get DE . 27194 = DH° – 298DS° ...(ii)
2C6 H 6 (l) + 15 O 2 (g) ¾
¾® From eqn. (i) & (ii) on calculation we find
12 CO2 + 6H2O (l) DS° = – 96.44 J K -1mol -1
DH – DE = DnRT
= (12 – 15) × 8.314 × 300 = – 7.483 kJ \ DH° = – 1.55 k J/ mol
11. (a) Since, in the first reaction gaseous 19. (b) For 5 moles of gas at temperature T,
products are forming from solid carbon PV1 = 5RT
hence entropy will increase i.e. DS = +ve. For 5 moles of gas at temperature T – 2,
PV2 = 5R(T – 2)
1 \ P (V2 - V1 ) = 5R(T – 2 – T);
C (gr.) + O2(g) ® CO(g); DS° = + ve
2
Since, DG° = DH° – TDS hence the value of PDV = –10R, – PDV = 10R
DG decrease on increasing temperature. When DV is negative, W is + ve.
20. (b) The more the energy, the lesser is the
1 1
12. (a) H2 + X2 ¾ ¾® HX stability.
2 2 21. (b) Lattice energy opposes dissoloution of a
Let the bond enthalpy of X – X bond be x. solvent. Lattice energy is the amount of
DH f (HX ) = – 50 energy required to dissociate 1 mole of
1 1 ionic crystals into ions. If hydration energy
= DH H–H + DH X - X – DH H - X of solid is more than lattice energy, then
2 2
1 1 -x solid dissolves.
= 2x + x – 2x = 22. (b) Lattice energy opposes dissoloution of a
2 2 2 solvent. Lattice energy is the amount of
\ x = 50 × 2 = 100 kJ mol–1 energy required to dissociate 1 mole of
13. (d) Condition of equilibrium, hence DG = 0
ionic crystals into ions. If hydration energy
14. ˆˆ† 2NH 3
(b) N 2 + 3H 2 ‡ˆˆ of solid is more than lattice energy, then
According to thermodynamics’s Ist law solid dissolves.
DH = DE + nRT 23. (d) Heat of reaction
EBD_7167
http://t.me/iitjeehelps

C – 16 BITSAT Topicwise Solved Papers


Chapter
Equilibrium
7
1. Which of the following can form buffer
solution? [2017] (c) ClO3- < ClO4- < ClO2- < ClO -
(a) aq.NH3 + NH4OH (b) KOH + HNO3 (d) ClO- > ClO3- > ClO 2- < ClO 4-
(c) NaOH + HCl (d) KI + KOH
2. Which of the following can act as both Bronsted 8. Which of the following according to Le-
acid and Bronsted base? [2017] Chatelier’s principle is correct? [2015]
(a) Na2CO3 (b) OH– (c) HCO3– (d) NH3 (a) Increase in temperature favours the
endothermic reaction
3. The degree of dissociation of PCl5 (a) obeying (b) Increase in temperature favours the
the equilibrium PCl 5 PCl 3 + Cl 2 is exothermic reaction
related to the equilibrium pressure by [2016] (c) Increase in pressure shifts the equilibrium
in that side in which number of gaseous
1 1 moles increases
(a) a µ
4
(b) a µ
P P (d) All of the above are true
9. For a given exothermic reaction, Kp and KP¢ are
1
(c) a µ 2 (d) a µ P the equilibrium constants at temperatures T1 and
P T2, respectively. Assuming that heat of reaction
4. In a closed system, A(s) 2B(g) + 3C(g), if is constant in temperature range between T1 and
partial pressure of C is doubled, then partial T2, it is readily observed that: [2014]
pressure of B will be [2016] (a) Kp > KP¢ (b) Kp < KP¢
(a) 2 2 times the original value 1
(c) Kp = KP¢ (d) Kp =
1 K¢p
(b) times the original value
2 10. If 1.0 mole of I2 is introduced into 1.0 litre flask at
(c) 2 times the original value 1000 K, at quilibrium (Kc = 10–6), which one is
1 correct? [2013]
(d) times the original value (a) [I2 (g)] > [I– (g)] (b) [I2 (g)] < [I– (g)]
2 2
5. The percentage hydrolysis of 0.15 M solution 1 -
(c) [I2 (g)] = [I– (g)] (d) [I2 (g)] = [I (g)]
of ammonium acetate, Ka for CH3COOH is 2
1.8 × 10–5 and Kb for NH3 is 1.8 × 10–5 [2016] 11. For the reaction [2013]
(a) 0.556 (b) 4.72 (c) 9.38 (d) 5.56 CO (g) + (1/2) O2 (g) ¾¾ ® CO2 (g), Kp / Kc is
6. For a sparingly soluble salt A p B q, the (a) RT (b) (RT)–1 (c) (RT)–1/2(d)(RT)1/2
relationship of its solubility product Ls ® Ksp 12. The pOH value of a solution whose hydroxide
with its solubility (S) is [2016] ion concentration is 6.2 × 10–9 mol/litre is [2012]
(a) Ls ® Ksp = S pq (pq) P + q (a) 8.21 (b) 6.21 (c) 7.75 (d) 7.21
13. Which of the following combinations would not
(b) Ls = Sp + q .p p q q result in the formation of a buffer solution?[2012]
(a) NH3 + HCl (b) NH4Cl + NH3
(c) Ls ® Ksp = Sp + q .p q q p (c) CH3COOH + NaCl (d) NaOH+ HC2H3O2
(d) Ls ® Ksp = S p q
pq p q 14. The reaction, SO 2 + Cl 2 ¾ ¾® SO 2 Cl 2 is
exothermic and reversible. A mixture of SO2 (g),
7. What is the correct increasing order of Bronsted
Cl2 (g) and SO2Cl2 (l) is at equilibrium in a closed
bases? [2015]
container. Now a certain quantity of extra SO2 is
(a) ClO 4- < ClO3- < ClO2- < ClO - introduced into the container, the volume
remaining the same. Which of the following
(b) ClO4- > ClO3- > ClO -2 > ClO- is/are true? [2012]
http://t.me/iitjeehelps

Equilibrium C – 17
(a) The pressure inside the container will not 21. The solubility product of AgCl is 4.0 × 10–10 at
change 298 K. The solubility of AgCl in 0.04 M CaCl2
(b) The temperature will not change will be [2008]
(c) The temperature will increase (a) 2.0 × 10–5 M (b) 1.0 × 10–4 M
(d) The temperature will decrease (c) 5.0 × 10–9 M (d) 2.2 × 10–4 M
15. H2S gas when passed through a solution of +
22. What is [H ] of a solution having 0.1 M HCN and
cations containing HCl precipitates th e 0.2 M NaCN? (Ka for HCN = 6.2 × 10–10) [2007]
cati ons of secon d group of qualitative (a) 3.1 × 1010 (b) 6.2 × 105
analysis but not those belonging to the fourth (c) 6.2 × 10 –10 (d) 3.1 × 10–10
group. It is because [2011] 23. When hydrogen molecule decompose into it’s
(a) presence of HCl decreases the sulphide atoms which conditions gives maximum yields
ion concentration. of H atoms ? [2007]
(b) solubility product of group II sulphides (a) High temperature and low pressure
is more than that of group IV sulphides. (b) Low temperature and high pressure
(c) presence of HCl increases the sulphide (c) High temperature and high pressure
ion concentration. (d) Low temperature and low pressure
(d) sulphides of group IV cations are 24. If K1 and K2 are respective equilibrium constants
unstable in HCl. for the two reactions [2006]
16. The pH of a solution is increased from 3 to 6; its ˆˆ† XeOF (g) + 2HF (g)
XeF6 (g) + H2O (g) ‡ˆˆ 4
H+ ion concentration will be [2011] XeO4 (g) + XeF (g)
(a) reduced to half ˆˆ† XeOF4 (g) + XeO3F2 (g)
‡ˆˆ
(b) doubled the equilibrium constant for the reaction
(c) reduced by 1000 times (d) increased by ˆˆ† XeO F (g) + H O (g)
XeO4 (g) + 2HF (g) ‡ˆˆ 3 2 2
1000 times will be
17. Which of these is least likely to act as Lewis K1 K1 K2
base? [2010] (a) 2 (b) K1.K 2 (c) (d)
(a) F– (b) BF3 (c) PF3 (d) CO K2 K2 K1
18. The Ksp of CuS, Ag2S and HgS are 10–31,10–44 25. Which equilibrium can be described as an acid-
and 10–54 respectively. The solubility of these base reaction using the Lewis acid-base
sulphides are in the order [2010] definition but not using the Bronsted-Lowry
(a) Ag2S > CuS > HgS definition? [2006]
(b) AgS > HgS > CuS (a) 2NH3 + H2SO4 ‡ˆˆ ˆˆ† 2NH + + SO 2–
4 4
(c) HgS > Ag2S > CuS (b) NH3 + CH3COOH ‡ˆˆ ˆˆ†
(d) CuS > Ag2S > HgS NH4+ + CH3COO–
19. If the equilibrium constant of the reaction (c) H2O + CH3COOH ‡ˆˆ ˆˆ†
ˆˆ† H2 + I2 is 0.25, then the equilibrium
2HI ‡ˆˆ H3O+ + CH3COO–
constant for the reaction H2 + I2 ‡ˆˆˆˆ† 2HI (d) [Cu(H2O)4]2– + 4 NH3 ‡ˆˆ ˆˆ†
would be [2009] [Cu(NH3)4]2+ + 4H2O
(a) 1 (b) 2 (c) 3 (d) 4 26. When H2S is passed in acidic medium in solution
20. For a system in equilibrium, DG = 0 under having CuS and ZnS,only CuS is precipitated
conditions of constant [2008] because [2005]
(a) Temperature and pressure (a) KspZnS = KspCuS
(b) Temperature and volume (b) KspCuS <<KspZnS
(c) Energy and volume (c) KspCuS >>KspZnS
(d) Pressure and volume (d) ZnS has lower melting point than CuS.

Hints & Solutions


1. (a) Ammonia is a weak base and a salt 2. (c) H2CO3 H+ + HCO3–
containing its conjugate acid, the ammonium -
cation, such as NH4OH functions as a buffer H + CO3 HCO 3 can donate and accept H+.
+ –.

solution when they are present together in a 3. (b) PCl 5 PCl 3 + Cl 2


solution. at eq. 1 – a a a
EBD_7167
http://t.me/iitjeehelps

C – 18 BITSAT Topicwise Solved Papers


Total no. of moles at eq. = 1 – a + a + a = 1 + a 12. (a) –log (OH) = pOH; – log 6.2 × 10–9 = pOH;
a a \ pOH = 8.21
P´ P a 2P
1 + a 1 + a 13. (d) Combination of NaOH and CH 3 COOH is
\ Kp = =
1- a 1- a2 the mixture of alkali and acetic acid. Therefore
P
1+ a this combination can not be buffer forming
solution.
Kp
or, Kp = a2P ; \ a = when 1 - a 2 = 1 14. (c) By addition of SO2, equilibrium will shift to
P RHS which is exothermic. Hence temp, will
2 3 increase.
4. (d) K p = p B1 ´ p C1 15. (a) IV th g r oup n eeds h i gh er S 2– i on
concentration. In presence of HCl, the
Again, K p = p 2B ´ (2p C1 )3 dissociation of H2S decreases hence produces
2
less amount of sulphide ions due to common
\ p 2B1 ´ p C3 1 = 2
PB2 ´ 8p C3 1 ion effect, thus HCl decreases the solubility
of H2S which is sufficient to precipitate IInd
p 2B1 p B1 group radicals.
\ = p2B2 or,, = p B2
8 2 2 16. (c) pH = 3. \ [H+] = 10–3; pH = 6,
\ [H+] = 10–6.
-14 Hence [H+] is reduced by 10–3 times.
Kw 1´10
5. (a) a= = 17. (b) BF3 is Lewis acid (e– pair acceptor)
Ka ´ K b 1.8 ´10-5 ´ 1.8 ´ 10-5 18. (a) For CuS, solubility = (10–31)1/2;
= 0.556 1
1
6. (b) ApBq pA+ + qB – æ K sp ö 3 æ 10 - 44 ö 3
For Ag2S = çç ÷ =ç ÷ and
Sp Sq ÷ ç 4 ÷
è 4 ø è ø
Let the solubility be S mol/liter
( )
1
+ p – q
Thus, Ksp = [A ] [B ] for HgS = 10 -54 2
p q p. q p+q 19. (d) When the rection is reversed,
= [Sp] [Sq] = p q (S) .
1 1
7. (a) ClO -4 < ClO3- < ClO-2 < ClO- is the correct K' = = =4
increasing order of Bronsted base. With K 0.25
increase in the number of oxygen atoms in the 20. (a) At constant T and P, DG = 0.
conjugate bases, the delocalisation of the p 21. (c) Solubility of AgCl
bond becomes more and more extended. This K sp 4 ´10 -10
results in decrease in the electron density. [Ag+] = = = 5.0 ´ 10 -9 M
Consequently basicity also decreases. [Cl - ] 0.08
8. (a) According to Le-Chatelier’s principle 0.08 M of concentration of [Cl–] comes from
increase in temperature favours the endothermic CaCl2.
reaction while decrease in temperature favour + [Acid]
the exothermic reaction. 22. (a) [H ] = K a ´
Increase in pressure shifts the equilibrium in [Salt]
that side in which number of gaseous moles -10 0.1
decreases. = 6.2 ´ 10 ´ = 3.1 ´10 -10
0. 2
9. (a) In exothermic reactions on increasing 23. (a) H2 ® 2H; DH = + ve
temperature value of Kp decreases According to Le-Chatelier’s Principal, to increase
\ Kp > Kp¢ (Assuming T1 < T2) the rate of forward reaction, temperature should
be increased and pressure should be decreased.
(2x ) 2
10. (b) I2 2I– Kc = = 10 -6 24. (d) Reaction (II) and reverse of reaction (I)
(1 - x) gives the desired reaction hence
1–x 2x 1 K
It shows that (1 – x) < 2x K = K2 ´ = 2 .
K1 K1
11. (b) K p = K c (RT) Dn ;. ˆˆ†
25. (d) [Cu(H2O)4]2+ + 4NH3 ‡ˆˆ
æ 1ö 3 1 Kp [Cu(NH3)4]2+ + 4H2O
Dn = 1 - ç1 + ÷ = 1 - = - \ = (RT ) -1 / 2 involves lose and gain of electrons.
è 2 ø 2 2 Kc 26. (b) The compound with lower Ksp is precipitated
first.
http://t.me/iitjeehelps

Chapter
Redox Reaction
8
1. A transition metal ion having metal in its highest 5. The oxidation state of chromium in potassium
oxidation state behaves as [2013] dichromate (K2Cr2O7) is [2008]
(a) –5 (b) +6
(a) Oxidising agent (b) Reducing agent
(c) +2 (d) –2
(c) Dehydrating agent (d) Bleaching agent 6. In which of the following pairs, there is greatest
2. In the reaction [2012] difference in the oxidation number of the
underlined elements ? [2007]
3Br2 + 6CO32- + 3H 2O ® 5Br - + BrO3– + 6HCO 3-
(a) NO 2 and N 2 O 4 (b) P 2 O5 and P 4 O10
(a) Bromine is oxidised and carbonate is
reduced. (c) N 2 O and NO (d) SO2 and SO3
(b) Bromine is reduced and water is oxidised 7. For the redox reaction
(c) Bromine is neither reduced nor oxidised
(d) Bromine is both reduced and oxidised MnO 4– + C 2 O 24 – + H + ® Mn 2+ + CO 2 + H 2 O
3. A gas X at 1 atm is bubbled through a solution the correct coefficients of the reactants for the
containing a mixture of 1 M Y– and 1 M Z– at balanced reaction are: [2006]
25°C. If the reduction potential is Z > Y > X, then MnO –4 C 2O 24 – H+
[2011] (a) 2 5 16
(a) Y will oxidise X and not Z (b) 16 5 2
(c) 5 16 2
(b) Y will oxidise Z and not X
(d) 2 16 5
(c) Y will oxidise both X and Z 8. In the reaction
(d) Y will reduce both X and Z
Cr2 O 72 - + 14H + + 6I - ¾¾
®
4. The oxidation states of sulphur in the anions
2Cr3+ + 7H 2 O + 3I 2
SO 32 - , S2 O 24- and S2 O 62 - follow the order Which element is reduced [2006]
[2009]
(a) I (b) O
(a) SO 32 - < S2 O 24 - < S2 O 62- (c) H (d) Cr
9. For the the reaction
(b) S2 O 24- < S2 O 62 - < SO 32-
CrO4–2 aq. + 2H+ ® Cr2 O 72- (aq.) + H2O,
(c) S2 O 62- < S2 O 24 - < SO 32- element oxidised is [2005]
(a) Oxygen (b) H
(d) S2 O 24- < SO 32 - < S2 O 62-
(c) Chromium (d) None
EBD_7167
http://t.me/iitjeehelps

C – 20 BITSAT Topicwise Solved Papers

Hints & Solutions


1. (a) Oxidising agent has metal ion in its highest O.N. of N in N2O is +1 and in NO, it is +2,
oxidation state. \ difference is 1
2. (d) 3Br2 + 6CO32 – + 3H 2 O ®
O.N. of S in SO2 is +4 and in SO3 , it is +6,
5Br – + BrO3– + 6HCO 3– \ difference is +2.
O.N. of Br2 changes from 0 to –1 and +5,
7. (a) The balanced equation is
hence it is reduced as well as oxidised.
3. (a) The more the reduction potential, the more 2MnO 4– + 5C 2 O 24 – + 16H + ®
the oxidising power.
4. (d) According to chemical bond method: 2Mn 2 + + 10CO 2 + 8H 2 O
– O O O O 8. (d) 2I - ® I 2 is oxidation (loss of electrons) ;
O – – – –
– S = O: , O – S – S – O : , O – S – S – O Cr (+6) changes to Cr (+3) by gain of
O +4 +3 electrons. Hence Cr is reduced.
+5 O O
5. (b) O.N. of Cr in K2Cr2O7 is +6. +6 - 2
9. (d) Cr O4 ( aqu.) + 2H + ¾¾®
6. (d) O.N. of N in NO2 and N2O4 is +4 +6
-2
\ difference is zero. Cr2 O 7 ( aqu.) + H 2 O
O.N. of P in P2O5 and P4O10 is +5 None of the species is lossing e–
\ difference is zero \ none is showing oxidation.
http://t.me/iitjeehelps

Chapter
Hydrogen
9
1. Identify the structure of water in the gaseous 5. In which of the following reactions, H2O2 is
phase. [2017] acting as a reducing agent [2010]
+
(a) H–O–H (b) H – O – H (a) H 2 O 2 + SO 2 ® H 2SO 4
2 d–
O 95.7 H (b) 2KI + H 2O 2 ® 2KOH + I 2
(c) pm
H
d+ 104.5° d+
H (c) PbS + 4H 2 O 2 ® PbSO 4 + 4H 2O
(d) None of these (d) Ag 2 O + H 2 O 2 ® 2Ag + H 2O + O 2
2. Which one the following removes temporary 6. 30 volume hydrogen peroxide means [2008]
hardness of water ? [2015] (a) 30% of H 2O 2 solution
(a) Slaked lime (b) Plaster of Paris
(c) Epsom (d) Hydrolith (b) 30 cm3 solution contains 1g of H 2O 2
3. The property of hydrogen which distinguishes (c) 1 cm3 of solution liberates 30 cm3 of O2 at
it from other alkali metals is [2013] STP
(a) its electropositive character (d) 30 cm3 of solution contains 1 mole of H2O2
(b) its affinity for non-metals 7. In the hydrogen peroxide molecule : [2005]
(c) its reducing character (a) O – H bonds are polar but molecule is
(d) its non-metallic character non polar.
4. The boiling point of water is exceptionally high (b) the four atoms are arranged in a non linear
because [2012] and non planar manner.
(a) there is covalent bond between H and O
(b) water molecule is linear (c) all the four atoms are in same plane.
(c) water molecules associate due to hydrogen (d) two hydrogen atoms are connected to one
bonding of the oxygen.
(d) water molecule is not linear

Hints & Solutions


2d– 3. (d) Hydrogen is non metallic in nature.
1. (c) O 95.7 pm O 4. (c) The high boiling point of water is due to
H
104.5°
H
d+
H
d+
H H-bonding.
(i) (ii) 5. (d) SO2 changes to H2SO4 (O.N. changes from
+4 to +6 oxidation)
2KI ® I2 (O.S. changes from –1 to 0
O oxidation)
d+
H
d+
H H
PbS ® PbSO 4 (O.S. changes from –2 to
H +6 oxidation)
(iii) Ag 2 O ® 2Ag (O.S. changes from +1 to 0
2. (a) This method is known as Clark's process. In
reduction)
this method temporary hardness is removed
byadding lime water or milk of lime. 6. (c) 30 vol of H2O2 means one volume of H2O2
on decom-position will give 30 volume of
Ca(OH) 2 + Ca(HCO 3 ) 2 ¾¾ ®
oxygen.
2CaCO3 ¯ +2H 2 O 7. (b) Statement (b) is correct. See structure of
ppt. H2O2.
EBD_7167
http://t.me/iitjeehelps

C – 22 BITSAT Topicwise Solved Papers

Chapter
The s-Block Elements
10
1. Dead burn plaster is [2017] 7. When a crystal of caustic soda is exposed to air,
(a) CaSO4.2H2O (b) MgSO4. 7H2O a liquid layer is deposited because : (2011)
(c) CaSO4.½ H2O (d) CaSO4 (a) Crystal loses water
(b) Crystal absorbs moisture and CO2
2. Which of the following is not present in Portland
(c) Crystal melts
cement? [2017] (d) Crystal sublimes
(a) Ca2SiO4 (b) Ca3SiO5 8. Sodium peroxide in contact with most air turns
(c) Ca3(PO4)2 (d) Ca3Al2O6 white due to formation of : [2010]
3. Lithophone is : [2015] (a) Na2CO3 (b) Na2O
(a) ZnSO4 + PbS (b) BaSO4 + ZnS (c) NaOH (d) NaHCO3
(c) PbO2 (d) ZnSO4 9. The value of x is maximum for (2009)
(a) MgSO4.x H2O (b) CaSO4.x H2O
4. Which of the following among alkali metal is
(c) BaSO4.x H2O (d) All have the same
most reactive ? [2014]
value of x.
(a) Na (b) K
10. Portland cement does not contain [2008]
(c) Rb (d) Cs
(a) CaSiO4 (b) Ca2SiO5
5. The alkali metals form salt-like hydrides by the
(c) Ca2Al2O6 (d) Ca3(PO4)2
direct synthesis at elevated temperature. The
11. In curing cement plasters water is sprinkled from
thermal stability of these hydrides decreases in time to time. This helps in [2006]
which of the following orders ? [2013] (a) developing interlocking needle-like crystals
(a) CsH > RbH > KH > NaH > LiH of hydrated silicates
(b) KH > NaH > LiH > CsH > RbH (b) hydrating sand and gravel mixed with
(c) NaH > LiH > KH > RbH > CsH cement
(d) LiH > NaH > KH > RbH > CsH (c) converting sand into silicic acid
6. Which of the following has correct increasing (d) keeping it cool
basic strength? [2012] 12. LiAlH4 is used as : [2005]
(a) MgO < BeO < CaO < BaO (a) An oxidizing agent
(b) BeO < MgO < CaO < BaO (b) A reducing agent
(c) BaO < CaO < MgO < BeO (c) A mordant
(d) CaO < BaO < BeO < MgO (d) A water softener
http://t.me/iitjeehelps

The s-Block Elements C – 23

Hints & Solutions


1. (d) CaSO4 7. (b) It is hygroscopic and deliquescent and
2. (c) There are four chief minerals present in a hence absorbs moisture and CO2 to form
Na2CO3
Portland cement tricalcium silicate
(Ca3SiO5), dicalcium silicate (Ca2SiO4), 2NaOH + CO2 ¾¾® Na2CO3 + H2O
tricalcium aluminate (Ca 3 Al 2 O6 ) and 1
calcium alumino-ferrite (Ca4AlnFe2-nO7). 8. (c) Na 2 O 2 + H 2 O ¾
¾® 2 NaOH + O2
2
3. (b) Lithophone is BaSO4 + ZnS, which is a 9. (a) Because of smaller size, Mg2+ ions are
white paint. extensively hydrated.
4. (d) Amongst alkali metals, Cs is most reactive 10. (d) Portland cement is a complex mixture of
because of its lowest IE. silicates and aluminates of Ca.
5. (d) The stability of alkali metal hydrides 11. (a) Setting of cement is exothermic process
decreases from Li to Cs. It is due to the which develops interlocking crystals of
hydrated silicates
fact that M–H bonds becomes weaker with
12. (b) LiH + AlCl3 ¾¾ ®
increase in size of alkali metals as we move
excess
down the group from Li to Cs. Thus the (AlH3)n ¾¾¾¾ ® LI[AlH4]
LiH
order of stability of hydrides is
Lithuim aluminium hydride is a most useful
LiH > NaH > KH > RbH > CsH organic reducing agent. It reduces
6. (b) The basic character of oxides increases functional groups but does not attack
down the group. double bonds.
EBD_7167
http://t.me/iitjeehelps

Chapter
The p-Block Elements
11
1. Which of the following forms the base of talcum 4. Which of the following is similar to graphite
powder? [2016] [2010]
(a) Zinc stearate (a) B (b) BN
(b) Sodium aluminium silicate (c) B2H6 (d) B4C
(c) Magnesium hydrosilicate 5. For making good quality mirrors, plates of float
(d) Chalk glass are used. These are obtained by floating
molten glass over a liquid metal which does not
2. Ordinary glass is : [2015]
solidify before glass. The metal used can be
(a) Sodium silicate
[2009]
(b) Calcium silicate
(a) tin (b) sodium
(c) Sodium and calcium silicate
(c) magnesium (d) mercury
(d) Mixed salt of Na and Ca
6. Tin plague is the [2007]
3. Which of the following is the most basic oxide?
(a) Conversion of stannous to stannic
[2015]
(b) Conversion of white tin to grey tin
(a) Sb2O3 (b) Bi2O3
(c) Emission of sound while bending a tin rod
(c) SeO2 (d) Al2O3
(d) Atmospheric oxidation of tin

Hints & Solutions


1. (c) Magnesium hydrosilicate forms base of central atom. Thus Al2O3 and Sb2O3 are
talcum powder. amphoteric and Bi2O3 is basic.
2. (c) Normal glass is calcium alkali silicate glass 4. (b) Graphite and boron nitride have similar
made by fusing the alkali metal carbonate, structure.
CaCO3 and SiO2. 5. (d) It is mercury, because mercury exists as
3. (b) More the oxidation state of the central atom liquid at room temperature.
(metal) more is its acidity. Hence SeO2 6. (b) It is the conversion of white tin to grey tin
(O. S. of Se = +4) is acidic. Further for a at low temperature which crumbles into
given O.S., the basic character of the oxides powder.
increases with the increasing size of the
http://t.me/iitjeehelps

Chapter
Organic Chemistry : Some
Basic Principles and Techniques 12
1. A compound of molecular formula of C7H16 (a) crystallisation (b) filtration
shows optical isomerism, compound will be (c) distillation (d) sublimation
[2017] 7. Which of the following compounds has wrong
(a) 2, 3-Dimethylpentane IUPAC name? [2014]
(a) CH3 – CH2 – CH2 – COO – CH2CH3
(b) 2,2-Dimethylbutane
® ethyl butanoate
(c) 3-Methylhexane
(d) None of the above (b) CH3 – CH – CH2 – CHO
|
2. Which of the following does not contain Plane ® 3-methyl-butanal
CH3
of symmetry? [2017]
(c) CH3 - CH - CH - CH3
(a) trans-1,3 dichloro cyclohexane
| |
(b) trans-1,2 dichloro cyclohexane
OH CH3
(c) cis-1,2 dichloro cyclohexane
(d) trans-1,3 cyclopentane ® 2-methyl-3-butanol
3. The correct statement about the compounds A,
O
B, and C [2017] ||
COOCH3 COOH COOH (d) CH3 - CH - C - CH 2 - CH3
|
H OH H OH H OH CH3 ® 2-methyl-3-pentanone
H OH H OH HO H
is 8. Consider the following carbocations [2014]
COOH COOCH3 COOCH3
(A) (B) (C) + +
(a) A and B are identical I. C6 H5 C H 2 II. C 6 H 5 CH 2 C H 2
+ +
(b) A and B are diastereomers III. C 6 H 5 C HCH 3 IV. C 6 H 5 C(CH 3 ) 2
(c) A and C are enantiomers
The correct sequence for the stability of these
(d) A and B are enantiomers is
4. Which of the following compounds has all the (a) II < I < III < IV (b) II < III < I < IV
four types (1°, 2°, 3° and 4°) of carbon atoms? (c) III < I < II < IV (d) IV < III < I < II
[2016] 9. Arrangement of (CH3)3 C –, (CH3)2 CH –, CH3
(a) 2, 3, 4-Trimethylpentane CH 2 – when attached to benzyl or an
(b) neo-Pentane unsaturated group in increasing order of
(c) 2, 2, 4-Trimethylpentane inductive effect is [2014]
(d) None of the three
5. Which of the following has two stereoisomers? (a) (CH3)3C – < (CH3)2 CH – < CH3 CH2–
[2016] (b) CH3CH2– < (CH3)2CH – < (CH3)3C –
CH =CH2 CH =CH 2 (c) (CH3)2CH – <(CH3)3C – < CH3CH2–
| | CH = CH2
CH3 - N +- H CH3 - N +- H
| (d) (CH3)3C – < CH3CH2 – < (CH3)2CH –
| | H3 C - N
..-H 10. The most stable carbanion among the following
CH2CH3 CH3
(I) (II) (III) is [2013]

(a) Ph – CH 2 – CH 2 (b) Ph – CH 2 –
(a) None of these (b) Only I
(c) Only III (d) I and III (c) p – CH3O Ph – CH 2–
6. The anthracene is purified by [2015] (d) p – NO 2 Ph – CH 2–
EBD_7167
http://t.me/iitjeehelps

C – 26 BITSAT Topicwise Solved Papers


11. Among the following four structures I to IV, 18. The correct nucleophilicity order is [2011]
[2013] (a) CH 3- < NH 2- < HO - < F -
CH3 O CH3 (b) CH 3- ~
- NH 2- > HO - -~ F-
| |
C 2 H 5 - CH - C3 H 7 , CH 3 - C - C H - C 2 H 5 (c) CH 3- > NH 2- > HO - > F -
(d) NH 2- > F - > HO - > CH 3-
( I) ( II )
19. In the anion HCOO– the two carbon-oxygen
CH3 bonds are found to be of equal length. what is
+ |
C H3 the reason for it ? [2011]
, C 2 H5 - CH - C2 H5
(a) The C = O bond is weaker than the C — O
(III) (IV) bond
it is true that (b) The anion HCOO– has two resonating
(a) only I and II are chiral compounds structures
(b) only III is a chiral compound (c) The anion is obtained by removal of a
(c) only II and IV are chiral compounds proton from the acid molecule
(d) all four are chiral compounds (d) Electronic orbitals of carbon atom are
12. The number of enantiomers of the compound hybridised
CH3CHBrCHBrCOOH is [2013]
20. The number of geometrical isomers of [2010]
(a) 0 (b) 1 (c) 3 (d) 4
13. The following two compounds are [2012] CH3CHCH – CH = CH – CHCHCl is
(a) 2 (b) 4 (c) 6 (d) 8
CH3 CH3
21. According to IUPAC system, the correct name
H OH OH H
of the compound having the formula [2010]
H OH OH H are
CH3 - C - CH 2 - CH 2
C2H5 C2H5 || |
(a) enantiomers (b) diastereomers CH 2 CH3
(c) identical (d) epimers
14. In paper chromatography [2012] (a) 2-ethyl-3-methyl but-1-ene
(a) Mobile phase is liquid and stationary phase (b) 2-methyl pent-3-ene
is solid (c) 2-methyl but-1-ene
(b) Mobile phase is solid and stationary phase (d) None of these
is liquid 22. Liebig’s method is used for the estimation of
(c) Both phases are liquids [2010]
(d) Both phases are solids
+ (a) N (b) S
15. In which case the NO2 will attack at the meta (c) C and H (d) Halogens
position +
[2012] 23. Hyperconjugation involves [2010]
PhCCl3 PhNO2 PhNH3 PhO– (a) s – p conjugation
(I) (II) (III) (IV) (b) s – p delocalisation
(a) I, II, III (b) II, IV (c) No bond resonance
(c) II and III only (d) II only (d) All
16. Which of the following compound is not chiral? 24. The intermediate formed during the addition of
[2011] HCl to propene in the presence of peroxide is
(a) DCH2CH2CH2Cl (b) CH3CHDCH2Cl [2009]
(c) CH3CHClCH2D (d) CH3CH2CHDCl · +
17. C 6 H 5C º N and C 6 H 5 N ®
= C exhibit which (a) CH 3 C HCH 2 CI (b) CH 2 C HCH 3
type of isomerism? [2011] · +
(a) Position (b) Functional (c) CH 3 CH 2 C H 2 (d) CH 3 CH 2 C H 2
(c) Demotropism (d) Position isomerism
http://t.me/iitjeehelps

Organic Chemistry : Some Basic Principles and Techniques C – 27


25. Which of the following has zero dipole moment? 31. The reaction : [2008]
[2009] H O
(a) 1, 1-dichloromethane (CH3)3C – Br ¾¾2¾® (CH3)3 – C –OH
(b) cis-1, 2-dichloroethene (a) elimination reaction
(c) trans-1, 2-dichloroethene (b) substitution reaction
(d) 1-chloroethane (c) free radical reaction
26. Keto-enol tautomerism is observed in [2009] (d) displacement reaction.
O O 32. The addition of HCN to a carbonyl compound is
|| || an example of [2007]
(a) C6H 5 - C- H (b) C 6 H 5 - C - CH 3 (a) nucleophilic substitution
(c) C 6 H 5 - C - C 6 H 5 (d) None (b) electrophilic addition
|| (c) nucleophilic addition
O (d) electrophilic substitution
27. The restricted rotation about carbon-carbon 33. Among the following compounds which is most
double bond in 2- butene is due to [2008] reactive towards nitration [2007]
(a) Overlap of one s- and one sp2-hybridized (a) Benzene (b) Nitrobenzene
orbitals (c) Toluene (d) Chloro benzene
(b) Overlap of two sp2-hybridized orbitals
(c) Overlap of one p-and one sp2-hybridized 34. – C-O- electrophilic
orbitals ||
(d) Sideways overlap of two p-orbitals O
28. IUPAC name of CH3CH(CH3)COOH is [2008] substitution will occur at [2007]
(a) Dimethyl acetic acid (a) meta in second ring
(b) 2 - Methylpropanoic acid (b) ortho/para in the first ring
(c) Propionic acid (c) meta in the first ring
(d) Butyric acid (d) ortho/para in the second ring
29. IUPAC name of the compound [2008]
35. Which is a pair of geometrical isomers ? [2007]
O Cl Br Cl Br
|| I. C=C II. C=C
O H Br H CH3

Cl CH3 H Br
III. C=C IV. Cl C = C
NO 2 Br H CH3
(a) I and II (b) I and III
(a) ethyl-3-methyl-2-(3-nitro) phenyl (c) II and IV (d) III and IV
butanoate 36. The IUPAC name of the compound is
(b) ethyl-2-methyl-2-(m-nitro) phenyl
propanoate
(c) ethyl-2-methyl-2-(3-nitro) phenyl [2007]
HO
propanoic acid (a) 3, 3-dimethyl - 1- cyclohexanol
(d) None of the above
(b) 1, 1-dimethyl-3-hydroxy cyclohexane
30. Example of chlorinolyis is [2008]
Cl
(c) 3, 3-dimethyl-1-hydroxy cyclohexane
(a) CH 2 = CH 2 ¾¾¾
2 ® C H Cl
2 4 2 (d) 1, 1-dimethyl-3-cyclohexanol
(b) CCl 4 + H 2 O ¾¾
® COCl2 + 2HCl 37. Which of the following compounds is isomeric
with 2, 2, 4, 4- tetramethylhexane? [2006]
(c) CHCl3 + 4NaOH
(a) 3-ethyl -2, 2- dimethylpentane
¾¾
® HCOONa + 3NaCl + 2H 2 O (b) 4-isopropylheptane
Cl (c) 4-ethyl-3-methyl-4-n-propyloctane
(d) C3 H 8 ¾¾¾
2 ® CCl + C Cl + 8HCl
4 2 6 (d) 4, 4-diethyl-3-methylheptane
EBD_7167
http://t.me/iitjeehelps

C – 28 BITSAT Topicwise Solved Papers

38. The systematic name of [2006] 41. Arrange the following in order of decreasing
basicity [2005]
CH3 - CH 2 - CH - NH 2 A. CH 3CH 2 MgBr
|
CH3 is B. HC º CMgBr
C. CH 3CH 2OMgBr
(a) 1-Methyl-1 aminopropane
(a) A > C > B (b) A > B > C
(b) 2-Butanamine
(c) C > B > A (d) B > A > C
(c) 2-Methyl-2 aminopropane 42. Th e order of decreasing stability of the
(d) None of these carbanions
39. The compound which gives the most stable (CH 3 )3 C - (I) ; (CH 3 ) 2 CH - (II) ; CH 3 CH -2 (III);
carbonium ion on dehydration : [2006]
C 6 H 5 CH 2- (IV) is [2005]
(a) 2-Methylpropanol
(a) I > II > III > IV (b) IV > III > II > I
(b) 2-Methylprop-2-ol
(c) IV > I > II > III (d) I > II > IV > III
(c) Butanol (d) But-2-ol 43. A compound with molecular formula, C7H16
40. Which of the followings is most stable ? [2006] shows optical isomerism, the compound will be
CH 3 CH 3 [2005]
| (a) 2, 3-dimethylpentane
+ (b) 2, 2-dimethylpentane
(a) H (b) (c) 2-methylhexane
+ (d) none of these
Y
44. The IUPAC name of the compound
H Y
CH 2 - CH - CH 2
CH 3 CH3 | | | is [2005]
| H |+ CN CN CN
(a) 1, 2, 3 - Tricyanopropane
(c) Y (d) (b) 3 - Cyanopentane - 1, 5 -dinitrile
+
(c) 1, 2, 3 - Cyanopropane
(d) Propane trinitrile - 1, 2, 3
H Y

Hints & Solutions


1. (a) A compound is said to exhibit optical
isomerism if it contains atleast one chiral COOH
carbon atom, which is an atom bonded to 4 H OH
Rotate through 180º
different atoms or groups. H OH ¾¾¾¾¾¾¾¾¾
®
with the plane of the paper
CH3
| COOCH3
(B)
(CH3 )2 CH - C* - CH 2 CH3
|
H COOCH3 COOCH3
2. (c) Meso compounds are characterized by an OH H H OH
internal plane of symmetry that renders HO H H OH
them achiral.
3. (d) Rotation of B through 180º within the plane COOH COOH
of the paper gives D which is an (D) (A)
enantiomer of A, hence A and B are
enantiomers
http://t.me/iitjeehelps

Organic Chemistry : Some Basic Principles and Techniques C – 29


a mobile phase. Thus, both phases are
CH3
liquids.
1 2 3 4 5 +
4. (c) H3C— C— CH2— CH— CH3 15. (a) -CCl 3 ,- NO 2 and – NH3 are

CH3 m -directing in nature.


CH3
16. (a) None of the carbon atoms in
C1 and C5 are 1°, C3 is 2°, C4 is 3°, and C2 is DCH2CH2CH2Cl is chiral i.e., each carbon
4°. atom is achiral (symmetric).
5. (b) A rapid umbrella type inversion rapidly
17. (b)
converts the structure III to its enantiomer;
18. (c) Nucleophilicity increases with th e
hence the two enantiomers are not
decrease in electronegativity of the central
separable.
atom. Since electronegativity follows the
6. (d) Anthracene is purified by sublimation. In
order: F > O > N > C; nucleophilicity of the
sublimation, a solid is converted directly
into gaseous state on heating without concerned group will follow the reverse
passing through liquid phase. order i.e., CH 3 - > NH 2 - > OH - > F -
7. (c)
O O
+ + || |
8. (a) I. C 6 H 5 C H 2 II. C 6 H 5 CH 2 C H 2 19. (b) H – C– O H – C=O
1º Benzylic 1º
+ +
20. (d) The given structure has three double
III. C 6 H 5 C H CH 3 IV. C 6 H 5 C (CH 3 ) 2 bonds whose each carbon atom is
2º benzylic 3º benzylic differently substituted hence number of
geometrical isomers will be 2n = 23’ = 8,
3º Benzylic (IV) > 2º Benzylic (III) > 1º
where n is the number of double bonds
Benzylic(I) > 1º (II)
whose each carbon atom is differently
9. (b) –CH3 group has +I effect, as number of –
CH3 group increases, the inductive effect substituted.
increases. CH3
10. (d) –NO 2 group, being strong electron- |
withdrawing, disperses the –ve charge, 21. (d) H 2 C = C - CH 2 - CH 2 - CH3
1 2 3 4 5
hence stabilizes the concerned carbanion. 2 - methylpent -1- ene
11. (a) A chiral object or structure has four
different groups attached to the 22. (c) 23. (a)
carbocation. 24. (b) The addition of HCl to propene proceeds
* * by ionic mechanism and not by free radical
12. (d) The structure CH 3 C H Br C H Br COOH mechanism. Hence it forms intermediate
has two different chiral carbon atoms, carbonium ion.
hence number of enantiomers (optically 25. (c)
active forms) is 2n = 22 = 4
O OH
13. (b) The given two structures are optical
isomers but as these are not mirror image 26. (b) C6H5 – C – CH3 C6H5 – C = CH2
of each other, hence they represent Keto enol
diastereomers of each other. 27. (d)
14. (c) Paper chromatography is a special case of 28. (b) The first carbon will be with the functional
partition chromatography where the special group. The CH3 group is attached on the
quality paper containing water trapped in second carbon.
it acts as a stationary phase and solvent as
EBD_7167
http://t.me/iitjeehelps

C – 30 BITSAT Topicwise Solved Papers


29. (a) It is an ethyl ester of butanoic acid. 38. (b) In this compound, the amine group is on
the second carbon so its systematic name
CH 3 O becomes 2-Butanamine.
| ||
CH 3 - CH - CH - C - O - C 2 H 5 39. (b) (CH 3 ) 3 COH gives tert ( CH 3 ) 3 C + . The
4 3 2 1
order of stability of carbonium ions is
6 1 2 +
3 3° > 2° > 1° > CH 3
5 NO 2 CH3
4
|+
ethyl-3-methyl-2-(3-nitro) phenyl
butanoate 40. (d) The structure carring +ve charge
30. (d) Chlorinolysis is substitution by chlorine
31. (b) The reaction is SN reaction.
H Y
32. (c) nearest to electron repelling -CH 3 group
33. (c) Due to hyperconjugation toluene is most is most stable.
reactive for electrophilic substitution -
reaction. 41. (b) CH 3C H 2 > HC º C - > CH 3CH 2O -
34. (d) The second ring has o/p directing O–atom basic character. The acid character being
and first ring has m-directing - C - group. CH 3CH 2OH > HC º CH > CH 3CH 3 .
|| The weaker the acid, the stronger the base
O
(conjugate pair).
o/p directing group activate the benzene 42. (b) C6H5CH2– > CH3CH2–
nucleus. > (CH3)2 CH– > (CH3)3C–
35. (c) Dispersal of Intensification of
36. (a) –ve charge –ve charge
due to resonance due to + I effect
1 3 and - I effect. of CH3 gps.
2
HO
43. (a) 2, 3-dimethylpentane due to presence of
two chiral carbon atom shows optical
3, 3-Dimethyl -1- cyclohexanol
isomerism.
37. (b) 2, 2, 4, 4 - Tetramethylhexane has 10 carbon
atoms, only 4-isopropylheptane has also CH3 CH3
10 carbon atoms so these two are isomers. | |
CH3 – CH – CH – CH2 – CH3
CH3 1 2 3 4 5
|
4
CH – CH3 44. (b) CH 2 - 3 CH - 2CH 2
| | | |
CH3 – CH2 – CH2 – CH – CH2 – CH2 – CH3 5
CN CN 1
CN
4-isopropylheptane 3 - Cyanopentane - 1, 5 -dinitrile
http://t.me/iitjeehelps

Hydrocarbons C – 31

Chapter
Hydrocarbons
13
1. In the following structures, which two forms
are staggered conformations of ethane ? [2017] D
SO3H
D D
H
H H H D
H (c) (d)
H D D

H H H D
H 4. An organic compound A (C4H9Cl) on reaction
H H
(1) with Na/diethyl ether gives a hydrocarbon which
(2)
on monochlorination gives only one chloro
H derivative, then A is [2014]
H (a) tert-butyl chloride
H
(b) sec-butyl chloride
H H (c) isobutyl chloride
H H
H H
H H H (d) n-butyl chloride
(3)
(4) 5. Which one of the following reactions is
(a) 1 and 4 (b) 2 and 3 expected to readily give a hydrocarbon product
(c) 1 and 2 (d) 1 and 3 in good yields ? [2013]
H 2 /Pt D /Pt Electrolyt ic
2. CH3C º CCH3 ¾¾¾¾ ® A ¾¾¾¾
2 ®B (a) RCOOK ¾¾ ¾ ¾¾®
oxidation
The compounds A and B, respectively are
[2016]
RCOO - Ag + ¾¾
(b) Br
¾2®
(a) cis-but-2ene and rac-2, 3-dideuterobutane
(b) trans-but-2ene an d rac-2, 3- Cl
dideuterobutane (c) CH 3 CH 3 ¾¾
¾2®
hu
(c) cis-but-2ene an d meso-2, 3-
dideuterobutane (d) C H OH
(CH 3 ) 3 CCl ¾¾2 ¾
5 ¾®
(d) trans-but-2ene an d meso-2, 3-
dideuterobutane 6. Which of the following chemical system is non
3. Give the possible structure of X in the following aromatic? [2013]
reaction : [2016]
C6H6 + D2SO4 ¾¾¾
2 ®
X
D O (a) (b)

SO3D SO3D
D (c) (d)
(a) (b) S
EBD_7167
http://t.me/iitjeehelps

C – 32 BITSAT Topicwise Solved Papers


7. Which alkene on ozonolysis gives CH3CH2CHO 11. Which one of the following contain isopropyl
and CH3CCH3 [2012] group? [2009]
|| (a) 2, 2, 3, 3-tetramethylpentane
O
(b) 2-methylpentane
CH3 (c) 2, 2, 3-trimethylpentane
(a) CH3CH2CH = C
CH3 (d) 3, 3-dimethylpentane
12. Compound which has one quaternary carbon
(b) CH3CH2CH = CHCH2CH3
(c) CH3 CH2 CH = CH CH3 atom is : [2008]
(a) (CH3)2 – C – (CH3)2
(d) CH 3 - C = CHCH 3
| (b) (CH 3 ) 2 - C = C - C - (CH 3 ) 2
CH3 |

8. What will be the product in the following H


reaction? (c) (CH3)3 CH
CH2 (d) CH3 CH2 – (CH3)CH – CH3
NBS 13. Methane and ethane both can be prepared in
[2011]
one step by which of the following compound?
Br [2007]
CH2 (a) C2H4 (b) CH3OH
CH3
(a) (b) (c) CH3Br (d) CH3CH2OH
Br 14. The most suitable catalyst for the
hydrogenation of [2007]
CH3
2-Hexyne ¾ ¾® 2-cis-Hexene is
CH2Br
(a) Pd–BaSO4 (b) (Ph3P)3RhCl
(c) (d) (c) 10% Pd—C (d) Raney Ni
Br 15. In preparation of alkene from alcohol using
9. Name of following reaction is [2010] Al2O3 which is effective factor ? [2006]
(a) Porosity of Al2O3
+ || ¾
¾® (b) Temperature
CN CN (c) Concentration
(a) Claisen Condensation (d) Surface area of Al2O3
(b) Diel’s Alder reaction 16. Marsh gas contains [2006]
(c) Dieckmann cyclisation (a) CO2 (b) CH4
(d) Michael addition reaction
(c) HCl (d) C2H2
10. The unsaturated hydrocarbon which on
ozonolysis gives one mole each of formaldehyde, 17. The reaction [2005]
acetaldehyde and methyl glyoxal (CH3COCHO) AlCl
C 6 H 6 + CH 3Cl ¾¾¾3 ® HCl + C 6 H 5CH 3
is [2010]
is called
(a) CH 3 – CH = C( CH 3 ) – CH 3
(a) addition reaction
(b) CH 2 = CH – CH 2 – CH = CH 2 (b) Friedel-craft acylation
(c) CH 2 = CH – C( CH 3 ) = CH – CH 3 (c) Friedel-craft alkylation
(d) Friedel-craft benzoylation
(d) (CH 3 ) 2 C = CH - CH 3
http://t.me/iitjeehelps

Hydrocarbons C – 33

Hints & Solutions


1. (c) Note that in structures 1 and 2, every two
CH 2 CH 2 Br
adjacent hydrogen atoms are at maximum NBS
possible distance from each other 8. (c) ¾¾¾
®
(staggered conformation).
2. (c) Catalytic hydrogenation of alkynes gives 9. (b) The given reaction is Diel’s Alder reaction.
cis-alkene which in turn adds deuterium 10. (c) CH 2 = CH - C(CH 3 ) = CH - CH 3
atoms in presence of H2 again in cis-manner O
forming meso-2, 3-dideuterobutane. ¾¾®
3 HCHO + CH3CHO + CH3COCHO
ZnO/H 2O
3. (d) D2SO4 transfers D+, an electrophile, to form
hexadeuterobenzene. 11. (b) CH 3
CH3 CH3
4. (a) CH -
CH3 – C – Cl + 2Na + Cl – C – CH3
CH 3
CH3 CH3 Isopropyl group
t-Butyl chloride
Wurtz

CH3
Rxn

|
CH3 CH3 CH3 CH3 CH3 - CH - CH 2 - CH 2 CH3
CH3 – C – C –CH2Cl CH – C – C – CH3 (2-methyl pentane)
Mono 3
It contains isopropyl group.
CH3CH3 Chlorination CH3 CH3
12. (a) It has quaternary carbon because central
5. (a) Electrolysis of a concentrated aqueous carbon atom is surrounded by 4 - carbon
solution of either sodium or potassium salts atoms.
CH 3
of saturated carboxylic acids yields higher |
alkane at anode. H 3 C - C - CH 3
|
2RCOOK ¾¾ ¾ ¾¾® 2RCOO - + 2K +
Electrolyt ic
CH 3
Oxidation Anode Cathode
2, 2-dimethyl propane
-
At anode 2RCOO ® 2RCOO + 2e – ¾
¾® 13. (c) CH3Br can form both methane and ethane
R — R + 2CO 2 in one step
+ –
At Cathode 2K + 2e ® 2K 2 H ( Zn + HCl )
CH 3 Br ¾¾ ¾ ¾ ¾
¾® CH 4 + HBr
2K + H 2 O ® 2KOH + H 2 ­
Wurtz Re action
CH 3 Br + Na + BrCH 3 ¾¾¾¾¾¾
¾®
(Kolbe's Method)
6. (a) Huckel rule is not obeyed. It has only four CH 3 - CH 3
electrons. Further it does not have 14. (a) Reduction of alkynes with Lindlar ’s
continous conjugation. catalyst (Pd-BaSO4) gives cis-alkenes.
7. (a) 15. (d) The amount of alcohol absorbed depends
H CH3 H O CH3 upon the surface area of Al2O3.
| | O 16. (b) Methane (CH4) is called as marsh gas.
3
CH 3 – CH2– C = C CH3 – CH2– C C
| | | 17. (c) When benzene react with alkyl chloride in
CH O–O CH3 the presence of anhydrous AlCl 3, then
O 3
|| methyl group is substituted in benzene ring.
CH3– C – CH3+ CH3CH2CHO This reaction is called as Friedel-Craft's
(–H2O)
alkylation.
EBD_7167
http://t.me/iitjeehelps

C – 34 BITSAT Topicwise Solved Papers

Chapter
Environmental Chemistry
14
1. Which of the following is not involved in the (a) Ozone layer is beneficial to us because
formation of photochemical smog? (2017, 2005) ozone cuts out the ultraviolet radiation of
(a) Hydrocarbon (b) NO the sun
(c) SO2 (d) O3 (b) The conversion of ozone to oxygen is an
2. Which of the following pollutants is main endothermic reaction
product of automobiles exhaust? [2015] (c) Ozone has a triatomic linear molecule
(a) C O (b) CO2 (d) None of the above
(c) NO (d) Hydrocarbons 10. Which of the following statements about polar
3. The disease caused the high concentration of stratosphere clouds (PSCs) is not correct?
hydrocarbon pollutants in atmosphere is/are [2011]
[2015] (a) PSCs do not react with chlorine nitrate and
(a) silicosis (b) TB HCl
(c) cancer (d) asthma (b) Type I clouds are formed at about –77ºC
4. Identify the wrong statement in the following: and contain solid HNO3 . 3H2O
[2014] (c) Type II clouds are formed at about –85ºC
(a) Chlorofluorocarbons are responsible for and contain some ice
ozone layer depletion (d) A tight whirlpool of wind called Polar Vortex
(b) Greenhouse effect is responsible for global is formed which surrounds Antarctica
warming 11. Minamata disease is due to pollution of[2010]
(c) Acid rain is mostly because of oxides of (a) Aresenic into the atmosphere
nitrogen and sulphur (b) Organic waste into drinking water
(d) Ozone layer does not permit infrared (c) Oil spill in water
radiation from the sun to reach the earth
5. When rain is accompanied by a thunderstorm, (d) Industrial waste mercury into fishing water
the collected rain water will have a pH value 12. Phosphate pollution is caused by [2010]
[2014] (a) Sewage and agricultural fertilizers
(a) Slightly lower than that of rain water (b) Weathering of phosphate rocks only
without thunderstorm (c) Agriculutral fertilizers only
(b) Slightly higher th an that when the (d) Phosphate rocks and sewage
thunderstorm is not there 13. Eutrophication causes reduction in [2010]
(c) Uninfluenced by occurrence of (a) Dissolved oxygen (b) Nutrients
thunderstorm (c) Dissolved salts (d) All the above
(d) Which depends upon the amount of dust 14. The statement which is not correct about
in air control of particulate pollution [2009]
6. The greenhouse effect is because of the (a) In electrostatic precipitator, the
(a) presence of gases, which in general are particulates are made to acquire positive
strong infrared absorbers, in the charge which are then attracted by the
atmosphere negative electrode and removed
(b) presence of CO2 only in the atmosphere (b) Gravity settling chamber removes larger
(c) presence of O3 and CH4 in the atmosphere particles from the air
(d) N2O and chlorofluorohydrocarbons in the (c) Cyclone collector removes fine particls in
atmosphere [2013] the diameter range 5-20 microns
7. Formation of ozone in the upper atmosphere (d) Wet scrubbers are used to wash away all
from oxygen takes place by the action of [2012] types of particulates
(a) Nitrogen oxides (b) Ultraviolet rays 15. Chief source of soil and water pollution is [2009]
(c) Cosmic rays (d) Free radicals (a) Mining
8. Which is the major air pollutant [2012] (b) Agro industry
(a) He (b) O2 (c) CO2 (d) CO
9. Which one of the following statements about (c) Thermal power plant
ozone and ozone layer is true [2011] (d) All the above
http://t.me/iitjeehelps

Environmental Chemistry C – 35
16. The false statement among the followings : 21. Presence of which fuel gas in the exhaust fumes
[2009] shows incomplete combustion of fuel. [2007]
(a) The average residence time of NO is one (a) Sulphur dioxide
month (b) Carbon monoxide and water vapour
(b) Limestone acts as a sink for SOx (c) Carbon monoxide
(c) SOx can be removed from flue gases by (d) Nitrogen dioxide
passing through a solution of citrate ions 22. The substance having the largest concentration
(d) Ammonia acts as a sink for NOx in acid rain? [2007]
17. Which of the following acts as a sink for CO? (a) H2CO3 (b) HNO3
[2008] (c) HCl (d) H2SO4
(a) Plants 23. The greatest affinity for haemoglobin is shown
(b) Haemoglobin by which of the following : [2006]
(c) Microorganisms present in the soil (a) NO (b) CO
(d) Oceans (c) O2 (d) CO2
18. Sewage mostly constitutes [2008] 24. Which of the following is not a consequence of
(a) Non-biodegradable pollutants greenhouse effect? [2006]
(b) Biodegradable pollutants (a) Climatic conditions will be changed
(c) Effluents resulting in
(d) Air pollutants (b) Plants in warmer climates with adequate
19. BOD is connected with [2008] rainfall would grow faster
(a) Microbes and organic matter (c) The incidence of infectious diseases is
(b) Organic matter likely to increase
(c) Microbes (d) Malaria will be controlled as the
(d) None of the above mosquitoes will not survive.
20. The type of pollution caused by spraying of 25. The substance which is not regarded as a
DDT [2007] pollutant? [2005]
(a) Air and soil (b) Air and water (a) NO2 (b) CO2
(c) Air (d) Air, water and soil (c) O3 (d) Hydrocarbons

Hints & Solutions


1. (c) Photochemical smog does not involve 9. (a) Statement (a) is correct.
SO2. 10. (a) PSCs react with chlorine nitrate and HCl
2. (c) NO pollutant is the main product of to give HOCl and Cl2.
automobiles exhaust. 11. (d) Minamata is caused by Hg poisoning.
3. (c) The high concentration of hydrocarbon 12. (a) Phosphate pollution is caused by sewage
pollutants in atmosphere causes cancer. and agricultural fertilizers.
4. (d) Ozone layer acts as a shield and does not 13. (a) Eutrophication causes reduction in D.O
allow ultraviolet radiation from sun to reach 14. (a) Particulates acquire negative charge and
earth. It does not prevent infra-red are attracted by the positive electrode.
radiation from sun to reach earth, thus 15. (d) is correct
option (d) is wrong statement and so it is
the correct answer. 16. (a) The average residence time of NO is 4
5. (a) Normal rain water has pH 5.6 Thunderstorm days.
results in the formation NO and HNO3 17. (c) CO is converted into CO 2 by
microorganism present in soil.
which lowers the pH. 18. (c) Domestic sewage constitute
6. (a) Green house gases such as CO2, ozone, biodegradable pollutants.
methane, th e chlorofluoro carbon 19. (a) BOD is connected with microbes and
compounds and water vapour form a thick organic matter
20. (d) DDT causes water, air and soil pollution.
cover around the earth which prevents the 21. (c) Prsence of CO in the exhaust fumes shows
IR rays emitted by the earth to escape. It incomplete combustion.
gradually leads to increase in temperature 22. (d) Acid rain contains H2SO4 > HNO3> HCl.
of atmosphere. 23. (b) Haemoglobin has great affinity for CO.
7. (b) In presence of U.V. rays O2 is converted 24. (d) The mosquitoes will increase their
into O3. population and spread malaria.
8. (d) CO is major air pollutant. 25. (b) CO2 is generally not as regarded as an
pollutant.
EBD_7167
http://t.me/iitjeehelps

Chapter
The Solid State
15
1. The number of atoms per unit cell of bcc (a) decreases (b) increases
structure is [2017] (c) does not change (d) changes
(a) 1 (b) 2 (c) 4 (d) 6 6. The fraction of total volume occupied by the
2. Al (at. wt 27) crystallizes in the cubic system atoms present in a simple cube is [2011]
with a cell edge of 4.05 Å . Its density is 2.7 g per p p
cm3. Determine the unit cell type calculate the (a) (b)
3 2 4 2
radius of the Al atom [2016]
p p
(a) fcc, 2.432 Å (b) bcc, 2.432 Å (c) (d)
(c) bcc, 1.432 Å (d) fcc, 1.432 Å 4 6
7. The atomic radius of atom is r.Total volume of
3. The non stoichiometric compound Fe 0.94O is
atoms present in a fcc unit cell of an element is
formed when x % of Fe 2 + ions are replaced by [2009]
24 3 12 3
as many 2 Fe 3 + ions, x is [2016] (a) pr (b) p r
3 3 3
(a) 18 (b) 12 (c) 15 (d) 6 16 3
4. Graphite is a [2015] (c) pr (d) None
3
(a) molecular solid (b) covalent solid
8. Which of the folliowing metal oxides is anti-
(c) ionic solid (d) metallic solid
ferromagnetic in nature? [2006]
5. Due to Frenkel defect, the density of ionic solids
(a) MnO2 (b) TiO2
[2013] (c) VO2 (d) CrO2

Hints & Solutions


1. (b) In bcc structure, But x/3 = 1 – 0.94 = 0.06,
no. of atoms at corner = 1/8 × 8 = 1 x = 0.06 × 3 = 0.18 = 18%
no. of atom at body centre = 1 4. (b) Graphite is covalent solid.
therefore, total no of atom per unit cell = 2. 5. (c) No change in density.
Z´ M 6. (d) Number of atoms per unit cell = 1
2. (d) r = , Atoms touch each other along edges.
No ´ a3
Z ´ 27 a
2.7 = Hence r =
6.02 ´ 10 ´ ( 4.05) 3 ´ 10 -24
23 2
( r = radius of atom and a = edge length)
\ Z=4
Hence it is face centred cubic unit lattice. 4 3
pr
p
Again 4r = a 2 = 5.727 Å Therefore % fraction = 3 3 =
(2r) 6
\ r = 1.432 Å
7. (c) 4 atom are present in fcc.
3. (a) The number of Fe 3+ ions replacing x Fe2+
é4 ù 16
ions = 2 x vacancies of cations So, V = 4 ê pr3 ú = pr 3
3 ë3 û 3
2x 8. (c) MnO2.
=x– = x /3
3
http://t.me/iitjeehelps

Chapter
Solutions
16
1. Raoult’s law becomes a special case of Henry’s 7. Coolant used in car radiator is aqueous solution
law when [2017] of ethylene glycol. In order to prevent the
(a) K H < p1° (b) K H = p1° solution from freezing at – 0.3°C. How much
ethylene glycol must be added to 5 kg of water ?
(c) K H ; p° (d) K H ³ p° (Kf = 1.86 K Kg mol–1) [2010]
1 1
(a) 50 g (b) 55 g (c) 45 g (d) 40 g
2. Identify the correct order of solubilty of Na 2S. 8. Which one of the following statements is FALSE?
CuS and ZnS in aqueous medium [2014] [2009]
(a) CuS > ZnS > Na2S (a) The correct order of osmotic pressure for
(b) ZnS > Na2S > CuS 0.01 M aqueous solution of each
(c) Na2S > CuS > ZnS compound is
(d) Na2S > ZnS > CuS
BaCl 2 > KCl > CH 3COOH > sucrose
3. Equal weights of NaCl and KCl are dissolved
separately in equal volumes of solutions. (b) The osmotic pressure (p) of a solution is
Molarity of the two solutions will be : [2013] given by the equation p = MRT, where M is
(a) Equal the molarity of the solution
(b) That of NaCl will be less than that of KCl (c) Raoult’s law states that the vapour pressure
(c) That of NaCl will be more than that of KCl of a component over a solution is
solution proportional to its mole fraction
(d) That of NaCl will be about half of that of (d) Two sucrose solutions of same molality
KCl solution prepared in different solvents will have the
4. The van’t Hoff factor i for an electrolyte which same freezing point depression
undergoes dissociation and association in 9. The degree of dissociation of Ca ( NO 3 ) 2 in
solvents are respectively [2012]
dilute aq. solution containing 7.0 g of salt per
(a) greater than 1 and greater than 1
100 g of water at 100° C is 70%. If vapour pressure
(b) less than 1 and greater than 1
of water at 100° C is 760 mm. The vapour
(c) less than 1 and less than 1
pressure of solution is [2009]
(d) greater than 1 and less than 1
(a) 735 (b) 730 (c) 760 (d) 746
5. Which of the following metal sulphides has
10. The volume of water added to 500 ml 0.5 M NaOH
maximum solubility in water? [2012]
so that its strength becomes 10 mg NaOH per ml
(a) HgS Ksp = 10–54 (b) CdS Ksp = 10–30
is : [2008]
(c) FeS Ksp = 10–20 (d) ZnS Ksp = 10–22
(a) 100 ml (b) 200 ml
6. 1.00 g of a non-electrolyte solute (molar mass
(c) 250 ml (d) 500 ml
250 g mol–1) was dissolved in 51.2 g of benzene.
11. In a 0.2 molal aqueous solution of a weak acid
If the freezing point depression constant, Kf of
HX the degree of ionization is 0.3. Taking kf for
benzene is 5.12 K kg mol–1, the freezing point of
water as 1.85, the freezing point of the solution
benzene will be lowered by [2011]
will be nearest to [2008]
(a) 0.3 K (b) 0.5 K (a) – 0.360º C (b) – 0.260º C
(c) 0.4 K (d) 0.2 (c) + 0.480º C (d) – 0.480º C
EBD_7167
http://t.me/iitjeehelps

C – 38 BITSAT Topicwise Solved Papers

12. The milliequivalent in 60 ml 4M H2SO4 is : [2007] (a) 0.1M NaCl (b) 0.1M KCl
(a) 240 (b) 480 (c) 0.1M CaCl2 (d) 0.1M glucose
(c) 24 (d) 48 16. An X molal solution in carbon tetrachloride has
13. The heat of vaporisation of benzene is 7353 cal the mole fraction of solute equal to 0.23527 . The
mol–1. The approximate bpt. of benzene is[2007] value of X is [2005]
(a) 77.1° C (b) 87.1° C (a) 1.55 (b) 1.82
(c) 101° C (d) 95° C (c) 2.00 (d) 2.16
14. Vapour pressure of chloroform (CHCl 3 ) 17. Which of the following solutions will have the
and dichloromethane (CH2Cl 2) at 25ºC are highest boiling point? [2005]
200 mm Hg and 41.5 mm Hg respectively. Vapour (a) 0.5 molal BaCl2
pressure of the solution obtained by mixing (b) 1.0 molal KBr
25.5 g of CHCl3 and 40 g of CH2Cl2 at the same (c) 1.8 × 1024 glucose molecules per litre.
temperature will be : (Molecular mass of CHCl3 (d) 100 g powdered glucose in one litre water
= 119.5 u and molecular mass of CH2Cl2 = 85 u). 18. To neutralise completely 20 mL of 0.1 M aqueous
[2007] solution of phosphorous acid (H3PO3), the value
(a) 173.9 mm Hg (b) 615.0 mm Hg of 0.1 M aqueous KOH solution required is
(c) 347.9 mm Hg (d) 285.5 mm Hg [2005]
15. Which of the following solution will have lowest (a) 40 mL (b) 20 mL
boiling point [2006] (c) 10 mL (d) 60 mL

Hints & Solutions


1. (a) Raoult’s law becomes special case of
1.86 ´ WB ´ 1000
Henry’s law when KH become equal to p °. 0.3 =
1 62 ´ 5000
2. (d) The correct order of solublity of sulphides
\ WB = 50 g
is Na2S > ZnS > CuS
3. (c) When equal weights of different solutes The amount used should be more than
are present in equal volumes of solution 50 g.
the molarity is inversely related to 8. (d) DTf = K f ´ m ´ i . Since Kf has different
molecular mass of the solute. Mol. mass of values for different solvents, hence even
NaCl is less than KCl. Hence, molarity of
NaCl solution will be more. if the m is the same DTf will be different
4. (d) When an electrolyte dissociates van’t Hoff 9. (d) Ca ( NO 3 ) 2 Ca + + + 2 NO - 3
factor i is greater than 1 and when it
associates the i is less than 1. 1 – 0.71 0.7 2 × 0.7
5. (c) The higher the value of Ksp, the more is 70
(Q a = = 0.7)
the solublity. 100
1 1000 \ i = 1 – 0.7 + 0.7 + 2 × 0.7 = 2.4
6. (c) DT = K f m = 5.12 ´ ´ = 0.4K
250 51.2 7
n1 = = 0.042
7. (b) DTf = 0.3° C 164
100
K f ´ WB ´ 1000 n2 = = 5.55
DTf = 18
M B ´ WA
http://t.me/iitjeehelps

Solutions C – 39
p0 - ps n 2 ´ i 760 - p s
25.5
= , 14. (N) n CHCl3 = = 0.213
p 0 n1 + n 2 760 119.5
0.042 ´ 2.4 40
= n CH 2Cl2 = = 0.47
5.55 + 0.042 85
\ ps = 746 mm PT = PA° X A + PB° X B
10. (d) 1 ml has 10 mg NaOH
1000 ml will have = 10 × 1000 mg = 10 gm 0.213 0.47
= 200 ´ + 41.5 ´
NaOH 0.683 0.683
gm / litre 10 = 62.37 + 28.55
Normality = = N = 0.25 N
eq.wt. 40 = 90.92
Now using normality equation, 15. (d) Elevation in boiling point is a colligative
property. Smaller the no. of solute particles
N1V1 = N 2 V2 lesser will be elevation in boiling point.
0.5 ´ 500 = 0.25 ´ V2 1000X 2
16. (c) Use the formula m = ;
(Q 0.5M NaOH º 0.5N NaOH ) X1M1
Given
0.5 ´ 500 X 2 = 0.23527, X1 = (1 - 0.23527) = 0.76473
V2 = = 1000 ml
0.25 Molecular weight of CCl4 = 153.82,
Volume of water added = 1000 – 500 = 500 ml 1000 ´ 0.23527
m= = 2.00
+ - 0.76473 ´ 153.82
11. (d) ( HX H + X , i = 1.3)
1- 0.3 0. 3 0.3 17. (c) For electrolytes multiply the concentration
by vant Hoff’s factor i
i = 1 – 0.3 + 0.3 + 0.3 = 1.3
(a) 0.5 × 3 = 1.5 for BaCl2 ;
DTf = Kf × m × i ; (b) 1.0 × 2 = 2 for KBr,
DTf = 1.85 × 0.2 × 1.3 = 0.480º C
1.8 ´ 10 24
\ Tf = 0 – 0.480 = – 0.480°C. (c) = 2.98 for glucose
6.023 ´ 10 23
12. (b) 4M H 2SO4 º 8N H 2SO 4
100
Number of milli equivalents (d) = 0.55 for glucose
= normality × volume in ml = 8 × 60 = 480 180
13. (a) Trouton’s rule is Hence highest bpt will be observed in case
of (c).
D H evp -1 18. (a) N1V1 = N2V2 (H3PO3 is dibasic
= 21 Cal. K mol -1
Tb
\ M = 2N)
7353 20 ´ 0.2 = 0.1´ V \ V = 40 ml
Tb = = 350.1 K = 77.1° C
21
EBD_7167
http://t.me/iitjeehelps

C – 40 BITSAT Topicwise Solved Papers

Chapter
Electrochemistry
17
1. Stronger is oxidising agent, more is [2017]
5. The EMF of the cell Tl/Tl+ (0.001M) || Cu2+
(a) standard reduction potential of that species (0.01M) /Cu is 0.83. The cell EMF can be
(b) the tendency to get it self oxidised increased by [2016]
(c) the tendency to lose electrons by that +
(a) Increasing the concentration of Tl ions.
species
(b) Increasing the concentration of Cu2+ ions.
(d) standard oxidation potential of that species
(c) Increasing the concentration of Tl + and
2. Which of the following relation represents
Cu2+ ions.
correct relation between standard electrode
potential and equilibrium constant? [2017] (d) None of these
6. Electrolysis is carried out in three cells [2016]
nFE°
I. log K = (A) 1.0 M CuSO 4 Pt electrode
2.303 RT
nFE ° (B) 1.0 M CuSO 4 copper electrodes
II. K = e RT (C) 1.0 M KCl Pt electrodes
-nFE° If volume of electrolytic solution is maintained
III. log K = constant in each of the cell, which is correct set
2.303 RT
of pH changes in (A), (B) and (C) cell
nFE° respectively ?
IV. log K = 0.4342
RT (a) decrease in all the three
Choose the correct statement(s). (b) increase in all the three
(a) I, II and III are correct (c) decrease, constant, increase
(b) II and III are correct (d) increase, constant, increase
(c) I, II and IV are correct 7. The equilibrium constant for the
(d) I and IV are correct disproportionation reaction [2016]
3. E° for the cell, 2Cu + (aq) ¾ 2+
¾® Cu (s) + Cu (aq)
Zn | Zn 2+ (aq) | | Cu 2+ (aq)| Cu is 1.10 V at at 25°C
25°C. The equilibrium constant for the cell (Eo Cu + / Cu = 0.52V, E o Cu 2+ / Cu = 0.16V) is
reaction [2017] (a) 6 ´ 10 4 (d) 6 ´ 10 6
ˆˆ† Cu + Zn 2+ (aq)
Zn + Cu 2+ (aq) ‡ˆˆ (c) 1 .2 ´ 10 6 (d) 1.2 ´ 10 -6
is of the order of 8. The efficiency of fuel cell is given by the
(a) 10–37 (b) 1037 expression, h is [2015]
(c) 10 –17 (d) 1017 nFE cell
4. Specific conductance of 0.1 M HA is (a) h=- ´ 100
DH
3.75 × 10–4 ohm–1 cm–1. If
nFE cell
l¥ (HA ) = 250 ohm -1 cm 2 mol -1 , the (b) h=- ´ 100
DS
dissociation constant Ka of HA is : [2017]
nFE cell
(a) 1.0 × 10 –5 (b) 2.25 × 10 –4
(c) h=- ´ 100
DA
(c) 2.25 × 10–5 (d) 2.25 × 10–13
(d) None of the above
http://t.me/iitjeehelps

Electrochemistry C – 41
9. The mass of the substance deposited when one Sn (s) + 2Fe 3+ (aq) ® 2Fe 2+ (aq) + Sn 2+ (aq) is
Faraday of charge is passed through its solution
is equal to [2015] [2012]
(a) relative equivalent weight (a) 0.91 V (b) 1.40 V
(b) gram equivalent weight (c) 1.68 V (d) 0.63 V
(c) specific equivalent weight 16. The number of coulombs required for the
(d) None of the above deposition of 108 g of silver is [2011]
10. In the cell reaction (a) 96500 (b) 48250
Cu(s) + 2Ag + (aq) ¾¾
® Cu 2 + (aq) + 2Ag(s) , (c) 193000 (d) 10000
17. Which of the following will form the cathode
E0cell = 0.46 V. By doubling the concentration of with respect to iron anode in an electrolyte cell?
Cu2+, E0cell is [2014] [2010]
(a) doubled (b) halved (a) Mg (b) Al (c) Cu (d) Zn
(c) increases but less than double 18. When the sample of copper with zinc impurity is
(d) decreases by a small fraction to be purified by electrolysis, the appropriate
11. Cuaq+ is unstable in solution and undergoes electrodes are [2009]
simultaneous oxidation and reduction according Cathode Anode
to the reaction : [2014] (a) Pure zinc Pure copper
+ ˆˆ† Cu (aq) + Cu(s)
2Cu (aq) ‡ˆˆ 2 + (b) Impure sample Pure copper
choose correct Eº for above reaction if (c) Impure zinc Impure sample
Eº Cu2+/Cu = 0.34 V and Eº Cu2+/Cu+ = 0.15 V (d) Pure copper Impure sample
(a) –0.38 V (b) +0.49 V 19. The conductivity of a saturated solution of
(c) +0.38 V (d) –0.19 V BaSO4 is 3.06 × 10–6 ohm –1 cm –1 and its
12. A current of 2.0 A passed for 5 hours through a equivalent conductance is 1.53 ohm –1 cm 2
molten metal salt deposits 22.2 g of metal (At wt. equiv–1. The Ksp for BaSO4 will be [2009]
= 177). The oxidation state of the metal in the (a) 4 × 10–12 (b) 2.5 × 10–9
metal salt is [2013] (c) 2.5 × 10–13 (d) 4 × 10–6
(a) +1 (b) +2 (c) +3 (d) +4 20. In a cell that utilises the reaction [2009]
13. The electrolytic cells, one containing acidified
ferrous chloride and another acidified ferric Zn(s) + 2H + (aq) ® Zn 2 + (aq) + H 2 (g)
chloride are connected in series. The ratio of addition of H2SO4 to cathode compartment, will
iron deposited at cathodes in the two cells when (a) increase the E and shift equilibrium to the
electricity is passed through the cells will be right
[2013] (b) lower the E and shift equilibrium to the right
(c) lower the E and shift equlibrium to the left
(a) 3 : 1 (b) 2 : 1 (c) 1 : 1 (d) 3 : 2 (d) increase the E and shift equilibrium to the
14. 1.08 g of pure silver was converted into silver left
nitrate and its solution was taken in a beaker. It 21. How long a current of 3 amperes has to be
was electrolysed using platinum cathode and passed through a solution of AgNO3 to coat a
silver anode. 0.01 Faraday of electricity was metal surface of 80 cm2 and 0.005 mm thick layer.
passed using 0.15 volt above the decomposition Density of Ag is 10.5 g cm–3. [2008]
potential of silver. The silver content of the
beaker after the above shall be [2012] (a) 125.1 seconds (b) 12.5 seconds
(a) 0 g (b) 0.108 g (c) 155.2 seconds (d) 200 seconds
(c) 0.108 g (d) 1.08 g 22. Two electrochemical cell Fe/Fe2+ || Cu2+ || Cu
15. Consider the following Eº values and Zn/Zn2+ || Cu2+ || Cu are connected in series.
(Fe2+/Fe = –0.41V, Zn2+/Zn = –0.076V, Cu2+/Cu =
EºFe3+ /Fe2 + = + 0.77V ; EºSn 2 + /Sn = - 0.14V +0.34V). The net emf of this cell is [2006]
Under standard conditions the potential for the (a) 1.10V (b) 0.75V
reaction. (c) 0.35V (d) 1.85V
EBD_7167
http://t.me/iitjeehelps

C – 42 BITSAT Topicwise Solved Papers

Hints & Solutions


1. (a) More is E°RP , more is the tendency to get Cathode :
itself reduced or more is oxidising power. Cu 2 + (aq) + 2e - ¾
¾® Cu(s)
2. (c) DG = –2.303 RT log K (pH = decrease)
-
–nFE° = –2.303 RT log K Cu 2 + (aq) + 2e ¾¾® Cu(s)
(pH = constant)
nFE°
log K =
2.303 RT 2 H 2 O + 2e - ¾
¾® H 2 + 2OH - (aq)
(pH = increase)
nFE°
= 0.4342 ........ (i) 7. (c) The reaction
RT
2 Cu + (aq) ¾
¾® Cu(s) + Cu 2+ (aq)
nFE°
ln K =
RT 0.0592 [Cu 2+ ]
E cell = E°cell – log
nFE ° 1 [Cu + ]2
....... (ii)
K = e RT
At equilibrium E cell = 0
0.059 1.10 ´ 2
3. (b) E ocell = log K c or = log K c \ E°cell = 0.0592 log K c
2 0.059
0.52 – 0.16
\ K c = 1.9 ´1037 or, log K c =
0.0592
1000k 1000 ´ 3.75 ´ 10 -4
4. (c) l m = = = 3.75; \ K c = 1.2 ´ 10
6
0.1 0.1
8. (a) Efficiency of fuel cell is:
l 3.75
a= m¥
= = 1.5 ´ 10-2 ; -nFE cell
l m 250 h= ´ 100
DH
Ka = Ca2 = 0.1 ×(1.5 ×10 –2)2 = 2.25 ×10–5 9. (b) The mass of the substance deposited
5. (b) The oxidation potential when one Faraday of charge is passed
through its solution is equal to gram
1
µ and reduction equivalent weight.
Concentration of ions 10. (d)
potential µ concentration of ions. The cell
voltage can be increased by decreasing the 11. (c) 2 Cu + ¾¾
® Cu +2 + Cu
concentration of ions around anode or by
increasing the concentration of ions 2 e - + Cu +2 ¾
¾® Cu ; E1º = 0.34V; ...(i)
around cathode
6. (c) Anode : e - + Cu + 2 ¾
¾® Cu + ; E º2 = 0.15V; ...(ii)
CuSO 4 / Pt. Cu + + e - ® Cu; E3º = ? ... (iii)
1
H2O ¾ ¾® 2H + + O 2 + 2e -
2 Now, DG1º = -nFE1º = -2 ´ 0.34F
CuSO 4 / Cu
DG º2 = -1´ 0.15F , DG 3º = -1´ E 3º F
Cu(s) ¾ 2 + (aq) + -
¾® Cu 2e
KCl/Pt Again, DG1º = DG º2 + DG 3º
2Cl - ¾
¾® Cl 2 (aq) + 2e - Þ -0.68F = -0.15F - E3º F
http://t.me/iitjeehelps

Electrochemistry C – 43
º 17. (a) In electrolytic cell the cathode is of higher
0 Þ E3 = 0.68 - 0.15 = 0.53V
reduction potential.
E ºcell = E cathode
º
(Cu + / Cu) 18. (d) In electrolytic purification cathode is of
º
- E anode (Cu +2 / Cu + ) pure metal and anode is of impure metal.
At anode : M ® M n + + ne
= 0.53 – 0.15 = 0.38 V.
At cathode : M n + + ne ® M .
E.wt ´ Q
12. (c) m= ; The metal thus deposited at cathode.
96500
conductivity ´ 1000
m ´ 96500 19. (d) Solublity =
\ E.wt = L eq
Q
22.2 ´ 96500
= = 60.3 3.06 ´ 10-6 ´ 1000
2 ´ 5 ´ 60 ´ 60 = = 2 × 10–3
1.53
At wt. 177
Oxidation state = = =3 Ksp = S2 = 4 × 10–6
Eq. wt . 60.3
20. (a) ˆˆ†
Zn (s) + 2H + + (aq) ‡ˆˆ
13. (d) At cathodes : Fe+ + 2e - ¾¾
® Fe;
3+ -
Zn 2 + (aq) + H 2 (g )
Fe + 3e ¾¾
® Fe
At. wt. At. wt. 0.059 [Zn 2 + ][H 2 ]
(E Fe )1 = ; (E Fe )2 = E cell = E°cell - log
2 3 2 [H + ]2
(E Fe )1 3 Addition of H2SO4 will increase [H+] and
Hence, =
(E Fe ) 2 2 Ecell will also increase and the equilibrium
will shift towards RHS
14. (a) Ag + + e - ¾
¾® Ag 21. (a) Volume = Area × thickness
1F 108 g
Mass = Volume × density
1 F = 108 g Ag
\ Mass of Ag to be deposited
0.01F = 1.08 g Ag
\Ag left = 1.08 – 1.08 = 0g 80 ´ 0.005
= ´ 10.5 = 0.42 g
10
15. (a) Fe 3+ + e ® Fe 2+ DG ° = -1 ´ F ´ 0.77 E.wt ´ i ´ t
Amount deposited =
Sn 2 + + 2e ® Sn (s)DG° = -2 ´ F(-0.14) 96500
108 ´ 3 ´ t
for Sn (s) + 2Fe 3+ (aq) ® 2Fe 2 + (aq) + Sn 2+ (aq) \ 0.42 =
96500
DG = -(-2F ´ (-0.14) + 2(-1´ F ´ 0.77) \ t = 125.1 seconds
= -1.82F 22. (d) For Fe & Cu cell,Fe is anode and Cu is
\ Standard potential for the given cathode,
-1.82F Ecell = 0.34 – (–0.41) = 0.75V and for
reaction = = 0.91V
- 2´ F Zn & Cu cell, Zn is anode and Cu is
E. wt ´ Q cathode,
16. (a) Amt. deposited = ;
96500 Ecell = 0.34 – (–0.76) = 1.10 V.
107.870 So net emf = 0.75V + 1.10V = +1.85V
107.870 = ´ Q; \ Q = 96500C
96500
EBD_7167
http://t.me/iitjeehelps

C – 44 BITSAT Topicwise Solved Papers

Chapter
Chemical Kinetics
18
1. For the reaction H2(g) + Br2 (g) ® 2HBr (g), the shown in the figure. The overall order of the
experimental data suggest, rate = k[H2][Br2]1/2. reaction is [2016]
The molecularity and order of the reaction are
respectively [2017] [Q]0
3 3 3
(a) 2, (b) ,
2 2 2 [Q]
1
(c) 1, 1 (d) 1, Time
2 (a) 2 (b) 3 (c) 0 (d) 1
2. The rate of reaction between two reactants A 5. The unit of rate constant for reactions of
an d B decreases by a factor of 4 if the second order is [2015]
concentration of reactant B is doubled. The (a) L mol–1s–1 (b) L–1 mol s–1
order of this reaction with respect to reactant (c) L mol s–1 (d) s –1
B is: [2017] 6. In a first order r eaction with time the
(a) 2 (b) -2 (c) 1 (d) -1 concentration of the reactant decreases[2015]
3. Consider the reaction : (a) linearly (b) exponentially
Cl2 ( aq ) + H 2S( aq ) ® (c) no change (d) None of these
7. The decomposition of A follows two parallel first
S(s ) + 2H + (aq) + 2Cl- (aq)
order reactions [2014]
The rate equation for this reaction is k1 = 1.26 ´ 10 -4 sec -1
B
rate = k[Cl2 ][H 2S] A k 2 = 3.8 ´ 10- 5 sec - 1
C
Which of these mechanisms is/are consistent
The percentage distribution of B and C are
with this rate equation? [2016]
(a) 90 % B and 10 % C
A. Cl 2 + H 2S ® H + + Cl - + Cl+ + HS- (slow) (b) 80 % B and 20 % C
(c) 60 % B abd 40 % C
Cl + + HS- ® H + + Cl- + S (fast)
(d) 76.83% B and 23.17 % C
B. H 2S ƒ H + + HS- (fast equilibrium) 1
8. A gaseous reaction X 2 (g) ¾ ¾® Y + Z (g)
2
- - + There is increase in pressure from 100 mm to 120
Cl 2 + HS ® 2Cl + H + S (Slow)
mm in 5 minutes. The rate of disappearance of
(a) B only (b) Both A and B
X2 is [2014]
(c) Neither A nor B (d) A only
(a) 8 mm min–1 (b) 2 mm min–1
4. In the reaction, (c) 16 mm min–1 (d) 4 mm min–1
P + Q ¾¾® ? R + S 9. Two substances R and S decompose in solution
The time taken for 75% reaction of P is twice the independently, both following first order
time taken for 50% reaction of P. The kinetics. The rate constant of R is twice that of
concentration of Q varies with reaction time as S. In an experiment, the solution initially
http://t.me/iitjeehelps

Chemical Kinetics C – 45
contained 0.5 millimoles of R and 0.25 of S. The The order of reaction is :
molarities of R and S will be equal just at the end (a) Zero (b) One
of time equal to [2014] (c) Two (d) Three
(a) twice the half life of R 15. The chemical reaction 2O 3 ¾ ¾® 3O 2
(b) twice the half life of S proceeds as follows:
(c) the half life of S Fast Slow
(d) the half life of R O 3 ¾¾ ¾® O 2 + O ; O + O 3 ¾¾¾® 2O 2
10. Velocity constant of a reaction at 290 K was found the rate law expression should be [2009]
to be 3.2×10–3. At 300 K it will be [2013] (a) r = k[O3]2 (b) r = k [O3]2[O2]–1
(a) 1.28 × 10 –2 (b) 9.6 × 10 –3
(c) r = k3 [O3][O2]2 (d) r = [O3][O2]2
(c) 6.4 × 10–3 (d) 3.2 × 10–4 16. For a reaction A ® B, the rate increases by a
11. The addition of a catalyst during a chemical factor of 2.25 when the concentration of A is
reaction alters which of the following quantities? increased by 1.5. What is the order of the
[2012] reaction? [2008]
(a) 3 (b) 0 (c) 2 (d) 1
(a) Entropy (b) Internal energy
17. For the reaction system :
(c) Enthalpy (d) Activation energy
¾® 2 NO 2 ( g ) volume is
2 NO ( g ) + O 2 (g ) ¾
12. During the kinetic study of the reaction,
suddenly reduced to half of its value by
2A + B ® C + D, following results were obtained:
increasing the pressure on it. If the reaction is of
Run [A]/mol L–1 [B]/mol L–1 Initial rate of
first order with respect to O2 and second order
with respect to NO, the rate of reaction will
formation of
[2007]
D/mol L–1min–1
(a) diminish to one-fourth of its initial value
I 0.1 0.1 6.0 × 10–1 (b) diminish to one-eighth of its initial value
II 0.3 0.2 7.2 × 10–1 (c) increase to eight times of its initial value
III 0.3 0.4 2.88 × 10–1 (d) increase to four times of its initial value
IV 0.4 0.1 2.40 × 10–1 18. ˆˆ† 2HBr(g) ,
For the reaction H 2(g) + Br2(g) ‡ˆˆ

Based on the above data which one of the the rate law is rate = k[H 2 ][Br2 ]1/ 2 . [2006]
following is correct? [2011] Which of the following statement is true about
this reaction?
2
(a) rate = k [A] [B] (b) rate = k[A] [B] (a) The reaction is of second order
(b) Molecularity of the reaction is 3/2
(c) rate = k [A]2 [B]2 (d) rate = k [A] [B]2 (c) The unit of k is s–1
13. The activation energy for a simple chemical (d) Molecularity of the reaction is 2
reaction A ® B is Ea in forward direction. The 19. The rate law for a reaction between the
activation energy for reverse reaction [2010] substances A and B is given by
(a) is always less than Ea Rate = k [A]n [B]m
(b) can be less than or more than E a On doubling the concentration of A and halving
(c) is always double of Ea the concentration of B, the ratio of the new rate
(d) is negative of Ea to the earlier rate of the reaction will be as [2005]
14. The following data are for the decomposition of 1
(a) ( m+ n ) (b) (m + n)
ammonium nitrite in aqueous solution : [2010] 2
Vol. of N2 in cc Time (min) (c) (n – m) (d) 2(n – m)
6.25 10 ˆˆ†
20. The reaction 2N2O5 ‡ˆˆ 2N2O4 + O2 is
9.00 15 [2005]
11.40 20 (a) Bimolecular and second order
13.65 25 (b) Unimolecular and first order
35.65 Infinity (c) Bimolecular and first order
(d) Bimolecular and zero order
EBD_7167
http://t.me/iitjeehelps

C – 46 BITSAT Topicwise Solved Papers

Hints & Solutions


3
1. (a) The order of reaction is and molecularity This is true only for first order reaction.
2 So, order with respect to P is 1.
is 2.
2. (b) Rate1= k [A]x[B]y ... (1) Further the graph shows that concentration
of Q decreases with time. So rate, with
Rate1
= k [A]x [2B]y ... (2) respect to Q, remains constant. Hence, it is
4 zero order wrt Q.
or Rate1 = 4k[A]x[2B]y So, overall order is 1+ 0 = 1
From (1) and (2) we get 5. (a) Unit of rate constant for second order
reaction is L mol–1 sec–1.
k[A]x [B]y
= k[A]x[2B]y 6. (b) For a first order reaction [A] = [A0]e–kt
4
\ The concentration of reactants will
[B]y
= [2B] y exponentially decreases with time.
4
7. (d) For two parallel reactions
y
1 æ 2B ö 1 kaverage = k1 + k2
or =ç ÷ Þ = 2y or (2)–2 = 2y
4 è Bø 4 = 1.26 × 10–4 + 3.8 × 10–5
y = –2. = 1.64 × 10–4sec–1
3. (d) Since the slow step is the rate determining kB
step hence if we consider option (A) we The fractional yield of B =
k av
find
Rate = k [ Cl2 ][ H 2S] 1.26 ´ 10 - 4
= = 0.7683 = 76.83 %
Now if we consider option (B) we find 1.64 ´ 10 - 4
The fractional yield of
Rate = k [ Cl2 ] éë HS ùû
-
...(i)
For equation, kC 3.8 ´ 10 - 5
ˆˆ† H+ + HS– C= =
H2S ‡ˆˆ k av 1.64 ´ 10 - 4
é H + ù é HS- ù = 0.2317 = 23.17 %
K = ë ûë û 8. (a) The increase in pressure shows the
H 2S
increase in conc. of Z. Rate of appearance
- K [ H 2 S] 120 - 100
or éë HS ùû = of Z = = 4 mm min–1
H+ 5
Substituting this value in equation (i) we
Rate of disappearance of X2 = 2 × rate of
find
appearance of Z
Rate = k [ Cl2 ] K
[ H 2S] = k ' [ Cl2 ][ H 2S] = 2 × 4 mm min–1 = 8 mm min–1
H+ éH+ ù 9. (a) Substance R Substance S
ë û 2k k rate constant
Thus slow step should involve 1 molecule t½ 2 t½ Half life period
of Cl2 and 1 molecule of H2S.
hence only , mechanism (A) is consistent T = n × t1/2
with the given rate equation. where n = number of half life period
4. (d) For P, if t50% = x 0.5
then t75% = 2x Amount of R left = ;
(2) T/ t½
http://t.me/iitjeehelps

Chemical Kinetics C – 47

0.25 16. (c) Let the rate law equation be


Amount of S left =
(2)T/2t½ r = k[A]x ....(i)
T/ t1 / 2
0.5 ( 2) 2.25r = k[1.5A]x ....(ii)
Equating both =
0.25 (2) T/ 2 t1 / 2
T /t Divide eq(ii) by eq(i)
or 2 = (2) 1/ 2
\ 2.25 = (1.5)x \ x = 2 ; Second order
\ T = 2 t 1 / 2 . 2t1/2 is half life of S and
17. (c) Rate law = k[O 2 ][ NO ] 2
twice the half -life of R
If volume is reduced to half, concentrations
10. (c) The velocity constant doubles for every
10°C rise in temperature will get double.
11. (d) A catalyst provides an alternative route for \ Rate = k[ 2O 2 ][ 2 NO ]2 = 8 times
the reaction with a lower activation energy.
12. (d) In case of (II) and (III) Keepin g 18. (d) ˆˆ† 2HBr(g)
H 2 (g) + Br2 (g) ‡ˆˆ
concentration of [A] constant, when the Rate law, R = k[H2] [Br2]½
concentration of [B] is doubled, the rate
quadruples. Hence it is second order with Order of reaction = 1 + ½ = 3/2
respect to B. In case of I & IV Keeping the Molecularity of reaction = 2
concentration of [B] constant, when the
R
concentration of [A] is increased four The unit of k =
times, rate also increases four times. Hence, [H 2 ] [Br2 ]½
the order with respect to A is one. hence
Rate = k [A] [B]2 mole. lit -1s -1
13. (b) Since the nature of reaction (i.e. exothermic =
[mole. lit -1 ] [mole. lit -1 ]½
or endothermic) is not given, Ea for reverse
reaction can be more or less. = mole–½ . lit½ . s–1
14. (b) NH 4 NO 2 ¾
¾® N 2 + 2 H 2 O
19. (d) Rate = k[ A ]n [B ]m .
Volume of N2 formed in successive five
minutes are 2.75 cc, 2.40 cc and 2.25 cc On doubling conc. of A and halving conc. of
which is in decreasing order. So rate of B
reaction is dependent on concentration of m
NH4 NO2. As decrease is not very fast so it é1 ù
New Rate = k[ 2 A ] n ê Bú
will be first order reaction. ë2 û
Fast
15. (b) O3 ¾¾¾ ® O 2 + O, 1
= k 2 n. [A ]n [B]m
Slow
O + O3 ¾¾¾® 2O 2 2m

[O 2 ][O] = k 2 n - m [ A ] n [ B] m
k= … (i)
[O 3 ]
New rate
Rate = k' [O3][O] … (ii) Ratio =
Old rate
put [O] from (ii)
k '[O3 ]k[O3 ] k 2n - m [A]n [B]m
r= = k[O3 ]2 [O2 ]-1 = = 2n - m
[O 2 ] k ¢[A]n [B]m
Note intermediates are never represented 20. (c) It is bimolecular first order reaction, since
in rate law equation. Rate µ [N2O5]
EBD_7167
http://t.me/iitjeehelps

Chapter
Surface Chemistry
19
1. Which of the following ionic substances will be (a) polar at outer surface and non-polar at inner
most effective in precipitating the sulphur sol? surface
[2015] (b) polar at inner surface and non-polar at outer
(a) KCl (b) BaCl2 surface
(c) Fe2(SO4)3 (d) Na3PO4 (c) distributed all over the surface
2. The isoelectric-point of a colloidially dispersed (d) present in the surface only
material is the pH value at which [2014] 6. For adsorption of a gas on a solid, the plot of
(a) the dispersed phase migrate in an electric log x/m vs log P is linear with slope equal to
field (n being whole number) [2011]
(b) the dispersed phase does not migrate in an (a) k (b) log k
electric field 1
(c) the dispersed phase has pH equal to 7 (c) n (d)
(d) the dispersed phase has pH equal to zero n
3. At high pressure, the entire surface gets covered 7. An example of autocatalysis is [2008]
by a monomolecular layer of the gas follows (a) Decomposition of KClO3 to KCl and O2
[2013] (b) Oxidation of SO2 to SO3
(a) three-halved order (b) second-order (c) Formation of NH3 by N2 and H2 in presence
(c) first-order (d) zero-order of Fe.
4. Which of the following gas molecules have (d) Oxidation of oxalic acid by acidic KMnO4
maximum value of enthalpy of physisorption? 8. Which property of colloidal solution is
[2012] independent of charge on the colloidal particles:-
(a) C2H6 (b) Ne (c) H2O (d) H2 [2007]
5. Position of non-polar and polar part in micelle (a) Electrophoresis (b) Electro-osmosis
is [2011] (c) Tyndall effect (d) Coagulation

Hints & Solutions


1. (c) x x 1
= kP1/ n or log = log k + log P;
2. (b) At isoelectric point there is no migration of m m n
dispersed phase in an electric field.
3. (d) At high pressure the extent of adsorption
follows zero order Kinetics. x/m
Ps
4. (c) The more the liquifiable nature of a gas,
the more is the enthalpy of adsorption. P
Water is more liquifiable. x
plot of log vs log P is linear with slope
m
1
5. (a) o- Polarhead =
n
n- Non-polar tail
(micelle) 7. (d) Mn 2+ ions formed during the reaction
catalyse the reaction.
6. (d) According to Freundlich adsorption 8. (c) Tyndall effect is an optical property, and it
isotherm. is independent of charge on colloidal
At in termediate pressure, extent of particles.
adsorption
http://t.me/iitjeehelps

Chapter
General Principles & Processes of
Isolation of Elements 20
1. Electrometallurgical process is used to extract 4. Among the following statements the incorrect
[2017] one is [2009]
(a) Fe (b) Pb (a) Calamine and siderite are carbonates
(c) Na (d) Ag (b) Argentite and cuprite are oxides
2. Semiconductor materials like Si and Ge are (c) Zinc blende and iron pyrites are sulphides
usually purified by [2015] (d) Malachite and azurite are ores of copper.
(a) distillation 5. Silver containing lead as an impurity is removed
(b) zone refining by [2008]
(c) liquation (a) poling (b) cupellation
(d) electrolytic refining (c) lavigation (d) distillation
3. Calcination is used in metallurgy for removal 6. The most electropositive metals are isolated
of? [2011] from their ores by [2006]
(a) Water and sulphide (a) high temperature reduction with carbon
(b) Water and CO2 (b) self reduction
(c) CO2 and H2S (c) thermal decomposition
(d) H2O and H2S (d) electrolysis of fused ionic salts

Hints & Solutions


1. (c) Because Na is very reactive and cannot be of impurities are formed and finally
extracted by means of the reduction by C, removed.
CO etc. So it is extracted by electrolysis. 3. (b) Calcination is used for removal of volatile
2. (b) Semiconductor materials like Si and Ge are impurities and decompose carbonates.
usually purified by zone refining. Zone 4. (b) Cuprite is Cu2O and Argentite is Ag2S.
refining is based on the principle of 5. (b) Silver containing lead is purified by
fractional crystallisation i.e. difference in cupellation (see text).
solubilities of impurities in solid and 6. (d) Most electropositive metals are obtained
molten states of metal, so that the zones by electrolysis of their fused ionic salts.
EBD_7167
http://t.me/iitjeehelps

C – 50 BITSAT Topicwise Solved Papers

Chapter
The p-Block Elements
(Group15,16,17and 18) 21
1. Which of the following shows nitrogen with (b) high electron gain enthalpy of phosphorus
its increasing order of oxidation number? (c) high oxidation state of phosphorus
[2017] (d) presence of two –OH groups and one P–H
(a) NO < N2O < NO2 < NO3– < NH4+ bond.
(b) NH4+ < N2O < NO2 < NO3– < NO 12. When chlorine water is exposed to sunlight, O2
is liberated. Hence [2012]
(c) NH4+ < N2O < NO < NO2 < NO3–
(a) Hydrogen has little affinity to O2
(d) NH4+ < NO < N2O < NO2 < NO3– (b) Hydrogen has more affinity to O2
2. Which of these doesn’t exist? [2017] (c) Hydrogen has more affinity to chlorine
(a) PH3 (b) PH5 (c) LuH3 (d) PF5 (d) It is a reducing agent.
3. Laughing gas is [2016] 13. Phosphine is not obtained by the reaction
(a) nitrogen pentoxide(b) nitrous oxide [2011]
(c) nitrogen trioxide (d) nitric oxide (a) White P is heated with NaOH
4. The by product of solvay-ammonia process is (b) Red P is heated with NaOH
[2016] (c) Ca3P2 reacts with water
(a) CO2 (b) NH3 (c) CaCl2 (d) CaCO3 (d) Phosphorus trioxide is boiled with water
5. Which of the following is a strong base? [2015] 14. Which liberates ammonia when treated with
(a) PH3 (b) AsH3 (c) NH3 (d) SbH3
water? (2010)
6. Which of the following fluorides of xenon is
impossible? [2015] (a) Li3N (b) Mg3N2
(a) XeF2 (b) XeF3 (c) XeF4 (d) XeF6 (c) CaCN2 (d) All
7. Which of the following halogens exhibit only 15. The correct order of reactivity of halogens with
one oxidation state in its compounds ? [2014] alkalies is [2010]
(a) Bromine (b) Chlorine (a) F > Cl > Br > I (b) F < Cl > Br < I
(c) Fluorine (d) Iodine (c) F < Cl < Br < I (d) F < Cl < Br > I
8. Starch can be used as an indicator for the 16. Sodium pyrophosphate is [2008]
detection of traces of [2014] (a) Na2P2O7 (b) Na4P2O7
(a) glucose in aqueous solution (c) NaPO4 (d) Na2PO2
(b) proteins in blood
(c) iodine in aqueous solution 17. Bleaching action of SO2 is due to its [2007]
(d) urea in blood (a) oxidising property (b) acidic property
9. Which one of the following arrangements (c) reducing property (d) basic property
represents the correct order of electron gain 18. Which of the following behaves as both
enthalpy (with negative sign) of the given oxidising and reducing agents ? [2006]
atomic species? [2014] (a) H2SO4 (b) SO2
(a) S < O < Cl < F (b) Cl < F < S < O (c) H2S (d) HNO3
(c) F < Cl < O < S (d) O < S < F < Cl 19. Gradual addition of electronic shells in the noble
10. Which of the following is incorrect with respect gases causes a decrease in their [2006]
to property indicated ? [2013] (a) Ionization energy (b) Atomic radius
(a) E.N : F > Cl > Br (c) Boiling point (d) Density
(b) E.A.: Cl > Br > < F 20. Atomic radii of fluorine and neon in Angstrom
(c) Oxidising power : F2 > Cl2 > Br2 units are respectively given by [2006]
(d) Bond energy : F2 > Cl2 > Br2 (a) 0.72 ; 1.60 (b) 1.60 ; 1.60
11. Strong reducing behaviour of H3PO2 is due to (c) 0.72 ; 0.72 (d) None
[2013] 21. Caro’s acid is [2005]
(a) presence of one –OH group and two P–H (a) H2SO3 (b) H2S2O5
bonds (c) H2SO5 (d) H2S2O8
http://t.me/iitjeehelps

The p-Block Elements (Group 15, 16, 17 and 18) C – 51

Hints & Solutions


1. (c) Compound O.S. of N atom to produce a monovalent anion is
N2O +1 called electron gain enthalpy.
Electron affinity value generally increase
NO +2
on moving from left to right in a period
NO2 +4 however there are exceptions of this rule in
NO3– +5 the case of those atoms which have stable
NH4+ –3 configuration. These atoms resist the
addition of extra electron, therefore the low
Therefore increasing order of oxidation value of electron affinity
state of N is:
O < S < F < Cl
NH +4 < N2O < NO < NO2 < NO 3- . -1.48 -2.0 -3.6 -3.8
2. (b) PH5 does not exist because d-orbital of ‘P’ On the other hand Cl, because of its
interacts with s-orbital of H. Bond formed comparatively bigger size than F, allow the
is not stable and not energetically addition of an extra electron more easily.
favorable. It depends on size and 10. (d) Bond energy follows the order Cl2 > F2 >
orientation of interaction. Br2
3. (b) Nitrous oxide (i.e., N2O) is the laughing 11. (a) The acids which contain P-H bond have
gas. strong reducing properties. Thus H3PO2
4. (c) CaCl2 is produced as a by product in solvay acid is good reducing agent as it contains
ammonia process. two P–H bonds. For example, it reduces
AgNO3 to metallic silver.
(i) NaCl + CO2 + NH3 + H2O ¾¾ ®
NaHCO3 + NH4Cl 4 AgNO3 + 2H2O + H3PO2 —®
(ii) CaCO3 ¾¾ ® CO2 + CaO 4Ag + 4HNO3 + H3PO4
(iii) 2NH2Cl + CaO ¾¾ ® 1
12. (c) Cl 2 + H 2 O ® 2HCl + O2
2NH3 + CaCl2 + H2O 2
By product Hydrogen has more affinity for chlorine.
5. (c) Order of basic character is NH3 > PH3 > 13. (b) Red P does not react with NaOH to give
AsH3 > SbH3. Basic-character decreases PH3.
down the group from N to Bi due to 14. (d) All nitrides react with H2O to give NH3 but
increase in atomic size. CaCN2 also react with H2O
6. (b) 673K
Xe + F2 ¾¾¾® XeF2 CaNCN + 3H2O ¾¾® CaCO3 + NH3
15. (a) Reactivity follows the order F > Cl > Br > I
2 :1 16. (b) The chemical formula of sodium
673K
Xe + 2F2 ¾¾¾® XeF4 pyrophosphate is Na2P2O7. It is a salt of
5 - 6Atm pyrophosphoric acid (Na4P2O7).
1:5 17. (c) SO 2 + 2H 2 O ® H 2SO4 + 2[H] .
573K
Xe + 3F2 ¾¾ ¾¾¾® XeF6 Bleaching action is due to reduction.
50- 60 Atm
1 : 20 18. (b) In SO2, S is in + 4 oxidation state which
can be increased or decreased by gain or
XeF3 is not possible.
loss of electrons. Hence SO2 can act as
7. (c) Fluorine, since it is the most oxidising and reducing agent.
electronegative element. 19. (a)
8. (c) I2 gives blue colour with starch. 20. (a) In inert gases we have van der waals radii
which is always higher than atomic radii
9. (d) The amount of energy released when an
electron is added to an isolated gaseous 21. (c)
EBD_7167
http://t.me/iitjeehelps

C – 52 BITSAT Topicwise Solved Papers

Chapter
The d-and f-Block Elements
22
1. Cadmium is used in nuclear reactors for? [2017] 7. K4[Fe(CN)6] is used in detecting. [2014]
(a) absorbing neutrons (a) Fe3+ ion (b) Cu+ ion
(b) cooling (c) Cu3+ ion (d) Fe2+ ion
(c) release neutrons 8. Which form coloured salts : [2014]
(d) increase energy (a) Non-metals
2. Knowing that the chemistry of lanthanoids(Ln) (b) Metals
is dominated by its + 3 oxidation state, which of (c) p-block elements
the following statements is incorrect? [2016] (d) Transitional elements
(a) The ionic size of Ln (III) decrease in general 9. An extremely hot copper wire reacts with steam
with increasing atomic number to give [2012]
(b) Ln (III) compounds ar e generally (a) CuO (b) Cu2O
colourless. (c) Cu2O2 (d) CuO2
(c) Ln (III) hydroxide are mainly basic in 10. Which of the following does not have valence
character. electron in 3d-subshell? [2012]
(a) Fe (III) (b) Mn (II)
(d) Because of the large size of the Ln (III) ions
(c) Cr (I) (d) P (0)
the bonding in its compounds is
11. Among the following the lowest degree of
predominantly ionic in character.
paramagnetism per mole of the compound at
3. Which of the following arrangements does not 298 K will be shown by [2012]
represent the correct order of the property stated
against it ? [2016] (a) MnSO 4 .4H 2 O (b) CuSO 4 .5H 2O
2+ 2+ 2+ 2+
(a) V < Cr < Mn < Fe :paramagnetic (c) FeSO 4 .6H 2 O (d) NiSO 4 .6H 2 O
behaviour 12. Which of the following halides is not oxidized
(b) i2+ < Co2+ < Fe2+ < Mn2+ : ionic size by MnO2 [2011]
(c) Co3+ < Fe3+ < Cr3+ < Sc3+ :
(a) F– (b) Cl– (c) Br– (d) I–
stability in aqueous solution
(d) Sc < Ti < Cr < Mn : number of oxidation 13. Which of the following exhibit only + 3 oxidation
states state ? [2011]
4. The number of elements present in the d-block (a) U (b) Th (c) Ac (d) Pa
of the periodic table is [2015] 14. Which of the following pairs has the same size?
(a) 40 (b) 41 (c) 45 (d) 46 [2011]
5. Which one of given elements shows maximum (a) Fe 2+ , Ni 2+ (b) Zr 4+ , Ti 4+
number of different oxidation states in its
compounds? [2015] (c) Zr 4+ , Hf 4+ (d) Zn 2+ , Hf 4+
(a) Am (b) Fm (c) La (d) Gd 15. In the manufacture of iron from haematite,
6. Thomas slag is [2015] limestone is added to act as : [2010]
(a) Ca3(PO4)2 (a) Flux (b) Slag
(b) CaSiO3 (c) A reducing agent (d) An oxidizing agent
(c) Mixture of (a) and (b) 16. Cinnabar is an ore of [2009]
(d) FeSiO3 (a) Hg (b) Cu (c) Pb (d) Zn
http://t.me/iitjeehelps

The d-and f-Block Elements C – 53


17. Which of the following is used in the preparation (a) The white solid dissolves to form a
of chlorine? [2009] colourless solution
(a) Only MnO2 (b) The white solid dissolves to form a blue
(b) Only KMnO4 solution
(c) The white solid dissolves to form a green
(c) Both MnO2 and KMnO4 solution
(d) Either MnO2 or KMnO4 (d) The white solid turns blue but does not
18. Which of the following elements does not dissolve.
belong to first transition series? [2009] 22. Titanium shows magnetic moment of 1.73 B.M.
(a) Fe (b) V (c) Ag (d) Cu in its compound. What is the oxidation number
19. Percentage of silver in the alloy ‘german silver’ is of Ti in the compound? [2006]
[2008]
(a) + 1 (b) + 4 (c) + 3 (d) + 2
(a) 1.5% (b) 2.5%
(c) 10% (d) Zero % 23. Blue virtriol is : [2005]
20. If an aqueous solution of KCN is added to a (a) MgSO 4 .5H 2O (b) CuSO 4 .5H 2O
solution of ferrous salt then the complex formed
(c) CaSO 4 .5H 2 O (d) ZnSO 4 .5H 2O
is represented by [2007]
4- 3-
24. Arrange Ce3+, La3+, Pm3+ and Yb3+ in increasing
(a) é Fe ( CN ) ù (b) é Fe ( CN ) ù order of their ionic radii. [2005]
ë 6û ë 6û
(a) Yb3+ < Pm3+ < Ce3+ < La3+
3+ 2+
(c) é Fe ( H 2 O ) ù (d) é Fe ( H 2 O ) ù (b) Ce3+ < Yb3+ < Pm3+ < La3+
ë 6û ë 6û
(c) Yb3+ < Pm3+ < La3+ < Ce3+
21. What is the effect of shaking dil. H2SO4 with a
small quantity of anhydrous CuSO4? [2006] (d) Pm3+ < La3+ < Ce3+ < Yb3+.

Hints & Solutions


1. (a) Control rods slowdown the motion of (b) For the same oxidation state, the ionic radii
neutrons and help in controlling the rate generally decreases as the atomic number
of fission. Cadmium is efficient for this increases in a particular transition series.
purpose. hence the order is
2. (b) Most of the Ln3+ compounds except La3+ Mn2+ > Fe2+ > Co2+ > Ni2+
and Lu3+ are coloured due to the presence (c) In solution, the stability of the compound
of f-electrons.
depends upon electrode potentials, SEP of
3. (a)
the transitions metal ions are given as
(a) V = 3d 3 4s 2 ; V2+ = 3d 3 = 3 unpaired
Co3+ / Co = + 1.97, Fe3+ / Fe = + 0.77 ;
electrons
Cr = 3d 5 4s 1 ; Cr 2+ = 3d 4 = 4 unpaired Cr3+ / Cr2+ = – 0.41, Sc 3+ is highly stable as
electrons it does not show + 2 O. S.
Mn = 3d 54s2 ; (d) Sc – (+ 2), (+ 3)
Mn2+ = 3d 5 = 5 unpaired electrons Ti – (+ 2), (+ 3), (+ 4)
Fe = 3d 6 4s 2 ; Fe2+ = 3d 6 = 4 unpaired Cr – (+ 1), (+ 2), (+ 3), (+ 4), (+ 5), (+ 6)
electrons Mn – (+ 2), (+ 3), (+ 4), (+ 5), (+ 6), (+ 7)
Hence the correct order of paramagnetic i.e. Sc < Ti < Cr < Mn
behaviour 4. (a) 40 elements are present in d-block.
V2+ < Cr 2+ = Fe2+ < Mn2+
EBD_7167
http://t.me/iitjeehelps

C – 54 BITSAT Topicwise Solved Papers

5. (a) Am shows maximum number of oxidation 15. (a) Limestone (CaCO3) is mixed with Fe2O3 and
states, + 3, + 4, + 5, + 6 it acts as flux to form slag (CaSiO3).
6. (c) Calcium silicophosphate (a mixture of 16. (a) Cinnabar (HgS) is an ore of Hg.
Ca3(PO4)2 & Ca2SiO4) is called Thomas 17. (c) Both MnO2 and KMnO4 used for the
slag. preparation of chlorine by the action of
7. (a) Fe3+ ion can be detected by K4[Fe(CN)6] conc. HCl
4Fe3+ + 3K 4 éë Fe ( CN )6 ùû ¾¾
® MnO2 + 4HCl ® MnCl2 + 2H 2 O + Cl 2

Fe 4 éë Fe ( CN ) 6 ùû + 12K + 2KMnO 4 + 16HCl ® 2KCl + 2MnCl 2


3 + 8H 2 O + 5Cl2
8. (d) Most of the transition metal compounds
Chlorine is not obtained by dil. HCl
(ionic as well as covalent) are coloured both
Ag 2S2 O3 + H 2 O ® Ag S + H SO
in the solid state and in aqueous solution 2 2 4
Black ppt.
in contrast to the compounds of s and
p-block elements due to the presence of 18. (c)
19. (d) German silver does not contain Ag at all.
incomplete d-subshell.
4-
9. (b) 2Cu + H 2 O ¾
¾® Cu 2 O + H 2 ­ . 20. (a) Fe2 + + 6KCN ® éë Fe ( CN ) 6 ùû + 6K +
Hot Steam
10. (d) P (At no. 15) has electronic configuration 21. (b) Anhydrous CuSO4(white) is reduced into
blue CuSO 4.H2 O which dissolves in
1s 2 , 2s 2 p6 , 3s 2 p3 , hence no electron in dil.H2SO4 to give blue solution.
d - subshell 22. (c) Magnetic moment m = n ( n + 2 ) BM
11. (b) Ion Mn2+ Cu2+ Fe2+ Ni2+
EC 3d5 3d9 3d6 3d8 1.73 = n ( n + 2 ) \ n = 1 , it has one
Number of unparied electrons hence elctronic
unpaired electron 5 1 4 2 configuration is [Ar]3d1 and electronic
Hence lowest paramagnetism is shown by configuration for Z = 22 is [Ar]3d25s2.
CuSO4.5H2O Hence charge on Ti is +3.
12. (a) F2 is strongest oxidising agent. F – is not 23. (b) Blue vitriol is CuSO4.5H2O.
oxidised by MnO2 24. (a) According to their positions in the periods,
13. (c) Ac (89) = [Rn] [6d1] [7s2] these values are in the order:
Yb3+ < Pm3+ < Ce3+ < La3+
14. (c) Due to lanthanide contraction, the size of
At. Nos. 70 61 58 57
Zr and Hf (atom and ions) become nearly
This is due to lanthanide contraction.
similar.
http://t.me/iitjeehelps

Chapter
Co-ordination Compounds
23
1. Which of the following complexes has square 10. The pair in which both species have same
planar structure? [2017] magnetic moment (spin only value) is : [2013]
(a) Ni(CO) 4 (b) [ Ni(CN)4 ]-2 (a) [Cr(H2O)6]2+ , [CoCI4]2–
(c) [ Ni(Cl) 4 ]
-2
(d) [ Zn(NH3 )4 ]+2 (b) [Cr(H 2O6 )2 + ,[Fe(H 2 O)6 ]3+
2. Which of the following complex shows sp3d2 (c) [Mn (H 2 O) 6 ) 2 + , [Cr(H 2 O) 6 ]2 +
hybridization? [2017]
(a) [Cr(NO2)6]3- (b) [Fe(CN)6]4– (d) [CoCl 4 ) 2 - , [Fe(H 2 O) 6 ]2+
(c) [CoF6]3- (d) [Ni(CO)4] 11. The complex used as an anticancer agent is
3. Which of the following is paramagnetic ? [2016] [2012]
(a) [Fe(CN)6]4– (b) [Ni(CO)4] (a) mer –[Co(NH3)3Cl3]
(c) [Ni(CN)4]2– (d) [CoF6]3– (b) cis –[PtCl2(NH3)2]
4. The hypothetical complex chloro- (c) cis–K2[PtCl2Br2]
diaquatriamminecobalt (III) chloride can be (d) Na2CoCl4
represented as [2016]
12. Which of the following is not considered as an
(a) [CoCl(NH3)3(H2O)2 ]Cl2
organometallic compound? [2011]
(b) [Co(NH3)3(H2O)Cl3] (a) cis-platin (b) Ferrocene
(c) [Co(NH3)3(H2O)2Cl] (c) Zeise's salt (d) Grignard reagent
(d) [Co(NH3)3(H2O)3]Cl3 13. The most stable ion is [2011]
5. The color of CoCl3.5NH3.H2O is [2015] (a) [Fe(OH)3]3- (b) [Fe(Cl)6]3-
(a) red (b) orange (c) [Fe(CN)6]3- (d) [Fe(H2O)6]3+.
(c) orange - yellow (d) pink
14. Which of the following has square planar
6. The common name of K[PtCl3(h2.C2H4)] is
[2015] geometry? [2010]
(a) potassium salt (b) Zeise’s salt (a) [PtCl 4 ]2 - (b) [ NiCl4 ]2 -
2-
(c) complex salt (d) None of these (c) [ ZnCl 4 ]2 - (d) [CoCl 4 ]
7. Which of the following represents hexadentate
ligand? [2015, 2009] 15. Which of the following is wrong statement?
(a) EDTA (b) DMG [2008]
(c) Ethylenediamine (d) None of the above (a) Ni(CO)4 has oxidation number +4 for Ni
8. Some salts containing two different metallic (b) Ni(CO)4 has zero oxidation number for Ni
elements give test for only one of them in (c) Ni is metal
solution, such salts are [2015] (d) CO is gas
(a) double salts (b) normal salts 16. CH3 – Mg – Br is an organo metallic compound
(c) complex salts (d) None of these due to [2006]
9. The correct order of magnetic moments (spin (a) Mg – Br bond (b) C – Mg bond
only values in B.M.) anong is [2014] (c) C – Br bond (d) C – H bond.
(a) [Fe(CN) 6 ]4- > [MnCl 4 ]2- > [CoCl 4 ] 2- 17. Which one of the following complexes is an outer
(b) [MnCl 4 ]2- > [Fe(CN ) 6 ]4- > [CoCl 4 ] 2- orbital complex ? [2005]
(a) [Co(NH3)6] 3+ (b) [Mn(CN)6]4–
(c) [MnCl 4 ]2- > [CoCl 4 ]2- > [ Fe(CN) 6 ] 4-
(c) [Fe(CN)6] 4– (d) [Ni(NH3)6]2+
(d) [Fe(CN) 6 ]4- > [CoCl 4 ]2- > [ MnCl 4 ] 2-
(Atomic nos. : Mn = 25, Fe = 26, Co = 27) (Atomic nos. : Mn = 25; Fe = 26; Co = 27, Ni = 28)
EBD_7167
http://t.me/iitjeehelps

C – 56 BITSAT Topicwise Solved Papers

Hints & Solutions


1. (b) In Ni(CO) 4 ; [ Ni ( Cl ) 4 ] - 2 ; 10. (b) [Cr(H2O)6]2+ Cr is in Cr2+ form
[ Zn ( NH 3 ) 4 ] + 2 , all the central atoms have
-2 Cr 2 +
sp3 hybridisation, while in [ Ni(CN) 4 ] , 24 3d 4s
2
its central atom has dsp hybridisation due
to strong field ligand (CN - ) . Hence Fe 3 +
26 3d 4s
[ Ni(CN) 4 ]- 2 has square planar structure.
2. (c) Among these ligands, ‘F’ is a weak field In [Fe(H2O)]2+ Fe2+ form. Both will have 4
ligand, makes only high spin complexes unpaired electrons.
which has sp3d2 hybridization. 11. (b) The complex used as an anticancer agent
is cis-platin.
3. (d) The electronic configuration of Fe2+ is [Ar]
3d6. Since CN is strong field ligand d 12. (a) The structural formula of cis-platin is
electrons are paired. In Ni(CO)4 O. S. of Ni NH 3 Cl
is zero, electronic configuration is [Ar]3d8
Pt
4s2. In presence of CO it is [Ar] 3d10 4s0,
NH 3 Cl
electrons are paired. Electronic
configuration of Ni2+ [Ar]3d8 4s0, due to Since no carbon is involved it is not a
CN– ligand all electrons are paired. Co3+ is organometallic compound.
[Ar] 3d6 since F is weak ligand hence 13. (b) A more basic ligand forms stable bond with
paramagnetic. metal ion, Cl- is most basic amongst all.
4. (a) The complex chlorodiaquatriammine cobalt 14. (a) [PtCl4]2– has square planar geometry.
(III) chloride can have the structure Pt : 5d96s1
[CoCl(NH3)3(H2O)2]Cl2 Pt2+ =
5. (d) CoCl3.5NH3.H2O is pink in colour 5d 6s 6p
6. (b) Zeise’s salt is common name of
K[Pt Cl3(h2 = C2H4)] Two electrons are removed from 5d shell
7. (a) EDTA is hexadentate ligand and 6s shell. So, hybridisation takes place
– –
is dsp2 i.e. square planar geometry.
OOCH2C •• •• CH2 – COO 15. (a) In Ni (CO)4, nickel has oxidation number of
– N – CH2 – CH2– N
OOCH2C CH2 – COO– zero. So option (a) is the answer.
8. (c) Complex compounds contains two 16. (b) Bond between C of organic molecule and
different metallic elements but give test metal atom.
only for one of them. Because complex ions 17. (d) Hybridisation
such as [Fe (CN)6]4– of K4 [Fe (CN)6], do [Fe(CN)6 ]4 -, [Mn(CN)6 ]4 -,
not dissociate into Fe2+ and CN– ions. d 2sp3 d 2sp3
9. (c) Number of unpaired electrons in central atom
[Co(NH3 )]3+ , [Ni(NH3 )6 ]2 +
4- 2- 2-
[Fe(CN)6 ] ,[CoCl4 ] ,[MnCl 4 ] d 2sp3 sp3d 2
zero three five
The greater the number of unpaired Hence [ Ni( NH 3 ) 6 ]2 + is outer orbital
electrons, the higher the value of magnetic complex.
moment
http://t.me/iitjeehelps

Haloalkanes and Haloarenes C – 57

Chapter
Haloalkanes andHaloarenes
24
1. An aromatic compound has molecular formula heating with Cu at 300°C gives an alkene C, what
C7H7Br. Give the possible isomers and the are A and C [2011]
appropriate method to distinguish them.[2016] (a) CH 3CH 2 Cl, CH 2 = CH 2
(a) 3 isomers; by heating with AgNO3 solution
(b) 4 isomers; by treating with AgNO3 solution (b) Me 3 CCl, MeCH = CH.Me
(c) 4 isomers; by oxidation (c) Me 3 CCl, Me 2 C = CH 2
(d) 5 isomers; by oxidation
2. The boiling point of alkyl halide are higher than (d) Me 2 CH. CH 2 Cl, Me 2 C = CH 2
those of corresponding alkanes because of 8. In which of the following conver sions,
(2015) phosphorus pentachloride is used as the
(a) dipole-dipole interaction reagent? [2010]
(b) dipole-induced dipole interaction
(a) H 2 C = CH 2 ¾¾® CH 3CH 2 Cl
(c) H-bonding
(d) None of the above (b) CH 3CH 2 OH ¾ ¾® CH 3CH 2 Cl
3. The number of double bonds in gammexane is :
[2014]
(c) H 3 C - O - CH 3 ¾
¾® CH 3Cl
(a) 0 (b) 1 (c) 2 (d) 3 (d) CH º CH ¾ ¾® CH 2 = CHCl
4. DDT is prepared by reacting chlorobenzene
9. Which of the following order is not correct ?
with : [2013]
[2009]
(a) CHCl3 (b) CCl3CHO
(c) CCl4 (d) C2H6 (a) MeBr > Me 2CHBr > Me3CBr >
5. In a SN2 substitution reaction of the type Et 3CBr(SN 2)
R - Br + Cl - ¾¾¾
DMF
® R - Cl + Br - (2013) (b) Me3CBr > Me 2 CHBr >
which one of the following has the highest Me2CH.CH 2 Br >
relative rate ? Me CH 2 CH 2 CH 2 Br.(E 2 )
(a) CH3 – CH2 – CH2Br
(c) PhCH 2 Br > Ph CHBrMe > PhCBrMe 2 >
(b) CH3 - CH - CH 2 Br
| PhCBrMePh(SN1)
CH3 (d) MeI > MeBr > MeCl > MeF (SN2)
10. Which of the following will give vinyl chloride ?
CH3
[2008]
| 600°C
(c) CH3 - C - CH 2 Br (a) CH 2 = CH 2 + Cl 2 ¾¾ ¾®
| KOH
CH (b) ClCH 2 - CH 2Cl ¾ethanol
¾¾ ¾®
3
(d) CH3CH2Br
Hg 2 +
6. The alkyl halide is converted into alcohol by : (c) CH º CH + HCl ¾¾¾®
[2012]
(a) elimination (b) halogenation (d) All
(c) addition (d) substitution 11. The following reaction proceeds through the
7. A is an optically inactive alkyl chloride which intermediate [2007]
on reaction with aqueous KOH gives B. B on ¾® RBr + CO 2 + AgBr
RCOOAg + Br2 ¾
EBD_7167
http://t.me/iitjeehelps

C – 58 BITSAT Topicwise Solved Papers


(a) RCOO · (b) R · (c) produce electrophile
(c) Br · (d) All (d) break bond
12. Function of AlCl3 in Friedel-Craft’s reaction is to 13. When CH3CH2Br reacts with sodium acetylide,
[2006] the main product is [2005]
(a) produce nucleophile (a) 1-butane (b) 1-butyne
(b) make new bond (c) 1-butene (d) 3-butane

Hints & Solutions


Br CH3 CH3 CH3 8. (b) When ethyl alcohol is treated with PCl 5 ,
Br then ethyl chloride is formed.
1. (c) , , , CH 3CH 2 - OH + PCl 5 ¾¾® D

Br Br
CH 3CH 2 - Cl + HCl + POCl 3
2. (a) Due to dipole-dipole interaction the 9. (c) The more is the stability of intermediate
boiling point of alkyl halide is higher as carbonium ion, the more is the chance of
compared to corresponding alkanes. S N 1 mechanism. The intermediates
3. (a) Gammexane is C6H6Cl6 or (6, 6, 6). It is a obtained will be
+ +
saturated compound so no double bond is Ph C H 2 (i), Ph C H - Me (ii),
there in it. Cl H
Cl + +
H Ph C- Me 2 (iii), Ph C MePh (iv).
H Cl The stabilty is of the order iv > iii > ii > i.
Cl H 10. (d) All given reactions give the vinyl chloride
H Cl by substitution.
Cl H
(a) by dehydrohalogen atom (b) and by
4. (b) DDT is pr epared by reaction of
addition (c).
chlorobenzene with CCl3CHO.
11. (d) Mechanism of Hunsdiecker’s reaction is
5. (d) SN2 mechanism is followed in case of
+ Br2
primary and secondary halides i.e., SN2
R– CO O Ag ¾–¾Ag¾¾ ® RCOOBr
reaction is favoured by small groups on Br
the carbon atom attached to halogens so · ·
CH3 CH2 Br > CH3 CH2 CH2 Br > ¾ ¾® R · + CO 2 ;
¾® RCO O + Br ¾
CH3 ·
| R · + RCOO Br ¾ ¾® R – Br + RCO O
CH3– CH – CH2Br > CH3 – C – Br 12. (c) Function of AlCl3 in Friedel craft’s reaction
| | is to produce electrophile. The mechanism
CH3 CH3
is followed
i.e. option (d) is correct. Cl
6. (d) Alkyl halide is converted into alcohol by | +
substitution. Cl - Al - + Cl - R + C6 H 6 ¾ ¾®
R - X + OH – ¾¾ ® R - OH + X - |
Cl
7. (c) Me3CCl ¾KOH ¾¾® Me 3 COH H
(A) tert . alcohol é d+ H d- ù |
Cu / 300° C H
ê 6 5 ......... Cl AlCl3ú ¾
¾® C 6+ H 5 + AlCl –
® CH 3 - CH = CH 2 + H 2O
¾¾¾¾ ë û | 4
| R
R
CH 3
or 13. (b) CH º C - Na + Br - CH 2 - CH 3 ¾
¾®
Me2 C = CH 2 CH º C - CH 2 - CH 3
(C) 1- Butyne
http://t.me/iitjeehelps

Alcohols, Phenols and Ethers C – 59

Chapter
Alcohols, Phenols and Ethers
25
CH3 (a) Benzene (b) Phenol
.. .. H+
1. OH
.. + O = C ® X [X] is
¾¾¾ (c) Thiophenol (d) Benzoic acid
..

CH3 4. Consider the following phenols :


[2016]
OH OH OH OH
CH3

(a) HO C CH3

OH NO2
CH3 NO2
CH3 (I) (II) (III) (IV)
..
(b) HO .O. C CH3
The decreasing order of acidity of the above
H phenols is [2014]
(a) III > IV > II > I (b) II > I > IV > III
CH3 (c) I > IV > II > III (d) III > IV > I > II
(c) HO C C CH3 5. The ionization constant of phenol is higher than
that of ethanol because : [2014]
OH
..
..

O (a) Phenoxide ion is bulkier than ethoxide


CH3
(b) Phenoxide ion is stronger base than
ethoxide
(d) HO C OH (c) Phenoxide ion is stabilized through
CH3 delocalization
(d) Phenoxide ion is less stable than ethoxide
2. Which of the following method gives better 6. Which of the following is not the product of
yield of p-nitrophenol? [2016]
dil.HNO 3 dehydration of OH ? [2013]
(a) Phenol ¾¾¾¾® p-Nitrophenol
20° C

(i) NaNO 2 + H 2SO 4 , 7 - 8° C


(b) Phenol ¾¾¾¾¾¾¾¾¾¾® (a) (b)
(ii) HNO 3

p-Nitrophenol
(i) NaOH (c) (d)
(c) Phenol ¾¾¾¾¾¾®
(ii) Conc. HNO 3
7. The compound which reacts with bleaching
(d) None of the three. powder to produce chloroform is [2012]
3. Sodium salt of benzene sulphonic acid on fusion (a) Methanol (b) Methyl formate
with caustic soda gives [2014] (c) Ethanol (d) Methyl chloride
EBD_7167
http://t.me/iitjeehelps

C – 60 BITSAT Topicwise Solved Papers


8. The reaction COOH
COOH
CH3 OH
(a) (b)
CH 3– C – ONa + CH3CH2Cl NaCl
CH3 CH3
H3C
CH3 – C – O – CH2 – CH3 (c) C O O H
H3C
CH3
is called : [2011]
(a) Williamson continuous etherification
process CH3
(b) Etard reaction
(c) Gatterman - Koch reaction (d) H3C C O O H
(d) Williamson Synthesis OH
9. Match List I (Reaction) with List II (Reagent)
and select the correct answer using the codes
given below the lists : [2010]
List I List II
I. Etard reaction A. Alcoholic KOH 13. In the reaction: [2008]
II. Hydroxylation B. Anhydrous AlCl3 CH3
III. Dehydro- C. Chromyl chloride |
Heated
halogenation CH3 - CH - CH 2 - O - CH 2 - CH3 + HI ¾¾¾¾ ®
IV. Friedel-Crafts D. Dilute alkaline Which of the following compounds will be
reaction KMnO4 formed?
(a) I-A, II-B, III-C, IV-B
(b) I-D, II-C, III-A, IV-B (a) CH3 - CH - CH3 + CH3CH 2 OH
(c) I-C, II-D, III-A, IV-B |
(d) I-B, II-A, III-D, IV-C CH3
10. Which of the following will not form a yellow (b) CH3 - CH - CH 2 OH + CH3CH3
precipitate on heating with an alkaline solution
|
of iodine? [2010] CH3
(a) CH3CH(OH)CH3
(b) CH3CH2CH(OH)CH3 CH3
|
(c) CH3OH (c) CH3 - CH - CH 2 OH + CH3 - CH 2 - I
(d) CH3CH2OH
CH3
11. Lucas reagent is [2008] |
(a) Conc. HCl + ZnCl2 (b) Dil. HCl + ZnCl2 (d) CH3 - CH - CH 2 - I + CH3CH 2OH
(c) H2SO4 + ZnCl2 (d) Conc. HCl + Zn
14. 3-methyl-2-butanol on treatment with HCl gives
12. Phenol is distilled with Zn dust followed by
predominantly : [2007]
Friedel Crafts alkylation with propyl chloride in
(a) 2-Chloro-2-methyl butane
the presence of AlCl3 to give a compound (B).
(b) 2-Chloro-3-methyl butane
(B) is oxidised in the presence of air to form the
(c) 2, 2-Dimethyl pentane
compound (C). The structural formula of (C) is
(d) None of above.
[2008]
http://t.me/iitjeehelps

Alcohols, Phenols and Ethers C – 61

Hints & Solutions


1. (d) Thus here, oxidation of phenol is minimised by
forming p-nitrosophenol.
3. (b)
H3 C .. H3 C H3 C Å ..
Å Å SO 3 - Na +
C=O +H C= O – H ¬¾® C– O
.. – H O - Na +
..

..
H3 C H3 C H3 C
NaOH + Na 2SO 3 + H 2 O
CH3 CH3 + 300°

.. ..
H3 C – C – OH
.. H3 C – C – O.. – H
H HCl

¬¾¾ ¬¾¾ OH

O
.. +O O
.. + NaCl
..

..

..

H H H
H+
4. (a) Electron withdrawing group (–NO 2 )
CH3 increases the acidity while electron
Å releasing group (–CH3, –H) decreases
H3C C OH2 acidity. Also effect will be more if functional
O H group is present at para position then ortho
and meta position.
5. (c) The acidic nature of phenol is due to the
OH CH3
formation of stable phenoxide ion in
solution
+
H3C C OH
ˆˆ† C6 H 5 O - + H 3O +
C6 H5OH + H 2O ‡ˆˆ
H
Phenoxide ion
The phenoxide ion is stable due to
resonance.
OH
O– O O
CH3 –
••
|
HO C OH
| ••

CH3

2. (b) OH O– O
d d
– –

OH OH OH ••

d

NaNO2, H2SO4 HNO3
¾¾¾¾¾¾® ¾¾® The negative charge is delocalized in the
7–8°C
benzene ring which is a stabilizing factor in
the phenoxide ion and increase in acidity
NO NO2
Phenol p-Nitrosophenol p-Nitrophenol of phenol. wheras no resonance is possible
in alkoxide ions (RO–) derived from alcohol.
EBD_7167
http://t.me/iitjeehelps

C – 62 BITSAT Topicwise Solved Papers


The negative charge is localized on oxygen 12. (c)
atom. Thus, alcohols are not acidic.
6. (b) OH

Zn AlCl 3
CH3CH2CH2Cl
OH + HÅ ® O—H
Å CH3
C O O H CH3 CH CH3
H CH3
¯ O2
H

Å
¬ 13. (c) In the cleavage of mixed ethers with two
different alkyl groups, the alcohol and alkyl
¬

iodide that form depend on the nature of


¯ alkyl group. When primary or secondary
alkyl groups are present, it is the lower alkyl
group that forms alkyl iodide therefore
D
CH3 - C H - CH 2 - O - CH 2 - CH 3 + HI ¾¾®
|
7. (c) CH3 – CH2OH + Cl2 (from bleaching powder) CH3
¾¾ ® CCl3 – CHO + 3HCl
CCl3 – CHO + Ca(OH)2¾¾ ® CH3
2CHCl3+ (HCOO)2Ca |
8. (d) Williamson synthesis is one of the best CH3 - CH - CH 2 OH + CH3CH 2 I
methods for the preparation of symmetrical
CH3
and unsymmetrical ethers. In this method,
an alkyl halide is allowed to react with 14. (b) CH3– CH – CH – CH3+ HCl
sodium alkoxide.
9. (c) OH
10. (c) CH3OH does not have – CH(OH)CH3 3-Methyl-2-butanol
group hence it will not form yellow
precipitate with an alkaline solution of
iodine (haloform reaction). CH3
11. (a) Lucas reagent is a mixture of conc. HCl and |
ZnCl2. CH3 - CH - CH - CH3 + HCl
|
Cl
2- chloro -3- methylbutane
http://t.me/iitjeehelps

Aldehycles, Ketones and Carboxylic acids C – 63

Chapter
Aldehycles, Ketones and
Carboxylic acids 26
1. The most likely acid-catalysed aldol (a) B > A > D > C (b) B > D > C > A
condensation products of each of the two (c) A > B > C > D (d) A > C > B > D
aldehydes I and II will respectively be [2016] 4. When 2-hydroxybenzoic acid is distilled with
zinc dust, it gives [2013]
CHO (I) and (II) (a) phenol
(b) benzoic acid
(c) benzaldehyde
(d) a polymeric compound
(a) and 5. Nucleophilic addition reaction will be most
favoured in [2013]
(a) (CH3)2C = O (b) CH3CH2CHO
(c) CH3CHO O
||
(d) CH3 – CH2 – CH2C – CH3
(b) and
6. CCl 3CHO (Chloral) does not give aldol
condensation because [2012]
(a) of three highly electronegative Cl atoms.
(b) of steric hinderance caused by Cl atoms.
(c) it does not have a-H atom.
(c) and (d) All of these
7. The preparation of ethyl acetoacetate involves
[2012]
(a) Wittig reaction
(b) Cannizzaro’s reaction
(d) and (c) Reformatsky reaction
(d) Claisen condensation.
8. Which one of the following pairs is not correctly
matched? [2012]
(a) > C = O ® > CH2 (Clemmensen reduction)
(b) > C = O ® > CHOH
(Wolf – Kishner reduction)
2. The reaction, [2014] (c) – COCl ® – CHO (Rosenmund reduction)
CO + H2O
CH3 - CH = CH 2 ¾¾¾¾¾® (d) – C º N ® – CHO (Stephen reduction)
H+ 9. Which of the following esters cannot undergo
CH3- CH - CH 3
| Claisen self condensation ? [2011]
COOH (a) CH3CH2CH2CH2COOC2H5
is known as
(a) Wurtz reaction (b) C6H5–COOC2H5
(b) Koch reaction (c) C6H11CH2COOC2H5
(c) Clemmensen reduction (d) C6H5CH2COOC2H5
(d) Kolbe’s reaction 10. Schotten-Baumann reaction is a reaction of
3. The correct order of decreasing acid strength of phenols with [2011]
trichloroacetic acid (A), trifluoroacetic acid (B), (a) Benzoyl chloride and sodium hydroxide
acetic acid (C) and formic acid (D) is : [2014] (b) Acetyl chloride and sodium hydroxide
EBD_7167
http://t.me/iitjeehelps

C – 64 BITSAT Topicwise Solved Papers


(c) Salicylic acid and conc. H2SO4 20. The correct statement regarding a carbonyl
(d) Acetyl chloride and conc H2SO4 compound with a hydrogen atom on its alpha
11. Identify X, [2011] carbon, is : [2006]
H3C (a) a carbonyl compound with a hydrogen atom
CH3MgI
C = O ¾¾¾¾ ® on its alpha-carbon never equilibrates with
H3C dry ether its corresponding enol.
H 2O
Intermediate ¾¾¾ (b) a carbonyl compound with a hydrgen atom
®X on its alpha-carbon rapidly equilibrates
(a) CH3OH (b) Ethyl alcohol with its corresponding enol and this
(c) Methyl cyanide (d) tert-Butyl alcohol process is known as aldehyde-ketone
12. The reagent (s) which can be used to distinguish equilibration.
acetophenone from benzophenone is (are) (c) a carbonyl compound with a hydrogen atom
[2011] on its alpha-carbon rapidly equilibrates
(a) 2,4- Dinitrophenylhydrazine with its corresponding enol and this
(b) Aqueous solution of NaHSO3 process is known as carbonylation.
(c) Benedict reagent (d) a carbonyl compound with a hydrogen atom
(d) I2and NaOH. on its alpha-carbon rapidly equilibrates
13. Formic acid and acetic acid can be distinguished with its corresponding enol and this
by [2010] process i s known as keto-en ol
(a) Phenyl hydrazine (b) NaHCO3 tautomerism.
(c) Tollen’s reagent (d) None of these 21. If acetylchloride is reduced in the presence of
14. When esters are hydrolysed the product gives BaSO4 and Pd, then [2006]
hydrogen ions. The product which gives (a) CH3CHO is formed
hydrogen ion is : [2009] (b) CH3COOH is formed
(a) Acid (c) CH3CH2OH is formed
(b) Alcohol (d) CH3COCH3 is formed
(c) Both 22. Benzoic acid reacts with conc. HNO3 and H2SO4
(d) Either acid or alcohol to give : [2005]
15. Which of the following compound can not used (a) 3-Nitrobenzoic acid
in preparation of iodoform? [2009] (b) 4-Benzene sulphonic acid
(a) CH3CHO (b) CH3COCH3 (c) 4-Nitrobenzoic acid
(c) HCHO (d) 2- propanol (d) 2-Nitrobenzoic acid
16. When sodium propionate is heated with sodalime 23. Oxidation of acetaldehyde with selenium dioxide
which product is formed : [2008] produces [2005]
(a) Propyl amine (b) Ethane (a) Ethanoic acid (b) Methanoic acid
(c) Acetone (d) Acetaldehyde (c) Glyoxal (d) Oxalic acid
17. When Acetic Acid is heated with P2O5 the 24. An organic compound 'X' having molecular
compound formed will be : [2007] formula C5H10O yields phenyl hydrazone and
(a) (CH3CO)2O (b) CH3COCH3 gives negative response to the Iodoform test
(c) CH3CHO (d) CH4 and Tollen's test. It produces n-pentane on
18. In which reaction, an aromatic aldehyde is reduction. 'X' could be :- [2005]
treated with acetic anhydride in presence of (a) 2-pentanone (b) 3-pentanone
corresponding salt of the acid to give an (c) n-amyl alcohol (d) pentanal
unsaturated aromatic acid ? [2007]
(a) Friedel-Crafts reaction
(b) Perkin’s reaction 25. Treatment of cyclopentanone =O with
(c) Wurtz reaction
methyl lithium gives which of the following
(d) None of these
species? [2005]
19. In the reduction
(a) Cyclopentanonyl cation
R - CHO + H 2 ¾¾
® RCH 2 OH
(b) Cyclopentanonyl radical
the catalyst used is : [2006] (c) Cyclopentanonyl biradical
(a) Ni (b) Pd
(c) Pt (d) Any of these (d) Cyclopentanonyl anion
http://t.me/iitjeehelps

Aldehycles, Ketones and Carboxylic acids C – 65

Hints & Solutions


O
1. (d) OH
+ C2H5ONa
H – H 2O
CHO ¾¾®CH COO C H + H.CH
CHO ¾¾®
.CO.OC H CH3C. CH2COOC2 H5+ C2H5OH
Ethyl acetoacetate
(b -ketoester)
I
8. (b) Like clemmensen reduction, Wolf-Kishner
OH reduction involves reduction of > C = O
–H O
CHO CHO to > CH2 , of course by different reagent.
or CHO 9. (b) Claisen condensation is given by esters
having two a-hydrogen atoms
10. (a)
Conjugated system (stable)
OH OCOC6H5

2 . + C6H5COCl
aq. NaOH

phenyl benzoate
It does not lose water because the double The function of NaOH is
bond so formed will be isolated. (i) To convert phenol to more stronger
2. (b) nucleophile PhO–
3. (a) CF3 COOH > CCl3 COOH > HCOOH > (ii) To neutralize the acid formed
CH3COOH (Ka order)
The halogenated fatty acids are much 11. (d) H3 C d- d+
stronger acids than the parent fatty acid C = O + CH 3 - MgI ¾
¾®
and more over the acidity among the H3 C
halogenated fatty acid increases almost
proportionely with the increase in H3 C CH3
electronegativity of the halogen present. H OH |
C H2 O
O MgI ¾¾¾ ® H3C – C – OH
Further formic acid having no alkyl group |
H3 C
is more acidic than acetic acid. CH3
4. (b) CH3
Tert butyl
5. (c) Aldehydes are more reactive than ketones + Mg(OH)I alcohol
due to +I effect of –CH3 group. There are 12. (d) I2 and NaOH react with acetophenone
two – CH3 group in acetone which reduces (C6H5COCH3) to give yellow ppt. of CHI3
+ve charge density on carbon atom of but benzophenone (C 6H5COC6H5) does
carbonyl group. More hindered carbonyl not and hence can be used to distinguish
group too becomes less reactive. So in the between them.
given case CH3CHO is the right choice. 13. (c) Formic acid(HCOOH) has aldehydic group.
6. (c) Aldol condensation is given by carbonyl 14. (a) When esters are hydrolysed , then acid and
compounds which have a-H atom. alcohol are formed, where acid gives
7. (d) In Claisen condensation Intermolecular hydrogen ion.
condensation of esters containing O
a-hydrogen atom in presence of strong ||
( Conc .H SO )
base produce b-keto ester R - C - O - R + HOH ¾¾ ¾ ¾2¾¾
4 ®

Ester
CH3COO C2H5 + H.CH2.CO.OC2H5
ethyl acetate d-
R C OO +d H + R – OH
EBD_7167
http://t.me/iitjeehelps

C – 66 BITSAT Topicwise Solved Papers

15. (c) Formaldehyde can not produce iodoform, O O


as only those compound which contains || ||
either CH3 — C — CH2 — C — CH3
Ketonic form
CH3 - CH - group or CH3 -CH - group OH O
| || | ||
OH O CH 3 — C CH — C — CH 3

||
on reaction with potassium iodide and sod. enolic form
hypochlorite yield iodoform. Pd / BaSO
21. (a) CH 3 COCl + 2 H ¾¾¾ ¾¾4®
16. (b) When sodium propionate is heated with
soda lime, then ethane is formed. CH 3CHO + HCl
D
22. (a) –COOH group when attached to benzene
CH 3CH 2 COONa + NaOH ¾¾ ® ring deactivates the ring and substitution
Soda lime
occurs at m-position. (HNO3 + H2SO4) is a
Sod. Propionate
source of NO2(+) (electrophile) which
CH3 - CH 2 - H + Na 2 CO3 attacks at m-position.
Ethane Sod. carbonate COOH
17. (a) P2O5 is a dehydrating agent. When it is
H2SO4
+ HNO3 ¾¾¾¾
heated with CH3COOH, it eliminates a water ®
molecule from CH3COOH and thus, acetic COOH
anhydride is formed.
CH 3CO OH CH 3CO NO2
+ P2 O5 3- nitrobenzoic acid
¾¾ ¾® O + H2O
CH 3COOH Heat
CH3CO
23. (c) Seleium dioxide oxidises aldehydes and
ketones at a-position to carbonyl group.
2 moles of acetic acid Acetic anhydride
CH 3CHO ® OHC.CHO
O
O O 24. (b) H3C—CH 2— C—CH2—CH3
H O (3-pentanone)
18. (b) + O
does not give iodoform test due to absence
aromatic aldehyde acetic anhydride
O
O of (CH3—C—) group. It also does, not
1 – NaOAC give Tollen’s test.
2 – base OH
On reduction it gives n-pentane
a-b -unsaturated acid reduction
H3C—CH2—C—CH2—CH3
Perkin’s Reaction Zn-Hg/HCl

O or NH2-NH2/OH
19. (d) Any one of Ni, Pt or Pd can be used in the
reduction of aldehydes. CH 3—CH 2—CH2—CH 2—CH 3
20. (d) Keto-enol tautomerism is possible only in n-pentane
those aldehydes and ketones which have –
O O
at least one a-hydrogen atom, which can 25. (d) CH3
convert the ketonic group to the enolic – Å
group. e.g. + CH3 Li ®

Cylopentanoyl anion
http://t.me/iitjeehelps

Chapter
Amines
CHO
27
1. Which of the following represents Schotten-
Baumann reaction? [2017] (d) + HCHO + NaOH
(a) formation of amides from amines and acid
chlorides/NaOH
OCH3 CH3OH
(b) formation of amines from amides and
LiAlH4 CHO
(c) formation of amines from amides and
Br2/NaOH
(d) formation of amides from oxime and H2SO4 CH3O
2. Which reagent converts nitrobenzene to
N-phenylhydroxylamine? [2017] 4. Aniline reacts with phosgene and KOH to form
(a) Zn/HCl (b) H2O2 [2014]
(c) Zn/NH4Cl (d) LiAlH4
3. The carbylamine reaction is [2015] OH

OH
(a)
(a) NaOH
+ CHCl3
OH O
CHO
C Cl
(b)
NH2

(b) D NHCOCl
+ CHCl 3 + 3KOH (c)
N C

NCO
(d)

COCH3 5. Which of th e following does not give


effervescences of CO 2 with aq. NaHCO 3
(c) Zn/Hg + Conc.HCl solution? [2014]
+ 4H (a) H2CO3
CH2CH3
OH
NO2

(b)
NO2
EBD_7167
http://t.me/iitjeehelps

C – 68 BITSAT Topicwise Solved Papers

OH 10. When ethylamine react with sodium metal, the


gas evolved is [2010]
O2N NO2
(c) (a) H2 (b) C2H5
(c) N2 (d) NH3
11. Which of the following compound is obtained
(d) None of these
by heating ammonium cyanate? [2009]
6. Identify the prdouct C in the series [2013] (a) alkane
Na / C H OH (b) urea
CH 3CN ¾¾¾¾¾¾
2 5
®A
(c) ethylamine
HNO Cu / 573K (d) ammonium thiocyanate
¾¾¾¾
2 ® B ¾¾¾¾¾
®C
12. Hydrolysis of acetonitrile in acid medium gives
(a) CH 3COOH
[2008]
(b) CH 3CH 2 NHOH (a) CH 3 NC (b) CH 3COOC 2 H 5
(c) CH 3CONH 2 (c) CH 3COOH (d) CH 3CH 2OH
(d) CH 3CHO SnCl
13. CH 3 NO 2 ¾ ¾ ¾
¾2®
CH 3 X, the ‘X’ contain
7. When NH3 is passed over heated metal A, its HCl
amide is formed. The metal is [2013] [2007]
(a) Mg (b) K (a) –NH2 (b) – COOH
(c) Al (d) Pb (c) – CHO (d) (CH3CO)2O
8. Stephen’s reduction is a method for preparation 14. Methyl cyanide can be converted into acetic
of [2012] acid by one of the following reaction : [2006]
(a) Amines (b) Alcohols (a) reduction (b) hydrolysis
(c) Aldehydes (d) Carboxylic acids (c) electrolysis (d) decarboxylation
9. Aniline reacts with nitrous acid to produce 15. The correct order of the increasing basicity of
[2011] methyl amine, ammonia and aniline is [2005]
(a) phenol (a) methyl amine < aniline < ammonia
(b) nitrobenzene (b) aniline < ammonia < methyl amine
(c) chlorobenzene (c) aniline < methyl amine < ammonia
(d) benzene diazonium chloride (d) ammonia < aniline < methyl amine

Hints & Solutions


1. (a) Schotten-Baumann Conditions 2. (c) Reducing reagent is needed, as shown in
given reaction.
O O
H
R NaOH
R NO2 N
+ H2N – R ¢
Zn OH
Cl N – R¢ aq. NH 4Cl
H
3. (b) Primary amines (aromatic or aliphatic) on
Th e use of added base to drive the warming with chloroform and alcoholic
equilibrium in the formation of amides from KOH, gives carbylamine having offensive
amines and acid chlorides. smell. This reaction is called carbylamine
reaction.
http://t.me/iitjeehelps

Amines C – 69

NH2 10. (a) When ethylamine is heated with sodium


metal, then hydrogen gas is evolved.
D 2C 2 H 5 NH 2 + 2Na¾ ¾¾®
+ CHCl3 + 3KOH
N C 2C 2 H 5 NHNa + H 2 ­
11. (b) Urea is obtained by heating ammonium
+ 3KCl + 3H2O cyanate
D
NH 4 CNO ¾¾
® NH 2CONH 2
4. (d) C6 H5 NH 2 + COCl2 ® C6 H5 NH.COCl Ammonium cyanate Urea
D 12. (c) Hydrolysis of acetonitrile in acid medium,
+ HCl ¾¾® C 6 H 5 NCO + HCl
5. (d) Each of them decompose aqueous solution gives acetic acid.
OH-
of NaHCO3. Recall that when at least two CH 3 - C º N + 2H 2O ¾¾¾®
NO2– groups are present in ortho and para
positions with respect to phenolic –OH CH 3COOH + NH 3
group, it becomes highly acidic and gives 13. (a) On hydrogenation of methyl nitrate in the
effervescences of CO2 with aq. NaHCO3 presence of SnCl2 and HCl, then methyl
solution. amine is formed.
Na / C H OH
6. (d) CH 3CN ¾¾ ¾2¾5¾
¾® CH 3CH 2 NH 2 SnCl
(A) CH 3 NO 2 ¾¾ ¾
¾2®
CH 3 NH 2 + 2H 2O
HCl
HNO Cu
¾¾ ¾
¾2 ® CH CH OH ¾¾® CH CHO
3 2 3
( B) 573 K ( C)
14. (b) CH3CN can be converted into acetic acid
by hydrolysis.
7. (b) When Potassium is treated with ammonia,
then potassium amide is obtained. H+ / H O
CH3 CN ¾¾¾¾
2 ® CH COOH
3
1
K + NH 3 ¾¾® KNH 2 + H 2
15. (b) In aniline the lone pair on N is involved in
Pot. amide 2
delocalization with benzene ring and is not
8. (c) In Stephen’s reduction cyanides are
reduced to aldehydes by SnCl2 +conc.HCl. available for protonation. Methyl amine is
9. (d) When aniline is treated with nitrous acid in a stronger base than ammonia because + I
the presence of HCl, then benzene effect of methyl group increases electron
diazonium chloride is obtained. density on N making it more basic than NH3.
NH 2 Å
NH 2 NH 2
CH3 ®– NH 2
+ NaNO2 + 2HCl¾¾ ¾ ® + I Effect increases
basicity.
C 6 H 5 N = NCl + NaCl + 2H 2O
Cannot be protonated. least basic
(Benzene diazonium chloride)
EBD_7167
http://t.me/iitjeehelps

C – 70 BITSAT Topicwise Solved Papers

Chapter
Biomolecules
28
1. Which has glycosidic linkage? [2017] (b) spectroscopic analysis shows planar
(a) amylopectin (b) amylase structure of - C - NH - bond
(c) cellulose (d) all of these ||
2. Which one of the following is an amine hormone ? O
[2017] (c) C–N bond length in proteins is smaller than
(a) Thyroxine (b) Oxypurin usual C–N bond length
(c) Insulin (d) Progesterone (d) none of these
10. The helical structure of protein is stabilised by
3. Fructose on reduction gives a mixture of two
[2012]
alcohols which are related as [2016]
(a) peptide bonds
(a) diastereomers (b) epimers
(b) dipeptide bonds
(c) both (a) and (b) (d) anomers. (c) hydrogen bonds
4. What will happen when D-(+)-glucose is treated (d) vander waals forces
with methanolic —HCl followed by Tollens’ 11. Complete hydrolysis of cellulose gives [2012]
reagent ? [2016] (a) D-ribose (b) D-glucose
(a) A black ppt. will be formed (c) L-glucose (d) D-fructose
(b) A red ppt. will be formed 12. The structural feature which distinguishes
(c) A green colour will appear proline from natural a-amino acids? [2011]
(d) No characteristic colour or ppt. will be (a) Proline is optically inactive
formed. (b) Proline contains aromatic group
5. The metal present in vitamin B12 is [2015] (c) Proline is a dicarboxylic acid
(a) magnesium (b) cobalt (d) Proline is a secondary amine
(c) copper (d) zinc 13. The secondary structure of a protein refers to
6. A sequence of how many nucleotides in [2010]
messenger RNA makes a codon for an amino (a) fixed configuration of the polypeptide back-
acid? [2015] bone
(a) Three (b) Four (b) a – helical backbone
(c) One (d) Two (c) hydrophobic interactions
7. Which functional group participates in (d) sequence of a – amino acids.
14. Which of the following statements about
disulphide bond formation in proteins? [2014]
vitamin B-12 is incorrect? [2009]
(a) Thioester (b) Thioether
(a) It has a cobalt atom
(c) Thiol (d) Thiolactone
(b) It also occurs in plants
8. Insulin production and its action in human body
(c) It is also present in rain water
are responsible for the level of diabetes. This
(d) It is needed for human body in very small
compound belongs to which of the following
amounts
categories ? [2013]
15. The hormone which controls the processes of
(a) An enzyme (b) A hormone
burning of fats, proteins and carbohydrates to
(c) A co-enzyme (d) An antibiotic
liberate energy in the body is [2008]
9. Which statement is incorrect about peptide
(a) Cortisone (b) Adrenaline
bond? [2013]
(c) thyroxine (d) Insulin
(a) C–N bond length in proteins is longer than
16. The water soluble vitamin is [2007]
usual C–N bond length
(a) B1 (b) K (c) E (d) D
http://t.me/iitjeehelps

Biomolecules C – 71
17. Scurvy is caused by the deficiency of vitamin (a) Thyroxine (b) Progesterone
[2006] (c) Adrenaline (d) Estron
(a) D (b) C (c) B6 (d) A 19. Deficiency of vitamin E causes [2005]
18. Which of the following hormones contain iodine? (a) Loss of fertility (b) Impaired clotting
[2005] (c) Scurvy (d) Night blindness

Hints & Solutions


1. (d) Glycosidic linkage is a type of covalent 8. (b) Insulin is a biochemically active peptide
bond that joins either two carbohydrate harmone secreted by pancreas.
(sugar) molecule or one carbohydrate to 9. (a) Due to resonance,
another group. All molecules show such –
O O
type of linkages. +
– C – NH – – C = NH – , C – N
2. (a) Thyroxine is an amine hormone. bond acquires some double bond character,
3. (c) Ketoses on reduction produce a new chiral hence shorter in length
carbon leading to the formation of two 10. (c) Fibrous proteins have thread like molecules
isomeric alcohols which are diastereomeric which lie side by side to form fibres. The
as well as C–2 epimers. various molecules are held together by
4. (d) Reaction of D-(+)-glucose with methanolic hydrogen bonds.
—HCl leads to formation of methyl H+
glucoside (C1—OH group is methylated) 11. (b) (C 6 H 10 O 5 ) n + nH 2 O ¾¾® nC 6 H 12 O 6
D - Glu cos e
which, being acetal, is not hydrolysable
12. (d) Proline contains imino (secondary amino),
by base, so it will not respond Tollens’
reagent. NH group
5. (b) Cobalt is present in vitamin B12. 13. (b) The secondary structure of a protein refers
6. (a) The sequence of bases in mRNA are read to the shape in which a long peptide chain
in a serial order in groups of three at a can exist. There are two different
time. Each triplet of nucleotides (having a conformations of the peptide linkage
specific sequence of bases) in known as present in protein are a – helix and b –
codon. Each codon specifies one amino conformation. The a – helix always has a
acid. Further since, there are four bases. right handed arrangement.
therefore, 43 = 64 triplets or codons are In b – conformation all peptide chains are
possible. streched out to nearly maximum extension
R -S-S- R and then laid side by side held together by
7. (c) 2R - S - H
Thiol Disulphide intermolecular hydrogen bonds. The
Example : structure resembles the pleated folds of
drapery and therefore is known as b –
2HO 2C CHC H 2SH pleated shelt.
| 14. (b) Vitamin B12 does not occur in plants.
NH2
Cysteine 15. (d) Insulin
16. (a) B1 is water soluble vitamin
HO 2CCHC H 2S - SCH 2 CHC O 2 H 17. (b) Scurvy is caused by the deficiency of
| | vitamin C
NH 2 NH2
Cystine
18. (a) Thyroxine contains iodine.
19. (a) Loss of fertility is caused by vitamin E
EBD_7167
http://t.me/iitjeehelps

Chapter
Polymers
29
1. Which of the following can be repeatedly soften (a) CF2 = CF2 (b) CH2 = CHCl
on heating? [2017] (c) CCl2 = CCl2
(i) Polystyrene (ii) Melamine Cl
(iii) Polyesters (iv) Polyethylene |
(v) Neoprene (d) CH 2 = C — CH = CH 2
(a) (i) and (iii) (b) (i) and (iv) 5. Ebonite is [2007]
(c) (iii), (iv) and (v) (d) (ii) and (iv) (a) Natural rubber
2. Formation of polyethylene from calcium carbide (b) Synthetic rubber
takes place as follows [2016] (c) Highly vulcanized rubber
CaC 2 + 2H 2 O ¾ ¾® Ca (OH ) 2 + C 2 H 2 (d) Polypropene
6. Biodegradable polymer which can be produced
C2 H2 + H2 ¾ ¾® C 2 H 4
from glycine and aminocaproic acid is : [2008]
nC 2 H 4 ¾
¾® ( - CH 2 - CH 2 - ) n (a) PHBV (b) Buna - N
The amount of polyethylene obtained from (c) Nylon 6, 6 (d) Nylon 2- nylon 6
64.1 kg of CaC2 is 7. Polymer formation from monomers starts by
(a) 7 kg (b) 14 kg (c) 21 kg (d) 28 kg [2006]
3. Polymer used in bullet proof glass is [2015] (a) condensation reaction between monomers
(a) Lexan (b) PMMA (b) coordinate reaction between monomers
(c) Nomex (d) Kevlar (c) conversion of monomer to monomer ions
4. Which one of the following monomers gives the by protons
polymer neoprene on polymerization ? [2014] (d) hydrolysis of monomers.

Hints & Solutions


1. (b) Polystyrene and polyethylene belong to Cl
|
the category of thermoplastic polymers K S O
4. (d) nCH 2 = CH - C = CH 2 ¾¾2 ¾
2¾8®
which are capable of repeatedly softening Chloropren e
on heating and harden on cooling. Cl
|
2. (d) The concerned chemical reactions are -( CH 2 - CH = C - CH 2 -
)n
Neoprene
(i) CaC 2 + 2H 2 O ® Ca (OH) 2 + C 2H2
64 kg Ethyne , 26kg
5. (c) Ebonite is a hard highly vulcanized rubber,
(ii) C 2 H 2 + H 2 ® C2H 4 containing 20-30%, rubber
Ethylene , 28 kg 6. (d) H2 N—CH2—COOH +
Glycine
(iii) nC2 H 2 ® [-CH 2 - CH 2 -]n
n´28 kg n´28 kg polythene H2 N — (CH2)5—COOH ®
or 28 kg or 28 kg Aminocapric acid
Thus 64 kg of CaC2 gives 26 kg of acetylene (HN—CH2—CO—NH— (CH2)5—CO) —
which in turn gives 28 kg of ethylene whose Nylon-2-nylon-6
28 kg gives 28 kg of the polymer, polythene. 7. (a) Polymerisation starts either by condensation
3. (b) PMMA is used in bullet proof glass or addition reactions between monomers
http://t.me/iitjeehelps

Chapter
Chemistry in Action
30
62. The important antioxidant used in food is [2016] (b) N-acetyl-para-aminophenol
(a) BHT (b) BHC (c) diazepam
(c) BTX (d) All the three (d) tetrahydrocatenol
47. Cobalt (60) isotope is used in the treatment of : 3. Bithional is generally added to the soaps as an
[2015] additive to function as a/an : [2008]
(a) Heart diseases (b) Skin diseases (a) Dryer (b) Buffering agent
(c) Diabetes (d) Cancer (c) Antiseptic (d) Softner
18. Which of the following can possibly be used 16. The substance which is not an artificial
as analgesic without causing addiction and sweetener [2007]
modification? [2014] (a) Sucralose (b) Alitame
(a) morphine (c) Saccharin (d) Sucrose

Hints & Solutions


62. (a) BHT is an important antioxidant used in 18. (b)
food. 3. (c) Bithionol is added to soaps to impart
47. (d) Cobalt (60) isotope is used in the treatment antiseptic properties.
of cancer.
16. (d) Sucrose is a natural sweetner.
EBD_7167
http://t.me/iitjeehelps

C – 74 BITSAT Topicwise Solved Papers

Chapter
Analytical Chemistry
31
1. Correct formula of the complex formed in the 8. In the reaction of KMnO4 with an oxalate in
brown ring test for nitrates is [2017] acidic medium, MnO-4 is reduced to Mn 2+ and
(a) FeSO4. NO (b) [Fe(H2O)5NO]2+ C 2 O 24 - is oxidised to CO2. Hence, 50 ml of 0.02
(c) [Fe(H2O)5NO]+ (d) [Fe(H2O)5NO]3+ M KMnO4 is equivalent to [2012]
2. Sometimes, the colour observed in Lassaigne’s (a) 100 ml of 0.05 M H2C2O4
test for nitrogen is green. It is because [2016] (b) 50 ml of 0.05 M H2C2O4
(a) of green colour of ferrous sulphate (c) 25 ml of 0.2 M H2C2O4
(b) ferric ferrocyanide is also green (d) 50 ml of 0.10 M H2C2O4
(c) of green colour of copper sulphate 9. Which of the following is soluble in yellow
(d) of excess of Fe3+ ions whose yellow colour ammonium sulphide? [2012]
makes the blue colour of ferric ferrocyanide (a) CuS (b) CdS (c) SnS (d) PbS
to appear green. 10. Which of the following cannot give iodometric
3. Which of the following ions can be separated titration? [2011]
by aq. NH4OH in presence of NH4Cl [2014] (a) Fe3+ (b) Cu2+ (c) Pb2+ (d) Ag2+
(a) Al3+ and Fe3+ (b) Cr3+ and Al3+ 11. Acetaldehyde and acetone can be distinguished
(c) Cu2+ and Al3+ (d) None of these by : [2011]
4. 3.92 g of ferrous ammonium sulphate react (a) Iodoform test
N (b) Nitroprusside test
completely with 50 ml KMnO4 solution.
10 (c) Fehlings solution test
The percentage purity of the sample is (d) C & P test
[2014, 2006] 12. When H2S gas is passed through the HCl
(a) 50 (b) 78.4 (c) 80 (d) 39.2 containing aqueous solution of CuCl2, HgCl2,
5. A mixture of chlorides of copper, cadmium, BiCl3 and CoCl2 it does not precipitate out
chromium, iron and aluminium was dissolved in [2010]
water acidified with HCl and hydrogen sulphide (a) CuS (b) HgS (c) Bi2S3 (d) CoS
gas was passed for sufficient time. It was 13. Which one of the following statements is correct ?
filtered, boiled and a few drops of nitric acid [2010]
were added while boiling. To this solution (a) From a mixed precipitate of AgCl and AgI,
ammonium chloride and sodium hydroxide were ammonia solution dissolves only AgCl
added and filtered. The filterate shall give test (b) Ferric ions give a deep green precipitate
for. [2013] on adding potassium ferrocyanide solution
(a) Sodium and iron (c) On boiling a solution having K+, Ca2+ and
(b) Sodium and aluminium
HCO 3- ions we get a precipitate of
(c) Aluminium and iron
K2Ca(CO3)2
(d) Sodium, iron, cadmium and Al
(d) Manganese salts give a violet borax bead
6. Volume of 3% solution of sodium carbonate
test in the reducing flame
necessary to neutralise a litre of 0.1 N sulphuric
acid [2013] 14. Three separate samples of a solution of a single
salt gave these results. One formed a white
(a) 176.66 ml (b) 156.6 ml
precipitate with excess ammonia solution, one
(c) 116.0 ml (d) 196.1 ml
formed a white precipitate with dil. HCl solution
7. 0.45 g of acid molecular weight 90 is neutralised
and one formed a black precipitate with H2S.
by 20 ml of 0.5N caustic potash. The basicity of
The salt could be [2010]
acid is [2012]
(a) 1 (b) 2 (c) 3 (d) 4 (a) AgNO3 (b) Pb(NO3)2
(c) Hg(NO3)2 (d) MnSO4
http://t.me/iitjeehelps

Analytical Chemistry C – 75
15. Experiment to study kinetics of the dissociation 22. A gas 'X' is passed through water to form a
of hydrogen peroxide must be performed by saturated solution. The aqueous solution on
group of two or three so that– [2010] treatment with silver nitrate gives a white
(a) when one is recording data other should precipitate. The saturated aqueous solution also
be swirling flask at constant rate dissolves magnesium ribbon with evolution of
(b) experiment can be performed by one student a colourless gas 'Y'. Identify 'X' and 'Y'. [2007]
only as outcomes are independent on rate (a) X = CO2, Y = Cl2 (b) X = Cl2, Y = CO2
of mixing of mixture 1 and 3 (c) X = Cl2, Y = H2 (d) X = H2, Y = Cl2
(c) for safety purpose 23. A one litre flask is full of brown bromine vapours.
(d) none of these The intensity of brown colour of vapour will
16. Ammonia forms the complex ion [Cu(NH3)4]2+ not decrease appreciably on adding to the flask
with copper ions in alkaline solutions but not in some [2007]
acidic solutions. What is the reason for it ? (a) Pieces of marble
[2009] (b) Animal charcoal powder
(a) In acidic solutions protons coordinate with (c) Carbon tetrachloride
(d) Carbon disulphide
ammonia molecules forming NH +4 ions and 24. An organic compound is treated with NaNO2
NH3 molecules are not available and dil. HCl at 0°C. The resulting solution is
(b) In alkaline solutions insoluble Cu(OH)2 is added to an alkaline solution of b-naphthol
precipitated which is soluble in excess of where by a brilliant red dye is produced. It shows
any alkali the presence of [2007]
(c) Copper hydroxide is an amphoteric (a) – NO2 group
substance (b) aromatic –NH2 group
(d) In acidic solutions hydration protects (c) – CONH2 group
copper ions (d) aliphatic – NH2 group
17. An aqueous solution of a substance gives a 25. The gas that turns lime water milky is [2006]
white precipitate on treatment with dil. HCl which (a) CO2 (b) SO2
dissolves on heating. When H2S is passed (c) Both of these (d) None of these
through the hot acidic solution, a black 26. Phenol can be distinguished from ethyl alcohol
precipitate is obtained. The substance is a by all reagents except [2006]
[2009] (a) NaOH (b) FeCl3
(c) Br2/H2O (d) Na.
(a) Hg 22 + salt (b) Cu2+ salt 27. The correct method of finding the pH of a
(c) Ag+ salt (d) Pb2+ salt solution is to [2005]
18. The weight of oxalic acid required to neutralise (a) heat the solution in a test tube and expose
100 ml of normal NaOH [2009] the pH paper to the vapours formed.
(a) 6.3 g (b) 126 g (c) 530 g (d) 63 g (b) pour few drops of the solution from the
19. How do we differentiate between Fe3+ and Cr 3+ test tube on the pH paper.
in group III? [2008] (c) drop the pH paper in the solution.
(a) by taking excess of NH4OH solution (d) put a drop of the solution on the pH paper
(b) by increasing NH +4 ion concentration using a dropper.
(c) by decreasing OH – ion concentration 28. Which of the following impart green colour to
(d) both (b) and (c). the burner flame? [2005]
20. Which of the following pair is not distinguished (a) B(OMe)3 (b) Na(OMe)
by passing H2S ? [2008] (c) Al(OR)3 (d) Sn(OH)2
(a) Hg, Pb (b) Cd, Pb 29. A red solid is insoluble in water. However it
(c) As, Cd (d) Zn, Mn becomes soluble if some KI is added to water.
21. A little dilute hydrochloric acid is dropped on a Heating the red solid in a test tube results in
pH paper. The colour of the pH paper turns to liberation of some violet coloured fumes and
[2008] droplets of a metal appear on the cooler parts of
(a) dark pink (b) light green the test tube. The red solid is [2005]
(c) light blue (d) bright yellow (a) HgI2 (b) HgO
(c) Pb3O4 (d) (NH4)2Cr2O7
EBD_7167
http://t.me/iitjeehelps

C – 76 BITSAT Topicwise Solved Papers

Hints & Solutions


1. (b) 14. (b) Pb(NO3)2 + 2NH4OH
2. (d) Although blue coloured ferric ® Pb(OH)2 ¯ + 2NH4NO3 (white ppt)
ferrocyanide is formed but due to the Pb(NO3)2 + 2HCl
presence of yellow coloured Fe3+ salts, the ® PbCl2 ¯ + 2HNO3 (white ppt)
blue colour gives the shade of green. Pb(NO3)2 + H2S ® PbS ¯ + 2HNO3 (black)
3. (c) Cu2+ is of group II and Al3+ is of group III 15. (a)
of cation analysis. ..
16. (a) N H 3 + H + (acidic medium) NH +4
50 ´ 1
4. (a) Eq of KMnO4 used = = 0.005 17. (d) PbCl2 is insoluble in cold water, soluble in
1000 ´ 10
hot water and PbS is black ppt in acidic
\ Eq of FAS reacted = 0.005
\ weight of FAS needed medium.
= 0.005 × 392 = 1.96 g 18. (a) Number of Eq. of NaOH = Number of Eq.
Thus percentage purity of FAS is 50% of oxalic acid
5. (b) CuS and CdS are precipitated by H2S. 1000 ´ 1 wt. of oxalic acid
Hydroxide of Al will pass into the solution Then =
1000 63
in the form of NaAlO2 being amphoteric in
\ W = 6.3 g
nature. Hence filtrate will give test for
sodium and aluminium 19. (d) NH +4 ions are increased to suppress
6. (a) Normality of 3% Na2CO3. release of OH – ions, hence solubility
product of Fe(OH)3 is attained. Colour of
3 ´ 100
N= = 0.566N precipitate is different.
53 ´ 100 20. (a) Both Hg and Pb give black ppt with H2S
For H2SO4 sol. N1 = 0.1, V1 = 100 ml (HgS and PbS).
For Na2CO3 sol. N2 = 0.566. V2 = 21. (a)
Now apply N1V1 = N2V2
22. (c) Cl2 + H 2O ® HOCl + HCl
N1V1 0.1 ´ 100ml
V2 = N = = 176.66ml 2HCl + Mg ® MgCl 2 + H 2 ­
2 0.566 23. (a) CCl4 and CS2 disslove bromine very much,
7. (b) Eq. of acid = Eq of base, animal charcoal will adsorb it. Only marble
0.45 20 ´ 0.5 will not effect it.
\ = , = E.wt = 45 OH
E.wt 1000 24. (b) N 2 Cl + ¾
¾®
M.wt 90
Basicity = = =2
E.wt 45 OH
8. (b)
N=N Red dye
9. (c) SnS +(NH4)2 S2 ® (NH4)2 SnS3 soluble
10. (a) There is no reaction between I– and Fe3+.
11. (c) Acetaldehyde is easily oxidised to acetic 25. (c) Both CO2 and SO2 turn lime water milky
acid by a mild oxidising agent like Fehling due to CaCO3 and CaSO3 formation.
solution. Acetone is not easily oxidised. 26. (d) 27. (d)
Both acetone and acetaldehyde give 28. (a) B(OMe)3 imparts green colour to the burner
iodoform test. Other two conditions are not flame.
relevant to aldehydes and ketones. 29. (a) HgI 2 + KI ® K 2 HgI 4
12. (d) CoS is not precipitated in acidic medium. (insoluble) (soluble)
13. (a) Ammonia can dissolve ppt. of AgCl only due On heating HgI2 decomposes as
to formation of complex as given below: HgI2 Hg + I2
AgCl + 2NH3 ® [Ag (NH3)2]Cl
http://t.me/iitjeehelps

SECTION III - MATHEMATICS

Chapter
Sets
(a) A ¢ (b) A
1
1. Let A, B, C be finite sets. Suppose that n (A) = 10,
n (B) = 15, n (C) = 20, n (AÇB) = 8 and n (BÇC) (c) B ¢ (d) None of these
= 9. Then the possible value of n (AÈBÈC) is 6. Let A = {x : x Î R, x < 1};
[2017]
(a) 26 (b) 27 B = {x : x Î R, x - 1 ³ 1} and A È B = R - D,
(c) 28 (d) Can be 26 or 27 or 28 then the set D is [2010]
2. Two finite sets have m and n elements. The
number of subsets of the first set is 112 more (a) {x :1 < x £ 2} (b) {x :1 £ x < 2}
than that of the second set. The values of m and (c) {x :1 £ x £ 2} (d) None of these
n respectively are, [2016] 7. If A = {1, 2, 3, 4, 5}, then the number of proper
(a) 4, 7 (b) 7, 4 (c) 4, 4 (d) 7, 7 subsets of A is [2009]
3. The set (A \ B) È (B \ A) is equal to [2014] (a) 31 (b) 38 (c) 48 (d) 54
(a) [ A \ ( A Ç B)] Ç [ B \ ( A Ç B)] 8. A set A has 3 elements and another set B has 6
(b) ( A È B) \ ( A Ç B) elements. Then [2008]
(a) 3 £ n (A È B) £ 6 (b) 3 £ n (A È B) £ 9
(c) A \ ( A Ç B) (c) 6 £ n (A È B) £ 9 (d) 0 £ n (A È B) £ 9
(d) A Ç B \ A È B 9. There are 20 students in a chemistry class and
30 students in a physics class. If ten students
4. A class has 175 students. The following data are to be enrolled in both the courses, then the
shows the number of students obtaining one or number of students which are either in physics
more subjects. Mathematics 100, Physics 70, class or chemistry class is [2007]
Chemistry 40; Mathematics and Physics 30, (a) 50, if two classes meet at the same hour.
Mathematics and Chemistry 28, Physics and (b) 40, if two classes meet at different hours
Chemistry 23; Mathematics, Physics and (c) both (a) and (b) correct
Chemistry 18. How many students have offered (d) (a) correct but (b) incorrect
Mathematics alone? [2013] 10. The number of elements in the set
(a) 35 (b) 48 (c) 60 (d) 22 {(a, b) : 2a2 + 3b2 = 35, a, b Î Z}, where Z is
5. Let A and B be two sets then (A È B)‘È (A ‘Ç B)
the set of all integers, is [2005]
is equal to [2012]
(a) 2 (b) 4 (c) 8 (d) 12

Hints & Solutions


1. (d) We have We have n (C Ç A) – n (A Ç B Ç C) ³ 0
n (A È B È C) = n (A) + n (B) + n (C) – ...(ii)
n (A Ç B) – n(BÇC) – n (C Ç A) From (i) and (ii)
+ n (AÇB Ç C) n (A È B È C) £ 28 ...(iii)
= 10 +15 + 20 – 8 – 9 – n (C Ç A) Now, n(A È B) = n (A) +n (B) – n (A Ç B)
+ n (A Ç B ÇC) = 10 + 15 – 8 = 17
= 28 – {n(C Ç A) – n (A Ç B Ç C)} ...(i) and n (B È C) = n (B) + n (C) – n (B Ç C)
Since n (C Ç A) ³ n (A Ç B Ç C) = 15 + 20 – 9 = 26
EBD_7167
http://t.me/iitjeehelps

M–2 BITSAT Topicwise Solved Papers

Since, n (A È B È C) ³ n (AÈC) and 6. (b) A = [ x : x Î R, -1 < x < 1]


n (AÈBÈC) ³ n (BÈC), we have
B = [ x : x Î R : x - 1 £ -1or x - 1 ³ 1]
n (AÈBÈC) ³ 17 and n (AÈBÈC) ³ 26
Hence n (AÈBÈC) ³ 26 ...(iv) = [ x : x Î R : x £ 0 or x ³ 2]

From (iii) and (iv) we obtain \ A È B = R - D , where


26 £ n (AÈBÈC) £ 28 D = [ x : x Î R,1 £ x < 2]
Also n (AÈBÈC) is a positive integer 7. (a) Note : Number of proper subsets of A = 2n –1
\ n(AÈBÈC) = 26 or 27 or 28 Given : A = {1, 2, 3, 4, 5}
m n n m- n Here n = 5
2. (b) 2 - 2 = 112 Þ 2 (2 - 1) = 16.7
\ no. of proper subsets = 25 – 1
\ n m -n 4 3
2 (2 - 1) = 2 (2 - 1)
8. (c) We have
Comparing we get n = 4 and m – n = 3 min n (A È B) = max {n(A), n (B)} = max {3,
Þ n = 4 and m = 7 6} = 6
3. (b) Given set can be written as Max n (A È B) = n (A) + n (B) = 9
(A – B) È (B – A) = (A È B) – (A Ç B)
\ 6 £ n (A È B) £ 9
(By definition of symmetric difference)
9. (c) Let C be the set of students in chemistry
Hence, (A \ B) È (B \ A) = (A È B) \ (A Ç B)
class and P be the set of students in
4. (c) n (M alone) physics class.
= n(M) – n(M ÇC) – n(M Ç P) + n(M ÇC Ç C)
Given n (C) = 20, n (P) = 30 and n (C Ç P) =
10. We have to find n (CÈP)
M P If two classes meet at the same hour, then
CÇP = f [No enrollment is possible in both
course in this case.]
So, n (C È P) = n (C) + n (P) = 50
If two classes meet at different hours, then
C n (CÇP) = 10 (given) So, n (CÈP) = n (C)
+n(P) – (CÇP) = 40
= 100 – 28 – 30 + 18 = 60 10. (c) Given set is

5. (a) From Venn-Euler’s Diagram. {(a, b) : 2a 2 + 3b2 = 35, a, b Î Z }

We can see that, 2(±2)2 + 3(±3)2 = 35


(A È B)'
U and 2(±4)2 + 3(±1) 2 = 35
\ (2, 3), (2, – 3), (–2, – 3), (–2, 3), (4, 1),
(A 'Ç B) (4, – 1),
(– 4, –1), (–4, 1) are 8 elements of the set .
A B \ n = 8.

\ (A È B) ' È (A 'Ç B) = A '


http://t.me/iitjeehelps

Relations and Functions M–3

Chapter
Relations and Functions
2
x (a) (– ¥, 3] (b) (–¥, ¥)
1. If x is real number, then 2 must lie
x - 5x + 9 é1 ù
between [2017] (c) [3, ¥) (d) ê , 3ú
ë3 û
1 1 6. The domain of the functions f (x) = log2 log3
(a) and 1 (b) –1 and
11 11
log4 x is [2007]
1 (a) [4, ¥ ) (b) (4, ¥)
(c) –11 and 1 (d) – and 1
11 (c) (– ¥ ,4) (d) None of these
2. The domain of the function 5
æ 1 ö ö 7. The range of the function f (x) = is [2006]
æ 3 - x2
f ( x ) = log 2 ç – log1/ 2 ç1 + 1/ 4 ÷ – 1÷ is [2014]
è è x ø ø
é5 ö æ5 ö
(a) (0, 1) (b) (0, 1] (c) [1, ¥) (d) (1, ¥) (a) (– ¥ , 0) È ê 3 , ¥÷ø (b) (– ¥ , 0) È çè 3 , ¥÷ø
ë
3. Let x and y be two natural numbers such that xy
= 12(x + y) and x £ y. Then the total number of é5 ö
(c) (– ¥ , 0] È ê 3 , ¥÷ø (d) None of these
pairs (x, y) is [2012] ë
(a) 8 (b) 6 (c) 4 (d) 16 1
4. If f(x) is a function that is odd and even 8. The domain of the function f (x) =
2
simultaneously, then f(C) – f(B) is equal to [2011] x - 3x + 2
(a) 1 (b) – 1 is [2005]
(c) 0 (d) None of these (a) ( – ¥, 1) (b) ( – ¥, 1) È (2, ¥)
(c) ( – ¥, 1] È [2, ¥) (d) (2, ¥)
x2 - x +1
5. The range of the function f (x) =
x2 + x +1
where x Î R, is [2009]

Hints & Solutions


x é -1 ù
1. (d) Let y = Þ y Î ê ,1ú
2 ë 11 û
x - 5x + 9
2
Þ x y – (5y + 1)x + 9y = 0 2. (a) f (x) is defined if – log1/2
for real x, Discriminant = b2 – 4ac ³ 0 æ 1 ö
çè 1 + 1/ 4 ÷ø –1 > 0
(5y + 1)2 – 36y2 > 0 x
æ 1 ö
Þ (5y + 1 – 6y) (5y + 1 + 6y) > 0 Þlog1/2 çè 1 + 1/ 4 ÷ø < –1
x
Þ (– y + 1) (11y + 1) ³ 0 –1
1 æ 1ö
Þ (y – 1) (11y + 1) £ 0 Þ1 + >ç ÷
x1/ 4
è 2ø
EBD_7167
http://t.me/iitjeehelps

M–4 BITSAT Topicwise Solved Papers

1 Þ(3y – 1) (y – 3) £ 0
Þ 1/ 4
>1
x
é1 ù é1 ù
Þ 0< x<1 Þ y Î ê , 3ú \ Range is êë 3 , 3úû
ë3 û
3. (a) xy – 12x – 12y = 0 Þ (x – 12) (y – 12) = 144
Now 144 can be factorised into two factors 6. (b) f (x) is defined if log3 log4 x > 0, log4 x > 0
x and y where x £ y and the factors are and x > 0
(1, 144), (2, 72), (3, 48), (4, 36), (6, 24), (8, 18), Þ log4 x > 3° = 1, x > 4° and x > 0
(9, 16), (12, 12). Þ x > 41, x > 1 and x > 0 Þ x > 4
Thus there are eight solutions. Domain of f = ( 4, ¥)
7. (a) For f (x) to be defined, 3 – x2 ¹ 0
4. (c) f (x) = 0 " x Î R Þ f (3) – f (2) = 0
i.e. x ¹ ± 3
2
x - x +1 \ Domain of f = R – {± 3 }
5. (d) Let y = 2
x + x +1
5 y(3y - 5)
Þx2(y – 1) + x(y + 1) + (y – 1) = 0 Now, let y = 2
Þ x2 =
3- x y2

- (y + 1) ± (y + 1) 2 - 4(y - 1) 2 For x to be real, x2 ³ 0 Þ y (3y – 5) ³ 0


Þx = Also y ¹ 0
2(y - 1)
5
\ y < 0 or y ³
3
-(y + 1) ± -3y 2 + 10y - 3
= is real iff é5 ö
2(y - 1) Hence, range of f = (– ¥, 0) È ê , ¥÷
ë3 ø
y–1¹ 0Þy¹1
8. (b) For f (x) to be defined, we must have
If y = 1 then original equation gives x = 0,
x2 – 3x + 2 = (x – 1) (x – 2) > 0 Þ x < 1 or > 2
so taking y = 1
Domain of f = (– ¥,1) È ( 2, ¥).
Also 3y2 – 10y + 3 £ 0
http://t.me/iitjeehelps

Trigonometric Functions M–5

Chapter
Trigonometric Functions
3
1. The number of roots of equation 5sin q - 3cos q
cos x + cos 2 x + cos 3x = 0 is (0 £ x £ 2p) 8. If 5 tan q = 4, then = [2014]
5sin q + 2 cos q
[2017, 2012] (a) 0 (b) 1 (c) 1/6 (d) 6
(a) 4 (b) 5 (c) 6 (d) 8
æ p ö æ p ö
2. If A and B are positive acute angles satisfying 9. cos 2 ç + q÷ - sin 2 ç - q÷ = [2014]
è6 ø è6 ø
3 sin A 2 cos B
3 cos 2 A + 2 cos 2 B = 4 and = , 1
sin B cos A (a) cos 2q (b) 0
Then the value of A + 2B is equal to : [2016] 2
p p p p 1 1
(a) (b) (c) (d) (c) - cos 2q (d)
6 2 3 4 2 2
3. If sin q1 + sin q2 + sin q3 = 3 , then 10. The solution of (2 cos x – 1) (3 + 2 cos x) = 0 in
the interval 0 £ x £ 2p is [2014]
cos q1 + cos q2 + cos q3 = [2016]
p p 5p
(a) 0 (b) 1 (c) 2 (d) 3 (a) (b) ,
3 3 3
4. If tan (cot x) = cot (tan x), then sin 2x is equal to : p 5p æ 3ö
[2016] (c) , , cos –1 ç – ÷ (d) None of these
3 3 è 2ø
2 4
(a) (b) 11. If x sin 3 q + y cos 3 q = sin q cos q and
(2n + 1)p (2n + 1)p
2 4 x sin q = y cos q , then x 2 + y 2 = [2013]
(c) (d) (a) 1 (b) 2
n (n + 1)p n (n + 1)p
(c) 0 (d) None of these
5. The general solution of the equation
sin 2x + 2sin x +2 cos x+ 1 = 0 is [2016] 12. If cos 7q = cos q - sin 4q, then the general value
p of q is [2013]
(a) 3np - np np p np np p
4 (a) , + ( -1) n (b) , + (-1) n
6 3 18 3 3 18
p np np p np np p
(b) 2np + (c) , ±
4 3 18
(d) ,
4 3
+ ( -1) n
18
4
13. If sin 2q + sin2f = 1/2, cos2q +cos2f = 3/2 then
n -1 æ 1 ö÷ cos2 (q – f) is equal to
(c) 2np + (-1) sin çç ÷ [2012]
è 3ø (a) 3/8 (b) 5/8 (c) 3/4 (d) 5/4
p 1 1
(d) np - 14. If, tan A = and tan B = , then find the value
4 2 3
cos A sin A of A + B [2011]
6. If = n, = m, then the value of (m2 – n2) p p –p
cos B sin B (a) p (b) (c) (d)
2 4 4
sin 2 B is [2015] 1
(a) 1 + n2 (b) 1 – n2 (c) n 2 (d) – n2 15. If sin q = – and tan q = 1 / 3 then q =
7. The period of tan 3q is [2015] 2
[2011]
(a) p (b) 3p/4 (a) 2np + p/6 (b) 2np +11p/6
(c) p/2 (d) None of these (c) 2np +7p/6 (d) 2np + p/4
EBD_7167
http://t.me/iitjeehelps

M–6 BITSAT Topicwise Solved Papers

cos q sin q p p
16. + is equal to [2011] (a) q = 2np ± (b) q = np + (–1)n
1 - tan q 1 - cot q 6 6
(a) sinq – cos q (b) sinq + cos q p p
(c) tanq + cot q (d) tanq – cot q (c) q = np + (d) q = np -
6 6
17. If 12cot 2 q - 31cosec q + 32 = 0, then the 25. The number of solutions of cos 2q = sin q in (0, 2p) is
[2007]
value of sin q is [2010] (a) 1 (b) 2 (c) 3 (d) 4
3 2 -2 26. If A = cos2 q +sin4 q , then for all real values of q
(a) or 1 (b) or [2007]
5 3 3
3
4 3 1 (a) 1 £ A £ 2 (b) £ A £1
(c) or (d) ± 4
5 4 2 13 13
3
18. tan 20° + tan 40° + 3 tan 20° tan 40° is equal to (c) £A£1 (d) £A£
10 4 16
[2010]
27. If sin q + cosec q = 2, then sin 2 q + cosec 2 q is
(a) 3 / 2 (b) 3 / 4 (c) 3 (d) 1 equal to [2006]
2sin a (a) 1 (b) 4
19. If y= , th en value of (c) 2 (d) None of these
1 + cos a + sin a
28. If cot x – cos x cot x = 1 – cos x then general
1 - cos a + sin a
is [2009] solution of this equation is [2006]
1 + sin a (a) 2np, n Î I
y 3
(a) (b) y (c) 2y (d) y p
3 2 (b) (4n + 1) , nÎI
4
sin q + sin 2q p
20. Period of is [2009] (c) (2n + 1) , n ÎI
cos q + cos 2q 4
2p p p
(a) 2p (b) p (c) (d) (d) 2np, (4k + 1) , n, k ÎI
3 3 4
x 1 q
21. The general solution of 8 tan 2 = 1 + sec x is 29. If sec q = 1 , then tan = [2005]
2 4 2
[2009]
1 3 1 5
-1 æ -1 ö p (a) (b) (c) (d)
(a) 2np ± cos ç ÷ (b) 2np ± 3 4 4 4
3
è ø 6 30. If 2 y cos q = x sin q and
-1 æ 1 ö
(c) 2np ± cos ç ÷ (d) None of these 2 x sec q - y cosec q = 3, then x 2 + 4 y 2 =
è 3ø
22. The value of 2 (cos 15° - sin15°) is equal to [2005]
(a) 4 (b) – 4
[2008] (c) ±4 (d) None of these
(a) 3 (b) 2 (c) 1 (d) 2
1
23. The value of 31. If tan q = – , then general solution of the
tan A + tan( 60 ° + A ) - tan( 60 ° - A ) is [2008] 3
(a) tan 3A (b) 2 tan 3A equation is [2005]
(c) 3 tan 3A (d) None of these p p
(a) 2np + , n Î I (b) np + , n Î I
1 6 6
24. If tan q = – , then the general solution of the
3 p p
(c) 2np - , n Î I (d) np - , n Î I
equation (2008) 6 6
http://t.me/iitjeehelps

Trigonometric Functions M–7

Hints & Solutions


1. (c) We have cos x + cos 2 x + cos 3x = 0 1 4
or (cos 3x + cos x ) + cos 2 x = 0 Þ sin 2 x = =
np p ( 2 n + 1) p
+
or 2 cos 2x. cos x + cos 2x = 0 2 4
or cos 2x (2 cos x + 1) = 0 5. (d) Given, sin 2 x + 2 sin x + 2 cos x + 1 = 0
We have, either cos 2x = 0 or 2cosx +1 = 0 Þ 1 + sin 2x + 2(sin x + cos x ) = 0
p Þ (sin x + cos x ) 2 + 2(sin x + cos x ) = 0
If cos 2x = 0, then 2 x = (2 m + 1)
2
Þ (sin x + cos x ) (sin x + cos x + 2) = 0
p
or x = (2m + 1) , m Î I \ sin x + cos x = 0 or sin x + cos x = -2
4
But, sin x + cos x = -2 is inadmissible.
1 2p
If 2 cos x + 1 = 0, then cos x = - = cos Since, | sin x | £1, | cos x | £ 1
2 3
p
Þ x = 2np ±
2p
,nÎI \ sin x + cos x = 0 Þ sin æç x + ö÷ = 0
3 è 4ø
Hence the required general solution are p p
Þ x+ = np Þ x = np -
p 2p 4 4
x = (2 m + 1) and x = 2np ± , m , n Î I
4 3 6. (b) cos A = n cos B and sin A = m sin B
2. (b) Given, 3 cos2A +2 cos2 B = 4 Squaring and adding, we get
1 = n2 cos2B + m2sin2B
Þ 2 cos 2 B - 1 = 4 - 3 cos 2 A - 1 Þ 1 = n2 (1 – sin2B) + m2sin 2B
\ (m2 – n2) sin2B = 1 – n2
Þ cos 2B = 3 (1 - cos 2 A) = 3 sin2A ...(i) 7. (d) tanq is of period p so that tan 3q is of
and 2 cos B sin B = 3 sin A cos A period p/3.
sin 2B = 3 sin A cos A ...(ii)
Now, cos (A + 2B) = cos A cos 2B 8. (c) 4
5 tan q = 4 Þ tan q =
– sin A sin 2B 5
= cos A (3 sin 2 A) – sin A (3 sin A cos A) = 0 4 5
[using eqs. (i) and (ii)] \ sin q = and cos q =
p 41 41
Þ A + 2B =
2 4 5
3. (a) Since, sin q1 + sin q2 + sin q3 = 3 5´ - 3´
5sin q - 3cos q 41 41 = 1
p Þ =
\ sin q1 = sin q2 = sin q3 = 1 Þ q1 = q2 = q3 = 5sin q + 2cos q 5 ´ 4 + 2 ´ 5 6
2
41 41
\ cos q1 + cos q2 + cos q3 = 0
æp ö æp ö æp ö
4. (b) Q tan(cot x ) = cot(tan x ) = tan ç - tan x ÷ 9. (a) cos 2 ç + q÷ - sin 2 ç - q÷
è6 ø è6 ø
è2 ø
æp p ö æp p ö
p p = cos ç + q + - q÷ cos ç + q - + q÷
Þ cot x = np + - tanx Þ cot x + tanx = np + è6 6 ø è 6 6 ø
2 2 2p 1
1 p 1 np p = cos cos 2q = cos 2q
Þ = np + Þ = + 6 2
sin x cos x 2 sin 2 x 2 4
EBD_7167
http://t.me/iitjeehelps

M–8 BITSAT Topicwise Solved Papers


10. (b) We have (2cos x – 1) (3 + 2 cos x) = 0 15. (c) We shall first consider values of q between 0
and 2p
1
If 2 cos x – 1=0, then cos x = æ pö
2 1 p
sin q = – = – sin = sin çè p + ÷ø
\ x = p/3, 5p/3 2 6 6
If 3 + 2 cos x = 0, the cos x = –3/2 or sin (2p – p/6) \ q = 7p/6 ; 11p/6
which is not possible. tan q = 1 / 3 = tan (p/6) or tan (p + p/6)
11. (a) x sin 3 q + y cos 3 q = sin q cos q ....(i) \ q = p/6 , 7p/6
The value of q which satisfies both the equations
and x sin q = y cos q ...(ii) is 7p/6
Equation (i) may be written as Hence the general value of q is 2np + 7p/6 where
nÎI
x sin q. sin 2 q + y cos 3 q = sin q cos q
cos q sin q cos q sin q
16. (b) + = +
Þ y cos q sin 2 q + y cos 3 q = sin q cos q 1 - tan q 1 - cot q sin q cos q
1- 1-
cos q sin q
Þ y cos q(sin 2 q + cos 2 q) = sin q cos q
Þ y cos q = sin q cos q \ y = sin q ...(iii) cos 2 q sin 2 q
= - = cosq + sin q
Putting the value of y from (iii) in (ii), we get cos q - sin q cos q - sin q
x sin q = sin q. cos q Þ x = cos q ...(iv) 17. (c) 12 cot 2 q - 31cosec q + 32 = 0
Squaring (iii) and (iv), and adding , we get Þ 12(cosec 2q - 1) - 31cosec q + 32 = 0
2 2 2 2
x + y = cos q + sin q = 1 Þ 12cosec 2 q - 31cosec q + 20 = 0
12. (d)
Þ 12cosec2 q - 16 cosec q - 15cosec q + 20 = 0
cos 7q = cos q - sin 4q Þ sin 4q= cos q - cos 7q
Þ (4cosec q - 5)(3cosec q - 4) = 0
Þ sin 4q = 2sin 4q sin 3q Þ sin 4q (1 - 2sin 3q) = 0
5 4 4 3
1 Þ cosecq = , ; \ sin q = ,
\ sin 4q = 0 or sin 3q = 4 3 5 4
2
tan 40° + tan 20°
p 18. (c) 3 = tan 60° = tan (40° + 20°) =
1 - tan 40° tan 20°
Þ 4q = np or 3q = np + (-1) n
6 \ 3 – 3 tan 40° tan 20° = tan 40° + tan 20°
np np p Hence tan 40° + tan 20° + 3 tan 40° tan20° = 3
Þ q= or + ( -1) n
4 3 18 1 - cos a + sin a
13. (b) Using cosine formula 19. (b)
1 + sin a
2 sin (q + f) cos (q – f) = 1/2 .....(1)
1 - cos a + sin a 1 + cos a + sin a
2 cos (q + f) cos (q – f) =3/2 .....(2) = .
Squaring (1) and (2) and then adding 1 + sin a 1 + cos a + sin a
1 9 5 5 (1 + sin a )2 - cos 2 a
4 cos2 (q – f) = + = Þ cos2 (q – f) = =
4 4 2 8 (1 + sin a ) (1 + cos a + sin a )
1 1 2sin a (1 + sin a )
+ = (1 + sin a ) (1 + cos a + sin a )
tan A + tan B 2 3 =1
14. (c) tan (A + B) = =
1 - tan A tan B 1 - 1 . 1
2sin a
2 3 = =y
1 + cos a + sin a
p
\ A + B = 45° =
4
http://t.me/iitjeehelps

Trigonometric Functions M–9

2
æ 3q ö æqö æ 2sin q cos q ö
2sin ç ÷ cos ç ÷
sin q + sin 2q è ø2 è 2 ø = tan æ 3q ö = cos2q sin2q = ç ÷
20. (c) = ç ÷ è 2 ø
cos q + cos 2q æ ö
3 q æqö è 2 ø
2 cos ç ÷ cos ç ÷ 1
è 2 ø è 2ø = (sin2q)2 Þ 4 (1 – A) = sin2 2q
4
2p 0 £ 4 (1 – A) £ 1 [Q 0 £ sin22q £ 1]
Hence period =
3 Þ 0 £ 1 – A £ 1/4 Þ 1 – A ³ 0 i.e.A £ 1
and 1 – A £ 1/4 i.e. 3/4 £ A \ 3/4 £ A £ 1
x
21. (c) We have, 8 tan 2 = 1 + sec x 27. (c) We have sin q + cosec q = 2
2 Squaring both the side
æ 1 - cos x ö 1 1 + cos x Þ (sin q + cosec q )2 = (2)2
Þ 8ç ÷ = 1+ =
è 1 + cos x ø cos x cos x Þ sin2 q + cosec2 q + 2 sin q .cosec q = 4
Þ sin2 q + cosec2 q + 2 = 4
Þ 8 cos x - 8 cos 2 x = (1 + cos x ) 2 Þ sin2 q + cosec2 q = 2
Þ 9 cos 2 x - 6 cos x + 1 = 0 28. (b) cot x (1 – cos x) = 1 – cos x
1 – cos x = 0 ; cos x = 1 ; x = 2np
Þ (3 cos x - 1) 2 = 0 Þ 3 cos x - 1 = 0 p
cot x = 1 ; x = np +
1 4
Þ cos x = = cos a (say ) Þ x = 2np ± a Þ x = 2np is not possible as cot x is not
3
defined at np.
-1 æ 1 ö p p
\ x = 2np ± cos ç ÷, where n Î I Hence, x = np + = (4n + 1) , n Î I
è3ø 4 4
22. (c) 2(cos15 ° - sin15 °) 5
29. (a) Given that sec q =
é p p ù 4
= 2 ´ 2 ê cos cos15° - sin sin15 °ú
ë 4 4 û 1 + tan 2 (q / 2) 5 1 + tan 2 (q / 2)
sec q = Þ =
= 2[cos 45° cos 15°– sin 45° sin 15°] 1 - tan 2 (q / 2) 4 1 - tan 2 (q / 2)
= 2cos (45° + 15°) = 2cos 60° = 1
23. (c) The given expression Þ 5 - 5 tan 2 (q / 2) = 4 + 4 tan 2 (q / 2)
3 + tan A 3 - tan A 2 1
= tan A + - Þ 9 tan (q / 2) = 1 Þ tan(q / 2) =
1 - 3 tan A 1 + 3 tan A 3
30. (a) Given that 2y cos q = x sin q ...(i)
8 tan A 9 tan A - 3 tan 3 3
= tan A + = and 2 x sec q - y cosec q = 3 ...(ii)
1 - 3 tan 2 A 1 - 3 tan 2 A
2x y
Þ - =3
(3 tan A - tan 3 A) cos q sin q
= 3. = 3 tan 3A
1 - 3 tan 2 A Þ 2 x sin q - y cos q - 3sin q cos q = 0 ...(iii)
1 p p Solving (i) and (iii), we get y = sin q and
24. (d) tan q = – = tan æç - ö÷ \ q = np - x = 2 cos q
3 è 6ø 6
25. 2 2
(c) 1 – 2 sin q = sin q Þ 2 sin q + sin q – 1 = 0 Now, x 2 + 4 y 2 = 4cos2 q + 4sin2 q
1 3p p 5p 2 2
Þ sin q = -1, Þ q= , , . = 4(cos q + sin q) = 4
2 2 6 6
26. (b) A = cos2q + sin4q 1 æ pö
31. (d) tan q = – = tan ç - ÷
= cos2q + (1 – cos2q)2 = cos2q + 1 – 2 cos2q+ cos4 q 3 è 6ø
= 1 – cos2q + cos4q
p
\ 1 – A = cos2q – cos4 q = cos2q [1– cos2q] \ q = np -
6
EBD_7167
http://t.me/iitjeehelps

M – 10 BITSAT Topicwise Solved Papers

Chapter
Principle of Mathematical Induction
4
4n (2n)! 5. 10n + 3(4n+2) + 5 is divisible by ( n Î N ) [2009]
1. If < , then P(n) is true for [2017]
n + 1 (n!)2 (a) 7 (b) 5 (c) 9 (d) 17

(a) n ³ 1 (b) n > 0


6. If an = 7 + 7 + 7 + ... ... having n radical
(c) n < 0 (d) n ³ 2
2. The greatest positive integer, which divides signs then by methods of mathematical induction
n (n + 1)(n + 2)(n + 3) for all n Î N , is 2014] which is true [2008]
(a) 2 (b) 6 (c) 24 (d) 120 (a) an > 7 " n ³ 1 (b) an < 7 " n ³ 1
3. Let T(k) be the statement 1 + 3 + 5 + ... +
(2k – 1)= k2 +10 (c) an < 4 " n ³ 1 (d) an < 3 " n ³ 1
Which of the following is correct? [2012] 2
7. Let S (k ) = 1 + 3 + 5... + (2k - 1) = 3 + k . Then
(a) T(1) is true which of the following is true? [2006]
(b) T(k) is true Þ T(k + 1) is true (a) Principle of mathematical induction can be
(c) T(n) is true for all n Î N used to prove the formula
(d) All above are correct (b) S (k ) Þ S (k + 1)
4. For n Î N, xn+1 + (x + 1)2n–1 is divisible by (c) S (k ) Þ
/ S (k + 1)
[2011]
(a) x (b) x + 1 (d) S (1) is correct
(c) x2 + x + 1 (d) x2 – x + 1 8. The inequality n! > 2n–1 is true for [2005]
(a) n > 2 (b) n Î N
(c) n > 3 (d) None of these

Hints & Solutions


4n (2n)! 4m +1 4m 4(m + 1)
1. (d) Let P(n) : < Now, = ·
n + 1 (n !)2 m + 2 m +1 m + 2
For n = 2, (2m)! 4(m + 1)
< ·
42
4! 16 24 (m !) 2 (m + 2)
P(2) : < Þ <
2 + 1 (2)2 3 4
(2m)!(2m + 1)(2m + 2)4(m + 1)(m + 1) 2
which is true. Let for n = m ³ 2, P(m) is true. =
(2m + 1)(2m + 2)(m !) 2 ( m + 1) 2 ( m + 2)
4m (2m)!
i.e. < ;
m + 1 ( m !) 2
http://t.me/iitjeehelps

Principle of Mathematical Induction M – 11

[2(m + 1)]! 2( m + 1) 2 [2(m + 1)]! 6. (b) a1 = 7 < 7. Let am < 7


= · <
[(m + 1)!] (2m + 1)(m + 2) [(m + 1)!]2
2
Then am + 1 = 7 + am Þ a2m + 1
Hence, for n ³ 2, P(n) is true. = 7 + am < 7 + 7 < 14.
2. (c) The product of r consecutive integers is
Þ am + 1 < 14 < 7; So by the principle of
divisible by r ! . Thus n (n+ 1 ) (n + 2) (n + 3)
is divisible by 4 ! = 24. mathematical induction an < 7 " n.
3. (b) When k = 1, LHS = 1 but RHS = 1 + 10 = 11 \ r = 0,8,16,24,........256 , total 33 values.
\ T(1) is not true
Let T(k) is true. That is 7. (b) S(k) = 1+3+5+...+(2k – 1) = 3 + k2
S (1) :1 = 3 + 1, which is not true
1 + 3 + 5 + ..... + (2 k - 1) = k 2 + 10
Q S (1) is not true.
Now, 1 + 3 + 5 + ..... + (2k - 1) + (2k + 1) \ P.M.I cannot be applied
2 2 Let S(k) is true, i.e.
= k + 10 + 2k + 1 = ( k + 1) + 10
1 + 3 + 5.... + (2k - 1) = 3 + k 2
\T(k +1) is true.
Þ 1 + 3 + 5.... + (2k - 1) + 2k + 1
That is T(k) is true Þ T(k + 1) is true.
But T(n) is not true for all n Î N , as T(1) is = 3 + k 2 + 2k + 1 = 3 + ( k + 1) 2
not true. \ S (k ) Þ S (k + 1)
4. (c) For n = 1, we have;
8. (a) Let P(n) º n! > 2n–1
xn+1 + (x + 1)2n–1 = x2 + (x + 1) = x2 + x + 1, P(3) º 6 > 4
which is divisible by x2 + x + 1 Let P(k) º k! > 2k–1 is true.
For n = 2, we have; xn+1 + (x + 1)2n–1 \ P(k + 1) = (k + 1)! = (k + 1)k!
> (k + 1) 2k–1
= x3 + (x + 1)3 = (2x + 1) (x2 + x + 1),
> 2k (as k + 1 > 2)
which is divisible by x2 + x + 1.

5. (c) 10n + 3(4n + 2 ) + 5 Taking n = 2;

10 2 + 3 ´ 44 + 5 = 100 + 768 + 5 = 873


Therefore this is divisible by 9.
EBD_7167
http://t.me/iitjeehelps

M – 12 BITSAT Topicwise Solved Papers

Chapter
Complex Numbers and Quadratic

1.
Equations
If a an d b are roots of the equation
5
(a) {1, 3} (b) {1, 2, 3}
3p (c) {0, 1, 3} (d) {0, 1, 2, 3}
x 2 + px + = 0 , such that | a - b |= 10, 1/ 3 x y
4 9. If z = x + iy, z = a – ib, then – = k (a2 – b2)
then p belongs to the set : [2017] a b
(a) {2, – 5} (b) {– 3, 2} where k is equal to [2014]
(c) {– 2, 5} (d) {3, – 5} (a) 1 (b) 2 (c) 3 (d) 4
7-z 1
2. If f(z) = , where z = 1 + 2i, then |f(z)| is 10. i57 + 25 , when simplified has the value [2014]
1 - z2 i
equal to : [2017] (a) 0 (b) 2i (c) – 2i (d) 2
11. If the roots of x2 + x + a = 0 exceed a then
|z| [2014]
(a) (b) | z |
2 (a) 2 < a < 3 (b) a > 3
(c) 2 | z | (d) None of these (c) – 3 < a < 3 (d) a < – 2
3. If z1 = 3 + i 3 and z 2 = 3 + i , then the z+2
12. If the real part of is 4, z ¹ 1, then the locus
50 z -1
æ z1 ö of the point representing z in the complex plane
complex number çç ÷÷ lies in the : [2016] is [2013]
è z2 ø
(a) first quadrant (b) second quadrant (a) a straight line parallel to x-axis
(c) third quadrant (d) fourth quadrant (b) a straight line equally inclined to axes
4. If a, b are the roots of the equations x2 – 2x– 1 = 0, then (c) a circle with radius 2
what is the value of a2 b–2+ a –2 b2 [2016] (d) a circle with radius
1
(a) –2 (b) 0 (c) 30 (d) 34 2
5. If a, b and c are real numbers then the roots of 13. If a and b are the roots of x2 – x + 1 = 0, then the
the equation (x – a) (x – b) + (x – b) (x – c) equation whose roots are a100 and b100 are
+ (x – c) (x – a) = 0 are always [2016] [2013]
(a) real (b) imaginary (a) x2 – x + 1 = 0 (b) x2 + x – 1 = 0
(c) positive (d) negative (c) x2 – x – 1 = 0 (d) x2 + x + 1 = 0
6. The root of the equation
p æ pö
2 (1 + i ) x 2 - 4 ( 2 - i ) x - 5 - 3i = 0 which has 14. The amplitude of sin + i ç1 - cos ÷ [2012]
5 è 5ø
greater modulus is [2015] (a) p/5 (b) 2p/5
3 - 5i 5 - 3i 3-i (c) p/10 (d) p/15
(a) (b) (c) (d) None 15. If x = w – w2 –2, then the value of x4 + 3x3 + 2x2
2 2 2
7. If complex number z1, z2 and 0 are vertices of – 11x – 6 is [2012]
(a) 1 (b) –1
equilateral triangle, then z12 + z 22 - z1z 2 is equal (c) 2 (d) None of these
to [2015] 16. If a, b are the roots of the equation ax2 + bx + c = 0,
(a) 0 (b) z1 – z2 (c) z1 + z2 (d) 1 then the roots of the equation ax2 + bx (x + 1)
8. Universal set, + c (x + 1)2 = 0 are [2011]
U = {x | x5 – 6x4 + 11x3 – 6x2 = 0} (a) a – 1, b – 1 (b) a + 1, b + 1
A = {x | x2 – 5x + 6 = 0} a b a b
B = {x | x2 – 3x + 2 = 0} (c) , (d) ,
What is (A Ç B)' equal to ? [2015] a -1 b -1 1- a 1- b
http://t.me/iitjeehelps

Complex Numbers and Quadratic Equations M – 13

17. If a > 0, a Î R, z = a + 2i and z | z | – az + 1 = 0 (a) z1 = z2 (b) |z2|2 = z1z2


then [2011] (c) z1z2 = 1 (d) None of these
(a) z is always a positive real number 3-i 3+ i
(b) z is always a negative real number 25. If z = + , then value of arg (zi) is
2+i 2-i
(c) z is purely imaginary number
[2007]
(d) such a complex z does not exist
p p p
18. The roots of the equation x2 – 2 2 x + 1 = 0 are (a) 0 (b) (c) (d)
[2010] 6 3 2
(a) Real and different (cos q + i sin q ) 4
(b) Imaginary and different 26. Value of is [2007]
(c) Real and equal (cos q - i sin q) 3
(d) Rational and different (a) cos 5q + i sin 5q (b) cos 7q + i sin 7q
1 - ia (c) cos 4q + i sin 4q (d) cos q + i sin q
19. If = A + iB, then A2 + B2 equals to
1 + ia
27. If the roots of the equation x 2 + 2ax + b = 0
[2010]
(a) 1 (b) a2 (c) –1 (d) – a2 are real and differ by at most 2m, m ¹ 0 then b
20. If the expression x2 – 11x + a and x2 – 14x + 2a lies in the interval [2007]
must have a common factor and a ¹ 0, then, the (a) (a 2 - m 2 , a 2 ) (b) [a 2 - m 2 , a 2 )
common factor is [2009]
(a) (x – 3) (b) (x – 6) (c) (a 2 , a 2 + m 2 ) (d) None of these
(c) (x – 8) (d) None of these 28. If the equation x2 + 2 (k + 1) x + 9k – 5 = 0 has
1 1 1 only negative roots, then – [2006]
21. For the equation - = , if the (a) k £ 0 (b) k ³ 0 (c) k ³ 6 (d) k £ 6
x+a x+b x+c
product of roots is zero, then the sum of roots is 29. The value of the expression x4 – 8x3 – 8x + 2
[2009] when x = 2 + 3 is – [2006]
2ab 2bc -2bc (a) 0 (b) 1 (c) 2 (d) 3
(a) 0 (b) (c) (d) 30. If a, b are the roots of x2 + px + q = 0, and w is an
b+c b+c b+c
imaginary cube root of unity, then value of
22. If arg(z1 ) = arg(z 2 ) , then [2009]
(a) z2 = kz1–1 (k > 0) (b) z2 = kz1(k > 0) (wa + w 2b) (w 2 a + wb) is [2006]
(a) p 2 (b) 3q
(c) | z2 | = | z1 | (d) None of these
(c) p2 – 2q (d) p2 – 3q
23. Let a, b, c Î R and ax² + bx + c = 0 has two 31. The minimum value of |z| + | z – i| is [2005]
negative roots, then – [2008] (a) 0 (b) 1 (c) 2 (d) none
(a) a, b, c are of same sign 32. The number of real roots of
(b) a, –b, c are of same sign
3
(c) a, b, –c are of same sign æ 1ö æ 1ö
(d) a, – c are of same sign ç x + ÷ + ç x + ÷ = 0 is [2005]
è xø è xø
z1 (a) 0 (b) 2 (c) 4 (d) 6
24. If = 1 and arg(z1 z2) = 0, then [2008]
z2

Hints & Solutions


2 3p Now, given | a – b | = 10 Þ a – b = ± 10
1. (c) Given quadratic eqn. is x + px + =0
Þ (a – b)2 = 10 Þ a2 + b2 – 2ab = 10
4
3p Þ (a + b)2 – 4ab = 10 Þ p2 – 3p – 10 = 0
So, a + b = – p, ab = Þ p = – 2, 5 Þ p Î {– 2, 5}
4
EBD_7167
http://t.me/iitjeehelps

M – 14 BITSAT Topicwise Solved Papers

2. (a) z = 1 + 2i Þ| z |= 1 + 4 = 5 4( 2 - i) ± 16( 2 - i) 2 + 8(1 + i )(5 + 3i )


6. (a) Roots =
7- z 7 - 1 - 2i 3-i 4(1 + i)
\ f (z) = = 2 =
2 1 - (1 + 2i ) 4-i -i 3 - 5i -1 - i
1- z 2 - 2i
= or = or
3-i | 3-i | 5 |z| 1+ i 1+ i 2 2
Þ| f ( z) | = = = = 7. (a) z1, z2, 0 are vertices of an equilateral triangle,
2 - 2i | 2 - 2i | 2 2
so we have z12 + z 22 + 02 = z1 z2 + z2. 0 + 0. z1
50 50 é 2 ù25
æ z1 ö æ 3 +i 3 ö æ ö Þ z12 + z 22 = z1 z 2 Þ z12 + z 22 - z1 z 2 = 0
ç ÷ = ç ÷ êç 3(1+ i) ÷ ú
3. (a) ç z2 ÷ ç 3 +i ÷ = êç 3 +i ÷ ú 8. (c) U = {x : x5 – 6x4 + 11x3 – 6x2 = 0}
è ø è ø ëè ø û Solving for values of x, we get
25 25 U = {0, 1, 2, 3}, A = {x : x2 – 5x + 6 = 0}
æ 3i ö 325 i 25 æ3ö
= çç ÷÷ = = – i.w ç ÷ Solving for values of x, we get A = {2, 3}
è 1 + 3i ø (-2w 2 ) 25 è2ø and B = {x : x2 – 3x + 2 = 0}
25 æ Solving for values of x, we get B = {2, 1}
æ -1 + 3 i ö æ 3 ö25 æ3ö ö
= -i çç ç ÷ ç 3 + 1 i÷ A Ç B = {2} \ (A Ç B)' = U – (A Ç B)
÷÷ ç ÷ = ç 2 2 ÷
è 2 øè 2 ø è2ø è ø = {0, 1, 2, 3} – {2} = {0, 1, 3}
50 9. (d) z1/ 3 = a - ib Þ z = ( a - ib ) 3
æz ö
Hence, çç 1 ÷÷ lies in the first quadrant \ x + iy = a 3 + ib3 - 3ia 2b - 3ab2 .
è z2 ø 3 2 x 2 2
as both real and imaginary parts of this Þ x = a - 3ab Þ = a - 3b
a
number are positive. y
4. (d) Since, a and b are the roots of the equation and y = b3 - 3a 2b Þ = b2 - 3a 2
b
x2 – 2x – 1 = 0, then x y 2
So, - = 4(a - b ) 2
Sum of roots, a + b = 2 and
product of the roots ab = – 1 a b
Since, (a + b) = a2 + b2 + 2ab 57 1 4 14 1
Þ 4 = a2 + b2 – 2 Þ a2 + b2 = 6 10. (a) i + 25 = (i ) .i + 4 6
i (i ) .i
a2 b2 a4 +b4 æ 1 ö
Now, a2b–2 + a–2b2 = + = 1
b a (ab) 2 2 2 =i+ (Qi 4 = 1) = i - i çèQ = -i÷ø
i i
Þ (a2 + b2)2 = 62 Þ a4 + b4 + 2a2b2 = 36 =0
Þ a4 + b4 + 2 = 36 Þ a4 + b4 = 34 ...(i) 11. (d) Let f (x) = x2 + x + a
1
a 4 + b4 34 D ³ 0 Þ 1 – 4a ³ 0 Þ a £
Þ = = 34 4
(ab)2 (-1) 2 f (a) > 0 Þ a2 + a + a > 0 Þ a(a + 2) > 0
[Putting value of a4 + b4 = 34 from Equation (i)] 1 1
5. (a) Given equation is (x – a) (x – b) + (x – b) (x Þ a < – 2 or a > 0 – >aÞa<–
– c) + (x – c) (x – a) = 0 2 2
Þ 3x2 – 2(b + a + c) x + ab + bc + ca = 0 The common values of a are a < – 2
Now, here A = 3, B = – 2 (a + b + c) z +2
12. (d) Real part of is given by
C = ab + bc + ca \ D = B – 4 AC 2 z -1

= (–2(a + b + c))2 – 4(3)(ab + bc + ca) 1 é z + 2 æ z + 2 öù


ê +ç ÷ú = 4
2 ë z -1 è z -1 ø û
= 2 a2 + b2 + c2 – ab – bc – ca
z +2 z+2
Þ + =8
1 z -1 z - 1
= 2 {(a – b) 2 – (b – c) 2 + (c – a)2} ³ 0
2
Þ z z - z + 2 z - 2 + z z + 2 z - z - 2 = 8( z z - z - z + 1)
http://t.me/iitjeehelps

Complex Numbers and Quadratic Equations M – 15

z z - z + 2 z - 2 + z z + 2 z - z - 2 = 8( z z - z - z + 1) 17. (d) Putting z = a + 2i in the given equation and


3 3 comparing imaginary parts, we get a2 + 4 = a2,
Þ zz - z- z +2=0 ..... (i) which is not possible.
2 2 18. (a) The discriminant of the equation
Comparing with the equation
3 ( -2 2 ) 2 – 4 (1) (1) = 8 – 4 = 4 > 0 and a
z z + a z + a z + b = 0 , we get a = – and b = 2. perfect square
2
so roots are real and different but we can't
Thus, the locus of z given by the equation
say that roots are rational because coeffi
3 1 cients are not rational therefore.
(i) is a circle with centre and radius =
2 2
( 2 2)
2
13. (d) We have x + 1 º ( x + 1) ( x - x + 1) .
3 2
2 2± -4 2 2 ± 2 = 2 ±1
Therefore, a and b are the complex cube =
2 2
roots of –1 so that we may take a = –w and
this is irrational \ the roots are real and different.
b = –w2, where w ¹ 1 is a cube root of unity..
1 - ia 1 + ia
Thus a100 = ( - w)100 = w and 19. (a) A + iB = Þ A – iB =
1 + ia 1 - ia
b100 = ( - w2 )100 = w2 so that the (1 - ia) (1 + ia)
Þ (A + iB) (A - iB) = =1
required equation is x2 + x + 1 = 0. (1 + ia) (1 - ia)
p æ pö Þ A2 + B2 =1
14. (c) sin + i ç1 - cos ÷ 20. (c) Here Let x – a is the common factor
5 è 5ø
then x = a is root of the corresponding
p p p equation \ a2 – 11a + a = 0
= 2 sin cos + i2sin2
10 10 10 a2 – 14a + 2a = 0
p æ p pö Subtracting 3 a – a = 0 Þ a = a/3
= 2sin çè cos + i sin ÷ø a2 a
10 10 10
Hence - 11 + a = 0, a = 0 or a = 24
p 9 3
sin
10 p since a ¹ 0, a = 24
For amplitude, tan q = = tan
p 10
ïì x - 11x + 24
2
cos
10 \ the common factor of 2 í is
p ïî x - 14x + 48
Þ q= clearly x – 8
10
15. (a) We have, x = w – w2 –2 or x + 2 = w – w2 1 1 1
21. (d) - =
Squaring, x2 + 4x + 4 = w2 + w4 – 2w3 x+a x+b x+c
= w2 + w3w. –2w3 = w2 + w – 2 [w3 = 1] or x2 + (a + b) x + ab = (b – a) x + (b – a) c
= –1 – 2 = – 3 Þ x2 + 4x + 7 = 0 or x2 + 2ax + ab + ca – bc = 0
Dividing x4 + 3x3 + 2x2 –11x – 6 byx2 + 4x + 7, Since product of the roots = 0
we get bc
x4 + 3x3 + 2x2 – 11x–6= (x2 +4x+7)(x2 – x–1) + 1 ab + ca – bc = 0 ; a =
b+c
= (0) (x2 – x – 1)+1 = 0 + 1 = 1
16. (d) The second equation can be rewritten as -2bc
Thus sum of roots = – 2a =
2 b+c
æ x ö æ x ö
aç ÷ + b ç x +1 ÷ + c = 0 z1 z1
è x +1 ø è ø 22. (a) z1 = = | z1 |2 z1-1
x z1
and hence its roots correspond to =a
x +1 Þ arg(z1–1) = arg (z1 ) Þ arg (z2)
x
and = b. Þ z2 = kz1–1 (k > 0)
x +1
a b -b
23. (a) Let a > 0, c > 0 , < 0 Þ -b < 0
Hence x = and . 2a
1- a 1-b
EBD_7167
http://t.me/iitjeehelps

M – 16 BITSAT Topicwise Solved Papers


Þ b > 0 Þ a > 0, b > 0, c > 0 combining we get,
Similarly, if a < 0 , we obtain b < 0, c < 0. a 2 - m 2 £ b < a 2 \ b Î [a 2 - m 2 , a 2 )
28. (c) Let f (x) = x2 + 2 (k +1) x + 9k – 5. Let a, b be
the roots of f (x) = 0. The equation f (x) = 0
will have both negative roots, if –
-b (i) Disc. ³ 0 (ii) a < 0, b < 0, i.e. (a + b) < 0 and
2a (0, c) (iii) f (0) > 0
O Now, D ³ 0 Þ 4 (k +1)2 – 36k + 20 ³ 0
\ a, b, c are of same sign Þ k2 – 7k + 6 ³ 0 Þ (k – 1) (k – 6) ³ 0
Þ k £ 1 or k ³ 6 ...(i)
24. (b) Let z1 = r1 (cos q1 + i sin q1 ) . (a + b) < 0 Þ – 2 (k +1) < 0
z1 Þ k +1 > 0 Þ k > – 1 ...(ii)
\ = 1 Þ | z1 | = | z 2 | Þ | z1 | = | z 2 | = r1 and, f (0) > 0 Þ 9k – 5 > 0 Þ k < 5/9 ...(iii)
z2 From (i), (ii), (iii), we get k ³ 6.
Now arg (z1 z2) = 0 Þ arg(z1) + arg(z2 ) = 0 29. (b) x = 2 + 3 (x–2)2=3 i.e.,x2 –4x+1=0 ...(i)
Þ arg(z 2 ) = -q1 Therefore, x4 – 8x3 + 24x2 – 32x + 16 = 9
x4 – 8x3 + 18x2 – 8x + 2 + 6 (x2 – 4x + 1) – 1 = 0
z 2 = r1 (cos( -q1 ) + i sin( -q1 ) From (i), x4 – 8x3 + 18x2 – 8x + 2 = 1
= r1 (cos q1 - isin q1) = z1 30. (d) We have a + b = - p, ab = q

( )
Þ z2 = z1 = z1 Þ | z 2 |2 = z1z 2
Now (wa + w 2 b) (w 2 a + wb )
= a 2 + b 2 + (w + w 2 ) ab = a 2 + b 2 - ab
3-i 3+i (3 - i)(2 - i) + (3 + i)(2 + i)
25. (d) z= + = = (a + b) 2 - 3ab = p 2 - 3q
2+i 2-i (2 + i) (2 - i)
31. (b)
Þ z = 2 Þ (iz) = 2i, which is the positive
p
imaginary quantity \ arg (iz) =
2
(cos q + i sin q )4
26. (b) = (cosq + i sinq) 4
(cos q - i sin q) 3
(cosq – i sinq)–3 Using the result |z1+z2 | £ |z1| + |z2|, we get |z|
= (cos 4 q + i sin 4q) {cos (–q) + i sin (– q)}–3 + |z – i| = |z| + |i – z| [since |z| = | – z|]
= (cos 4 q + i sin 4q) {cos(– 3) (– q) £ | z + i – z | = |i| = 1
+ i sin (–3) (– q)} \ minimum value of |z| + |z–i| is 1
= (cos 4q + i sin 4q) {cos3q + i sin 3q} æ 1ö æ 1ö
3
= cos 4q cos 3q – sin 4q sin 3q 32. (a) We have, ç x + ÷ + ç x + ÷ = 0
x è ø
x è ø
+ i (sin 4q cos 3q + sin3q cos 4q)
1 ö éæ ù
2
= cos (4q + 3q) + i sin (4q + 3q) æ 1ö
= cos 7q + i sin 7q Þ ç x + ÷ êç x + ÷ + 1ú = 0
è x ø êè
ë
xø úû
27. (b) Let the roots be a an d b , then
1
a + b = -2a and ab = b By hypothesis Þ either x + =0 Þ x2 = –1 Þ x = ± i
x
| a - b | £ 2 m Þ (a - b ) 2 £ 4 m 2 æ 1ö
2
or çx + ÷ + 1 = 0
Þ (a + b) 2 - 4ab £ 4m 2 è xø
1
Þ 4a 2 - 4b £ 4m 2 Þ b ³ a 2 - m 2 Þ x2 + + 3 = 0 Þ x4 + 3x2 + 1 = 0
x2
Again , the roots are real and distinct \D> 0
–3 ± 9 – 4 –3 ± 5
Þ 4a 2 - 4b > 0 Þ b < a 2 . On Þ x2 = = <0
2 2
\ There is no real root.
http://t.me/iitjeehelps

Linear Inequalities M – 17

Chapter
Linear Inequalities
6
1. The necessary condition for third quadrant (c) (3, 0) (d) None of these
region in xy-plane, is [2014]
2x + 3 4 x - 1
(a) x > 0, y < 0 (b) x < 0, y < 0 4. If < , then x lies in the interval
5 2
(c) x < 0, y > 0 (d) x < 0, y = 0 [2009]
2. If x satisfies | 3 x – 2 | + | 3x – 4 | + | 3x – 6 | ³ 12, then
(a) é 11 ö (b) é 11 , ¥ ö÷
(2013) ê 0, 16 ÷ ê16
ë ø
8 8 -4 ë ø
(a) 0 £ x ³ (b) x ³ or
3 3 3 æ 11 ö æ 11 ö
8 (c)ç 0, ÷ (d) ç , ¥ ÷
(c) x £ 0 or x ³ (d) x ³ 2 only è 16 ø è 16 ø
3 5. Number of solution for the system of inequalities
3. Which of the following is not a vertex of the
positive region bounded by the inqualities 2x +
3y £ 6, 5x + 3y £ 15 and x, y ³ 0 (2011) { 2x + 1 < x + 2,
x - 1 > 2x (2007)
(a) (0, 2) (b) (0, 0) (a) 1 (b) 2 (c) 3 (d) 0

Hints & Solutions


1. (b) It is obvious. Y
(0, 5) 5x + 3y = 15
1 4 2x + 3y = 6
2. (c) Dividing R at , and 2, analyse 4 cases.
2 3
(0, 2)
2
When x £ , the inequality becomes
3
(3,0)
x
2 – 3x + 4 – 3x + 6 – 3x ³ 12. O
implying – 9x ³ 0 Þx £ 0.
2x + 3 4x - 1
when x ³ 2 the ineqality becomes 4. (d) < Þ – 16x < – 11
5 2
3x – 2 + 3x – 4 + 3x – 6 ³ 12, 11
Þ 16x > 11 Þ x >
Implying 9x ³ 24 Þ x ³ 8/3 16
The inequality in invalid in the other two æ 11 ö
Hence, x Î ç , ¥÷
sections. è 16 ø
5. (d) The given system of inequalities is equiva-

{
\ either x £ 0 or x ³ 8/3 x < 1,
3. (d) Here (0, 2), (0, 0) and (3, 0) all are vertices of lent to the system -1 > x
feasible region. Hence option (d) is correct. since either inequality of the system is re-
placed by an equivalent inequality. The ob-
tained system has no solution, consequently,
the given system has no solution either.
EBD_7167
http://t.me/iitjeehelps

M – 18 BITSAT Topicwise Solved Papers

Chapter
Permutations & Combinations
7
1. How many different nine digit numbers can be 8. In how many ways can a committee of 5 made
formed from th e number 223355888 by out 6 men and 4 women containing atleast one
rearranging its digits so that the odd digits woman? [2014]
occupy even positions ? [2017] (a) 246 (b) 222
(a) 16 (b) 36 (c) 60 (d) 180 (c) 186 (d) None of these
2. Statement 1 : A five digit number divisible by 3 9. In how many ways can 5 boys and 5 girls be
is to be formed using the digits 0, 1, 2, 3, 4 and 5 seated at a round table so that no two girls may
with repetition. The total number formed are 216. be together ? [2013]
Statement 2 : If sum of digits of any number is (a) 4! (b) 5! (c) 4! + 5! (d) 4! × 5!
divisible by 3 then the number must be divisible 10. A box contains two white balls, three black balls
by 3. [2017] and four red balls. In how many ways can three
(a) Statement-1 is true, Statement-2 is true and balls be drawn from the box if at least one black
is a correct explanation for Statement -1 ball is to be included in the draw? [2013]
(b) Statement -1 is true, Statement -2 is true (a) 64 (b) 129
and is NOT a correct explanation for (c) 84 (d) None of these
Statement - 1 11. In how many ways can 5 prizes be distributed
(c) Statement - 1 is true, Statement- 2 is false among 4 boys when every boy can take one or
(d) Statement -1 is false, Statement -2 is true more prizes ? [2012]
(a) 1024 (b) 625 (c) 120 (d) 600
n n
C
3
3. If å (-1)r r +3 Cr
r= 0 a + 3
=
, then a – n is equal to 12. The number of positive integral solution of abc
= 30 is [2012]
r
[2017] (a) 30 (b) 27
(a) 0 (b) 1 (c) 8 (d) None of these
(c) 2 (d) None of these 13. If 20Cr = 20Cr–10 then 18Cr is equal to (2011)
4. The number of values of r satisfying the equation (a) 4896 (b) 816
39
C 3r -1 - 39 C = 39 C - 39 C 3r is [2016] (c) 1632 (d) None of these
r2 r 2 -1 14. In a polygon no three diagonals are concurrent.
(a) 1 (b) 2 (c) 3 (d) 4 If the total number of points of intersection of
5. All the words that can be formed using diagonals interior to the polygon be 70 then the
alphabets A, H, L, U and R are written as in a number of diagonals of the polygon is [2010]
dictionary (no alphabet is repeated). Rank of (a) 20 (b) 28
the word RAHUL is [2016] (c) 8 (d) None of these
(a) 71 (b) 72 (c) 73 (d) 74
15. With 17 consonants and 5 vowels the number of
6. The total number of 4-digit numbers in which
words of four letters that can be formed having
the digits are in descending order, is [2015]
two different vowels in the middle and one
(a) 10C4 × 4! (b) 10C4
consonant, repeated or different at each end is
10! [2010]
(c) (d) None of these (a) 5780 (b) 2890 (c) 5440 (d) 2720
4!
7. The number of all three elements subsets of the 16. The letters of the word TOUGH are written in all
set {a1, a2, a3 . . . an} which contain a3 is[2014] possible orders and these words are written out
(a) nC 3 (b) n – 1C3 as in a dictionary, then the rank of the word
TOUGH is [2009]
(c) n – 1C2 (d) None of these (a) 120 (b) 88 (c) 89 (d) 90
http://t.me/iitjeehelps

Permutations & Combinations M – 19

17. If 2n +1Pn–1 : 2n – 1Pn = 3 : 5 then the value of n 20 The number of arrangements of the letters of the
is equal to [2008] word
(a) 4 (b) 3 (c) 2 (d) 1 BANANA is which the two ‘N’s do not appear
18. 3 integers are chosen at random from the set of adjacently is [2007]
first 20 natural numbers. The chance that their (a) 40 (b) 60 (c) 80 (d) 100
product is a multiple of 3, is – [2008] 21. If nCr – 1 = 36, nCr = 84 and nCr + 1 = 126, then
(a) 194/285 (b) 1/57 (c) 13/19 (d) 3/4 find r. (2006)
19. If the total number of m elements subsets of the (a) 2 (b) 1 (c) 3 (d) 5
set A = {a1, a2, a3, ...., an} is l times the number 22. A bag contains 3 one rupee coins, 4 fifty paisa
of 3 elements subsets containing a4, then n is coins and 5 ten paisa coins. How many selection
[2007] of money can be formed by taking atleast one
(a) (m – 1) l (b) ml (c) (m + 1) l (d) 0 coin from the bag. (2005)
(a) 90 (b) 140 (c) 119 (d) 125

Hints & Solutions


1. (c) X - X - X - X - X. The four digits 3, 3, 5,5 can 6 æ ( n + 3)( n + 2) ö
4! = ç1 - n - 3 + ÷
be arranged at (-) places in = 6 ways. ( n + 1)(n + 2)(n + 3) è 2 ø
2!2!
The five digits 2, 2, 8, 8, 8 can be arranged
3 3
5! = (n 2 + 3n + 2) =
at (X) places in ways = 10 ways (n + 1)(n + 2)(n + 3) n+3
2!3!
Total no. of arrangements = 6 ´ 10 = 60 ways 3 3
Given, = Þn=aÞa–n =0
2. (d) Number form by using 1, 2, 3, 4, 5 = 5! = 120 n+3 a+3
Number formed by using 0, 1, 2, 4, 5
39
4. (b) C 3r -1 - 39 C = 39 C - 39 C 3r
r2 r 2 -1
4 4 3 2 1 = 4.4.3.2.1 = 96
Þ 39 C 3r -1 + 39
C3r = 39 C + 39 C
Total number formed, divisible by 3 (taking r 2 -1 r2
40 40
numbers without repetition) = 216 Þ C 3r = C 2 2
Þ r = 3r or r = 40 - 3r
r2
Statement 1 is false and statement 2 is true.
Þ r = 0, 3 or – 8, 5
n n
3. (a) Cr 1 Cr 3 and 5 are the values as the given equation
= 3! .
r+3 (r + 3)(r + 2) (r + 1) is not defined by r = 0 and r = –8. Hence, the
Cr
n +1 number of values of r is 2.
1 C r +1 5. (d) No. of words starting with A are 4 ! = 24
= 3! .
(r + 3)(r + 2) (n + 1) No. of words starting with H are 4 ! = 24
n +2 No. of words starting with L are 4 ! = 24
1 C r +2 These account for 72 words
= 3!
(r + 3)(n + 1) n+2 Next word is RAHLU and the 74th word
RAHUL.
3! n +3
= C r +3 6. (b) Total number of arrangements of 10 digits
( n + 1)( n + 2) ( n + 3) 0, 1, 2, ...., 9 by taking 4 at a time = 10C4 × 4!
n n
C We observe that in every arrangement of 4
\ å
(-1) r r + 3 r
Cr selected digits there is just one
r =0 n
6 arrangement in which the digits are in
=
( n + 1)( n + 2)(n + 3) å (-1) r n +3
C r+3 descending order. \ Req. number of 4-digit
r=0
10
6 C 4 ´ 4! 10
= [ n +3 C 0 - n +3C1 + n + 3C 2 ] numbers. = = C4
( n + 1)( n + 2)( n + 3) 4!
EBD_7167
http://t.me/iitjeehelps

M – 20 BITSAT Topicwise Solved Papers


7. (c) The number of three elements subsets ways, as repetition is allowed for conso-
containing a3 is equal to the number of nants. The two vowels in the middle are
ways of selecting 2 elements out of n – 1 distinct so that the number of ways of fill-
elements. So, the required number of ing up the two places is 5P2 = 20.
subsets is n –1C2. The no. of different words = (17)2 . 20 = 5780.
8. (a) A committee of 5 out of 6 + 4= 10 can be 16. (c) Rank = (4!´ 3) + (3!´ 2) + (2!´ 2) + 1
made in 10C5 = 252 ways.
= 72 + 12 + 4 + 1 = 89
If no woman is to be included, thennumber
of ways = 5C5 = 6 2n +1
3
Pn -1 (2n + 1)! (n - 1) 3
17. (a) Þ = ´ =
\ the required number = 252 –6 = 246 2n -1 (n + 2)! (2n - 1)! 5
Pn 5
9. (d) Leaving one seat vacant between two boys, 2
5 boys may be seated in 4! ways. Then at Þ 3n – 11n – 4 = 0 Þ (3n + 1) (n – 4) = 0
remaining 5 seats, 5 girls any sit in 5! ways. æ 1ö
Hence the required number = 4! × 5! \ n = 4 çè n ¹ - ÷ø
3
10. (a) Atleast one black ball can be drawn in the
following ways 18. (a) Total number of ways of selecting 3 inte-
gers from 20 natural numbers = 20C3 = 1140
(i) one black and two other colour balls
their product is a multiple of 3 means, at
= 3C1 × 6C2 = 3 × 15 = 45
least one number is divisible by 3.
(ii) two black and one other colour balls
The numbers which are divisible by 3 are 3,
= 3C2 × 6C1 = 3 × 6= 18
(iii) All the three are black = 3C3 × 6C0 = 1 6, 9, 12, 15, 18 and the number of ways of
\ Req. no. of ways = 45 + 18 + 1 = 64 selecting atleast one of them is
6
11. (a) First prize may be given to any one of the 4 C1 ´ 14 C2 + 6 C2 ´ 14 C1 + 6 C3 = 776
boys, hence first prize can be distributed in
4 ways. 776 194
Probability = =
similarly every one of second, third, fourth 1140 285
and fifth prizes can also be given in 4 ways. 19. (b) Total number of m-elements subsets
\ the number of ways of their distribution of A = nCm ...(i)
= 4 × 4 × 4 × 4 × 4 = 45 = 1024 and number of m-elements subsets of A
12. (b) We have : 30 = 2 × 3 × 5. So, 2 can be each containing the element a4 = n–1Cm–1
assigned to either a or b or c i.e. 2 can be According to question, nCm = l. n–1Cm–1
assigned in 3 ways. Similarly, each of 3 and n n –1 n
5 can be assigned in 3 ways. Thus, the Þ . Cm –1 = l.n –1 Cm –1 Þ l = or n = ml.
m m
number of solutions is 3 × 3 × 3 = 27.
13. (b) 20Cr = 20Cr–10 Þ r + (r – 10) = 20 Þ r = 15 6! 5!
20. (a) Required number = 2!3! - 3! = 40
18.17.16
\ 18Cr = 18C15 = 18C3 = = 816
1.2.3 n
Cr
84
14. (a) A combination of four vertices is equivalent =n - r +1 7
21. (c) n Þ =
to one interior poin of intersection of Cr -1 36 r 3
diagonals. Þ 3n – 3r + 3 = 7r Þ 10r – 3n = 3 . . . (i)
n
C r +1 n - ( r + 1) + 1 126
and = =
n
Cr ( r + 1) 84
n-r 3
Þ = Þ 2n – 2r = 3r + 3
r +1 2
\ No. of interior points of intersection = Þ 5r – 2n = – 3 Þ 10 r – 4n = – 6
nC = 70
4 Substracting (ii) from (i) n = 9
Þ n (n – 1) (n – 2) (n – 3) = 5. 6. 7. 8 10r – 27 = 3 Þ 10 r = 30 Þ r = 3
\n=8 22. (c) Here are 3 things of first kind, 4 things of
So, number of diagonals = 8C2 – 8 = 20 second kind and 4 things of third kind so
15. (a) The two letters, the first and the last of the the total number of selections
four lettered word can be chosen in (17)2 = (3 + 1)(4 + 1)(5 + 1) – 1 = 119
http://t.me/iitjeehelps

Bionomial Theorem M – 21

Chapter
Bionomial Theorem
8
(c) 1 (d) None of these
1. The integer just greater than (3 + 5)2 n is
10. The term independent of x in the expansion of
divisible by (n Î N) [2017]
18
(a) 2n–1 (b) 2n+1 æ 1 ö
(c) 2n+2 (d) Not divisible by 2 çè 9x - ÷ , x > 0 , is a times the
3 xø
n r+2 n 8 corresponding binomial coefficient. Then a is
2. If å C r = 2 – 1 , then n = [2016]
r = 0 r +1 6 [2011]
(a) 3 (b) 1/3
(a) 8 (b) 4 (c) 6 (d) 5
(c) –1/3 (d) None of these
3. If the sum of odd numbered terms and the sum
11. In the binomial (21/3 + 3-1/3)n, if the ratio of the
of even numbered terms in the expansion of (x +
seventh term from the beginning of the
a)n are A and B respectively, then the value of
expansion to the seventh term from its end is
(x2 – a2)n is [2016]
1/6 , then n equal to [2011]
(a) A2 – B2 (b) A2 + B2
(a) 6 (b) 9 (c) 12 (d) 15
(c) 4AB (d) None of these
12. The coefficient of x32 in the expansion of :
4. If the third term in the expansion of
15
[ x + x log10 x ]5 is 106, then x may be [2016] æ 4 1 ö
ç x – 3 ÷ is:
-2 / 5 è x ø
(a) 1 (b) 10 (c) 10 (d) 10
(a) –15 15 –15 15
5. The coefficient of x4 in the expansion of C3 (b) C4 (c) C5 (d) C2
(1 + x + x2 + x3)11, is [2014] n
(a) 440 (b) 770 (c) 990 (d) 1001 æ x 1ö
13. If in the expansion of ç 2 + x ÷ , T3 = 7T2
6. If T0 , T1 , T2 ....Tn represent the terms in the è 4 ø
expansion of (x + a)n, then (T0 –T2 + T4 – .......)2 and sum of the binomial coefficients of second
+ (T1 – T3 + T5 – .....)2 = [2014] and third terms is 36, then the value of x is –
[2009]
(a) ( x 2 + a 2 ) (b) ( x 2 + a 2 )n (a) –1/3 (b) – 1/2 (c) 1/3 (d)1/2
(c) ( x 2 + a 2 )1/ n (d) ( x 2 + a 2 )-1/ n 14. If the second, third and fourth terms in the
expansion of (a + b)n are 135, 30 and 10/3
7. The coefficient of the middle term in the expan- respectively, then find the value of n. [2008]
sion of(2 + 3x)4 is : [2013] (a) 3 (b) 4 (c) 5 (d) 6
(a) 6 (b) 5! (c) 8! (d) 216
n
8. If C0, C1, C2, .............. Cn denote the binomial æ 2 1ö
coefficients in the expansion of (1 + x)n, then the 15. If the middle term in the expansion of ç x + ÷
è xø
value of [2013] 6
is 924 x , then find n. [2008]
C0 + (C0 + C1) + (C0+ C1 + C2) + .... + (C0+ C1 + (a) 58 (b) 18 (c) 12 (d) 20
.....+ Cn–1) 16. If the third term in the expansion of
(a) n.2n–1 (b) n.2n
n–1 [ x + x log10 x ]5 is 106, then x may be [2007]
(c) (n – 1).2 (d) (n – 1).2n
(d) 10 -2 / 5
9. 20
The coefficient of x in the expansion of (a) 1 (b) 10 (c) 10

( ) ( )
-5 5 5
æ 2 1ö 17. x + x3 - 1 + x - x3 - 1 is a
(1 + x2)40. çè x + 2 + 2 ÷ø is [2012]
x polynomial of the order of [2007]
(a) 30 C 10 (b) 30 C 25 (a) 5 (b) 6 (c) 7 (d) 8
EBD_7167
http://t.me/iitjeehelps

M – 22 BITSAT Topicwise Solved Papers


18. If (1 – x + x2)n = a0 + a1x + a2x2 + ...... + a2n x2n 19. Find the term independent of x in the expansion
then 9
a0 + a2 + a4 + .... + a2n equals [2006] æ4 2 3 ö
of ç x - ÷ . [2005]
è3 2x ø
3n + 1 3n - 1 1 - 3n 1 n
(a) (b) (c) (d) 3 + (a) 4 (b) 7 (c) 8 (d) 10
2 2 2 2

Hints & Solutions


1. (b) R = (3 + 5)2n , G = (3 - 5)2 n 5
Þ log10 x = 1, = - \ x = 10, = (10)–5/2
Let [R] + 1 = I ( Q [ . ] GIF) 2
5. (c) We have Coefficient of x4 in (1 + x + x2 + x3)11
Þ R + G = I Þ (3 + 5)2n + (3 - 5)2n = I
= coefficient of x4 in (1 + x2)11 (1 + x)11
seeing the option put n = 1 = coefficient of x4 in (1 + x)11 + coefficient
I = 28 is divisible by 4 i.e. 2n+1 of x2 in
n
r+2 n 28 - 1 11. (1 + x)11 + constant term is 11C2.
2. (d) å r +1
Cr =
6
(1 + x)11
r =0 = 11C4 + 11. 11C2 + 11C2 = 990.
n
é 1 ùn 28 - 1 6. (b) From the given condition, replacing a by ai
Þ å ê1 + C
ú r
r + 1û
=
6
and – ai respectively, we get
r =0 ë
( x + ai )n = (T0 - T2 + T4 - ......) + i (T1 - T3 + T5 - ....)
n 8
1 2 -1
Þ 2n + å n + 1 . n + 1Cr + 1 = 6
.........(i)
and ( x - ai )n = (T0 - T2 + T4 - ......) - i(T1 - T3 + T5 - ....)
r =0
.........(ii)
2n + 1 - 1 28 - 1
Þ 2n + = Multiplying (ii) and (i) we get required result
n +1 6
i.e., ( x 2 + a 2 ) n = (T0 - T2 + T4 - ....) 2 + (T1 - T3 + T5 - ....) 2
2 n ( n + 1 + 2) - 1 2 5 (6 + 2) - 1 7. (d) When exponent is n then total number of
Þ = terms are n + 1. So, total number of terms in
n +1 6 (2 + 3x)4 = 5
Comparing we get n + 1 = 6 Þ n = 5 Middle term is 3rd. Þ T3 = 4C2(2)2 . (3x)2
3. (a) (x + a ) n = n C 0 x n + n C1 x n -1 a + n C 2 x n -2 a 2 4 ´ 3 ´ 2 ´1
= ´ 4 ´ 9x 2 = 216 x2
+ n C3 x n -3 a 3 + n C 4 x n -4 a 4 + .... 2 ´1´ 2
8. (a) C0 + (C0 + C1) + (C0 + C1 + C2) + ................
= ( n C 0 x n + n C 2 x n -2 a 2 + n C 4 x n -4 a 4 + ....) + + (C0 + C1 + ............ Cn–1)
= nC0 + (n - 1)C1 + (n - 2)C2 + .......Cn -1
+ ( n C1 x n -1a + n C3 x n -3a 3 + n C5 x n -5 a 5 ) + ....
=A+ B ....(1) = C1 + 2C2 + 3C3 + 4C 4 .......nC n = n.2n -1
Similarly, (x – a)n = A – B .....(2)
Multiplying eqns. (1) and (2), we get -10
9. (b) Expression = (1 + x 2 ) 40 æ 1ö
( x 2 - a 2 ) n = A 2 - B2 . çè x + ÷ø
x
4. (c) Put log 10 x = y, the given expression (1 + x2)30. x10
=
becomes ( x + x y ) 5 . The coefficient of x20 in x10 (1 + x2)30
= the coefficient of
T3 = 5C2 .x 3(x y )2 = 10x3+2y = 106(given) x10 in (1 + x2)30 = 30C5 = 30C30-5 = 30C25
Þ (3 + 2 y) log10 x = 5 log10 10 = 5 æ 1 ö
r
5 10. (d) Tr + 1 = 18C (9x)18 – r
r ç- ÷
Þ (3 + 2y) y = 5 Þ y = 1, - è 3 xø
2
http://t.me/iitjeehelps

Bionomial Theorem M – 23

18-
3r
18-
3r n
C1ab n -1 135
Dividing (i) by (ii), we get =
= (– r)r 18Cr
9 x 2 2 n
C 2 a 2b n - 2 30
is independent ofx provided r = 12 and then a = 1.
11. (b) Tr + 1 = nCr an - r . br where a = 21/3 and b n b 9
=
= 3 -1/3 n
( n - 1) a 2 …(iv)
T7 from beginning = nC6 an - 6 b6 and 2
T7 from end = nC6 bn - 6 a6 b 9
= (n –1) …(v)
a n - 121 n - 12 n - 12 a 4
Þ n -12 = Þ 2 3 . 3 3 = 6 -1 Dividing (ii) & (iii), we get
b 6
Þ n - 12 = - 3 Þ n = 9 n (n - 1)
12. (b) We know by Binomial expansion, that (x + a)n 2 b
. =9
= nC0 xn a0 + nC1xn–1.a + nC2 xn – 2 a2 + nC3 n(n - 1) (n - 2) a …(vi)
xn–3 a3. 3.2
+ nC4xn–4. a4 +..... + nCnx0an Eliminating a and b from (v) and (vi), we get
15 n=5
æ 4 1 ö th
Given expansion is ç x – 3 ÷ æn ö
è x ø 15. (c) Since n is even therefore ç + 1÷ term is
è2 ø
On comparing we get n = 15, x = x4, n/2
n 2 n/2 æ 1ö
æ 1ö middle term, hence Cn/2 (x ) ç ÷
a = ç– ÷ è xø
è x3 ø = 924 x6 Þ xn/2 = x6 Þ n = 12
16. (c) Put log 10x = y, the given expression
15 0
æ 1ö 4 15 æ1ö becomes ( x + x y ) 5 .
\ ç x4 – 3 ÷ = 15
C 0 (x ) ç – 3 ÷
è x ø è x ø
T3 = 5 C 2 . x 3 ( x y ) 2 = 10x 3+ 2 y = 106 (given )
2
æ 1ö æ 1ö Þ (3 + 2 y) log10 x = 5 log10 10 = 5
+15 C1 (x 4 )14 ç – 3 ÷ +15 C 2 (x 4 )13 ç – 3 ÷
è x ø è x ø 5
Þ (3 + 2y) y = 5 Þ y = 1, -
3 4 2
æ 1ö æ 1ö
+15 C3 (x 4 )12 ç – ÷ +15 C4 (x 4 )11 ç – ÷ +... 5
è x3 ø è x3 ø Þ log10 x = 1 or log10 x = -
2
15 æ 1ö
r
\ x = 10 or x = (10)–5/2
Tr +11 = Cr (x 4 )15–r × ç – ÷ = –15 Cr x 60–7r
è x3 ø
( ) ( )
5 5
3 3
Þ x 60 –7r = x32 Þ
60 – 7r = 32 17. (c) x + x - 1 + x - x - 1
Þ 7r = 28 Þ r = 4 = 2 [x5 + 5C2 x3(x3–1) + 5C4 x (x3 – 1)2]
So, 5th term, contains x32 = 2 [x5 + 10x3 (x3 – 1) + 5x (x6 – 2x3 + 1]
4 = 10x7 +20 x6 + 2x5 – 20 x4 – 20x3 + 10 x
æ 1ö
= 15 C 4 (x 4 )11 ç – ÷ =15 C 4 x 44 x –12 \ polynomial has order of 7.
è x3 ø 18. (a) Put x = 1 Þ 1 = a0 + a1 + a2 + .... + a2n
Put x = –1 Þ 3n = a0 – a1 + a2 – a3
= 15 C4 x32 . + ...... + a2n
Thus, coefficient of x32 = 15C4. Adding, 3n + 1 = 2 (a0 + a2 + a4 + .... + a2n)
13. (a) C1 + n C 2 = 36 Þ n = 8
n
9
æ4 2 3 ö
T3 = 7 T2 Þ (2x)3 = 1/2 19. (b) Here comparing çè x - ÷ø with
3 2x
1 n
3x = –1 Þ x = – æ a 1ö
3 çè x ± b ÷ø
x
14. (c) T2 = nC1 abn –1 = 135 …(i) 9(2)
T3 = nC2 a2bn –2 = 30 …(ii) We get a = 2 , b = 1, n = 9 and r = =6
10 2 +1
T4 = nC3 a3bn –3 = …(iii) \ ( 6 + 1) = 7th term is independent of x.
3
EBD_7167
http://t.me/iitjeehelps

M – 24 BITSAT Topicwise Solved Papers

Chapter
Sequences & Series
9
1. If A, B, C are the angles of a triangle and 2 2 2
æ 1ö æ 2 1 ö æ 1 ö
iA iB iC
e , e , e are in A.P. Then the triangle must 8. ç x + ÷ + çç x + 2 ÷÷ + çç x 3 + ÷÷ ....upto
è x ø è x ø è x3 ø
be [2017] n terms is [2016]
(a) right angled (b) isosceles
x 2n – 1 +2
(c) equilateral (d) None of these x 2n +1
(a) 2 × 2n
+ 2n
n x –1 x
2. If å k ( k + 1) ( k - 1) = pn4 + qn3 + tn2 + sn,
k =1 x 2n + 1 x 2n + 2 – 1
(b) × – 2n
where p, q, t and s are constants, then the value x 2 +1 x 2n
of s is equal to [2017]
x 2n – 1 x 2n – 1
1 1 1 1 (c) × – 2n
(a) - (b) - (c) (d) x 2 –1 x 2n
4 2 2 4
3. After striking the floor a certain ball rebounds (d) None of these
9. If w is the complex cube root of unity, then the
4
th of its height from which it has fallen. The æ 1 3 9 27 ö
5 ç + + + +.... ÷
value of w + wè 2 8 32 128 ø
is [2015]
total distance that the ball travels before coming (a) – 1 (b) 1 (c) – i (d) i
to rest if it is gently released from a height of
120m is [2017] 3 15 63
10. The value of + + +... upto n terms is
(a) 960 m (b) 1000 m (c) 1080 m (d) Infinite 4 16 64
4. 21/4. 22/8. 23/16. 24/32......¥ is equal to- [2017] [2015]
(a) 1 (b) 2 (c) 3/2 (d) 5/2 4n 1 4- n 1
5. The fourth term of an A.P. is three times of the (a) n- - (b) n + -
3 3 3 3
first term and the seventh term exceeds the twice
of the third term by one, then the common 4n 1 4- n 1
difference of the progression is [2016] (c) n+ - (d) n - +
3 3 3 3
3 11. If binomial coefficients of three consecutive
(a) 2 (b) 3 (c) (d) –1 terms of (1 + x)n are in HP, then the maximum
2 value of n is [2015]
6. The sum to n terms of the series (a) 1 (b) 2
1 3 7 15 (c) 0 (d) None of these
+ + + + .............. is [2016] 12. If the (2p)th term of a H.P. is q and the (2q)th term
2 4 8 16 is p, then the 2(p + q)th term is- [2014]
(a) n – 1 – 2 – n (b) 1 pq 2pq
(a) 2(p + q) (b) p + q
(c) n – 1 + 2 –n
(d) 1 + 2 – n
pq p+q
7. If log a, log b, and log c are in A.P. and also log a (c) p + q (d) pq
– log 2b, log 2b – log 3c, log 3c – log a are in A.P.,
then [2016]
are A. P., then æç + - ö÷
1 1 1 1 1 1
(a) a, b, c, are in H.P. 13. If , ,
a b c èa b cø
(b) a, 2b, 3c are in A.P.
(c) a, b, c are the sides of a triangle æ 1 1 1 ö is equal to [2014]
(d) None of these ç + - ÷
èb c aø
http://t.me/iitjeehelps

Sequences & Series M – 25

4 3 b 2 – ac 19. If m arithmetic means are inserted between 1 and


(a) – (b) 31 so that the ratio of the 7th and (m – 1)th means
ac b2 a 2b 2c2 is 5 : 9, then find the value of m. [2010]
4 1 (a) 14 (b) 24 (c) 10 (d) 20
(c) – (d) None of these
ac b2 20. The 100th term of the sequence 1, 2, 2, 3, 3, 3, 4,
14. The product of n positive numbers is unity, then 4, 4, 4,... is [2009]
(a) 12 (b) 13 (c) 14 (d) 15
their sum is : [2014] 21. The nth term of a GP is 128 and the sum of its n
(a) a positive integer (b) divisible by n terms is 255. If its common ratio is 2 then find the
1 first term. [2008]
(c) equal to n + (d) never less than n (a) 1 (b) 2 (c) 3 (d) 4
n 22. Find the geometric mean of numbers 7, 72, 73
15. Let Tr be the rth term of an A.P. for r = 1, 2, 3, .... If for ,.......,7n . [2008]
1 (n +1) (n +1) (n +1)
some positive integers m, n we have Tm = and (n +1)
n (a) 7 2 (b) 5 2 (c) 2 2 (d) 3 2
1
Tn = , then Tmn equals [2013] 23. Sum to 20 terms of the series 1.32 + 2.52 + 3.72 +...
m
is [2007]
1 1 1 (a) 178090 (b) 168090
(a) (b) + (c) 1 (d) 0
mn m n (c) 188090 (d) 190090
16. If x is positive then the sum to infinity of the 24. The sum of infinite terms of the geometric
series 2 +1 1 1
progression , , ........ is [2007]
1 1 - 3x (1 - 3x )2 (1 - 3x )3 2 -1 2 - 2 2
– + –
1 + 3x (1 + 3x )2 (1 + 3x )3 (a) 2( 2 + 1) 2 (b) ( 2 + 1)2
(
1 + 3x ) 4

........... ¥ is (2012) (c) 5 2 (d) 3 2 + 5


1 25. If the non-zero numbers x, y, z are in A. P. and
(a) 1/2 (b) tan – 1 x, tan – 1 y,, tan – 1 z are in A. P. then [2006]
6x
1 1
(c) 6x (1 + 3x ) (d) 2 (1 + 3x ) (a) x = y = z (b) y = zx
(c) x = yz (d) z = xy
26. th
8 term of the series 2 2, 2,0,..... will be –
17. If pth,qth and r th terms of H.P. are u,v,w respec-
tively, then find the value of the expression [2006]
(q – r) vw + (r – p) wu + (p – q) uv. [2011] (a) -5 2 (b) 5 2
(a) 2 (b) 0 (c) 4 (d) 8 (c) 10 2 (d) -10 2
18. If the sum of the first 2n terms of 2, 5, 8, ....... is 27. a, b, c are first three terms of a G.P. If HM of a and
equal to the sum of the first n terms of 57, 59, b is 12 and that of b and c is 36, then find the
61......., then n is equal to [2011] value of a. [2005]
(a) 10 (b) 12 (c) 11 (d) 13 (a) 6 (b) 12 (c) 10 (d) 8

Hints & Solutions


iB iA iC
1. (c) 2e = e + e n n
Þ 2cos B = cos A + cos C ...(i) 2. (b) å (k)(k + 1)(k - 1) = å k(k 2 - 1)
& 2 sin B = sin A + sin C ...(ii) k =1 k =1
Squaring and adding we get n 2
æ n(n + 1) ö n(n + 1)
cos( A - C) = 1 Þ A - C = 0
= å (k 3 - k) =ç
è 2 ø
÷ -
2
k =1
\ A = C, From (i) and (ii) cos B = cos A
and sin B = sin A So, A = B. Þ A = B = C n2 + n æ n2 + n - 2 ö
= ç ÷
2 è 2 ø
EBD_7167
http://t.me/iitjeehelps

M – 26 BITSAT Topicwise Solved Papers


5. (a) Let the progression be a, a + d, a + 2d,
n4 + n3 - 2n2 + n3 + n2 - 2n
= Then x 4 = 3x1 Þ a + 3d = 3a Þ 3d = 2a ...(i)
4
Again x 7 = 2x 3 + 1
n 4 n3 n2 n 1
= + - - Þs=- Þ a + 6d = 2(a + 2d ) + 1 Þ 2d = a + 1 ...(ii)
4 2 4 2 2
Solving (i) and (ii), we get a = 3, d = 2
3. (c) Clearly, the total distance described
1 3 7 15
é 4 4 4 6. (c) + + + + ...
= 120 + 2 ê120 ´ + 120 ´ ´ 2 4 8 16
ë 5 5 5
æ 1 ö æ 1 ö æ 1ö æ 1ö
4 4 4 ù = ç1 - ÷ + ç1 - ÷ + ç1 - ÷ + ç1 - ÷ + ....
+120 ´ ´ ´ + ......to ¥ú è 2 ø è 4 ø è 8 ø è 16 ø
5 5 5 û
Except in the first fall the same ball will travel 1ì 1 ü
twice in each step the same distance one upward í1 - ý
2 î 2n þ
and second downward travel. =n- = n - 1 + 2-n
1
\ Distance travelled 1-
2
First fall 7. (c) log a, log b, log c are in A.P.
Rebound and Þ 2 log b = log a + log c Þ log b 2 = log(ac)
second fall
120m 4
Þ b 2 = ac Þ a, b, c, are in G.P..
120´ m
5 log a –log 2b, log 2b–log 3c, log 3c – log a are in A.P.
Þ 2 (log 2b – log 3c) = (log a – log 2b)
+ (log 3c – log a)
Þ 3 log 2b = 3 log 3c Þ 2b = 3c
é 4 æ 4 ö2 ù
= 120 + 2 ´120 ê + ç ÷ + .......to ¥ ú 2 2b 2a 4a
êë 5 è 5 ø úû Now, b2 = ac Þ b = a . Þb = , c=
3 3 9

é 4 ù 2a 4a
i.e. a = a, b = , c=
ê ú 3 9
= 120 + 240 ê 5 ú = 120 + 240 ´ 4 = 1080 m
ê1 - 4 ú 2 4
ë 5û Þ a : b : c = 1: : = 9:6:4
3 9
4. (b) The given product Since, sum of any two is greater than the 3rd, a,
1 2 3 4 b, c, form a triangle.
+ + + +....
= 24 8 16 32 = 2s (say) 8. (a) The series is
1 2 3 4 æ 1 1 1 ö
Now S = + + + + ..... ...(i) ( x 2 + x 4 + x 6 + ...) + çç + + + .... ÷÷
2 4 6
4 8 16 32 èx x x ø
1 1 2 3 + ( 2 + 2 + .....)
Þ S= + + +... ...(ii)
2 8 16 32 1 æ 1 ö
Apply; (i) – (ii) 2 2n
2 ç1 - 2n ÷
x (x - 1) x è x ø + 2n
= +
1 1 1 1 x2 -1 1
Þ S = + + + ..... 1-
2 4 8 16 x2
1/ 4 1
= = \ S=1 Þ Product = 21 = 2 x 2n - 1 x 2 n + 2 + 1
1 - 1/ 2 2 = ´ + 2n
x 2 -1 x 2n
http://t.me/iitjeehelps

Sequences & Series M – 27

1 3 9 27 Þ n2 – 4nr + 4r2 + n = 0 Þ (n – 2r)2 + n = 0


9. (a) Consider + + + + .... which is not possible for any value for n.
2 8 32 128 12. (d) If a is the first term and d is the common
Which can be written as difference of the associated A.P.
30 31 32 33 1é 3 32 33 ù 1 1 1 1 1
+ + + + .... = ê1 + + + + ....ú
q
= + (2p – 1)d,
p
= + (2q – 1)d Þ d =
2pq
21 23 25 27 2 ê 2 2 2 4 26 a a
ë ûú
1 1
æ 3 32 ö If h is the 2(p + q)th term = + (2p + 2q – 1)d
a
Since çç 1 + 2 + 4 + _ _ _ ÷÷ is a G.P. therfore by h
è 2 2 ø 1 1 p+q
sum of infinite G.P, we have = + = .
q p pq
é ù é ù 1 1 1 1
1ê 1 ú 1ê 1 ú 13. (a) - = -
= ê ú= ê ú=2 a b b c
2 ê1 - 3 ú 2 ê1 - 3 ú
êë 22 úû ë 4û æ 1 1 1 öæ 1 1 1 ö
\ ç + - ÷ç + - ÷
\ Given expression è a b c øè b c a ø
æ 1 3 9 27 ö æ 2 1 öæ 2 1 ö 4 1 æ 2 2 ö 1
ç + + + +... ÷ = ç - ÷ç - ÷ = - ç + ÷ +
w+wè 2 8 32 128 ø = w + w2 = –1 è a b ø è c b ø ac b è a c ø b 2
3 15 63 4 2æ2ö 1 4 3
10. (b) Consider + + + ...upto n terms = - ç ÷+ = -
4 16 64 ac b è b ø b 2 ac b 2
14. (d) Since, product of n positive number is unity.
2 2 - 1 2 4 - 1 26 - 1 Þ x1 x2 x3 ....... xn = 1 ..(i)
= + + upto n terms
22 24 26 Using A.M. ³ GM
1
æ 1 ö æ 1 ö æ 1 ö x1 + x2 + ..... + xn
= ç1 - 2 ÷ + ç1 - 4 ÷ + ç1 - 6 ÷ + ...upto Þ ³ ( x1x2 .....xn ) n
è 2 ø è 2 ø è 2 ø n
n terms 1
= (1 + 1 + 1 + ....upto n terms ) Þ x1 + xn + ....... + xn ³ n (1) n [From eqn (i)]

æ 1 1 1 ö 1
-ç + + + ...upto n terms ÷ 15. (c). Tm = a + (m - 1)d = and
2 4 6 n
è2 2 2 ø
1
Tn = a + (n - 1)d =
é æ 1 ö nù m
ê1 - ú
ç ÷ 4-n 1
1 1
1 ê è 22 ø ú On solving, a = and d =
=n- ê ú =n+ - mn mn
22 ê 1 - 1 ú 3 3
1 1
ê 22 ú \ Tmn = a + (mn – 1) d = + (mn - 1) =1
ë û mn mn
11. (d) Let the coefficients of rth, (r + 1)th, and 16. (a) The series is a G.P. with common ratio
(r + 2)th terms be in HP. æ 1 - 3x ö 1 - 3x
2 1 1 =ç ÷ and | r | = is less than 1 since x is
Then, = + è 1 + 3x ø 1 + 3x
n n n
Cr Cr -1 Cr + 1 1
n n a 1 + 3x 1
Cr Cr n - r + 1 r +1 positive S¥ = = =
Þ 2= = Þ 2= + 1- r ì æ 1 - 3x ö ü 2
n
Cr -1 n
Cr +1 r n-r 1 - í- ç ÷ý
î è 1 + 3x ø þ
EBD_7167
http://t.me/iitjeehelps

M – 28 BITSAT Topicwise Solved Papers

1 1 1 23. (c) We have.


17. (b) Let H.P. be + + + ....... tn = [nth term of 1, 2, 3, ...] × [ nth term of 3, 5, 7, ....]2
a a + d a + 2d
= n (2n + 1) 2 = 4n 3 + 4n 2 + n
1 1
\u = a + (p - 1)d , v = a + (q - 1)d , \ Sn = St n = 4Sn 3 + 4Sn 2 + n
1 1 2
w= Þ a + (p – 1) d = ì n (n + 1) ü n (n + 1)(2n + 1) n (n + 1)
a + (r - 1)d u = 4.í ý + 4. +
î 2 þ 6 2
1 1
a + (q – 1) d = , a + (r –1)d = 2 1
v w = n 2 (n + 1) 2 + n (n + 1)(2n + 1) + n (n + 1)
Þ (q – r) {a + (p – 1)d} + (r – p) {a + (q – 1)d} + ...... 3 2
1 1 2 1
= (q – r) + (r – p) + .... Þ (q – r) vw + ..... = 0 \ S20 = 20 2.212 + .20 . 21 41 + × 20 × 21
u v 3 2
18. (c). Given, = 188090.
2n n 2 +1 1 1
{2.2 + (2n - 1)3} = {2.57 + (n - 1)2} , , ...........
2 2 24. (a)
2 -1 2( 2 - 1) 2
or 2 (6n + 1) = 112 + 2n or 10n = 110, \ n = 11
19. (a) Let the means be x1,x2,.....xm so that 1
1,x1,x2,....xm, 31 is an A.P. of (m + 2) terms. Common ratio of the series =
Now, 31 = Tm+2 = a + (m + 1)d = 1 + (m + 1)d 2( 2 + 1)
a æ 2 +1ö æ 1 ö
30 x7 5 \ sum = =ç ÷ / çç 1 - ÷
\d= Given : x = ç ÷
1– r è 2 - 1 ø è 2( 2 + 1) ÷ø
m +1 m -1 9
T8 a + 7d 5 ( 2 + 1) 2( 2 + 1)
\ T = a + (m - 1)d = 9 = . = 2( 2 + 1) 2
m ( 2 - 1) (1 + 2)
Þ 9a + 63d = 5a + (5m – 5)d 25. (a) We have 2 y = x + z and
30
Þ 4.1 = (5m – 68) 2 tan -1 y = tan -1 x + tan -1 z
m +1
Þ 2m + 2 = 75m – 1020 Þ 73m = 1022 2y x+z
Þ = Þ y 2 = xz .
1022 1- y 2 1 - xz
\ m= = 14
73
20. (c) 1 term ® 1, 2nd term = 2, 4th term ® 3,
st 26. (a) The given series 2 2 + 2 + 0 + ..... is an A.P..
7th term ® 4, 11th term ® 5,... Here a = 2 2, d = - 2
Series is 1, 2, 4, 7, 11,... Hence 8th term of the series
n(n - 1) n 2 - n + 2 = 2 2 + (8 - 1)(- 2) = -5 2
an = 1 + =
2 2 27. (d) Let given three terms be br, b, b/r
If n = 14, then an = 92,If n = 15, then an = 106.
21. (a). Let a be the first term. Then as given 2(br)b 2br
\ 12 = = ...(i)
Tn = 128 and Sn = 255 br + b r + 1
rTn - a 2(128) - a æ bö
But Sn = Þ 255 = Þ a =1 2b ç ÷
r -1 2 -1 è rø 2b
22. (a) Geometric mean of number 7, 7 , 7 ,.......,7n
2 3 and 36 = = ...(ii)
æ bö r + 1
= (7. 72. 73 ........7n)1/n = (71 + 2 + 3 + ....+n)1/n b+ç ÷
1/ n
è rø
é n (n +1) ù (n +1)
= ê7 2 ú =7 2 (i) / (ii) Þ r = 1/3
êë úû Then from (2) b = 24 \ a = br = 8
http://t.me/iitjeehelps

Chapter
Straightlines &Pairof straightlines
10
1. Given the system of straight lines a(2x + y – 3) + (c) x 2 - Ax + 2 A = 0
b(3x + 2y – 5) = 0, the line of the system situated (d) None of these
farthest from the point (4, –3) has the equation 7. If 4a2 + b2 + 2c2 + 4ab – 6ac – 3bc = 0, the family
[2017] of lines ax + by + c = 0 is concurrent at one or the
(a) 4x + 11y – 15 = 0 (b) 7x + y – 8 = 0 other of the two points- [2013]
(c) 4x + 3y – 7 = 0 (d) 3x – 4y + 1 = 0
2. The angle between the lines whose intercepts æ 1ö
(a) ç -1, - ÷ , (–2, –1)
on the axes are a, –b and b, –a respectively , is è 2ø
[2017] æ 1ö
-1 a 2 - b2 b2 - a 2 (b) (–1, –1), ç -2, - ÷
(a) tan (b) tan -1 è 2ø
ab 2 æ1 ö
-1 b 2
- a 2 (c) (–1, 2), ç , - 1÷
(c) tan (d) None of these è 2 ø
2ab
3. A ray of light coming from the point (1, 2) is æ1 ö
(d) (1, 2), ç , - 1÷
reflected at a point A on the x-axis and then è2 ø
passes through the point (5, 3). The co-ordinates 8. The nearest point on the line 3x + 4y = 12 from
of the point A is [2016] the origin is [2012]
æ 13 ö æ 5 ö æ 36 48 ö æ 3ö
(a) ç , 0÷ (b) ç , 0 ÷ (a) çè , ÷ø (b) ç 3, ÷
è 5 ø è 13 ø 25 25 è 4ø
(c) (–7, 0) (d) None of these æ 3ö
(c) çè 2, ÷ø (d) None of these
4. The eq. x 2 - 2 3xy + 3 y 2 - 3x + 3 3 y - 4 = 0 2
9. The distance of the point (–1, 1) from the line
represents [2016] 12(x + 6) = 5 (y – 2) is [2011]
(a) a pair of intersecting lines (a) 2 (b) 3 (c) 4 (d) 5
(b) a pair of parallel lines with distance between 10. The family of straight lines (2a + 3b) x + (a – b)
5 y+2a–4b= 0 is concurrent at the point [2011]
them
2 æ 2 -14 ö æ -2 -14 ö
(c) a pair of parallel lines with distance between (a) ç , ÷ (b) ç , ÷
them 5 2 è5 5 ø è 5 5 ø
(d) a conic section, which is not a pair of æ -2 14 ö æ 2 14 ö
straight lines (c) ç , ÷ (d) ç , ÷
è 5 5ø è5 5 ø
5. If P1 and P2 be the length of perpendiculars from 11. The reflection of the point (4, –13) in the line
the origin upon the straight lines x secq + y 5x + y + 6 = 0 , is [2010]
cosecq = a and x cosq – y sinq = a cos2q (a) (–1, –14) (b) (3, 4)
respectively, then the value of 4P12 + P22. [2014] (c) (1, 2) (d) (–4, 13)
(a) a 2 (b) 2a2 (c) a2/2 (d) 3a2 12. If the equations of the pairs of opposite sides of
6. The quadratic equation whose roots are the x a parallelogram are x2 – 5x + 6 = 0 and y2 – 6y +
and y intercepts of the line passing through (1, 1) 5 = 0, then equations of its diagonals are [2010]
and making a triangle of area A with the co- (a) x + 4y = 13, y = 4x – 7
ordinate axes is [2013] (b) 4x +y = 13, 4y = x – 7
(a) x 2 + Ax + 2A = 0 (c) 4x + y = 13, y = 4x – 7
(b) x 2 - 2Ax + 2A = 0 (d) y – 4x = 13, y + 4x = 7
EBD_7167
http://t.me/iitjeehelps

M – 30 BITSAT Topicwise Solved Papers


13. The line 3x – 4y + 7 = 0 is rotated through an (a) 3 x + y = 14 (b) 3 x – y = 14
p (c) 3 x + y + 14 = 0 (d) 3 x – y + 14 = 0
angle in the clockwise direction about the
4 16. The distance between the parallel lines
point (–1, 1). The equation of the line in its new
9 x 2 – 6 xy + y 2 + 18 x - 6 y + 8 = 0 is [2006]
position is [2009] 2 1
(a) 7y + x – 6 = 0 (b) 7y – x – 6 = 0 (a) (b)
(c) 7y + x + 6 = 0 (d) 7y – x + 6 = 0 10 10
4
14. Among the lines passing through C (3, 1) BA is (c) (d) None of these
farthest from the origin and cuts the x-axis and 10
y- axis at A and B respectively. Then BC:CA is 17. Find the distance of the line 2x – 3y = 4 from the
[2008] point (1, 1) in the direction of the line x + y = 1.
[2006]
(a) 9 : 1 (b) 1 : 9 (c) 3 : 1 (d) 3 : 1
(a) 3 (b) 5 (c) 2 (d) 2
15. The length of the perpendicular from the origin
18. If the lines 3y + 4x = 1, y = x + 5 and 5y + bx = 3 are
to a line is 7 and line makes an angle of 150° with
concurrent, then value of b is equal to
the positive direction of y-axis the equation of [2005]
the line is [2007] (a) 1 (b) 3 (c) 6 (d) 0

Hints & Solutions


1. (d) The given system of lines passes through Y
the point of intersection of the straight lines
2x + y – 3 = 0 and 3x + 2y –5 = 0 [L1 + lL2 =
0 form], which is (1, 1).
(5,3)
The required line will also pass through this
(1,2)
point. Further, the line will be farthest from
point (4, –3) if it is in direction perpendicular
to line joining (1, 1) and (4, –3). q q
O A(a,0) X
\ The equation of the required line is
-1 Qtan q + tan(p - q) = 0
y -1 = ( x - 1) Þ 3x - 4 y + 1 = 0
- 3 -1
3 2
4 -1 Þ + = 0 Þ 3 - 3a + 10 - 2a = 0
x y 5 - a 1- a
2. (c) Equation of lines are - = 1 and
a b 13
x y b a Þa=
- = 1 Þ m1 = and m2 = 5
b a a b æ 13 ö
Thus, the co-ordinates of A are ç , 0 ÷ .
è 5 ø
b a 3
- (b) We have a = 1, h = - 3 , b = 3, g = -
-1 a b b2 - a 2 4. ,
\ q = tan = tan -1 2
b a 2ab 3 3
1+ . f= , c = –4.
a b 2
3. (a) Let the co-ordinates of A be (a, 0). Then the
slope of the reflected ray is Thus abc + 2fgh - af 2 - bg 2 - ch 2 = 0
Hence the equation represents a pair of
3- 0
= tan q (say) straight lines.
5-a
Then the slope of the incident ray a h g 1
Again = = =-
2-0 h b f 3
= = tan( p - q)
1- a
http://t.me/iitjeehelps

Straight lines & Pair of straight lines M – 31

\ the lines are parallel. The distance 8. (a) If ‘D’ be the foot of altitude, drawn from
between them origin to the given line,y then ‘D’ is the required
point.
9
+4 Let Ð OBA = q B (0, 3)
g 2 - ac 5
=2 =2 4 = Þ tan q = 4/3 q
a (a + b ) 1(1 + 3) 2 Þ Ð DOA = q D

5. (a) We have P1 = length of perpendicular from we have A


(0, 0) on x secq + y cosecq = a OD = 12/5. O (4, 0)
x

If D is (h, k) then h = OD cosq, k = OD sinq


a Þ h = 36/25, k = 48/25.
i.e. P1 = = | a sinq cosq |
sec θ + cos ec 2 θ
2 9. (d) The given line is 12 (x + 6) = 5(y – 2)
Þ 12x + 72 = 5y – 10
a
= sin 2q or 2 P = | a sin2 q | or 12x – 5y + 72 + 10 = 0 Þ 12x – 5y + 82= 0
2 1
The perpendicular distance from (x1, y1) to
P2= Length of the perpendicular from (0, 0) on
x cosq – y sin q = a cos2q (ax1 + by1 + c)
the line ax + by + c = 0 is .
a 2 + b2
a cos 2q The point (x1, y1 ) is (–1, 1), therefore,
P2 = = | a cos 2q |
cos 2 q + sin 2 q perpendicular distance from (–1, 1) to the
Now, 4P12 + P22 = a2 sin2 2q + a2 cos2 2q = a2. line 12x – 5y + 82 = 0 is
6. (b) Equation of the line making intercepts a and | -12 - 5 + 82 | 65 65
= = = =5
x y
2
12 + (-5) 2 144 + 25 169
b on the axes is + = 1 .
a b 10. (a) Rewriting the equation (2x + y + 2) a +
Since, it passes through (1, 1) (3x – y – 4)b = 0 and for all a, b the straight
lines pass through the intersection of 2x +
1 1 y + 2 = 0 and 3x – y – 4 = 0
Þ + =1 ....(i)
a b
æ 2 14 ö
Also the area of the triangle formed by the i.e., the point çè , - ÷ø .
5 5
line and the axes is A.
11. (a) Let Q(a, b) be the reflection of P(4, – 13)
1 in the line 5x + y + 6 = 0
\ ab = A Þ ab = 2 A ....(ii)
2
æ a + 4 b - 13 ö
From eqs. (i) and (ii), we get, a + b = 2A Then the mid-point R ç , ÷ lies
Hence, a and b are the roots of the eq. è 2 2 ø
on 5x + y + 6 = 0
x 2 - ( a + b) x + ab = 0 Þ x 2 - 2Ax + 2A = 0
7. (a) 4a2 + b2 + 2c2 + 4ab – 6ac – 3bc æ a + 4 ö b - 13
\ 5ç + +6= 0
º (2a + b)2 – 3(2a + b) c + 2c2 = 0 è 2 ÷ø 2
Þ (2a + b – 2c) (2a + b – c) = 0 Þ c = 2a + b Þ 5a + b + 19 = 0 ...(i)
1 Also PQ is perpendicular to 5x + y + 6 = 0
or c = a + b
2 b + 13 æ 5 ö
The equation of the family of lines is Therefore ´ç- ÷ =–1
a - 4 è 1ø
æ 1ö Þ a – 5b – 69 = 0 ..(ii)
a (x + 2) + b(y+ 1) = 0 or a(x + 1) + b ç y + ÷ = 0 Solving (i) and (ii), we get a = – 1, b = – 14
è 2ø
giving the point of concurrence (–2, –1) or 12. (c) Equations of the sides of the
parallelogram are
æ 1ö (x – 3) (x – 2) = 0 and (y – 5) (y – 1) = 0
ç -1, - ÷ . i.e. x = 3, x = 2 ; y = 5, y = 1
è 2ø
EBD_7167
http://t.me/iitjeehelps

M – 32 BITSAT Topicwise Solved Papers


Hence its vertices are : A (2 ,1 ); B (3, 1) ; \ Equation of the
C (3, 5) ; D (2, 5) required line is xcos30° + ysin30° = 7
Equation of the diagonal AC is
3 1
4 or x + y ´ = 7or 3 x + y = 14
y – 1 = (x – 2) Þ y = 4x – 7 2 2
1 16. (a) 9 x 2 – 6 xy + y 2 + 18 x - 6 y + 8 = 0
Equation of the diagonal BD is
Þ ((3x)2 - 2 ´ (3x ) ´ y + y 2 )
4 + 6(3 x - y ) + 8 = 0
y– 1 = (x – 3) Þ 4x + y = 13
-1 Þ (3x - y )2 + 6(3 x - y ) + 8 = 0
13. (a) As (–1, 1) is a point on 3x – 4y + 7 = 0, the Let 3x - y = z \ z 2 + 6 z + 8 = 0
rotation is possible. Slope of the given line Þ z2 + 4z + 2 z + 8 = 0
= 3/4. Slope of the line in its new position Þ z ( z + 4) + 2( z + 4) = 0
3 Þ ( z + 2)( z + 4) = 0 Þ z = -2, z = -4
-1
4 1 3x - y + 2 = 0 ... (i)
= = –
3 7 or 3x - y + 4 = 0
1+
4 If P1 be the distance of line (i) from the
The required equation is 2 2
origin, then P1 = =
1 9 +1 10
y– 1= – (x + 1) or 7y + x – 6 = 0. Also, if P2 be the distance of line (ii) from
7 4
14. (a) If BA is farthest from the origin, then OC must the origin, then P2 =
10
be perpendicular to BA and has a slope = – 3. So, distance between lines
Equation of BA is y – 1 = – 3 (x – 3)
4 2 2
P = P2 - P1 = - =
æ 10 ö 10 10 10
3x + y = 10 \ A = ç , 0 ÷ and B = (0, 10).
è 3 ø 17. (c) Equation of line passing through (1, 1) and
B x -1 y -1
parallel to x + y = 1 is = =r
1/ 2 -1/ 2
\ any point at r distance from (1, 1) is
C(3, 1) æ r r ö
çè 1 + , 1- ÷ and it lie on the line 2x – 3y = 4
2 2ø
2r 3r
O A i.e., 2 + - 3+ = 4 i.e., r = 2
10 2 2
BC l l. + 1.0 18. (c) 3 lines, a1x + b1y + c1 = 0, a2x + b2y + c2 = 0
If = , then 3 =3
CA 1 and a3x + b3 y + c 3 = 0 are said to be
l +1
concurrent if :
\l =9 a1 (b 2 c3 - b3c 2 ) - b1 (a 2 c3 - a 3c 2 )
15. (a) Here p = 7 and a = 30° + c1 (a 2 b3 - a 3 b 2 ) = 0
Given equation of lines are
y
150° 3y + 4x = 1 ...(i)
y–x=5 ...(ii)
30° and 5y + bx = 3 ...(iii)
Therefore, given lines are concurrent if
Q 4 (–3 + 25) –3(3 + 5b) –1 (–5 –b) = 0
60° Þ 4 (22) – 9 – 15b + 5 + b = 0
30°
x¢ O x Þ 88 – 4 – 14b = 0 Þ 84 – 14b = 0
y¢ Þ 14b = 84 Þ b = 6
http://t.me/iitjeehelps

Conic Sections M – 33

Chapter
Conic Sections
11
1. The length of the semi-latus rectum of an ellipse æ3 ö
is one thrid of its major axis, its eccentricity would (a) (2, 2) (b) ç , 2 ÷
è2 ø
be [2017]
æ1 ö æ2 ö
2 2 1 1 (c) ç , 2÷ (d) ç 3 , 2 ÷
(a) (b) (c) (d) è2 ø è ø
3 3 3 2
10. Let S be the focus of the parabola y2 = 8x and
2. An equilateral triangle is inscribed in the circle PQ be the common chord of the circle
x2 + y2 = a2 with one of the vertices at (a, 0). x2 + y2 – 2x – 4y = 0 and the given parabola. The
What is the equation of the side opposite to this
area of DPQS is [2015]
vertex ? [2017]
(a) 4 sq units (b) 3 sq units
(a) 2x – a = 0 (b) x + a = 0
(c) 2 sq units (d) 8 sq units
(c) 2x + a = 0 (d) 3x – 2a = 0
3. The equation of one of the common tangents to 11. The eccentricity of an ellipse, with its centre at
the parabola y2 = 8x and x 2 + y 2 - 12x + 4 = 0 the origin, is 1/2. If one of the directrices is
is [2017] x = 4 , then the equation of the ellipse is:[2015]
(a) y = –x + 2 (b) y = x – 2 (a) 4x2 + 3y2 = 1 (b) 3x2 + 4y2 = 12
2 2
(c) y = x + 2 (d) None of these (c) 4x + 3y = 12 (d) 3x2 + 4y2 = 1
4. The line joining (5, 0) to ( (10 cos q, 10 sin q) is 12. Area of the circle in which a chord of length
divided internally in the ratio 2 : 3 at P. If q varies, 2 makes an angle p/2 at the centre, is[2015]
then the locus of P is [2016] (a) p/2 sq units (b) 2p sq units
(a) a pair of straight (b) a circle lines (c) p sq units (d) p/4 sq units
(c) a straight line (d) None of these 13. The angle of intersection of the two circles
5. The number of integral values of l for which x2 + y2 – 2x – 2y = 0 and x2 + y2 = 4, is -[2014]
x 2 + y 2 + lx + (1 - l) y + 5 = 0 is the equation of (a) 30º (b) 60º (c) 90º (d) 45º
a circle whose radius cannot exceed 5, is [2016] 14. An arch of a bridge is semi-elliptical with major
(a) 14 (b) 18 (c) 16 (d) None of these axis horizontal. If the length the base is 9 meter
6. The lengths of the tangent drawn from any point and the highest part of the bridge is 3 meter
on the circle 15x 2 + 15y 2 - 48x + 64 y = 0 to the from the horizontal; the best approximation of
two circles 5x2 + 5y2 – 24x + 32y + 75 = 0 and 5x2 + the height of the arch. 2 meter from the centre of
5y2 – 48x + 64y + 300 = 0 are in the ratio of [2016] the base is [2014, 2012]
(a) 1 : 2 (b) 18 (c) 16 (d) None of these (a) 11/4 m (b) 8/3 m (c) 7/2 m (d) 2 m
7. The length of the chord x + y = 3 intercepted by 15. A pair of tangents are drawn from the origin to
the circle x2 + y2+ 20 (x + y) + 20 = 0, then the
the circle x 2 + y 2 - 2x - 2 y - 2 = 0 is [2016] equation of the pair of tangent are [2013]
7 (a) x 2 + y 2 - 5xy = 0
3 3 7
(a)
2
(b) (c) 14 (d) (b) x 2 + y2 + 2x + y = 0
2 2
8. The locus of the point of intersection of two (c) x 2 + y 2 – xy + 7 = 0
tangents to the parabola y2 = 4ax, which are at
(d) 2x 2 + 2y2 + 5xy = 0
right angle to one another is [2016]
(a) x2 + y2 = a2 (b) ay2 =x 16. An ellipse has OB as semi minor axis, F and F '
(c) x + a = 0 (d) x + y ± a = 0 its focii and the angle FBF ' is a right angle.
9. The parabola having its focus at (3, 2) and Then the eccentricity of the ellipse is [2013]
directrix along the y-axis has its vertex at [2016]
EBD_7167
http://t.me/iitjeehelps

M – 34 BITSAT Topicwise Solved Papers

1 1 1 1 25. If the line 2x – 1 = 0 is the directrix of the parabola


(a) (b) (c) (d) y2 – kx + 6 = 0 then one of the values of k is
2 2 4 3
[2010]
17. If the line 2x – 3y = k touches the parabola (a) – 6 (b) 6 (c) 1/4 (d) – 1/4
y2 = 6x, then find the value of k. [2013]
(a) –15/4 (b) –27/4 (c) –1/4 (d) –3/4 26. The line ax + by = 1 cuts ellipse cx2 + dy2 = 1
18. S and T are the foci of an ellipse and B is an end only once if [2010]
of the minor axis. If STB is an equilateral triangle, c d
then the eccentricity of the ellipse is [2013] (a) ca 2 + db 2 = 1 (b) 2
+ =1
a b2
1 1 1 2
(a) (b) (c) (d) a 2 b2
4 3 2 3 (c) + =1 (d) ac 2 + bd 2 = 1
19. The length of the tangent drawn from any point c d
on the circle x2 + y2 + 2fy + l = 0 to the circle x2 27 Find the equation of chord of the circle x2 + y2
+ y2 + 2fy + m = 0, where m > l > 0, is [2012] = 8x bisected at the point (4, 3) [2010]
(a) y = 3 (b) y = 1 (c) y = 6 (d) y = 7
(a) m-l (b) m+l 28. Find the vertex of the parabola x2 – 8y – x + 19 = 0.
[2009]
(c) m2 - l2 (d) m + l
æ 1 75 ö æ 1 65 ö
20. Find the eccentricity of the conic represented (a) çè , ÷ø (b) çè , ÷ø
2 32 5 32
by x2 – y2 – 4x + 4y + 16 = 0 [2012]
(a) 2 (b) 2 (c) 2 2 (d) 3 2 æ 1 65 ö æ 1 35 ö
21. The length of the latus-rectum of the parabola (c) çè , ÷ø (d) çè , ÷ø
3 22 3 12
æ u2 u2 ö
whose focus is ç sin 2α, – cos 2α÷ and 29. Which of the following lines, is a normal to the
è 2g 2g ø parabola y2 = 16x ? [2008]
u2 (a) y = x – 11 cosq – 3 cos3q
directrix is y = , is [2011]
2g (b) y = x – 11 cosq – cos3q
u2 u2 (c) y = (x – 11) cosq + cos3q
(a) cos 2 α (b) cos 2α
g g (d) y = (x – 11) cosq – cos3q
2u 2 2u 2 30. For what value of l does the line y = x + l touches
(c) cos 2 2α (d) cos2 α
g g the ellipse 9x2 + 16y2 =144. [2008]
22. The equation of the ellipse with focus at (±5, 0) (a) ± 5 (b) ± 2 (c) ± 8 (d) ± 7
36 31. Find the equation of the hyperbola whose
and x = as one directrix is [2011] directrix is 2x + y = 1, focus (1, 2) and
5
x 2 y2 x 2 y2 eccentricity 3 . [2008]
(a) + =1 (b) + =1
36 25 36 11 (a) 7x2 – 2y2 + 12xy – 2x – 14 – 22 = 0
x 2 y2 (b) 7x2 + 2y2 – 12xy – 2x + 14 – 22 = 0
(c) + =1 (d) None of these (c) 7x2 + 2y2 + 12xy – 2x – 14 – 22 = 0
25 11
23. For what value of k the circles x2 + y2 + 5x + 3y + (d) 7x2 – 2y2 + 12xy – 2x + 14 – 22 = 0
7 = 0 and x2 + y2 – 8x + 6y + k = 0 cuts 32. Find the parametric coordinates of any point of
orthogonally [2011] the circle x2 + y2 + 2x – 3y – 4 = 0 [2007]
(a) 16 (b) –18 (c) – 13 (d) – 10
24. If the lines 3x – 4y + 4 = 0 and 6x – 8y – 7 = 0 are æ 7 3 7 ö
(a) çè -1 + cos q, + sin q÷ø
tangents to a circle, then the radius of the circle 3 2 3
is [2011]
(a) 3/2 (b) 3/4 (c) 1/10 (d) 1/20 æ 5 3 7 ö
(b) çè -1 + 3 cos q, 2 + 3 sin q÷ø
http://t.me/iitjeehelps

Conic Sections M – 35

æ 5 1 7 ö
(c) (-¥ - 7 ] È [ 7 , ¥ )
(c) çè -1 + cos q, + sin q÷ø (d) None of these.
3 2 3
35. The total number of common tangents to the
æ 2 3 7 ö two circles x2 + y2 – 2x – 6y + 9 = 0 and
(d) çè -1 + 3 cos q, 2 + 3 sin q÷ø
x2 + y2 + 6x – 2y + 1 = 0, is - [2005]
33. If the tangent at the point P(x1, y1) to the parabola (a) 1 (b) 2 (c) 3 (d) 4
y2 = 4ax meets the parabola y2 = 4a(x + b) at Q 36. The equation of the tangent to the elipse
and R, then the mid-point of QR is [2007] x 2 + 4 y 2 = 25 at the point whose ordinate is 2
(a) (x1 + b, y1 + b) (b) (x1 – b, y1 – b)
(c) (x1, y1) (d) (x1 + b, y1) is [2005]
34. If the line 3x + ay – 20 = 0 cuts the circle (a) x + 2y = 5 or 2x - y = 5
x 2 + y 2 = 25 at real distinct or coincident (b) 3x + 8y = 25 or 8 y - 3x = 25
points, then a belongs to the interval [2006]
(a) [- 7 , 7 ] (c) 3x + 2y = 15 or 3y - 2x = 15
(d) None of these
(b) (- 7 , 7 )

Hints & Solutions


3. (c) Any tangent to parabola y2 = 8x is
x2 y2
1. (c) Let eq. of ellipse be + = 1 , length of 2
a2 b2 y = mx + ...(i)
m
b2 a 2 (1 - e 2 ) 2 2
It touches the circle x + y - 12x + 4 = 0 , if the
semi-latus rectum = = = a (1 - e2 )
a a length of perpendicular from the centre (6, 0) is
equal to radius 32 .
1
Given a (1 - e 2 ) = (2a ) 2
3 6m + 2
m = ± 32 Þ æ 3m + 1 ö = 8(m 2 + 1)
2 2 1 1 çè ÷
Þ 1 - e2 = Þ e2 = 1- = Þ e = m2 + 1 mø
3 3 3 3
2. (c) Since the equilateral triangle is inscribed in Þ (3m2 + 1)2 = 8(m4 + m2 )
the circle with centre at the origin, centroid lies
on the origin. Þ m 4 - 2m 2 + 1 = 0 Þ m = ±1
Hence, the required tangents are y = x + 2 and
AO 2 y = –x – 2.
So, = Þ OD = 1 AO = a
OD 1 2 2 4. (b) Let P(x, y) be the point dividing the join of
So, other vertices of triangle have coordinates, A and B in the ratio 2 : 3 internally, then
20 cos q + 15
æ a 3 ö æ a 3 ö x= = 4 cos q + 3
C ç- , a and B ç - , -
ç 2 2 ÷÷ è 2 2 ø
a÷ 5
è ø x -3
y Þ cos q = ....(i)
C 4
20 sin q + 0 y
A y= = 4 sin q Þ sin q = ...(ii)
x 5 4
D O (a, 0) Squaring and adding (i) and (ii), we get the
B required locus ( x - 3) 2 + y 2 = 16 , which is a
\ Equation of line BC is : circle.
a
x =- Þ 2x + a = 0
2
EBD_7167
http://t.me/iitjeehelps

M – 36 BITSAT Topicwise Solved Papers


5. (c) Radius £ 5
7
AB = 2AD = 2 = 14
2
l2 (1 - l ) 2 8. (c) Let the two tangents to the parabola y2 =
+ - 5 £ 5 Þ l2 + (1 - l ) 2 - 20 £ 100
4 4 4ax be PT and QT which are at right angle
1 - 239 1 + 239 to one another at T(h, k). Then we have to
Þ 2l2 - 2l -119 £ 0 \ £l£ find the locus of T(h, k).
2 2 a
Þ -7.2 £ l £ 8.2 (approx.) We know that y = mx + , where m is the slope
m
\ l = -7, - 6, - 5, .......,7, 8 , in all 16 values is the equation of tangent to the parabola
6. (a) Let P(h, k) be a point on the circle y2 = 4ax for all m.
Y
15x 2 + 15y 2 - 48x + 64 y = 0
Then the lengths of the tangents from P(h, k) to (h,k) P
T
5 x 2 + 5 y 2 - 24 x + 32 y + 75 = 0 and
A
5x 2 + 5y 2 - 48x + 64y + 300 = 0 are
X

DIRECTRIX
24 32
PT1 = h 2 + k 2 - h + k + 15 Q
5 5
48 64
and PT2 = h 2 + k 2 - h+ k + 60 Since this tangent to the parabola will pass
5 5 through T(h, k) so
48 64 24 32 a
or PT1 = h- k- h + k + 15 k = mh + ; or m 2 h - mk + a = 0
15 15 5 5 m
32 24 This is a quadratic equation in m so will have
= k- h + 15 two roots, say m1 and m2, then
15 15
(Since (h, k) lies on k a
m1 + m 2 = , and m1 : m 2 =
15x 2 - 15y 2 - 48x + 64y = 0 h h
Given that the two tangents intersect at right
48 64
\h2 + k2 - h+ k = 0) and a
15 15 angle so m1. m2 = –1 or = -1 or h + a = 0
h
48 64 48 64 The locus of T(h, k) is x + a = 0, which is the
PT2 = h- k- h+ k + 60 equation of directrix.
15 15 5 5
9. (b) Vertex of the parabola is a point which lies
24 32 on the axis of the parabola, which is a line ^ to
=2 - h+ k + 15 = 2PT1 the directrix through the focus, i.e., y = 2 and
15 15 equidistant from the focus and directrix x = 0, so
Þ PT1 : PT2 = 1 : 2
æ3 ö
7. (c) The centre of the circle is C(1, 1) and radius that the vertex is ç , 2 ÷ .
of the circle is 2, perpendicular distance from C è2 ø
on AB, the chord x + y = 3 Y

C y=2
A S (3,2)
1+1 - 3 1
CD = = 2
2 2 O X
A D B 10. (a) The parametric equations of the parabola
x+y=3 y2 = 8x arex = 2t 2 and y = 4t.
1 7 and the given equation of circle is
\AD = 4 - = [AD = AC 2 - CD 2 ] x2 + y2 – 2x – 4y = 0
2 2 On putting x = 2t2 and y = 4t in circle
Hence, the length of the chord
http://t.me/iitjeehelps

Conic Sections M – 37

we get
4t4 + 16t2 – 4t2 – 16t = 0 x2 y2
The equation of the ellipse is + =1.
Þ 4t2 + 12t2 – 16t = 0 Þ 4t (t3 + 3t – 4) = 0 æ9ö
2
9
Þ t(t – 1) (t2 + t + 4) = 0 ç ÷
è2ø
Þ t = 0, t = 1 éQ t 2 + t + 4 ¹ 0ù Where centre is assumed as origin and base as
ë û
x-axis. Put x = 2, we get
Thus the coordinates of points of intersection
of the circle and the parabola are Q (0, 0) and 16 y 2 65 8
P(2, 4). Clearly these are diametrically opposite + =1 Þ y = » m (approximately)
81 9 3 3
points on the circle. 15. (d) Equation of pair of tangents is given by
The coordinates of the focus S of the parabola SS1 = T2,
are (2, 0) which lies on the circle. or S = x2 + y2 + 20 (x + y ) + 20, S1 = 20,
1 1 T = 10 (x + y) + 20 = 0
\ Area of DPQS = × QS ×SP = × 2 × 4
2 2 \ SS1 = T2
= 4 sq. units. Þ 20 (x2 + y2 + 20 (x + y ) + 20) = 102
1 a (x + y + 2)2
11. (b) e = . Directrix , x = = 4 Þ 4x + 4y2 + 10xy = 0 Þ 2x2 + 2y2 + 5xy = 0
2
2 e
1 1 16. (a) Q ÐFBF ' = 90° Þ FB 2 + F ¢B 2 = FF ¢ 2

( ) ( )
\a = 4´ = 2 \b = 2 1- = 3 2 2
2 4 \ a 2e2 + b2 + a 2e2 + b2 = (2ae) 2
Equation of ellispe is
x 2 y2 2 b2
+ = 1 Þ 3x 2 + 4 y 2 = 12 Þ 2(a 2 e 2 + b2 ) = 4a 2 e 2 Þ e = 2 ...(i)
4 3 a
12. (c) Let AB be the chord of length 2 . Let O B (0, b)
be the centre of the circle and let OC be the
perpendicular from O on AB.
2 1 F' ( - ae, 0) O F (ae, 0)
Then, AC = BC = =
2 2
In DOBC, we have O Also, e 2 = 1 - b 2 / a 2 = 1 - e2
OB = BC cosec 45° 1
2
(By using equation (i)) Þ 2e = 1 Þ e = .
45

1
°

= ´ 2 =1 A C B 2
2 3y + k
17. (b) Given x = ........ (1)
\ Area of the circle = p(OB)2 = p sq units 2
2
and y = 6x ........(2)
13. (d) Here circles are
x2 + y2 – 2x – 2y = 0 ...(1) æ 3y + k ö
x2 + y2 = 4 ...(2) Þ y2 = 6 çè ÷
2 ø
Now, C1 (1, 1), r1 = 12 + 12 = 2 Þ y2 = 3 (3y + k) Þ y2 – 9y – 3k = 0 ........(3)
C2 (0, 0), r2 = 2 If line (1) touches parabola (2) then roots of qua-
If q is the angle of intersection then dratic equation (3) is equal
\ (–9)2 = 4 × 1 × (– 3k) Þ k2 = – 27/4
r12 + r22 - (c1c2 )2 x y2
cos q = 18. (c) Let eq. of ellipse be 2 + 2 = 1 ,
2 r1r2 a b
2 + 4 - ( 2 )2 1 S is (–ae, 0), T is (ae, 0) and B is (0, b).
= = Þ q = 45º
2. 2.2. 2 Y
B p B
14. (b)
3m
A¢ A X
C 2mN S T
9m
EBD_7167
http://t.me/iitjeehelps

M – 38 BITSAT Topicwise Solved Papers


Thus, the required equation of the ellipse is
Þ SB = (0 + ae) 2 + b 2
x 2 y2
Also SB2 = ST2 Þ 4a 2e 2 = a 2 e 2 + b 2 + =1
36 11
Þ 3a 2e 2 = a 2 (1 - e 2 ) = a 2 - a 2 e 2 23. (b) Let the two circles be x2 + y2+ 2g1 x + 2f1y +
1 1 c1 = 0 and x2 + y2 + 2g2 x + 2f2y + c2 = 0
Þ 4a 2 e 2 = a 2 Þ e 2 = Þ e = where g1 = 5/2, f1 = 3/2, c1 =7, g2 =–4, f2 =3
4 2 and c2 = k
19. (a) Let the radius of the first circle be CT = r1. Also,
let the radius of the second circle be CP = r2. If the two circles intersects orthogonally, then
æ 9ö
In the triangle PCT, T is a right angle 2 (g1g2 + f1f2) = c1 + c2 Þ 2 ç -10 + ÷ = 7 + k
è 2ø
Þ 11 = 7 + k Þ k = – 18
24. (b). The diameter of the circle is perpendicular
distance between the parallel lines (tangents)
7
3x – 4y + 4 = 0 and 3x – 4y – = 0 and so it is equal
2
4 7/ 2 3 3
So, PT = PC2 - CT 2 = r12 - r22 to + = Hence radius is
9 + 16 9 + 16 2 4
= (f 2
) (
-l - f2 -m = m-l ) 25. (a) Given eqn of parabola is y2 – kx + 6 = 0
2 æ 6ö
20. (b) We have x2 – y2 – 4x + 4y + 16 = 0 Þ y2 = kx – 6 Þ y = k ç x - ÷
è kø
Þ (x2 – 4x) – (y2 – 4y) = 16 6 k 6 k
Þ (x2 – 4x + 4) – (y2 – 4y + 4) = – 16 Now, directrix, x - = - Þ x = - ..... (i)
k 4 k 4
Þ (x – 2)2 – (y – 2)2 = – 16 1
But directrix is given Þ x = .... (ii)
(x - 2) 2 (y - 2) 2 2
Þ - =1 6 k 1
42 42 \ - = Þ 24 – k2 = 2k Þ k2 + 2k – 24 = 0
This is rectangular hyperbola, whose eccentric- k 4 2
ity is always 2 . Þ (k + 6) (k – 4) = 0 Þ k = – 6, k = 4
21. (d) According to the figure, the length of latus 26. (c) Clearly ax + by = 1
rectum is a 1
i.e y = - x + is tangent to
b b
u2 2u 2 cos2 α
2(SM) = 2 ´ (1 + cos 2α) = . x 2 y2
2g g cx 2 + dy 2 = 1 Þ + =1
1 1
æ u 2 sin 2α u 2 ö c d
ç , ÷
è 2g 2g ø 2 2
æ1ö æ 1 öæ a ö æ 1 ö 2 2
Y M \ ç ÷ = ç ÷ ç - ÷ + ç ÷ Þ1 = a + b
èbø è c øè b ø è d ø c d
X 27. (a) T = S1 Þ x (4) + y (3) – 4 (x + 4) = 16 + 9 – 32
S Þ 3y – 9 = 0 Þ y = 3
æ u 2 sin 2α – u 2 cos 2α ö 28. (a) The given equation of Parabola can be writ-
ç , ÷
è 2g 2g ø ten as
2
æ 1ö 1
çè x - ÷ø – 8y + 19 – = 0
22. (b) We have ae = 5 [Since focus is (±ae, 0)] 2 4
2
a 36 é aù æ 1ö 76 - 1
çè x - ÷ø = 8y –
and = êsince directrix is x = ± e ú 2 4
e 5 ë û
On solving we get a = 6 2
æ 1ö æ 75 ö æ 1 75 ö
5 Þ ç x - ÷ = 8 çè y - ÷ø \ vertex = çè , ÷ø
æ 25 ö è 2 ø 32 2 32
and e = Þ b 2 = a 2 (1 - e 2 ) = 36ç1 - ÷ = 11
6 è 36 ø
http://t.me/iitjeehelps

Conic Sections M – 39

29. (d) Here a = 4 yy’ – 2ax – 2ax’ – 4ab = y’2 – 4ax’ – 4ab
Condition of normality is c = – 2am – am3 (i.e.) yy’ – 2ax – (y’2 – 2ax’) = 0 ..........(2)
(a) and (b) are not clearly the answer as m = 1 Equations (1) and (2) represent the same line.
for (c), (d) m = cosq y1 2a 2ax1
c = – 2 (4) cos q – 4cos3q \ y' = = 2
2a y ' - 2ax '
= – 8 cos q – (3 cosq + cos3q) This gives y’ = y1 and then
= – 11 cosq – cos 3q
30. (a) If the line y = x + l touches the ellipse 2ax1 = y’2 – 2ax’ = y12 – 2ax’ = 4ax1 – 2ax’
9x2 + 16y2 = 144, then c2 = a2m2 + b2 \ x’ = x1 \ mid-point (x’ , y’) = (x1, y1).
Þ l2 = 16 × 12 + 9 Þ l2 = 25 \ l = ± 5 34. (c) The length of the ^ from the centre (0, 0) of
31. (d) Let S(1, 2) be the focus and P (x, y) be a point the given circle to the line 3x + ay – 20 = 0 is
on the hyperbola. Draw PM perpendicular from | 3(0) + a ( 0) - 20 | 20
= =
P on the directrix. 9+a2 9 + a2
Then by definition. SP = ePM Radius of the given circle = 5
z Since the line cuts the circle at real distinct or
M P coincident point,
2x+y=1 20
\ £ 5 Þ a 2 + 9 ³ 16 Þ a 2 - 7 ³ 0
S(1, 2) 2
z’ Focus 9+a
Þ ( a + 7 )(a - 7 ) ³ 0
2x + y - 1
Þ (x - 1)2 + (y - 2)2 = 3
22 + 12 Þ a Î (-¥,- 7 )] È [ 7 , ¥ )
35. (d) Here C1 (1,3), r1 = 1 + 9 - 9 = 1
(2x + y - 1)2
Þ (x – 1)2 + (y – 2)2 = 3 C2 (–3,1), r2 = 9 + 1 - 1 = 3
5
Þ (x – 1)2 + (y – 2)2 = 3 {2x + y – 1}2 Now C1 C2 = (1 + 3)2 + ( 3 - 1)2 =
Þ 7x2 – 2y2 + 12xy – 2x + 14 – 22 = 0
This is the required equation of the hyperbola. 16 + 4 = 20 = 2 5 = 4.47 (approx)
æ 3ö C1C2 > r1 + r2
4 7
32. (a) Centre = çè -1, ÷ø , radius = 1 + + 4 = Hence the circles are non- intersecting externally.
2 9 3 Hence 4 tangents, two direct and two transverse
\ Parametric coordinates of any point are tangents may be drawn.
7 3 7 36. (b) The point (x, 2) is on the ellipse implies
(-1 + cos q, + sin q) 2
3 2 3 x 2 + 4.4 = 25 ; \ x = 9; \ x = ±3 .
34. (c) Equation of the tangent at P(x1, y1) to y2 = So the points are (3, 2) and (–3, 2)
4ax is The equation of the tangent at (3, 2) to the ellipse
yy1 – 2ax – 2ax1 = 0 ..........(1) is x.3 + 4.y.2 = 25 Þ 3x + 8y = 25
Equation of the chord of y2 = 4a(x + b) whose
mid-point is (x’ , y’ ) is The equation of the tangent at (–3, 2) to the ellipse
is 3(-3) + 4 y.2 = 25; or 8 y - 3x = 25.
EBD_7167
http://t.me/iitjeehelps

M – 40 BITSAT Topicwise Solved Papers

Chapter

The value of
Limits and Derivatives
12
1.
1 + sin 3x - 1
æ a1/ x + a1/ x nx
1/ x ö 8. Evaluate lim ; [2011]
2 + .......... + an x ®¥ in(1 + tan 2x)
lim ç 1 ÷
x®¥ è n ø (a) 1/2 (b) 3/2
ai > 0, i = 1, 2, ...... n, is [2017] (c) 3/4 (d) 1/4
(a) a1 + a2 + ........... + an (1 + x 2 ) - 1 - x 2
9. Find the value of lim
x ®0 x2
(b) e a1 +a2 +¼an
a1 + a2 + ..... + a n [2010]
(c) (a) 1 (b) 2
n (c) 3 (d) 5
(d) a1a2 a3 .......an
1- t
a n + bn 10. If f (t) = , then f ' (1/t) is equal to [2009]
lim 1+ t
2. , where a > b > 1, is equal to [2016]
n ®¥ a n - bn 1 1
(a) –1 (b) 1 (a) 2 (b)
(1 + t) (t - 1) 2
(c) 0 (d) None of these
-2t 2 2
æ1ö æ1ö (c) (d)
3. If lim x sin ç ÷ = A and lim x sin ç ÷ = B , (t + 1) 2
(t - 1) 2
x ®¥ x
è ø ® èxø
( ).
x 0
then which one of the following is correct? [2015] sin p cos 2 x
(a) A =1 and B = 0 (b) A =0 and B = 1 11. Evaluate lim 2
[2008]
x ®0 x
(c) A =0 and B = 0 (d) A =1 and B = 1
(a) p/2 (b) p
4. lim (cosec x )1/ log x is equal to : [2014] (c) p/4 (d) p/3
x®0
(a) 0 (b) 1 x 2 - 3x + 2
12. lim [2007]
1 x ®2 x2 + x - 6
(c) (d) None of these (a) 1/2 (b) 1/5
e
5. Let f (x) = (x – 1)(x +1), g(x) = (x2 – 1)(x2 – x+1)
5 3 (c) 0 (d) infinite
and let h (x) be such that f(x) = g(x)h(x). Then lim 1
13. Evaluate x®0 5 (sin3x – tan3x) [2006]
lim h(x) is [2013] x
x®1 (a) 0 (b) 1
(a) 0 (b) 1 ,3
(c) 3 (d) 5 (c) –1 (d)
2
14. If y = (1 + x1/4) (1 + x1/2) (1 – x1/4), then dy/dx
æ æ xöö
çè 1 - tan çè 2 ÷ø ÷ø (1 - sin x) equals – [2006]
6. lim =? [2012] (a) – 1 (b) 1
x ®p /2 æ æ xöö 3 (c) x (d) x
çè1 + tan çè 2 ÷ø ÷ø ( p - 2x)
(a) 1/8 (b) 0 1, cos3 x
15. The value of lim is [2005]
(c) 1/32 (d) ¥ x ®0 x sin x cos x
7. Let f (x + y) = f (x) . f (y) for all x, y where f (0) ¹ 0. 2 3
If f (5) = 2 and f ' (0) = 3, then f ' (5) is equal to – (a) (b)
[2012] 5 5
(a) 6 (b) 0 3 3
(c) (d)
(c) 1 (d) None of these 2 4
http://t.me/iitjeehelps

Limits and Derivatives M – 41

Hints & Solutions


1. (d) Putting x =
1
, we get logcosec x é ¥ ù
log y = lim ê formú
y x ®0 log x ë ¥ û
n/ y
æ a y + a y + ..... + a y ö - cot x
n
L = lim ç 1 2 ÷ = lim (By L' Hopital rule)
y® 0 ç n ÷ x ® 0 1/ x
è ø
(Q x®¥ y®0) x æ 1 ö
æ a y log a + a y log a + ..... + a y log a ö = - lim çèQ cot x = ÷
1 1 2 2 n n x ® 0 tan x tan x ø
ç y y y ÷ Þ log y = –1
è a 1 + a 2 + ...... + a n ø
= n lim 1
y ®0 1 Þ y = e -1 =
[using L¢Hopital rule] e
log(a1a2 ....an ) 1
= n. Hence, required limit =
n e
5. (d) Given f (x) = g(x) h(x)
\ log L = log( a1. a2 .....an ) Þ L = a1. a2 .a3......an
n
f ( x)
æbö Þ h (x) =
1+ ç ÷ g ( x)
(b) limit = lim èaø
2. = 1, f ( x)
n ®¥ æbö
n Þ lim h( x) = xlim
1- ç ÷ x ®1 ®1 g ( x)
èaø
n ( x5 - 1)( x3 + 1)
b æbö Þ xlim
because 0 <
a
< 1 implies ç ÷ ® 0 as n ® ¥
èaø
®1 ( x2 - 1)( x 2 - x + 1)
3. (a) As given, x 5 - 15
= lim = 5 ´ 14 = 5
æ1ö x ®1 x -1
sin ç ÷
æ1ö èxø p p
A = lim x sin ç ÷ = lim 6. (c). Put x = - h as x ® , h ® 0
x ®¥ x
è ø x ®¥ æ 1 ö 2 2
ç ÷ \ Given limit
èxø
1 æ p hö
Let t = when x ® a, t ® 0 1 - tan ç - ÷
x è 4 2 ø (1 - cosh)
lim .
sin t = h®0 æ p h ö (2h)3
Þ A = lim =1 1 + tan ç - ÷
t ®¥ t è 4 2ø
é sin x ù h
2 sin 2
êQ tlim = 1ú h 2
ë ®0 x û = lim tan
h®0 2 8h 3
æ1ö
and B = lim x sin ç ÷ 2
x®0 èxø hæ hö
tan
sin
1 2 ç 2÷ 1
æ1ö lim . ç h ÷ ´4
Þ B = lim x. lim sin ç ÷ = h®0 4 h
x ®0 x ®0 èxø ´2ç ÷
2 è 2 ø
ÞB=0
\ A = 1 and B = 0 is correct æ hö æ hö
2
1/ log x tan sin
4. (c) Let y = lim(cosec x) 1 ç 2÷ ç 2÷ 1
x ®0 lim .
= h ®0 32 ç h ÷ ç h ÷ = 32
Taking log on both sides, we get ç ´ 2÷ ç ÷
è2 ø è 2 ø
EBD_7167
http://t.me/iitjeehelps

M – 42 BITSAT Topicwise Solved Papers

7. (a) f ¢(5) = lim


h®0
f (5 + h) - f (5)
h 11. (b) lim
(
sin p cos 2 x )
x ®0 2
x
f (5 + h) - f (5 + 0)
= lim
h®0 h = lim
{
sin p(1 - sin 2 x ) }
x ®0 2
f (5).f (h) - f (5) + f (0) x
= lim
h®0 h
(Q f (x + y) = f (x). f (y) for all x, y) = lim
(
sin p - p sin 2 x ) = lim
(
sin p sin 2 x )
x ®0 2 2
x x ®0 x
æ f (h) - f (0) ö
= çè lim
h ®0 h
÷ø .f (5) = f ¢ (0).f (5)
lim
ï
í
(
ì sin p sin x
´ ´

p sin 2 x ï
ý
)
=3×2=6 = x ®0 2
ïî p sin x 1 x2 ï
1 + sin 3x - 1 þ
8. (c) xlim
®0 ln(1 + tan 2x)
= lim
(
sin p sin 2 x ) ´ p ´ lim sin 2
x
=1×
(1 + sin 3x) - 1 x ®0 p sin x 2 x ®0 x 2
lim
= x®0 { 1 + sin 3x + 1 ln(1 + tan 2x) } p×1= p
1 sin 3x x 2 - 3x + 2
= lim . × 12. (b) lim
x®0
( 1 + sin 3x + 1 ) 1
ln(1 + tan 2x) tan 2x
x ®2 x2 + x - 6
(x - 2) (x - 1) 2 - 1 1
= lim = =
1 x ® 2 (x - 2) (x + 3) 2+3 5
tan 2x 13. (d) sin3x – tan3x = (sin x – tan x) (sin2x + sinx
lim 1 tanx + tan2 x)
= x®0 . æç sin 3x ö÷ æç 2x ö÷
(
1 + sin 3x + 1 ) è 3x ø è tan 2x ø æxö
= –tan3x. 2sin2 ç ÷ .(cos2x + cos x + 1)
è2ø
1
× 3. 1 1 tan 3
2 lim lim
(sin3x – tan 3x) = –2 x®
ln(1 + tan 2x) tan 2x x®0 x5 0
x3
æ 1 ö æ3ö 1 æxö
3 sin 2 ç ÷
=ç ÷ (1) (1) ç ÷ = . lim è2ø lim (cos2x + cosx + 1)
è1+1ø è ø ln e 4
2 x®0 x®0
2
x
1 + x 2 - 1 - x2 1+ x2 + 1- x2 2
9. (a) lim 2
. æ1ö 3
x® 0 x 1+ x2 + 1- x2 = 2.13. ç ÷ .3 = –
è2ø 2
1+ x2 - 1+ x2 14. 1/2 1/2
(a) y = (1 + x ) (1 – x ) =1 – x
( )
lim
Þ x ®0 x 2 1+ x2 + 1- x2 \ dy/dx = – 1
1 - cos 3 x
2x 2
2 2 15. (c) Lim
Þ lim = = =1 x ® 0 x sin x cos x
x® 0
x 2
( 1+ x 2
+ 1- x 2
) 1+ 1 2
= Lim
(1 - cos x ) (1 + cos x + cos 2 x )
10. (c) x ®0 x sin x cos x
d é1 - t ù (1 + t ) (-1) - (1 - t) ´ (1) æxö
f '(t) = = 2 sin 2 ç ÷
dt êë1 + t úû (1 + t)2 = Lim è 2ø ´
(1 + cos x + cos 2 x )
x ®0 æxö æxö cos x
-1 - t - 1 + t -2 x.2 sinç ÷ cos ç ÷
= 2
= 2
è ø 2
è ø
(1 + t) (1 + t) 2
æxö
sin ç ÷
-2 -2t 2 2 1 + cos x + cos 2 x 1 3
f '[1/ t] = = = Lim è ø ´ = ´3 = .
æ 1ö
2
( t + 1) 2 x ®0 æ x ö
2ç ÷
æxö
cos ç ÷ cos x
2 2
çè1 + ÷ø è ø2 2
è ø
t
http://t.me/iitjeehelps

Mathematical Reasoning M – 43

Chapter
Mathematical Reasoning
13
1. In the truth table for the statement (p Ù q) ® (a) Raju is not tall or he is intelligent.
(q Ú ~ p), the last column has the truth value in (b) Raju is tall or he is intelligent
the following order is [2013]
(c) Raju is not tall and he is intelligent
(a) TTFF (b) FTTT
(d) Raju is not tall implies he is intelligent
(c) TFTT (d) TTTT
4. Which of the following is a statement? [2008]
2. Negation of “Paris in France and London is in
(a) May you live long!
England” is [2011]
(b) May God bless you!
(a) Paris is in England and London is in France
(c) The sun is a star
(b) Paris is not in France or London is not in
England (d) Hurrah! we have won the match
5. Which of the following is not a proposition
(c) Paris is in England or London is in France [2006]
(d) None of these (a) 3 is a prime
3. If : p Raju is tall and q: Raju is intelligent, then (b) 2 is irrational
the symbolic statement ~ pÚ q means [2009] (c) Mathematics is interesting
(d) 5 is an even integer

Hints & Solutions


1. (d) T T T T 3. (a) ~ p Ú q : Raju is not tall or he is intelligent.
2. (b) Let p : Paris is in France, q : London is in 4. (c) “The sun is a star” is a statement.
England 5. (c) “Mathematics is interesting” is not a logical
sentence. It may be interesting for some
\ we have p Ù q persons may not be interesting for others.
Its negation is ~ ( p Ù q) =~ pÚ ~ q \ This is not a propositions.
i.e., Paris is not in France or London is not
in England.
EBD_7167
http://t.me/iitjeehelps

M – 44 BITSAT Topicwise Solved Papers

Chapter

1.
Statistics
The marks obtained by 60 students in a certain
14
18 and 10 respectively, the standard deviation
test are given below : of this set of observations is [2015]
(a) 3 (b) 2
No. of No. of (c) 1 (d) None of these
Marks Marks
students students 5. If M. D. is 12, the value of S.D. will be [2014]
10 - 20 2 60 - 70 12 (a) 15 (b) 12
20 - 30 3 70 - 80 14 (c) 24 (d) None of these
6. If the value of mode and mean is 60 and 66
30 - 40 4 80 - 90 10 respectively, then the value of median is [2013]
40 - 50 5 90 - 100 4 (a) 70 (b) 64 (c) 90 (d) 50
7. Find the variance of the data given below [2013]
50 - 60 6 Size of item 3.5 4.5 5.5 6.5 7.5 8.5 9.5
Median of the above data is [2017] Frequency 3 7 22 60 85 32 8
(a) 68.33 (b) 70 (a) 1.29 (b) 2.19
(c) 68.11 (d) None of these (c) 1.32 (d) None of these
2. The arithmetic mean of numbers a, b, c, d, e is M.
8. If sample A contains 100 observations 101, 102, ....
What is the value of (a – M) + (b – M) + (c – M) 200 and sample B contains 100 obsections 151, 152,
+ (d – M) + (e – M) ? [2016] .......... 250, then ratio of variance vA/vB = [2012]
(a) M (b) a + b + c + d + e (a) 1 (b) 9/4 (c) 4/9 (d) 2/3
(c) 0 (d) 5 M 9. Find the A.M. of the first ten odd numbers. [2011]
3. The arithmetic mean of the data 0, 1, 2, ......, n (a) 10 (b) 20 (c) 15 (d) 25
with frequencies 1, nC1, nC2,....., nCn is [2015]10. Mean of 25 observations was found to be 78.4.
2n n But later on it was found that 96 was misread 69.
(a) n (b) (c) n + 1 (d) The correct mean is [2010]
n 2 (a) 79.24 (b) 79.48
4. The mean square deviation of a set of n (c) 80.10 (d) None of these
observation x1, x2, .... xn about a point c is defined
11. If the mean, mode and S.D. of a frequency
1 n distribution are 41,45 and 8 respectively, then its
as n å ( xi - c ) .
2
Pearson’s coefficient of skewness is [2010]
i =1 1 -1 2
The mean square deviations about – 2 and 2 are (a) (b) (c) (d) 1
3 2 3
12. Given below is a frequency distribution with median 46. In this distribution, some of the frequencies are
missing : Determine the missing frequencies. [2009]
Marks 10-20 20-30 30-40 40-50 50-60 60-70 70-80 Total
No. of students 12 30 ? 65 ? 25 18 229
(a) 34, 45 (b) 25, 40 15. Consider the following statements
(c) 12, 18 (d) 30, 35 (a) Mode can be compound from histogram
13. The mean deviation from the median of the (b) Median is not independent of change of scale
following set of observations 5, 3, 9, 12, 3, 10, 12, (c) Variance is independent of change of origin
21, 18, 12, 21 is [2007] and scale
(a) 5.113 (b) 4.606 (c) 4.134 (d) 4.909 Which of these is/are correct – [2005]
14. The quartile deviation of the following items : (a) (a), (b) and (c) (b) only (b)
(c) only (a) and (b) (d) only (a)
12, 7, 15, 10, 16, 17, 25 is [2006] 16. Find the harmonic mean of 2, 4, 5. [2005]
(a) 4.5 (b) 13.5 (c) 9 (d) 3.5 (a) 2.12 (b) 1.34 (c) 3.16 (d) 5.14
http://t.me/iitjeehelps

Statistics M – 45

Hints & Solutions


1. (a) We construct the following table taking n
assumed mean a = 55 (step deviation n å n -1 Cr -1
method). r =1 n.2n -1 n
x -a = = =
n n 2
Class xi fi c.f. ui = i fiu i 2
10–20 15 2 2 –4
10
–8
å n Cr
r =0
20–30 25 3 5 –3 –9
30–40 35 4 9 –2 –8 1 n
å ( x i + 2 ) = 18 and
2
40–50 45 5 14 –1 –5 4. (a) We have
n i=1
50–60 55 6 20 0 0
60–70 65 12 32 1 12 1 n
å ( x i - 2 ) = 10
2
70–80 75 14 46 2 28
n i=1
80–90 85 10 56 3 30
90–100 95 4 60 4 16 n
Total 60 56
å ( xi + 2)
2
Þ = 18 n and

\ The mean = a +
åfiui ´ h i=1

åfi n
å ( xi - 2)
2
= 10 n
56 56
= 55 + ´ 10 = 55 + = 64.333 i=1
60 6
n n
å ( x i + 2 ) + å ( xi - 2 )
2 2
n Þ = 28n
Here n = 60 Þ = 30 , therefore, 60–70 is
2 i =1 i =1
the median class
n n n
-C
and å ( x i + 2 ) - å ( xi - 2 )
2 2
30 - 20 = 8n
\ M =l + 2 ´ c = 60 + ´ 10 i =1 i =1
f 12
100 n n
= 60 + = 60 + 8.333 = 68.333
Þ 2å ( x i + 4 ) = 28n and 2å 4x i = 8n
2
12
i =1 i =1
a+b+c+d+e
2. (c) Given M = n n
Þ å x i + 4n = 14n and å xi = n
5 2
Þ a +b+ c+ d+ e=5M
Þ a +b+c+ d+ e–5M= 0 i =1 i =1
Þ (a – M) + (b – M) + (c – M) + (d – M) n n
+ (e – M) = 0
Hence, required value = 0 Þ å x i2 = 10 n and å xi = n
i =1 i =1
å fi x i
3. (d) Since, Mean = where x i are 2
f å 1
n æ1 n ö 10n æ n ö
2
i
observations with frequencies fi, i = 1, 2, ........n \s =
n å xi2 -ç
çn å xi ÷ =
÷
-ç ÷ = 3
n ènø
The required mean is giveny by i =1 è i =1¢ ø
0.1 + 1.n C1 + 2.n C2 + ...... + n.n C n 5 5
X= 5. (a) We know that Q.D = ´ M .D. = ´12 = 10
6 6
1 + n C1 + n C2 + ... + n Cn
3 3
n n
n \ S.D = ´ Q.D. = ´ 10 Þ S .D. = 15.
å r. n
Cr å r. r n -1Cr -1 2 2
r =0
= r =1 6. (b) Mode = 3 Median – 2 Mean
= n n
å n Cr å n
Cr 1
\ Median = (mode+ 2 mean)= (60+2× 66)=64
1
r=0 r=0 3 3
EBD_7167
http://t.me/iitjeehelps

M – 46 BITSAT Topicwise Solved Papers


7. (d) n 229
Class Frequency Cumulative = = 114.5 , Median = 46
frequency 2 2
5 – 10 5 5 \ Median class = 40 – 50
10 – 15 6 11 \ l = 40 , c.f. = 42 + x, f = 65, h = 10
15 – 20 15 26 æn ö
- c.f.
20 – 25 10 36 ç2 ÷
Median = l + ç ´h
f ÷
25 – 30 5 41
30 – 35 4 45 ç ÷
è ø
35 – 40 2 47
114.5 - (42 + x)
40 – 45 2 49 46 = 40 + ´ 10
N 49 65
Here N = 49. \ = = 24.5 (114.5 - 42 - x)
2 2 or 46 - 40 = ´2
13
The cumulative frequency just greater than (72.5 - x)
N/2 is 26 and corresponding class is 15–20. 6= ´ 2 or 78 = 145 – 2x
Thus 15–20 is themedian classsuch that l 13
67
= 15, f = 15, F = 11, h = 5 2x = 145 – 78 = 67 or x = = 33.5
2
N/2-F 24.5 - 11 \ x = 34 (Q Number of students cannot be in
\ Medium= l+ × h= 15+ ×5
f 15 fraction) Now Sfi = 29 \ x + y + 150 = 229
13.5 x + y = 229 – 150 = 79 .......... (i)
= 15 + = 19.5 Putting the value of x in (i), we get
3 34x + y = 79 Þ y = 79 – 34 = 45
8. (a) \ x = 34, y = 45
9. (a) First ten odd numbers are 1, 3, 5, 7, 11, 13, 13. (d) There are 11 observations, on arranging
15, 17, 19 respectively. So A.M. these observations in ascending order of
1 + 3 + 5 + 7 + 9 + 11 + 13 + 15 + 17 + 19 magnitude, we get
(x) = 3, 3, 5, 9, 10, 12, 12, 12, 18, 21, 21
10 The number of observations n ( = 11) is an
100 odd number, therfore, median
= = 10
10 11 + 1
Sx M= the observation = 6th observation = 12
10. (b) Mean x = or S x = n x 2
n For the mean deviation (about median),
S x = 25 × 78.4 = 1960 construct the following table :
But this Sx is incorrect as 96 was misread as 69. xi 3 3 5 9 10 12 12 12 18 21
21 Total
\ correct S x = 1960 + (96 – 69) = 1987 xi–M –9 –9 –7 –3 –2 0 0 0 6
1987 9 9 –6
\ correct mean = = 79.48 =xi–12
25 xi - M 9 9 7 3 2 0 0 0 6 9
1 Mean - Mode 9 54
11. (b) - , We have SK = = x i - 12
2 S.D.
\ Mean deviation (about median)
41 - 45 1
=- å x i - M å x i - 12 54
8 2 = = = = 4.909
12. (a) Marks No. of students c.f.
n n 11
14. (d) The given data in ascending order of
10-20 12 12 magnitude is
7, 10, 12, 15, 16, 17, 25
20-30 30 42 Here, lower quartile Q1 = 10,
30-40 ? 42+x median = 15 and upper quartile Q3 = 17
Q - Q1 17 - 10
40-50 65 107+x \ Q.D. = 3 = = 3. 5
2 2
50-60 ? 107+x+y 15. (c)
60-70 25 132+x+y
16. (c) The harmonic mean of 2,4 and 5 is
70-80 18 150+x+y 3 60
= = = 3.16
1 1 1 19
Total 229 + +
2 4 5
http://t.me/iitjeehelps

Probability M – 47

Chapter
Probability
15
1. If three vertices of a regular hexagon are chosen 5. A black die and a white die are rolled. Find the
at random, then the chance that they form an probability that the number shown by the black
equilateral triangle is : [2016] die will be more than twice that shown by the
white die. [2010]
1 1
(a) (b) (a) 1/8 (b) 1/6
3 5
1 1 (c) 1/3 (d) 1/4
(c) (d)
10 2 6. Five dice are tossed. What is the probability that
2. A bag contains 5 brown and 4 white socks. A the five numbers shown will be different?
man pulls out 2 socks. Find the probability that [2008]
they are of the same colour. [2014]
5 5
4 2 (a) (b)
(a) (b) 54 18
9 9
5 5
5 7 (c) (d)
(c) (d) 27 81
9 9
7. Find the probability of drawing a jack or an ace
3. If A and B are mutually exclusive events and if from a pack of playing cards. [2007]
1 13 (a) 1/8 (b) 1/6
P(B) = , P(A È B) = , then P(A) is equal to
3 21 (c) 1/3 (d) 2/13
[2011] 8. 3 integers are chosen at random from the set of
(a) 1/7 (b) 4/7 first 20 natural numbers. The chance that their
product is a multiple of 3, is. [2005]
(c) 2/7 (d) 5/7
4. A die is loaded such that the probability of 194 1
(a) (b)
throwing the number i is proportional to its 285 57
reciprocal. The probability that 3 appears in a
single throw is- [2011] 13 3
(c) (d)
(a) 3/22 (b) 3/11 19 4

(c) 9/22 (d) None of these


EBD_7167
http://t.me/iitjeehelps

M – 48 BITSAT Topicwise Solved Papers

Hints & Solutions

6 5. (b) The number of favourable cases are shown


1. (c) Three vertices can be selected in C3 below:
ways. Number on white die Number on black die
A5 A4 1 3
1 4
1 5
A6 1 6
A3
2 5
2 6
There are 6 favourable cases in which the
A1 A2 number on black die is more than twice the
The only equilateral triangles possible are number on the white die.
A1A3A5 and A2A4A6 \ m=6
n = Total number of cases = 6 × 6
2 2 1 (\ with each die there are six possibilities)
p= 6
= =
C3 20 10
m 6 1
\ Probability p = = =
2. (a) Let A º event of two socks being brown. n 6´6 6
B º event of two socks being white. 6. (a) Total number of cases = 65
5
Number of favourable cases = 6 ! = 720
C2 5.4 5
Then P (A) = 9
= = , P(B) 720 5
C2 9.8 18 Required probability = =
5
6 54
4
C2 4.3 3 7. (d) As there are four jacks and four aces, the
= 9
= = number of favourable cases = 8
C2 9.8 18
8 2
\ The required probability = p = =
Now, since A and B are mutually exclusive 52 13
events, so required probability 8. (a) Total number of ways of selecting 3 integers
5 3 4 from 20 natural numbers = 20C3 = 1140
= P (A) + P (B) = + =
18 18 9 Their product is a multiple of 3 means, at
3. (c) For mutually exclusive events least one number is divisible by 3.
2 The numbers which are divisible by 3 are 3,
P(A È B) = P(A) + P(B) Þ P(A) =
7 6, 9, 12, 15, 18 and the number of ways of
6 6 selecting atleast one of them is
k 1 49
4. (d) P(i) = Þ 1 =
i
å P(i) = k å i
=k
20 6
C1 ´ 14 C2 + 6 C2 ´ 14 C1 + 6 C3 = 776
i =1 i =1

20 20
Þk= . P(3) = 776 194
49 147 Required Probability = =
1140 285
http://t.me/iitjeehelps

Relations and Functions M – 49

Chapter

1.
Relations and Functions
Let f and g be functions from R to R defined as
16
(a) 1 (b) x
ì 7 x 2 + x - 8, x £ 1 ì | x |, x < -3 (c) 1 + x (d) None of these
ï ï
f ( x) = í 4 x + 5, 1 < x £ 7 , g ( x) = í0, - 3 £ x < 2 1- x
ï 8 x + 3, x > 7 ï 2 8. If f (x) = the domain of f–1 (x) is [2011]
î î x + 4, x ³ 2 1+ x
Then [2017] (a) R (b) R – {– 1}
(c) (– ¥, – 1) (d) (–1, ¥)
(a) (fog) (–3) = 8 (b) (fog) (9) = 683 9. Let E = {1, 2, 3, 4} and F = {1, 2}. Then the
(c) (gof) (0) = – 8 (d) (gof) (6) = 427 number of onto functions from E to Fis [2010]
(a) 14 (b) 16
ax + b (c) 12 (d) 8
2. Let f (x) = , then fof (x) = x, provided that :
cx + d
x
[2016] 10. If f(x) = , then (fof of) (x) is [2010]
(a) d = – a (b) d = a 1+ x2
(c) a = b = 1 (d) a = b = c = d = 1
3x x
3. If r = {(x, y) |x2 + y2 = 1; x, y Î R}. Then, r is (a) (b)
2
[2015] 1+ x 1 + 3x 2
(a) reflexive (b) symmetric
(c) transitive (d) anti-symmetric (c)
3x (d) None of these
4. Let f : R ® R be a fun ction defined by 1 - x2
x-m 11. If the function f : (– ¥ ,¥) ® B defined by
f (x) = , where m ¹ n , then [2014]
x-n f (x) = – x2 + 6x – 8 is bijective, then B = [2009]
(a) f is one-one onto (b) f is one-one into (a) [1,¥) (b) (– ¥, 1]
(c) f is many-one onto (d) f is many-one into (c) (– ¥, ¥) (d) None of these
5. The relation ‘less than’ in the set of natural 12. x2 = xy is a relation which is [2008]
numbers is : [2013] (a) Symmetric (b) Reflexive
(a) only symmetric (c) Transitive (d) None of these
(b) only transitive
(c) only reflexive 13. Let R = {(1,3),(4, 2),(2,4),(2,3),(3,1)} be a
(d) equivalence relation relation on the set A = {1, 2,3, 4}. . The relation
6. If f is an even function and g is an odd function,
R is [2007]
then the function fog is [2012]
(a) reflexive (b) transitive
(a) an even function
(c) not symmetric (d) a function
(b) an odd function
14. If f : R ® R, f (x) = 2 x + 3 then f –1 (x) = [2006]
(c) neither even nor odd
(d) a periodic function x-3 x+3
(a) (b)
2 2
ì x; when x is rational
7. If f(x) = í , then fof (x)
î1 - x; when x is irrational x+2 x-2
(c) (d) .
is given as [2011] 3 3
EBD_7167
http://t.me/iitjeehelps

M – 50 BITSAT Topicwise Solved Papers


2 2
15. Let P = {( x, y ) :| x + y | = 1, x, y Î R}. Then P
-1 æ x - 2 ö p
is [2005] (b) sin çè ÷+
2 ø 6
(a) Reflexive (b) Symmetric
(c) Transitive (d) Anti-symmetric
2p æ x - 2ö
+ cos -1 ç
é p 2p ù
(c)
3 è 2 ÷ø
16. Let f : ê - , ® [0, 4] be a function defined as
ë 3 3 úû
(d) None of these
f (x) = 3 sin x - cos x + 2 . Then f –1(x) is given by
[2005]

æ x - 2ö p
sin -1 ç -
(a) è 2 ÷ø 6

Hints & Solutions


1. (b) We have g (–3) = 0 =0 Þ a+d =0
Þ f (g(–3)) = f (0) = 7 (0)2 +0 – 8 = – 8 3. (b) Obviously, the relation is not reflexive and
\ fog (–3) = –8 transitive, but it is symmetric, because
g (9) = 92 + 4 = 85 Þ f (g(9)) = f (85) = 8(85) x2 + x2 = 2x2 ¹ 1
+ 3= 683 and x2 + y2 = 1, y2 + z2 = 1
\ fog (9) = 683 Þ x2 + z2 = 1
f (0) = 7.02 + 0 – 8 = – 8 Þ g (f (0)) = g (–8) = But x2 + y2 = 1 Þ y2 + x2 = 1
| –8 | = 8 4. (b) Let f : R ® R be a function defined by
\ gof (0) = 8
f (6) = 4(6) +5 = 29 Þ g (f (6)) = g (29) = (29)2 x-m
f (x) =
+ 4 = 845 x-n
\ gof (6) = 845 For any (x, y) Î R
ax + b Let f (x) = f (y)
2. (a) f (x) =
cx + d x-m y-m
Þ = Þ x=y
x-n y-n
ì ax + b ü
aí ý+b \ f is one – one
î cx + d þ
fof (x ) =
ì ax + b ü Let a Î R such that f (x) = a
cí ý+d
î cx + d þ x-m
Þ a= Þ (x – n) a = x – m
x-n
a 2 x + ab + bcx + bd
Þ =x Þ xa –na =x–m
acx + bc + cdx + d 2 Þ xa - x = na - m
Þ (ac + dc) x 2 + (bc + d 2 - bc - a 2 ) x Þ x(a - 1) = na - m
– ab – bd = 0, " x Î R na - m
Þ x= . for a = 1, x Ï R
a -1
Þ (a + d)c = 0, d 2 - a 2 = 0 and (a + d)b So, f is not onto.
http://t.me/iitjeehelps

Relations and Functions M – 51

5. (b) Given R is a relation from N to N


x
i.e. (x, y) Î R Û x < y, where x Î N, y Î N 10. (b) f (x) =
Since x < x is not possible for all x Î N 1+ x2
\ R is not reflexive.
Also, x < y ¹ y < x Þ R is not symmetric for x
all x, y Î N 1+ x2 x
Now, if x < y and y < z, x, y Î N So xRy fof = =
2
Then x < z, " x, y , z Î N x 2x 2 + 1
1+
\ xRy and yRz Þ xRz 1 + x2
\ Relation is transitive.
6. (a) We have, fog (–x) = f [g (–x)] = f [–g(x)] (Qg x
is odd)
2x 2 + 1 x
= f[g (x)] ( Q f is even) fofof = =
2
= fog (x) " x Î R. x 1 + 3x 2
1+
\ fog is an even function. 2x 2 + 1

ìf (x) ;when f(x) is rational 11. (b) Since the function f is bijective, therefore f
7. (b) fof (x) = í1 - f(x); when f(x) is irrational is onto. Therefore range of f = B.
î
Let y = – x2 + 6x – 8
ìx ; when x is rational Þ x2 – 6x + (8 + y ) = 0
= í 1 - (1 - x); when x is irrational= x
î 6 ± 36 - 4 (8 + y) 6 ± 4 - 4y
Þ x= =
1- x 2 2
8. (b) Let f(x) = y. Then =y For x to be real, 4 – 4y ³ 0 Þ y £ 1
1+ x
\ B = range of F = [– ¥, 1]
1- y 1- y 12. (b)
Þx= Þ f –1 (y) = 13. (c) (2,3) Î R but (3, 2) Ï R
1+ y 1+ y
\ R is not symmetric
1- x 14. (a) Since f is a bijection therefore its inverse
Thus, f–1 (x) = mapping exists and
1+ x
Clearly, f –1 (x) is defined for 1 + x ¹ 0. x-3
y = 2x + 3 Þ x = 2y + 3 Þ y =
Hence, domain of f –1 (x) is R – {– 1} 2
9. (a) If set A has m elements and set B has n x - 3
elements then number of onto functions \ f –1 (x) =
2
from A to B is 15. (b) Obviously, the relation is not reflexive and
n transitivebut it issymmetric, becausex2 +
å (-1)n-r n
Cr r m where 1 £ n £ m y2 = 1 Þ y2 + x2 = 1
r =1 é p 2p ù
16. (b) f : ê- , Þ [0, 4]
Here E = {1, 2,3, 4}, F = {1, 2} ë 3 3 úû
m = 4, n = 2 f (x) = 3 sin x - cos x + 2
\ No. of onto functions from E to F
2 æ pö
f(x) = 2sin ç x - ÷ + 2
= å (-1) 2-r 2
Cr (r ) 4 è 6ø
æ x - 2ö p
r =1
Þ (f (x)) = sin -1 ç
-1
+
= (-1) 2C1 + 2C2 (2) 4 = – 2 +16 = 14 è 2 ÷ø 6
EBD_7167
http://t.me/iitjeehelps

M – 52 BITSAT Topicwise Solved Papers

Chapter

1.
Inverse Trigonometric Functions
The domain of the function
17
æ 1ö æ 2ö
9. tan -1 ç ÷ + tan -1 ç ÷ equal to – [2012]
ì æ 1 2 öü è 4ø è 9ø
f (x) = sin–1 ílog 2 ç x ÷ ý is [2017]
î è2 øþ 1 æ 3ö 1 -1 æ 3 ö
(a) cos -1 ç ÷ (b) sin ç ÷
(a) [– 2, – 1) È [1, 2] (b) (– 2, – 1] È [1, 2] 2 è 5ø 2 è 5ø
(c) [– 2, – 1] È [1, 2] (d) (– 2, – 1) È (1, 2) 1 æ 3ö
tan -1 ç ÷ -1 æ 1 ö
(c) (d) tan ç ÷
2. The value of cot -1 7 + cot -1 8 + cot -1 18 is [2017] 2 è 5ø è 2ø
p 10. If k £ sin -1 x + cos -1 x + tan -1 x £ K , then –
(a) p (b) (c) cot -1 5 (d) cot -1 3 [2012]
2
3. If (a) k = 0, K = p. (b) k = 0, K = p/2
(c) k = p/2, K = p (d) None of these
–1 æ 1 – b ö
2
–1 æ 2a ö –1 æ 2 x ö 11. The value of
sin ç 2 ÷
– cos ç 2 ÷
= tan ç 2 ÷
,
è 1+ a ø è 1+ b ø è1– x ø æ 1ö æ 1ö
sin ç 4 tan -1 ÷ - cos ç 2 tan -1 ÷ is [2011]
then what is the value of x? [2016] è 3ø è 7ø
(a) a / b (b) ab (a) 3/7 (b) 7/8
a–b (c) 8/21 (d) None of these
(c) b / a (d) 12. The value of cos –1
x + cos –1
1 + ab
y æx 1 ö 1
4. If cos -1 x - cos -1 = a, then 4 x 2 cos -1
- 4 xy x + cos -1 ç + 3 - 3 x 2 ÷ ; £ x £ 1 is [2010]
2 è 2 2 ø 2
p p 3 3
cos a + y 2 is equal to [2015] (a) - (b) (c) (d) -
(a) 2 sin 2a (b) 4 3 3 p p
13. Find the value of
(c) 4 sin a 2
(d) – 4 sin a 2
1 1 1
-1
2 tan -1 + tan -1 + 2 tan -1 [2009]
sin (x - 3) 5 2 8
5. The domain of the function f (x) = is (a) p/4 (b) p/2
9 - x2 (c) 3p/4 (d) None of these
[2015] -1 1 1
(a) [1, 2] (b) [2, 3) (c) [1, 2 ] (d) [2, 3] 14. Find the value of sin {sin + cos-1 } [2007]
2 2
é -1 1 p ù (a) 3 (b) 1 (c) 2 (d) 4
6. Find the value of tan ê 2 tan - [2014]
ë 5 4 úû æ 7p ö
15. Find the value of cos–1 cos ç ÷ [2006]
(a) – 1/3 (b) – 7/17 (c) – 1/2 (d) – 1/4 è 6ø
-1 x p (a) p/4 (b) p/2 (c) 5p/6 (d) p/6
7. If tan < , x Î N , then the maximum value
p 3 16. A value of x for which sin (cot–1(1 + x)) = cos
of x is [2014] –1
(tan x), is : [2005]
(a) 2 (b) 5 1 1
(c) 7 (d) None of these (a) - (b) 1 (c) 0 (d)
8. The value of 2 2
17. Find the value of cos –1x + cos–1
é1 æ æ 63 ö ö ù
cos ê cos-1 ç cos ç sin -1 ÷ ú is – æx 3 - 3x 2 ö æ1 ö
ëê 2 è è 8 ÷ø ø úû cos-1 x + cos -1 ç + ÷ where çè 2 £ x £ 1÷ø [2005]
çè 2 2 ÷ø
[2013, 2006]
(a) 3/16 (b) 3/8 (c) 3/4 (d) 3/2 (a) p/4 (b) p/3 (c) 5p/6 (d) p/6
http://t.me/iitjeehelps

Inverse Trigonometric Functions M – 53

Hints & Solutions


1. (c) For f (x) to be defined, we must have y
-1 -1
4. (c) cos x - cos =a
æ1 2ö 2
– 1 £ log2 ç x ÷ £ 1
è2 ø
æ æ y 2 ö ö÷
-1 xy
Þ cos ç + (1 - x ) çç1 -
2
1 ÷ =a
Þ 2–1 £ x2 £ 21 [Q the base = 2 > 1] ç 2 è 4 ÷ø ÷
2 è ø
Þ 1 £ x2 £ 4 ....... (1) æ xy + 4 - y 2 - 4 x 2 + x 2 y 2 ö
Now, 1 £ x2 Þ x2 – 1 ³ 0 i.e. (x – 1) ( x +1) Þ cos
-1
ç ÷ =a
³0 çè 2 ÷ø
Þ x £ – 1 or x ³ 1 ...... (2) Þ 4 - y 2 - 4 x2 + x 2 y 2
Also, x2 £ 4 Þ x2 – 4 £ 0 i.e. (x – 2) (x + 2) £ 0
Þ – 2 £ x £ 2 ...... (3) = 4cos 2 a + x 2 y 2 - 4 xy cos a
Form (2) and (3), we get the domain of
f = {(– ¥, – 1] È [1, ¥)} Ç [–2, 2] Þ 4 x 2 + y 2 - 4 xy cos a = 4sin 2 a
= [– 2, – 1] È [1, 2]
sin -1 (x - 3)
2. (d) We have cot -1 7 + cot -1 8 + cot -1 18 5. (b) f (x) = is defined
9 - x2
æ 1 1 ö
çQ xy = . < 1÷ if (i) - 1 £ x - 3 £ 1 Þ 2 £ x £ 4 and
è 7 8 ø
1 1 1 (ii) 9 - x 2 > 0 Þ -3 < x < 3
\ tan -1 + tan -1 + tan -1 Taking common solution of (i) and (ii), we
7 8 18
get 2 £ x < 3 \ Domain = [2, 3)
æ 1 1 ö
+ -1 1 1 1
ç ÷
= tan -1 ç 7 8 ÷ + tan -1
1
= tan -1
15
tan -1
1 6. (b) 2 tan = tan -1 + tan -1
1 1 18 55 18 5 5 5
çç 1 - ´ ÷÷
è 7 8ø 1 1
+
-1 5 5 -1 2 / 5 5
æ3 1 ö = tan = tan = tan -1
also;ç × < 1÷ 1 1 24 / 25 12
è 11 18 ø 1- .
5 5
æ 3 1 ö
+ æ -1 1 p ö æ 5 pö
-1 3 -1 1 -1 ç 11 18 ÷ = tan çè 2 tan - ÷ = tan ç tan -1 - ÷
tan + tan = tan ç ÷ 5 4ø è 12 4 ø
11 18 çç 1 - 3 ´ 1 ÷÷
è 11 18 ø æ 5ö p 7
tan ç tan -1 ÷ - tan -
65 1 è 12 ø 4 = 12 = - 7
= tan -1 = tan -1 = cot -1 3 = 17
æ 5ö p 17
195 3 1 + tan ç tan -1 ÷ tan 12
3. (d) Given, è 12 ø 4

æ 2a ö æ 1 - b2 ö -1 æ 2 x ö -1 x p
sin -1 ç 2÷
- cos -1 ç ÷ = tan ç ÷ 7. (b) We have, tan <
è 1+ a ø ç 2÷
è 1 - x2 ø p 3
è1+ b ø
\ 2tan–1 a – 2tan–1b = 2 tan–1 x æ xö p x
Þ tan ç tan -1 ÷ < tan Þ < 3
Þ tan–1 a – tan–1 b = tan–1 x è pø 3 p
æ a -b ö a -b
Þ tan -1 ç -1 Þ x < 3p = 5.5 (approx.)
÷ = tan x Þ x = 1 + ab
è 1 + ab ø \ the maximum value of x is 5.
EBD_7167
http://t.me/iitjeehelps

M – 54 BITSAT Topicwise Solved Papers


8. (c) The given trigonometric ratio \ the given expression is equal to
é1 æ æ -1 1 ö ö ù p p
= cos ê ç cos ç cos ÷ ú y+ - y, i.e.,
ë2è è 8 ø ÷ø û 3 3
1 1 1
13. (a) 2 tan -1 + tan -1 + 2 tan -1
5 2 8
æ1 1ö 1 1
= cos ç cos -1 ÷ +
è2 8ø 1
= 2 tan -1 5 8 + tan -1
1 1 7
1- .
æ 1ö 5 8
1 + cos ç cos -1 ÷ 1
è 8ø 3 2.
= = -1 1 -1 1 -1 3 + tan -1 1
2 4 = 2 tan + tan = tan
3 7 1 7
1-
æ 1ö æ 2ö 9
9. (d) tan -1 ç ÷ + tan -1 ç ÷
è 4ø è 9ø 3 1
+
é 1 2 ù -1 3 -1 1 -1 4 7
+ = tan + tan = tan
ê ú é1ù 4 7 3 1
= tan -1 ê 4 9 ú = tan -1 ê ú 1- .
1 2
ê1 - ´ ú ë2û 4 7
êë 4 9 úû
25 p
p = tan -1 = tan -1 1 = 45° =
-1 -1 -1 -1 25 4
10. (d) sin x + cos x + tan x = + tan x
2
-1 1 1 æ pö
Since domain of the function x Î[-1,1] 14. (b) sin {sin + cos -1 } = sin ç ÷ = 1
2 2 è 2ø
p p æ 7p ö 7p
\- £ tan -1 x £ . 15. (c) cos–1 çè cos ÷¹
4 4 6ø 6
p 3p æ 7 p ö é æ 5p ö ù
Hence, k = and K = Now, cos -1 ç cos ÷ = cos -1 êcos ç 2p - ÷ ú
4 4 è 6ø ë è 6 øû
æ -1 1 ö -1 æ 5p ö 5p
11. (d) sin çè 4 tan ÷= = cos çè cos ÷ø =
3ø 6 6
æ 1ö æ 1ö 16. (a) sin (cot–1 (1 + x)) = cos (tan –1 x)
2sin ç 2 tan -1 ÷ cos ç 2 tan -1 ÷ Þ cosec2 (cot–1 (1 + x)) = sec2 (tan–1 x)
è 3ø è 3ø
Þ 1 + [cot (cot–1 (1 + x))]2 = 1 + [sec (tan–1 x)]2
æ -1 3ö æ -1 3 ö 3 4 24
= 2 sin çè tan ÷ = cos çè tan ÷ = 2. . = Þ (1 + x)2 = x2 Þ x = -
1
4ø 4ø 5 5 25
2
æ 1ö æ 7 ö 24 p
cos ç 2 tan -1 ÷ = cos ç tan -1 ÷ = . 17. (b) Let x = cos q , where 0 £ q £ because
è 7ø è 24 ø 25 3
The given expression = 0 1
£ x £1 Given expression is
12. (b) Let cos–1x = y 2
1
Þ x = cos y, so that £ x £ 1 or 0 £ y £ p -1 -1 é 1 3 ù
2 = cos cos q + cos ê cos q + sin q ú
3 ë2 2 û
x 1 1 3 -1 æ p ö
and + 3 - 3x 2 = cos y + sin y = q + cos cos ç q - ÷ ;
2 2 2 2 è 3ø
p -p p
p p æp ö 0£q£ Þ £ q- £ 0
= cos cos y + sin sin y = cos ç - y÷ 3 3 3
3 3 è3 ø
æ p ö p p
æx 1 ö p = q + cos -1 cos ç - q÷ = q + - q =
Þ cos-1 ç + 3 - 3x 2 ÷ = - y è3 ø 3 3
è2 2 ø 3
http://t.me/iitjeehelps

Matrices M – 55

Chapter
Matrices
18
é cos t sin t ù é 0 2ù é 0 2ù
1. If R (t) = ê ú , then R(s) R(t) equals ê3 7ú ê3 7ú
ë- sin t cos t û (a) ê ú (b) ê ú
[2017] êë 6 5úû êë 5 6úû
(a) R (s + t) (b) R (s – t) é0 1ù é 0 -2ù
ê3 7ú ê -3 -7ú
(c) R(s) + R(t) (d) None of these (c) ê ú (d) ê ú
ëê 5 6ûú ëê -5 -6ûú
éa b ù 6. If A and B are 2 × 2 matrices, then which of the
2. If ê g -a ú is square root of identity matrix of
ë û following is true ? [2009]
order 2 then – [2014] (a) (A + B) 2 = A 2 + B2 + 2AB
(a) 1 + a2 + bg = 0 (b) 1 + a2 – bg = 0 (b) (A - B) 2 = A 2 + B2 - 2AB
(c) 1 – a2 + bg = 0 (d) a2 + bg = 1 (c) (A - B)(A + B) = A 2 + AB - BA - B2
3. If the trace of the matrix [2013] (d) (A - B)(A + B) = A 2 - B2
éx - 1 0 2 5 ù é -1 2 ù 2
ê ú 7. If A = ê ú then element a21 of A is
ê 3 2
x -2 4 1 ú ë 3 -4û
A= is 0, then x [2008]
ê -1 -2 x-3 1 ú
ê ú (a) 22 (b) – 15
êë 2 0 4 x 2 - 6 úû (c) – 10 (d) 7
is equal to é1 -2 ù é -1 4 ù é 0 1ù
8. If A = ê ú ,B=ê ú and C = ê ú
(a) –2, 3 (b) 2, –3 ë3 0 û ë 2 3û ë -1 0 û
(c) –3, 2 (d) 3, –2 then 5A – 3B + 2C is equal to [2007]
4. The matrix A2 + 4A – 5I, where I is identity matrix
é8 20 ù é8 -20 ù
(a) (b) ê7 -9 ú
and A = éê
1 2ù ê7 9 ú
ú , equals : [2011] ë û ë û
ë 4 -3û
é -8 20 ù é 8 7ù
é 0 -1ù (c) ê -7 9 ú (d) ê -20 -9 ú
é2 1ù ë û ë û
(a) 4ê ú (b) 4 ê ú
ë2 0û ë2 2 û
é1 2 2ù
ê ú
é2 1ù é1 1 ù 9. If A = ê 2 1 2ú then A2 – 4A = [2006]
(c) 32 ê ú (d) 32 ê ú
ë2 0û ë1 0 û êë 2 2 1úû
(a) 3I (b) 4I
é 1 3ù é -1 -2ù (c) 5I (d) None of these
ê 3 2ú ê0 5ú
5. If A = ê ú and B = ê ú and A + B – D éa 0ù é1 0 ù
êë 2 5úû êë 3 1 úû 10. If A = ê ú and B = ê ú , then value of a
ë 1 1û ë5 1 û
= 0 (zero matrix), then D matrix will be - for which A2 = B, is [2005]
[2010] (a) 1 (b) –1
(c) 4 (d) No real values
EBD_7167
http://t.me/iitjeehelps

M – 56 BITSAT Topicwise Solved Papers

Hints & Solutions


é cos s sin s ù é cos t sin t ù 7. (b) The element a21 is product of second row
1. (a) R( s) R(t ) = ê ú´ê ú of A to the first column of A.
ë - sin s cos s û ë - sin t cos t û
é -1ù
\ a21 = [3 – 4] ê 3 ú = – 3 – 12 = – 15
é cos s cos t - sin s sin t cos s sin t + sin s cos t ù ë û

ë - sin s cos t - cos s sin t - sin s sin t + cos s cos t úû é- 1 4 ù
é1 - 2 ù
é cos( s + t ) sin( s + t ) ù 8. (b) Given A = ê ú, B=ê ú,
=ê ú = R (s + t ) ë3 0 û ë 2 3û
ë- sin( s + t ) cos( s + t ) û
é 0 1ù
éa b ù C =ê ú
2. (d) êg
ë -a úû ë - 1 0û
Consider 5A – 3B + 2C
éa b ù éa b ù é1 0ù
= I2 ; ê g -a ú ê g -a ú = ê 0 1 ú =5ê
é1 -2 ù é -1 4 ù é0 1ù
ë ûë û ë û
3 0 ú - 3 ê 2 3 ú + 2 ê -1 0 úû
Þ a2 + bg = 1 ë û ë û ë
3. (c) Trace of matrix is defined as é 5 - 10ù é- 3 12ù é 0 2ù
=ê ú-ê ú+ê
n
ë15 0 û ë 6 9 û ë- 2 0 úû
å a ii = 2x 2 + 2x - 12 = 0 Þ x = –3, 2
i =1 é8 - 22ù é 0 2 ù é8 - 20ù
=ê ú+ê ú =ê
- 9 úû
4. (a) A2 + 4A – 5I = A × A + 4A – 5I
ë9 - 9 û ë- 2 0 û ë7
é1 2 ù é1 2ù é1 2ù é1 0ù
êë= ´ +4ê -5ê é 1 2 2ù é1 2 2ù
4 -3úû êë 4 -3úû ë4 -3úû ë0 1 úû
ê ú
-4 ù é 4 9. (c) Here A2 = ê 2 1 2ú êê 2 1 2 úú
é9 8 ù é5 0ù
= ê -8 + - êë 2 2 1 úû ëê 2 2 1 ûú
ë 17 úû êë16 -12 úû êë 0 5 úû
é 9+4-5 -4 + 8 - 0 ù é8 4ù é2 1ù é 1 + 4 + 4 2 + 2 + 4 2 + 4 + 2ù é 9 8 8 ù
= 4
= ê-8 + 16 - 0
ë 17 - 12 - 5 úû êë8 0 úû = êë 2 0úû = êê 2 + 2 + 4 4 + 1 + 4 4 + 2 + 2úú = êê8 9 8 úú
éa b ù ëê 2 + 4 + 2 4 + 2 + 2 4 + 4 + 1 ûú ëê8 8 9 úû
ê ú
5. (c) Let D = ê c d ú é9 - 4 8 - 8 8 - 8 ù
ëê e f úû A – 4A = êê 8 - 8 9 - 4 8 - 8 úú
2

é 1 3 ù é - 1 -2ù é a b ù ëê 8 - 8 8 - 8 9 - 4 ûú
ê ú ê ú ê ú é 1 0 0ù
\ A + B – C = ê 3 2ú + ê 0 5 ú - êc d ú
êë 2 5 úû êë 3 1 úû êë e f úû = 5 êê0 1 0úú
= 5I
ëê0 0 1 úû
Þ éê 1 - 1 - a 3 - 2 - bùú éê 0 0 ùú
3+ 0- c 2 +5- d = 0 0 éa 0ù é1 0 ù
ê ú ê ú 10. (d) Given that A = ê ú and B = ê ú
êë 2 + 3 - e 5 + 1 - f úû êë 0 0 úû ë 1 1 û ë5 1 û
– a = 0 Þ a = 0, 1 – b = 0 Þ b = 1, and A2 = B
3 – c = 0 Þ c = 3, 7 – d = 0 Þ d = 7, éa 0 ù éa 0 ù é1 0 ù
5 – e = 0 Þ e = 5, 6 – f = 0 Þ f = 6, Þ ê úê ú=ê ú
ë 1 1 û ë 1 1 û ë5 1 û
é0 1 ù
ê ú é a2 0 ù é1 0 ù
\ D = ê3 7 ú Þê ú=ê ú
êë5 6 úû ëêa + 1 1 ûú ë5 1 û
Þ a2 = 1, a + 1 = 5
6. (c) For two 2 × 2 matrices, A & B Þ a = + 1, a = 4
(A – B) × (A + B)
Q There is no common value
=A×A+A × B – B ×A– B × B \ There is no real value of a for which A2 = B
= A2 – B2 + AB – BA
Hence, (A – B) (A + B) = A2 + AB – BA – B2
http://t.me/iitjeehelps

Determinants M – 57

Chapter
Determinants
19
1. If a system of equation – ax + y + z = 0 7. If the lines p1x + q1y = 1, p2x + q2y = 1 and p3x +
x – by + z = 0; x + y – cz = 0 (a, b, c ¹ –1) q 3 y = 1 be concurrent, then the points
has a non-zero solution then (p1, q1), (p2, q2) and (p3, q3) [2015, 2010]
1 1 1 (a) are collinear
+ + = [2017] (b) form an equilateral triangle
1+ a 1+ b 1+ c (c) form a scalene triangle
(a) 0 (b) 1 (c) 2 (d) 3 (d) form a right angled triangle
p q-y r-z 8. One of the roots of
2. If p-x q r - z = 0 , then the value of
x+a b c
p-x q-y r
a x+b c = 0 is : [2014]
p q r
+ + is [2017] a b x+c
x y z
(a) 0 (b) 1 (c) 2 (d) 4pqr (a) abc (b) a + b + c
(c) –(a + b + c) (d) –abc
é1 3 l + 2ù 9. The value of l , for which the lines
3. If the matrix ê2 4 ú
8 ú is singular, then l =
ê 3x - 4 y = 13, 8 x - 11y = 33 and 2 x - 3 y + l = 0
êë3 5 10 úû are concurrent is [2014]
[2016] 1
(a) –2 (b) 4 (c) 2 (d) – 4 (a) –1 (b) –7 (c) (d) 9
7
4. Let a1, a2 and b1, b2 be the roots of ax2 + bx + c
1 ( x – 3) ( x – 3)
2
= 0 and px2 + qx + r = 0 respectively. If the system
of equations a1y + a2z = 0 and b1y + b2z = 0 has 10. The determinant 1 ( x – 4 ) ( x – 4) 2
a non-trivial solution, then [2016]
b 2 ac c 2
ab vanishes for
1 ( x – 5) ( x – 5) 2
(a) = (b) 2 = [2013]
q 2 pr r pq
(a) 3 values of x (b) 2 values of x
a2 bc (c) 1 values of x (d) No value of x
(c) = (d) None of these 11. If the lines lx + my + n = 0, mx +ny + l = 0 and
2 qr
p nx + ly + m = 0 are concurrent then [2013]
5. If [ ] denotes the greatest integer less than or (a) l + m + n = 0 (b) l – m – n = 0
equal to the real number under consideration (c) l + m – n = 0 (d) m + n – l = 0
and –1 < x < 0; 0 < y < 1; 1 < z < 2 , then the value 12. The equations 2x + 3y + 4 = 0; 3x + 4y + 6 = 0 and
of the determinant [2016] 4x + 5y + 8 = 0 are [2012]
(a) consistent with unique solution
[x] + 1 [ y] [z] (b) inconsistent
[ x ] [ y ] + 1 [ z ] is (c) consistent with infinitely many solutions
[x] [ y ] [ z] + 1 (d) None of the above
(a) [z] (b) [y] 13. The value of the determinant
(c) [x] (d) None of these 265 240 219
6. Let M be a 3 × 3 non-singular matrix with det
240 225 198
(M) = a. If [M–1 adj (adj (M)] = KI, then the is [2012, 2007]
value of K is [2015] 219 198 181
(a) 1 (b) a (c) a2 (d) a3 (a) 1000 (b) 779 (c) 679 (d) 0
EBD_7167
http://t.me/iitjeehelps

M – 58 BITSAT Topicwise Solved Papers


20. If the three linear equations [2008]
é 2 0 0ù
ê ú x + 4ay + az = 0; x + 3by + bz = 0
14. If A = ê 2 2 0ú , then adj ( adj A) is equal to -
êë 2 2 2 úû
x + 2cy + cz = 0
[2011] have a non-trivial solution, where a ¹ 0, b ¹ 0,
c ¹ 0, then ab + bc is equal to
é1 0 0 ù é1 0 0 ù
(a) 2ac (b) – ac (c) ac (d) – 2ac
(a) 8 êê1 1 0úú (b) 16 êê1 1 0úú
21. The system of equations a x + y + z = a – 1,
êë1 1 1 úû êë1 1 1 úû
x + a y + z = a – 1, x + y + a z = a – 1 has no
é1 0 0ù
ê solution if a= [2007]
(c) 64 ê1 1 0 úú (d) None of these
(a) –2 (b) a ¹ – 2
êë1 1 1 úû
(c) either – 2 or 1 (d) a = 1
1 2 3
22. If 1, w, w 2 are the cube roots of unity, the[2006]
15. The value of -4 3 6 is [2010]
2 -7 9 1 wn w 2n
(a) 213 (b –231 (c) 231 (d) 39
16. If a > 0, b > 0, c > 0 are respectively the pth, qth, D = wn w2n 1 is equal to
rth terms of G.P., then the value of the 2n n
w 1 w
log a p 1 (a) 2 (b) 0 (c) 1 (d) w
w
determinant log b q 1 is [2009]
log c r 1 é0 1ù
é 0 -1 2ù ê ú
(a) 0 (b) 1 23. If A = ê ú ,B= ê1 0 ú and M = AB,
(c) –1 (d) None of these ë 2 -2 0û êë1 1 úû
17. The digits A, B and C are such that the three then find M–1. [2006]
digit numbers A88, 6B8, 86C are divisible by 72
A 6 8 é1 -1ù é1 1ù
ê6 3ú ê6 3ú
then the determinant
8 B 6
is divisible by (a) ê1 1ú (b) ê1 1ú
8 8 C ê ú ê ú
ë3 3û ë3 6û
[2009]
(a) 72 (b) 144 (c) 288 (d) 216 é1 1ù é1 1ù
é cos a - sin a 0ù ê3 6ú ê3 6ú
ê sin a cos a 0ú (c) ê1 1ú (d) ê1 1ú
18. If M (a) = ê ú ; ê ú ê ú
êë 0 ë6 3û ë6 6û
0 1úû
é cos b 0 sin b ù é 2 -3ù
ê 0 24. Inverse matrix of ê ú [2005]
1 0 úú ë -4 2 û
M (b) = ê then [M(a) M (b)]–1
êë - sin b 0 cos búû 1 é 2 3ù 1 é 2 4ù
is equal to - [2009] (a) – ê ú (b) – ê ú
8 ë 4 2û 8 ë 3 2û
(a) M(b) M (a) (b) M (–a) M(–b)
(c) M(–b) M(–a) (d) –M(b) M(a) 1 é 2 3ù é 2 3ù
19. If a + b + c = 0 then determinant [2008] (c) ê ú (d) ê ú
8 ë 4 2û ë 4 2û
a -b-c 2a 2a
2b b-c-a 2b is equal to,
2c 2c c -a -b
(a) 0 (b) 1 (c) 2 (d) 3
http://t.me/iitjeehelps

Determinants M – 59

Hints & Solutions


-a 1 1 -q r
and b1 + b2 = , b 1b 2 = ...(2)
1. (b) D= 1 -b 1 =0 for non-zero p p
1 1 -c Since the given system of equation has a
non-trivial solution
solution Þ abc – a – b – c – 2 = 0
Þ abc = a + b + c + 2 a1 a 2
\ = 0 i.e. a1b2 – a2b1 = 0
1 1 1 b1 b 2
Now, + +
1+ a 1+ b 1+ c
a1 a 2 a1 + a 2 a1a 2
3 + 2 (a + b + c) + (ab + bc + ac) or = = =
= b1 b 2 b1 + b 2 b1b 2
1 + (a + b + c) + (ab + bc + ac) + abc
pb pc b 2 ac
3 + 2 ( a + b + c) + (ab + bc + ac) Þ = Þ =
= =1 qa ra q 2 pr
1 + 2 ( a + b + c ) + 2 + ab + bc + ac 5. (a) Since, –1 £ x < 0 \ [x] = –1
0 £ y < 1 \ [y] = 0; 1 £ z < 2 \ [z] = 1
p q-y r-z
p-x q r-z =0 0 0 1
2. (c)
\ Given determinant = -1 1 1 = 1 = [z]
p-x q-y r -1 0 2
Apply R1 ® R1 – R3 and R2 ® R2 – R3, 6. (b) We know that, M (adj M) = |M| I
we get Replacing M by adj M, we get
x 0 -z adj M [adj (adj M) = det (adj M) I
= det (M) M–1 [adj (adj M) = a2l
0 y -z = 0 Þ a M–1 [adj (adj M)] = a2I
p-x q-y r Þ M–1 [adj (adj M)] = a I
Þ x[ yr + z(q - y)] - z[0 - y(p - x)] = 0 But M–1 [adj (adj M)] = KI Hence, K = a
7. (a) The equations of the lines are
Þ xyr + xzq - xzy + yzp - zyx = 0 p1x + q1y - 1 = 0 ...(i)
p q r p2 x + q 2 y - 1 = 0 ...(ii)
Þ xyr + zxq + yzp = 2xyz Þ + + = 2
x y z and p3 x + q 3 y - 1 = 0 ...(iii)
3. (b) | A | = 0 as the matrix A is singular As they are concurrent,
1 3 l+2
p1 q1 -1 p1 q1 1
\ | A |= 2 4 8 =0
p2 q2 -1 = 0 Þ p2 q2 1 = 0
3 5 10
p3 q3 -1 p3 q3 1
Apply R 2 ® R 2 - 2R1 and
R 3 ® R 3 - 3R1 and expand. This is also the condition for the points
(p1, q1), (p2, q2) and (p3, q3) to be collinear.
-2(4 - 3l) + 4(4 - 2l) = 0
x+a b c
Þ 8 - 2l = 0 Þ l = 4
For l = 4 , the second and the third column a x+b c
8. (c) =0
are proportional. a b x+c
4. (a) Since a1, a2 and b1, b2 are the roots of ax2 Applying, C1 ® C1 + C2 + C3
+ bx + c = 0 and px2 + qx + r = 0 respectively,
(a + b + c + x) b c
-b c
therefore a1 + a2 = , a1a2 = ...(1) (a + b + c + x) x + b c
a a =0
(a + b + c + x) b c+x
EBD_7167
http://t.me/iitjeehelps

M – 60 BITSAT Topicwise Solved Papers

1 b c -4 3 2 -4
Dx = =2 and Dy = =0
1 x+b c -6 4 3 -6
(a + b + c + x) =0
1 b c+x \ x = –2 and y = 0
C2 ® C2 – C1, C3 ® C3 – C1 Now this solution satisfies the third, so the
equations are consistent with unique
1 b c solution.
0 x 0 13. (d) Applying C1 – C2 and C2 – C3, we get
(a + b + c + x) =0
0 0 x 25 21 219 4 21 9
(a + b + c + x) 1. x2 = 0; x = 0, – (a + b + c) Det. = 15 27 198 = -12 27 -72
9. (b) For concurrency of 3 lines the determinant
21 17 181 4 17 11
of coefficients of equations should be 0.
3 -4 -13 [by C1 – C2, C3 –10C2]
8 -11 -33 = 0 4 21 9
i.e.,
2 -3 l = 0 90 -45 [By R2 + 3R1,R3–R1]
Þ 3(-11l - 99) + 4(8l + 66) -13(-24 + 22) = 0 0 -4 2
Þ -33l - 297 + 32l + 264 + 312 - 286 = 0 = 4 (180 – 180) = 0
Þ -l - 583 + 576 = 0 Þ l = -7 é 2 0 0ù
10. (d) The given determinant vanishes, i.e., ê 2 2 0ú
14. (b) | A | = ê ú = (2) (2) (2) = 8
1 x - 3 ( x - 3)2
ëê 2 2 2úû
1 x - 4 ( x - 4)2 = 0 Now adj (adj A) = |A|3–2A
1 x - 5 ( x - 5)2 é 2 0 0ù é1 0 0ù
ê 2 2 0ú ê ú
Expanding along C1, we get =8ê ú = 16 ê1 1 0ú
(x – 4)(x – 5)2 – (x – 5)(x – 4)2 – {(x – 3)(x –5)2 êë 2 2 2úû êë1 1 1úû
– (x – 5)(x – 3)2} + (x – 3)(x – 4)2
1 2 3
– (x – 4)(x – 3)2 = 0 3 6 -4 6
Þ (x – 4)(x – 5)(x – 5 – x + 4) -4 3 6
15. (c) =1 –2 +3
– (x – 3)(x – 5)(x – 5 – x + 3) 2 -7 9 -7 9 2 9
+(x – 3)(x – 4) (x – 4 –x + 3) = 0
Þ – (x – 4)(x – 5) + 2(x – 3)(x – 5) – (x – 3) -4 3
(x – 4) = 0 2 -7
Þ – x2 + 9x – 20 + 2x2 – 16x + 30 – x2 + 7x –
= 1 (3 × 9 – 6 (–7)) –2 (–4 × 9 –2 × 6) +3 ((–4)
12 = 0 (–7) – 3 × 2)
Þ – 32 + 30 = 0 Þ –2 = 0 = (27 + 42) –2 (–36 – 12)+3 (28 – 6) = 231
Which is not possible, hence no value of x
16. (a) Let A be the 1st term and R the common
satisfies the given condition.
ratio of G.P., then
11. (a) Since the lines are concurrent, so
l m n a = Tp = AR p-1
m n l = 0 Þ 3lmn – l3 – m3 – n3 = 0 \ log a = log A + ( p - 1) log R
n l m Similarly, log b = log A + (q - 1) log R
Þ (l + m + n) (l2 + m2 + n2 – lm – mn – nl) = 0 and log c = log A + (r - 1) log R
Þ l + m + n = 0 [Q l2 + m2 + n2 > lm + mn + n]
12. (a) Consider first two equations : log A + ( p - 1) log R p 1
2x + 3y = –4 and 3x + 4y = –6 \D = log A + ( q - 1) log R q 1
log A + ( r - 1) log R r 1
2 3 Split into two determinants and in the first
We have D = = -1 ¹ 0
3 4 take logA common and in the second take
http://t.me/iitjeehelps

Determinants M – 61

log R common 1 1 1
1 p1 p -1 p 1
D = log A 1 q 1 + log R q - 1 q 1 = (a + b + c ) 2 b b – c – a 2b
1 r 1 r -1 r 1 2c 2c c–a–b
Apply C1 ® C1 - C 2 + C3 in the second =0 ( Q a + b + c = 0)
0 p1 20. (a) Given system has non trivial solution
D = 0 + log R 0 q 1 = 0 1 4a a
0 r 1
Þ 1 3b b = 0, (a ¹ 0, b ¹ 0, c ¹ 0)
17. (a) R3 ® 100R1 + 10R 2 + R 3
1 2c c
A 6 8 A 6 8 on expanding along C1, we get
(3bc – 2bc) – (4ac – 2ac) + 4ab – 3ba = 0
Þ 8 B 6 = 8 B 6
Þ bc + ab = 2ac
8 8 C A88 6BC 86C
a 1 1
which is divisible by 72. 21. (a) D =0Þ 1 a 1 =0
18. (c) [M(a) M(b)]–1 = M(b)–1 M(a)–1] 1 1 a
é cos a sin a 0ù Þ (a – 1)2 (a + 2) = 0 Þ a = 1, –2
ê - sin a cos a 0ú But for no solution, at least one of D1, D2
Now M(a)–1 = ê ú and D3 should not be equal to 0.
êë 0 0 1úû \ a¹1 \ a = –2
é cos(-a ) - sin(-a ) 0ù
ê sin( -a) cos(-a ) 0ú 1 wn w 2n
= ê ú = M(–a) D = wn w 2n 1
êë 0 0 1 úû 22. (b)
2n
w 1 wn
é cos b 0 - sin bù
ê 0
M(b)–1 = ê
1 0 úú ( 3n
)
n 2n
= 1 w -1 - w w - w
2n
(
+ w 2n w n - w 4n ) ( )
êë sin b 0 cos b úû
= w 3n - 1 - 0 + w 3n - w 6 n
é cos(-b) 0 sin(-b) ù = 1 - 1 + 1 - 1 = 0 éQ w3n = 1ù
ê ë û
0 1 0 úú
= ê = M (–b)
êë - sin(-b) 0 cos(-b) úû é0 1ù
é 0 -1 2 ù ê 1 0ú é 1 2ù
[M(a) M(b)]–1 = M(–b) M(–a) 23. (a) M = ê 2 -2 0 ú ê ú = ê
-2 2 úû
19. (a) We have
ë û ê1
ë 1 úû ë

a-b-c 2a 2a é 1 -2 ù
|M| = 6, adj M = ê 2 2 ú
2b b-c-a 2b ë û
2c 2c c-a-b 1 é 1 -2 ù é1/ 6 -1/ 3ù
M–1 = ê ú = ê ú
and (a + b + c) = 0 6 ë 2 2 û ë1/ 3 1/ 3 û
24. (a) Let the given matrix is A, then | A | = – 8
Applying R1 ® R1 + R 2 + R 3
T
é 2 4ù é 2 3ù
a +b+c a +b+c a +b+c and adj A = ê ú = ê ú
ë 3 2û ë 4 2û
= 2b b–c–a 2b
2c 2c c–a–b 1 1 é 2 3ù
\ A–1 = adj A = – ê ú
|A| 8 ë 4 2û
Take (a + b + c) common from R1
EBD_7167
http://t.me/iitjeehelps

M – 62 BITSAT Topicwise Solved Papers

Chapter
Continuity & Differentiability
20
(a) has no limit
1. The function f ( x ) = x - | x - x 2 |, - 1 £ x £ 1 is
(b) is not continuous
continuous on the interval [2017] (c) is continuous but not differentiable
(a) [–1, 1] (b) (–1, 1) (d) is differentiable
(c) {–1, 1] – { 0 } (d) (–1, 1) – {0} 8. If g is the inverse of function f and f ¢(x) = sin x,
ì x log cos x then g¢(x) is equal to [2015]
ï ,x ¹ 0 (a) cosec {g(x)} (b) sin {g(x)}
2. If f (x) = í log(1 + x 2 ) then f(x) is 1
ï 0 ,x = 0 (c) sin {g ( x )} (d) None of these
î
[2017] 9. The number of real roots of the equation
(a) continuous as well as differentiable at x = 0 ex–1 + x – 2 = 0 is [2015]
(b) continuous but not differentiable at x = 0 (a) 1 (b) 2 (c) 3 (d) 4
(c) differentiable but not continuous at x = 0 d2y dy
(d) neither continuous nor differentiable at x = 0 10. If y = (x + 1 + x ) , then (1 + x ) 2 + x
2 n 2 is
dx dx
é1 - (log x) 2ù
[2015, 2007]
3. If f (x) = cos–1 ê ú
2 then the value of f ' (e)
ëê1 + (log x) úû (a) n2y (b) – n2y (c) –y (d) 2x2y
is equal to [2017] d
11. (cos–1 x + sin–1 x) is [2014]
dx
(c) 2 (d) 22
1
(a) 1 (b) (a) p/2 (b) 0
e e e
4. The number of points at which the function (c) 2 / 1 - x 2 (d) None of these
1 ì 1
f ( x) = is discontinuous is : [2016] ï( x –1) sin if x ¹ 1
log | x | 12. Let f ( x) = í x –1
ïî 0 if x = 1
(a) 1 (b) 2 (c) 3 (d) 4
5. Let f ( x) = x - 1 + x + 24 - 10 x - 1; Then which one of the following is true? [2014]
1 < x < 26 be real valued function. Then f ¢(x) (a) f is differentiable at x = 0 and x =1
for 1 < x < 26 is [2016] (b) f is differentiable at x = 0 but not at x = 1
1 (c) f is differentiable at x = 1 but not at x = 0
(a) 0 (b) (d) f is neither differentiable at x = 0 nor at x =1
x -1
x 2 x3 dy
(c) 2 x - 1 - 5 (d) None of these 13. If y =1 + x + + +....¥, then is equal to
6. For any differentiable function y of x, 2! 3! dx
[2013]
3
d 2 x æ dy ö d2y (a) x (b) 1
ç ÷ + = [2016]
dy 2 è dx ø dx 2 (c) y (d) None of these
(a) 0 (b) y (c) – y (d) x ì x 2 + 3x - 10
7. If a function f(x) is given by ï , when x ¹ -5
x x 14. If f (x) = í x 2 + 2x - 15 [2013]
f (x) = + ï
1 + x ( x + 1)( 2x + 1) î a , when x = -5
x is continuous at x = –5, then the value of ‘a’ will be
+ + ....¥ , then at x = 0, f(x) [2015] (a) 3/2 (b) 7/8 (c) 8/7 (d) 2/3
( 2x + 1)( 3x + 1)
http://t.me/iitjeehelps

Continuity & Differentiability M – 63

22. The set of points of discontinuity of the function


d2y 1/log | x | is – [2009, 2005]
15. If x = a sin q and y = b cos q , then is
dx 2 (a) {–1, 0, 1} (b) {0}
[2012, 2010] (c) {0, 1} (d) None of these
a -b dy
(a)
2
sec 2 q (b) sec 2 q 23. If y = (cos x2)2 then is equal to : [2008]
b a dx
-b 3 b (a) – 4x sin 2x2 (b) – x sin x2
(c) sec q (d) 2 sec3 q
a2 a (c) – 2x sin 2x2 (d) – x cos 2x2
16. If f(x) = xa log x and f(0) = 0, then the value of a 1
for which Rolle’s theorem can be applied in [0, 1] 24. If f(x) = , then the points of discontinuity
1- x
is [2012] of the function f [ f {f(x)}] are [2008]
(a) –2 (b) –1 (c) 0 (d) 1/2 (a) {0, –1} (b) {0,1}
ì 1 , x£2 (c) {1, –1} (d) None of these
ï ì x sin1 / x , x ¹ 0
17. If the function f(x) = íax + b , 2 < x < 4
ï 7 25. If f (x) = í is continuous at
, x³4 î k , x= 0
î
is continuous at x = 2 and 4, then the values of a x = 0, then the value of k will be [2007]
and b are [2012] (a) 1 (b) –1
(a) 3, 5 (b) 3, – 5 (c) 0, 3 (d) 0, 5 (c) 0 (d) None of these
26. Let h(x) = min {x, x2}, for every real number of x,
x2 dy Then [2006]
18. If y = x , then is equal to [2011]
1]
dx (a) h is continuous for all x
(2 ln x + 1) (b) h is differentiable for all x
(a) (2 ln x) (b)
(c) h’(x) = 2, for all x > 1
2 (d) h is not differentiable at three values of x.
(c) (ln ln x + 1) x x (d) None of these
27. If f(x) = (x + 1)cot x is continuous at x = 0, then
19. The function f (x) = (x - 1) | ln x | is at x = 1 f(0) is equal to [2006]
(a) 0 (b) 1 (c) 1/e (d) e
(a) discontinuous [2011]
(b) continuous but not differentiable dv
28. If 2t = v2, then is equal to [2006]
(c) differentiable with f ' (1) = 0 dt
(d) differentiable with f ¢ (1) ¹ 0 (a) 0 (b) 1/4 (c) 1/2 (d) 1/v
ì 2 29. If y2 = P (x), is a polynomial of degree, then
20. Let f (x) = íax + 1, x > 1 . Then f (x) is
îx + a , x £ 1 [2005]
derivable at x = 1, if [2010] æ d ö æ d yö
2
(a) a = 2 (b) a = 1 (c) a = 0 (d) a = 1/2 2 ç ÷ ç y3 . 2 ÷ equals
è dx ø è dx ø
d4y
21. If y = e–x cos x and y4 + ky = 0, where y4 = , (a) P'''(x) + P' (x) (b) P''(x). P'''(x)
dx 4 (c) P (x). P''' (x) (d) a constant
then k = [2009]
(a) 4 (b) – 4 (c) 2 (d) – 2

Hints & Solutions


1. (a) we have, f ( x) = x - | x - x 2 |= x - | x (1 - x ) | LHL
= x - | x || 1 - x |, = lim f (0 - h) = lim - h - | -h | | 1 + h |
\ Continuity is to be checked at x = 0 and h ®0 h ®0
x = 1. At x = 0
EBD_7167
http://t.me/iitjeehelps

M – 64 BITSAT Topicwise Solved Papers

= lim - h - h (1 + h ) = 0 =cos–1 [cos 2 q] = 2 q = 2 tan–1 t = 2 tan–1 (log x)


h ®0 Diff. both side w.r.t 'x', we get
RHL = lim f (0 + h ) = lim h - | h | | 1 - h | 1 1
h ®0 h ®0 f ' (x) = 2. 2
.
= lim h - h (1 - h ) = 0 1 + (log x ) x
h ®0 1 1 1
and f(0) = 0 Since LHL = RHL = f(0), Now, f ' (e) = 2. 2
. = (Q log e = 1)
1 + (log e) e e
\ f(x) is continuous at x = 0.
At x = 1; LHL 4. (c) The function log|x| is not defined at x = 0,
so, x = 0 is a point of discontinuity. Also for
= lim f (1- h) = lim(1- h)- |1- h ||1 - (1- h)|
h®0 h®0
f(x) to be defined, log |x| ¹ 0 Þ x ¹ ± 1.
Hence, 0, 1, –1 are three poin ts of
= lim (1 - h ) - h (1 - h ) = 1 discontinuity..
h ®0
h ® 0 f (1 + h ) = 1
Similarly RHL = lim 5. (a) f ( x) = x -1 + 25 + ( x -1) - 10 x - 1
and f (1) = 1- | 1 | . | 1 - 1 |= 1 = x - 1 + (5 - x - 1)2
\ f(x) is continuous at x = 1
Hence f(x) is continuous for all x Î [-1, 1] = x - 1+ |5 - x - 1 | = 5\ f '( x) = 0
2. (a) We have, -2
dy æ dx ö
-1 d2y æ dx ö ìï d æ dx öïü
6. (a) =ç ÷ Þ = -1çç ÷÷ í çç ÷÷ý
f ( 0 - h ) - f ( 0) - h log cosh dx çè dy ÷ø dx 2
è dy ø ïî dx è dy øïþ
Lf ¢(0) = lim = lim
h ®0 -h h ® 0 - h log(1 + h 2 )
-2
log cosh d2y æ dx ö ìï d æ dx ö dy ïü
= lim æ0 ö Þ = ( -1) çç ÷÷ í çç ÷÷ ý
ç form ÷ 2
h ®0 log(1 + h 2 ) è0 ø dx è dy ø ïî dy è dy ø dx ïþ
2 ì 2
æ dy ö ï d x dy üï
- tan h = (-1) ç ÷ í . ý
= lim = -1 / 2 è dx ø ïî dy 2 dx ïþ
h ®0 2h /(1 + h 2 )
3ì 2
ï d x üï d 2 x æ dy ö
3
f (0 + h ) - f ( 0) h log cosh æ dy ö d2 y
= -ç ÷ í 2 ýÞ 2 ç ÷ + 2 = 0
Rf ¢( 0) = lim = lim è dx ø ïî dy ïþ dy è dx ø dx
®
h 0 h h 0 h log(1 + h 2 )
®
7. (b) Let
log cos h
æ0 ö
= lim ç form ÷
h ®0 log(1 + h 2 ) è 0 ø x x x
f ( x) = + + + ...¥
- tan h -1 1 + x ( x + 1)( 2x + 1) ( 2x + 1)( 3x + 1)
= lim =
2 2
h ®0 2h /(1 + h ) n
x
Since Lf ¢(0) = Rf ¢(0), therefore f(x) is differen-
= lim
n ®¥
å é( r - 1) x + 1ù ( rx + 1)
r =1 ë û
tiable at x = 0
n é
Since differentiability x 1 ù
Þ continutity, therefore f(x) is continuous at = lim å ê é( r - 1) x + 1ù - rx + 1ú
x = 0.
x ®¥ r =1 êë ë û úû

é 1 – (log x )2 ù é 1 ù
= lim ê1 - =1
3. (b) Let f (x) = cos–1 ê 2ú n ®¥ ë nx + 1úû
ë 1 + (log x ) û
For x = 0, we have f(x) = 0
é1 - t 2 ù
Put log x = t in f (x) \ f (x) = cos–1 ê ì1, x ¹ 0
2ú Thus, we have f ( x ) = í
ë1 + t û î0, x = 0

Clearly, lim- f ( x ) = lim+ f ( x ) ¹ f ( 0 )


é1- tan2 qù
Now, put t = tan q , we get; f (x) = cos–1 ê ú x ®0 x ®0
2
êë1+ tan qúû So, f(x) is not continuous at x = 0.
http://t.me/iitjeehelps

Continuity & Differentiability M – 65

8. (c) Since, g is the inverse of function f. 1


Therefore, g(x) = f–1(x) h sin –0
h 1
= lim = lim sin
Þ f[g(x)] = x Þ fog ( x ) = x, for all x h®0 h h® 0 h
Differentiate both side,w.r.tx which does not exist.
d d \ f is not differentiable at x = 1
Þ
dx
{fog ( x )} =
dx
( x ) , for all x Also
Þ f ¢[g(x)] g¢(x) = 1, for all x 1 x –1 æ 1 öù
Þ sin {g ( x )} g ' ( x ) = 1, for all x f '(0) = sin –
( x –1) ( x –1) 2
cos ç ú
è x –1÷ø ú
û x= 0
1
Þ g' (x) = = –sin 1 + cos 1
sin {g ( x )} \ f is differentiable at x = 0
9. (a) Let f(x) = ex–1 + x – 2 dy
check for x = 1 Then, f (1) = e0 + 1 – 2 = 0 13. (c) y = ex Þ = ex = y
dx
So, x = 1 is a real root of the equation f(x) = 0 (x - 2) (x + 5) -7 7
Let x = a be the other root such that a > 1 or 14. (b) lim f (x) = = =
a < 1. Consider the interval [1, a ] or [ a,1] . x ®-5 (x + 5) (x - 3) -8 8
Clearly f(1) = f(a) = 0 15. (b) Given x = a sin q and y = b cos q
By Rolle’s theorem f¢(x) = 0 has a root in (1, a)
dx dy
or in (a, 1). Þ = a cos q and = - b sin q
But f¢(x) = ex–1 + 1 > 0, for all x. Thus, dq dq
f ¢ ( x ) ¹ 0 , for any x Î (1, a ) or x Î ( a,1) , dy dy dq b d 2 y -b
which is a contradiction. \ = ´ = - tan q Þ 2 = sec2 q
dx dq dx a dx a
Hence, f(x) = 0 has no real root other than 1.
16. (d) For Rolle’s theorem in [a, b], f(a) = f(b),
10. (a) y = (x + 1 + x ) 2 n In [0, 1] Þ f(0) = f(1) = 0
Q the function has to be continuous in [0, 1]
dy æ 1 ö
= n( x + 1 + x 2 )n -1 ç1 + (1 + x 2 ) -1/ 2 . 2 x÷; a
Þ f (0) = lim f ( x ) = 0 Þ lim x log x = 0
dx è 2 ø x ®0+ x ®0
log x
dy ( 1 + x 2 + x) Þ lim =0
= n( x + 1 + x 2 ) n -1 x ®0 x -a
dx 1 + x2
1/ x
2 n
Applying L.H. Rule lim =0
n( 1 + x + x ) x ®0 - ax - a-1
=
1 + x2 - xa
Þ lim =0Þa >0
x ®0 a
dy
or 1 + x 2 = ny or 1 + x 2 y 1= ny 17. (b) Since f (x) is continuous at x = 2
dx
Squaring, (1 + x 2 ) y12 = n2 y 2 \ f (2) = lim f (x) Þ 1 = lim (ax + b)
x ® 2+ x ® 2+
Differentiating, \ 1 = 2a + b ..... (1)
2 Again f(x) is continuous at x = 4,
(1 + x )2 y1 y2 + y12 .2 x
= n .2 yy 1 2

or (1+x2)y2 + xy1 = n2y \ f (4) = lim- f (x) Þ 7 = lim- (ax + b)


x®4 x®4
d d æ pö \ 7 = 4a + b .... (2)
11. (b) (cos–1 x + sin–1x) = çè ÷ø = 0 Solving (1) and (2), we get a = 3, b = – 5
dx dx 2
x 2
2
ì æ 1 ö 18. (d) Consider y = x Þ ln y = x ln x
ï( x –1)sin ç if x ¹ 1
12. (b) We have; f ( x ) = í è x –1÷ø 1 dy 1
ï0 = 2 x ln x + x 2 . = x (1 + 2 ln x )
î if x = 1 y dx x
f (1 + h) – f (1) dy 2 2
Rf '(1) = lim = x x . x (1 + 2 ln x ) = x x +1 (1 + 2 ln x )
h®0 h dx
EBD_7167
http://t.me/iitjeehelps

M – 66 BITSAT Topicwise Solved Papers


19. (c) Continuous as well as differentiable so 26. (a) From the figure it is clear that
f '(1) = 0
ì x, if x £ 0
f (1 - h) - f (1) ï 2
20. (d) Lf ' (1) = lim h (x) = í x , if 0 < x < 1
h®0 -h
ï x, if x ³ 1
(1 - h + a) - (1 + a) -h î
= lim = lim =1
h®0 -h h ®0 -h Y y=x
2

f (1 + h) - f (1) y=x
Rf ¢ (1) = lim (1, 1)
h®0 h
[a (1 + h) 2 + 1] - (1 + a) lim (ah + 2a) = 2a
= lim = h®0 O(0, 0) X
h ®0 h
Since f ' (1) exists, \ L f ' (1) = R f ' (1) Þ a = 1/2
21. (a) Let y = e–x cos x
y1 = – e–x sin x – e–x cos x = – e–x sin x – y From the graph it is clear that h is
y2 = – e–x cos x + e–x sin x – y1 continuous for all x Î R , h' (x) = 1 for all x
Þ y2 = – y – y1 + e–x sin x = – 2 (y + y1)
> 1 and h is not differentiable at x = 0 and 1.
Þ y3 = – 2 (y1 + y2) = – 2 (e–x sin x – y)
27. (d) Q f (x) is continuous at x = 0, so
Þ y4 = 4y1 + 2y2 = 4y1 – 4y – 4y1 or y4 + 4y = 0
Þ k=4 f (0) = lim f (x)
x ®0
1
22. (a) Let f (x) = Let A = xlim
®0
f (x) = xlim
®0
(x + 1)cot x
log | x |
The points of disconinuity of f (x) are those
points where Þ log A = xlim ®0
cot x. log (1 + x)
f (x) is undefined or infinite. It is undefined log(1 + x) æ 0 ö
when x = 0 and is infinite when = xlim
®0 tan x ç
è0
form÷
ø
log | x | = 0, | x | = 1, i.e. x = ± 1. x2
\ Set of points of discontinuity = {–1, 0, 1}. x - + ....
23. (c) As given : y = (cos x2)2 2 =1
Diff both side w.r.t 'x' = xlim
®0 x 3 \A= e1 = eÞ f (0) = e.
x + + ....
dy 3
= 2 cos x2 (– sin x2) 2x 28. (d) 2t = v2 ; Differentiating w.r.t. to v, we get
dx dv dv 1
= – 4x cos x2 sin x2 = – 2x sin 2x2 2v = 2 or =
dt dt v
1 dy
24. (b) We have, f (x) = . 29. (c) y2 = P (x) Þ 2y = P ¢ (x)
1- x dx
As at x = 1, f (x) is not defined, x = 1 is a
2 2
point of discontinuity of f (x).
or 2 çæ dy ÷ö + 2y d y = P ¢¢(x)
æ 1 ö 1 x -1 è dx ø dx 2
If x ¹1, f [f(x)] = f çè ÷ø = =
1- x 1 - 1/ (1 - x) x 2
d2 y æ dy ö P¢ 2
\ x = 0, 1 are points of discontinuity of f [f(x)] . or 2y = P ¢¢ - 2 çè ÷ø = P ¢¢ -
If x ¹ 0, x ¹1 dx 2 dx 2y 2

æ x - 1ö 1 d2 y 1 2 1
f [f { f (x) }] = f ç ÷ = =x. Q 2y3 = y 2 P ¢¢ - P ¢ = P ¢¢ - P ¢ 2
è x ø (x - 1) dx 2 2 2
1-
25. (c) Since f(x) is continuous at xx= 0, so æ 3 d2y ö
d
\2 çy ÷ = P ¢P ¢¢ + PP ¢¢¢ - P ¢P ¢¢ = PP ¢¢¢
f (0) = lim f (x) Þ k = lim x sin 1/x
x ®0 x ®0
dx è dx 2 ø
= 0 × (a finite quantity) = 0
http://t.me/iitjeehelps

Application of Derivatives M – 67

Chapter
Application of Derivatives
21
1. If f (x) = xx, then f (x) is increasing in interval : 6 1
[2017] (c) (d)
1 7 2
(a) [0, e] (b) [0, ] 6. A wire 34 cm long is to be bent in the form of a
e quadrilateral of which each angle is 90°. What is
(c) [0, 1] (d) None of these
2. A cylindircal gas container is closed at the top the maximum area which can be enclosed inside
and open at the bottom. if the iron plate of the the quadrilateral? [2016]
5 (a) 68 cm2 (b) 70 cm2
top is time as thick as the plate forming the (c) 71.25 cm2 (d) 72. 25 cm2
4 7. Consider the following statements in respect of
cylindrical sides. The ratio of the radius to the the function [2016]
height of the cylinder using minimum material
for the same capacity is [2017] f (x) = x3 – 1, x Î [ -1, 1]
2 1 4 1 I. f (x) is increasing in [– 1, 1]
(a) (b) (c) (d) II. f (x) has no root in (– 1, 1).
3 2 5 3
3. The set of all values of a for which the function Which of the statements given above is/are
f(x) = (a2 – 3a + 2) (cos2x/4 –sin2x/4) + (a –1) x + correct?
sin 1does not possess critical points is [2016] (a) Only I (b) Only II
(a) [1, ¥) (b) (0, 1) È (1, 4) (c) Both I and II (d) Neither I nor II
(c) (–2, 4) (d) (1, 3) È (3, 5) 8. At an extreme point of a function f (x), the tangent
4. Match List I with List II and select the correct to the curve is [2016]
answer using the code given below the lists: (a) parallel to the x-axis
[2016] (b) perpendicular to the x-axis
List I List II (c) inclined at an angle 45° to the x-axis
(A) f (x) = cos x 1. The graph cuts y-axis in (d) inclined at an angle 60° to the x-axis
infinitenumber of points 9. The curve y = xex has minimum value equal to
(B) f (x) = ln x 2. The graph cuts x - axis [2016]
in two points 1 1
(a) - (b)
(C) f (x) = 3. The graph cuts y-axis e e
x2 – 5x + 4 (c) – e (d) e
in only one point 10. The line which is parallel to X-axis and crosses
(D) f (x) = ex 4. The graph cuts x-axis i n the curve y = x at an angle of 45°, is [2015]
only one point 1 1
5. The graph cuts x-axis in (a) x = (b) y =
infinite number of points 4 4
1
Codes: (c) y = (d) y = 1
(A) (B) (C) (D) 2
11. Tangents are drawn from the origin to the curve
(a) 1 4 5 3
y = cos x. Their points of contact lie on [2015]
(b) 1 3 5 4
(a) x2y2 = y2 – x2 (b) x2y2 = x2 + y2
(c) 5 4 2 3
(c) x2y2 = x2 – y2 (d) None of these
(d) 5 3 2 4
12. The slope of the tangent to the curve y = ex cos
5. What is the x-coordinate of the point on the curve
x is minimum at x = a, 0 £ a £ 2p, then the value of
f (x) = x (7x – 6), where the tangent is parallel a is [2015]
to x-axis? [2016, 2008] (a) 0 (b) p (c) 2p (d) 3p/2
1 2 x 2
13. The function f ( x ) = + has a local minimum
(a) – (b) 2 x
3 7 at [2015]
EBD_7167
http://t.me/iitjeehelps

M – 68 BITSAT Topicwise Solved Papers


(a) x = 2 (b) x = –2 23. If the normal to the curve y = f (x) at the point (3,
(c) x = 0 (d) x = 1 4) makes an angle 3p/4 with the positive x-axis,
14. If a and b are non-zero roots of x2 + ax + b = 0 then f ' (3) = [2012]
then the least value of x2 + ax + b is [2015] (a) –1 (b) – 3/4 (c) 4/3 (d) 1
2 9 24. The function f(x) = sin x – kx – c, where k and
(a) (b) – c are constants, decreases always when
3 4
9 [2011]
(c) (d) 1 (a) k > 1 (b) k ³ 1 (c) k < 1 (d) k £ 1
4
p 25. The minimum value of f (x) = sin4 x + cos4 x in
15. If 0 < x < , then [2015, 2010]
2 p
(a) tan x < x < sin x (b) x < sin x < tan x the interval æç 0, ö÷ is [2011]
(c) sin x < tan x < x (d) None of these è 2ø
16. The interval in which the function 2x3 + 15 1
increases less rapidly than the function 9x2 – (a) (b) 2 (c) 2 (d) 1
2
12x, is – [2014, 2011] 26. xy
The curve y –e + x = 0 has a vertical tangent at
(a) (–¥, 1) (b) (1, 2) [2011]
(c) (2, ¥) (d) None of these (a) (1, 1) (b) (0, 1)
17. The fuel charges for running a train are (c) (1, 0) (d) no point
proportional to th e square of the speed 27. The function f(x) = 2x3 – 3x2 – 12x + 4, has
generated in miles per hour and costs [2011]
` 48 per hour at 16 miles per hour. The most (a) two points of local maximum
economical speed if the fixed charges i.e. salaries (b) two points of local minimum
etc. amount to ` 300 per hour is [2014] (c) one maxima and one minima
(a) 10 (b) 20 (c) 30 (d) 40 (d) no maxima or minima
18. The equation of all lines having slope 2 which 28. If a circular plate is heated uniformly, its area
1 expands 3c times as fast as its radius, then the
are tangent to the curve y = , x ¹ 3 , is value of c when the radius is 6 units, is
x -3
[2013, 2009] [2010]
(a) y = 2 (b) y = 2x (a) 4 p (b) 2 p (c) 6 p (d) 3 p
(c) y = 2x + 3 (d) None of these 29. The function f(x) = tan x – 4x is strictly decreasing
19. The function f (x) = (x(x–2))2 is increasing in the on [2010, 2006]
set [2013]
(a) (– ¥, 0) È (2, ¥) (b) (– ¥, 1) æ p pö æ p pö
(a) ç- , ÷ (b) ç , ÷
(c) (0, 1) È (2, ¥) (d) (1, 2) è 3 3ø è3 2ø
20. If a2 x4 + b2 y4 = c4, then the maximum value of æ p pö æp ö
xy is [2013] (c) ç - , ÷ (d) ç , p ÷
c c2 è 3 2ø è2 ø
(a) (b) 30. The slope of the tangent to the hyperbola
ab 2 ab 2x2 – 3y2 = 6 at (3, 2) is [2010]
c c2 (a) –1 (b) 1 (c) 0 (d) 2
(c) (d)
2 ab 2ab 31. The minimum value of the function y = x4 – 2x2
é1 ù
a 2 -1 + 1 in the interval ê , 2ú is [2009]
21. If f ( x ) = x 3 - 3x + 5 is a decreasing ë2 û
2
a +1 (a) 0 (b) 2 (c) 8 (d) 9
function of x in R then the set of possible values 32. The value of a in order that f (x) = sin x – cos x –
of a (independent of x) is [2012] ax + b decreases for all real values is given by
(a) (1, ¥) (b) (-¥, - 1) [2009]
(a) a ³ 2 (b) a < 2
(c) [-1, 1] (d) None of these (c) a ³ 1 (d) a < 1
22. The diagonal of a square is changing at the rate 33. The equation of tangent to the curve y = sin x at
of 0.5 cm/sec. Then the rate of change of area, the point (p, 0) is [2009]
when the area is 400 cm2, is equal to[2012] (a) x + y = 0 (b) x + y = p
(a) 20 2 cm 2 / sec (b) 10 2 cm 2 / sec (c) x – y = p (d) x – y = 0
1 10 34. If f(x) = cos x, then [2008]
(c) cm2 / sec (d) cm 2 / sec (a) f(x) is strictly decreasing in (0, p)
10 2 2
http://t.me/iitjeehelps

Application of Derivatives M – 69

(b) f(x) is strictly increasing in (0, 2p) 40. For all values of x, function f (x) = 2x3 + 6x2 + 7x
(c) f(x) is neither increasing nor decreasing in – 19 is [2006]
(p, 2p) (a) Monotonic increasing
(d) All the above are correct (b) Monotonic decreasing
35. The greatest value of f (x) = cos (xe[x] + 7x2 – 3x), (c) Not monotonic
x Î [–1, ¥) is – [2008] (d) None of these
(a) –1 (b) 1 log(p + x )
(c) 0 (d) None of these 41. f (x) = is [2005]
36. The function f (x) = cot –1 x + x increases in the log(e + x )
interval [2007] (a) increasing in [0, ¥)
(a) (1, ¥) (b) (–1, ¥) (c) (–¥, ¥) (d) (0, ¥) (b) decreasing in [0, ¥)
37. The total revenue in rupees received from the é pù
sale of x units of a product is given by R(x) (c) decreasing in ê0, ú & increasing in
= 13x2 + 26x + 15. Then the marginal revenue in ë eû
ép ö
rupees, when x = 15 is [2007] ,
êe ÷ ¥
(a) 116 (b) 126 (c) 136 (d) 416 ë ø
38. If at any point S of the curve by2 = (x + a)3, the é pù
(d) increasing in ê0, e ú & decreasing in
relation between subnormal SN and subtangent ë û
ST be p (SN) = q (ST)2 then p/q is equal to ép ö
[2007] ê e ,¥÷
ë ø
(a) 8b/27 (b) 8a/27 42. The largest value of y = 2x3 – 3x2 – 12x + 5 for
(c) b/a (d) None of these – 2 £ x £ 2 occurs at x is equal to : [2005]
39. The equation of one of the tangents to the curve (a) –2 (b) –1 (c) 2 (d) 4
y = cos(x +y), -2p £ x £ 2p that is parallel to 43. Function f(x) = cos x – 2lx is monotonic
the line decreasing when [2005]
x + 2y = 0, is [2006] (a) l > 1/2 (b) l < 1/2
(a) x + 2y = 1 (b) x + 2y = p/2 (c) l < 2 (d) l > 2
(c) x + 2y = p/4 (d) none of these

Hints & Solutions


1. (b) A function f(x) is said to be increasing
function in d 2S æ 4V 5 ö 15
=kç + p ÷ , = kp = positive
[a, b] if f '(x) > 0 in [a, b]. dr 2
è r3 2 ø 2
Given f (x) = xx .... (i) r 4
Differentiate equation (i) When r3 = 4V/5p or 5pr3 = 4pr2h.\ = .
h 5
f '(x) = xx (1 + log x) 3. (b) f(x) = (a2 – 3a + 2) (cos2x/4 – sin2x/4) + (a –
Put f '(x) = 0; 0 = xx (1 + log x) 1) x + sin 1
-1 1 Þ f(x) = (a –1) (a –2) cos x/2 + (a –1) x +sin1
Þ x = 0, log x = – 1 Þ x = e Þ x = , 0
e 1 x
é 1ù Þ f '(x) = – (a –1) (a – 2) sin + (a –1)
Now, in ê 0, e ú , f '(x ) > 0 2 2
ë û é ( a - 2) xù
é 1ù Þ f ' ( x ) = (a - 1) ê1 - sin ú
\ f (x) is increasing in interval ê 0, ú ë 2 2û
ë eû If f(x) does not possess critical points, then
2. (c) V = pr2h = constant. If k be the thickness of f’(x) ¹ 0 for any x Î R
the sides then that of the top will be (5/4)k. é ( a - 2) xù
\ S = (2prh)k + (pr2). (5/4)k Þ (a - 1) ê1 - sin ú ¹ 0 for any x Î R
ë 2 2û
(‘S’ is vol. of material used)
V 5 2 æ 2V 5 2 ö æa -2ö x
or S = 2prk. + pr k = kç + pr ÷ Þ a ¹ 1 and 1 - ç ÷ sin = 0
pr 2 4 è r 4 ø è 2 ø 2
dS æ 2V 5 ö must not have any solution in R.
\ = k çç - + pr ÷÷, \r 3 = 4 V / 5p
dr 2 2 ø
è r
EBD_7167
http://t.me/iitjeehelps

M – 70 BITSAT Topicwise Solved Papers


x 2 2
d y
Þ a ¹1 and sin = is not solvable Now, = e x + e x (1 + x ) = e x ( x + 2 )
2 a -2 dx 2
in R
2 æ d2 y ö 1
Þ a ¹ 1 and >1 ç 2÷ = +0 > 0
a-2 ç dx ÷
è ø( x =-1) e
éFor a = 2, f(x) = x + sin1ù
êë \ f' (x) = 1 ¹ 0 úû Hence, y = xex is minimum function and
Þ a ¹1 and | a – 2| < 2 Þ a ¹ 1 and –2 < a –2 < 2 1
y min = - .
Þ a ¹1 and 0 < a < 4 Þ a Î (0, 1) È (1,4). e
4. (c) (A) Graph of f(x) = cos x cuts x-axis at 10. (c) Given equation of a line parallel to X-axis
infinite number of points. (5 of list II) is y = k.
(B) Graph of f(x) = In x cuts x-axis in only Given equation of the curve is y = x ,
one point. (4 of list II) On solving equation of line with the
(C) Graph of f(x) = x2 – 5x + 4 cuts x axis in equation of curve, we get x = k2
two points (2 of list II) Thus the intersecting point is (k2, k)
(D) Graph of f(x) = ex cuts y-axis in only It is given that the line y = k intersect the
one point. (3 of list II)
curve y = x at an angle of p/4. This
5. (b) f(x) = x (7x – 6) = 7x3/2 – 6x1/2 means that the slope of the tangent to
3 1 æ pö
f '(x) = 7 ´ x1 / 2 - 6 ´ x -1 / 2
2 2 y= x at (k2, k) is tan ç ± 4 ÷ = ±1
When tangent is parallel to x axis f '(x) = 0 è ø
21 1/ 2 21 3 æ dy ö æ 1 ö
or, x - 3x -1/ 2 = 0 or 2 x = Þç ÷ = ±1 Þ ç ÷ = ±1
2 x è dx ø ( k ,k )
2 è2 x ø ( k2 ,k )
2 1
or, 7x = 2 Þ x = Þ k=±
7 2
6. (d) Let one side of quadrilateral be x and
another side be y 11. (c) Let (x1, y1) be one of the points of contact.
so, 2(x + y) = 34 Given curve is y = cos x
or, (x + y) = 17 ...(i) dy dy
Þ = - sin x Þ = - sin x1
We know from the basic principle that for dx dx ( x ,y )
a given perimeter square has the maximum 1 1
area, so, x = y and putting this value in
equation (i) Now the equation of the tangent at (x1,y1) is
17 æ dy ö
x=y=
2
y - y1 ç ÷ ( x - x1 )
è dx ø( x ,y )
17 17 289 1 1
Area = x . y = ´ = = 72.25
2 2 4 Þ y - y1 = - sin x1 ( 0 - x1 )
7. (a) Since f (x) is an increasing function in [– 1, Since, it is given that equation of tangent
1] and it has a root in (– 1, 1). passes through origin.
\ Only statement I is correct.
8. (a) At an extreme point of a function f (x), slope \ 0 - y1 = - sin x1 ( 0 - x1 )
is always zero. Þ y1 = – x1 sin x1 ...(i)
Thus, At an extreme point of a function f (x), Also, point (x1, y1) lies on y = cos x.
the tangent to the curve is parallel to the x- \ y1 = cos x1
axis. From Eqs. (i), (ii) , we get
9. (a) Let y = xex.
Differentiate both side w.r.t. ‘x’. y12
sin 2 x1 + cos 2 x1 = + y12 = 1
dy x12
Þ = e x + xe x = e x (1 + x )
dx
dy Þ x12 = y12 + y12 x12
Put = 0 Þ ex (1 + x) = 0 Þ x = – 1 Hence, the locus of (x1, y1) is
dx
x2 = y2 + y2x2 Þ x2y2 = x2 – y2
http://t.me/iitjeehelps

Application of Derivatives M – 71

12. (b) Let m be the slope of the tangent to the Þ tan x – x > 0 and x – sin x > 0,
curve: y = ex cos x.
æ pö
dy "x Î ç 0, ÷
Then, m = = e x ( cos x - sin x ) è 2ø
dx
Diff. w.r.t ‘x’ æ pö
Þ tan x > x and x > sin x, "x Î ç 0, ÷
dm è 2ø
Þ = e x ( cos x - sin x )
dx æ pö
Þ sin x < x < tan x, "x Î ç 0, ÷
+ e x ( - sin x - cos x ) = -2e x sin x è 2ø
16. (b) Let f (x) = 2x3 + 15 and g (x) = 9x2 – 12x then
d2m f ' (x) = 6x2 x Î R
and = -2e x ( sin x + cos x )
2 \ f (x) is increasing function x Î R
dx
dm 2
Put = 0 Þ sin x = 0 Þ x = 0, p, 2p Also, g ' (x) > 0 Þ 18x – 12 > 0 Þ x >
dx 3
d2 m 2
Clearly, > 0 for x = p Thus, f (x) and g (x) both increases for x >
dx 2 3
Thus, y is minimum at x = p. Let F (x) = f (x) – g (x) , F ' (x) < 0
Hence the value of a = p. (Q f (x) increases less rapidly than the
function g (x))
x 2 1 Þ 6x2 – 18x + 12 < 0 Þ 1 < x < 2
13. (d) + is of the form x + ³ 2 and
2 x x 17. (d) Let the speed of the train be v and distance
equality holds for x = 1 to be covered be s so that total time taken is
14. (b) As given a and b are the roots of the s/v hours. Cost of fuel per hour = kv2 (k is
equation: x2 + ax + b = 0 constant)
Þ sum of roots, a + b = – a 3
Þ b = – 2a ...(1) Also 48 = k. 162 by given condition \ k =
16
and product of roots, ab = b 3 2
Þ ab – b = 0 Þ b (a – 1) = 0 \ Cost to fuel per hour v .
16
if b = 0 then a = 0; if b ¹ 0 then a = 1 and b = – 2 Other charges per hour are 300.
so, the expression will be; f (x) = x2 + x – 2 Total running cost ,
2 2
1 æ1ö æ1ö æ 3 ös 3s 300 s
= x2 +2. x + ç ÷ - ç ÷ - 2 C = ç v 2 + 300 ÷ = v +
2 è2ø è2ø è 16 øv 16 v
2
æ 1ö 9 dC 3s 300 s
Þ f (x) = ç x + ÷ - = - 2 = 0 Þ v = 40
è 2ø 4 dv 16
2
v
æ 1ö
So, f (x) will be minimum, if ç x + ÷ = 0 d 2C 600 s
è 2ø = > 0 \ v = 40 results in
2
1 dv v3
i.e. when x = - minimum running cost
2
9 18. (d) The equation of the given curve is
Þ minimum value of function = - 1
4 y= , x ¹ 3.
15. (d) Let us assume the functions f(x) and g(x) x -3
given by The slope of the tangent to the given curve
f(x) = tanx – x and g(x) = x – sinx, for
dy -1
p at any point (x, y) is given by =
0<x< dx ( x - 3) 2
2
Now, f¢(x) = sec2x – 1 and g¢(x) = 1 – cos x For tangent having slope 2, we must have
æ pö -1
Þ f¢(x) > 0 and g¢(x) > 0, "x Î ç 0, ÷ 2= 1
è 2ø 2 Þ 2(x – 3)2 = – 1Þ (x – 3)2 = -
( x - 3) 2
æ pö
Þ f(x) > f(0) and g(x) > g(0) " x Î ç 0, ÷ which is not possible as square of a real
è 2ø number cannot be negative.
EBD_7167
http://t.me/iitjeehelps

M – 72 BITSAT Topicwise Solved Papers


Hence, there is no tangent to the given curve 25. (a) Let y = sin4x + cos4x
having slope 2. dy
19. (c) Here, f(x) = (x(x – 2))2 = 4 sin 3 x cos x + 4 cos 3 x (- sin x )
dx
Þ f ¢(x) = 4x (x – 2) (x – 1)
For f(x) as increasing, f ¢(x) > 0 = 4 sin xcox (sin 2 x - cos 2 x)
So, 4x (x – 1) (x – 2) > 0 Þ x(x – 1) (x – 2) > 0 = (2 sin 2x) (– cos 2x) = – sin 4x
dy
\ = 0 Þ sin 4 x = 0 Þ 4 x = 0, p, 2p, 3p
dx
p p 3p p
From the above figure required interval is, or x = 0, , , ,..........., Þ x =
(0, 1) È (2, ¥) 4 2 4 4
20. (d) If the sum of two positive quantities is a 26. (c) y – exy + x = 0
constant, then their product is maximum, when dy xy æ dy ö
they are equal . \ - e ç y + x ÷ +1 = 0
\ a2 x4. b2y2 is maximum when dx è dx ø
dy dy
1 2 4 1 2 4 2 4 c4 i.e., - y(x + y) - x(x + y) + 1 = 0
a2 x4 = b2y4 = (a x = (a x + b y ) = dx dx
2 2 2 dy
\ maximum value of i.e., [1 – x (x + y)] = y (x + y) – 1
c4 c4 c8 dx
a2 x4. b2y4 = . = for the vertical tangents
2 2 4 1 - x2
æ c8 ö
1/4 1 – x (x + y) = 0 i.e., y=
c2 x
Maximum value of xy = ç 2 2 ÷ = \ x = 1 and y = 0
è 4a b ø 2ab 27. 3 2
(c) f(x) = 2x – 3x – 12x + 4
æ a 2 -1 ö 2 Þ f ¢(x) = 6x2 – 6x – 12 = 6(x2 – x – 2)
21. (c) f ' ( x ) = 3çç 2 ÷÷ x - 3 = 6(x – 2) (x + 1)
è a + 1ø For maxima and minima f ¢(x) = 0
\ 6(x – 2)(x + 1) = 0 Þ x = 2, – 1
f ' ( x) < 0 for all x if a 2 - 1 £ 0 Þ -1 £ a £ 1
Now, f ¢¢ ( x ) = 12x - 6
22. (b) Diagonal D = 2.a
dD da At x = 2; f ¢¢ ( x ) = 24 - 6 = 18 > 0
Differentiating w.r.t. t: = 2
dt at \ x = 2 , local min. point
da
=
1 da
=
1
´ 0.5 cm / s At x = – 1; f ¢¢ ( x ) = 12 ( -1) - 6 = -18 < 0
or
dt 2 dt 2 \ x = –1 local max. point
Let Area is denoted by A 28. (a) Let A sq. units in the area measure when
dA da the radius is r units.
= 2a ...(i) their A = pr2
dt dt Differentiate both side w.r.t ‘t’
2
when area A is 400 cm then a = 20 dA dr
dA 0.5 = 2pr ...(i)
\ = 2 ´ 20 ´ = 10 2 cm 2 / sec dt dt
dt 2 dA dr
-1 We have, = 3c
23. (d) Slope of normal to y = f(x) at (3, 4) is . dt dt
f '(3) From eqn (i), we get
-1 æ 3p ö æp pö dr dr
Thus, = tan ç ÷ = tan ç + ÷ 3 c. = 2pr. Þ 3c = 2pr
f '(3) è ø4 è2 4ø dt dt
p 2
= – cot = – 1 Þ f ¢ (3) = 1. Now, c = p (6) = 4p when r = 6
4 3
29. (a) f(x) = tanx – 4x Þ f ¢(x) = sec2x – 4
24. (b) Let f (x) = sin x – kx – c where k and c are
constants. f '(x) = cos x – k -p p
When < x < , 1 < sec x < 2
\ f decreases if cos x £ k 3 3
Thus, f (x) = sin x – kx – c decrease always Therefore, 1 < sec2x < 4
when k > 1.
http://t.me/iitjeehelps

Application of Derivatives M – 73

Þ – 3 < (sec2x – 4) < 0 2


-p p æ dy ö
< x < , f ¢(x) < 0 y/
Thus, for
3 3 dy 3(x + a) p çè dx ÷ø
2
y 8b
= Þ = = 3
=
æ -p p ö dx 2by q æ dy ö æ dy ö 27
Hence, f is strictly decreasing on ç , ÷ yç ÷
è dx ø çè ÷ø
è 3 3ø dx
30. (b) Differentiating the given equation of the curve
39. (b) y = cos( x + y) …(1)
4x – 6y. (dy/dx) = 0 \ dy/dx = 2x / 3y
æ dy ö dy ì dy ü
2 3 \ = - sin( x + y ) í1 + ý
çè dx ÷ø = . =1 dx î dx þ
(3, 2) 3 2
dy d 4 dy sin( x + y) 1
31. (a) = (x - 2x 2 + 1) = 4x (x2 – 1) \ =- =-
dx dx dx 1 + sin( x + y ) 2
dy Þ sin(x + y) = 1 , so cos( x + y) = 0
For max. or min, =0
dx p
\ from (1) y = 0 and ( x + y) = 2np +
4x (x – 1) = 0; either x = 0 or x = ± 1
2
2
é1 ù æp ö p
x = 0 and x = – 1 does not belong to ê , 2ú Tangent at ç , 0 ÷ is x + 2 y =
ë2 û è2 ø 2
d2 y æ d 2 ö
y 40. (a) Qf ' (x) = 6x2 + 12x + 7 = 6 (x2 +2x) + 7
= 12x 2 - 4 \ ç ÷ = - = > = 6 (x +1)2 + 1
dx 2 ç 2 ÷
è dx øx = 1 which is positive for all value of x. Hence
= 12(1)2 – 4 = 8 > 0 f(x) is monotonic increasing function.
\ there is minimum value of function at x = 1 41. (b) We have e < p and
\ minimum value is 1 1
log( e + x ) - log( p + x )
y(1) = 14 – 2(1)2 + 1 = 1 – 2 + 1 = 0
32. (a) We have ; f (x) = sin x – cos x – ax + b f ¢( x ) = p + x e+ x
{log(e + x )}2
Þ f ' (x) = cos x + sin x – a
Þ f ' (x) < 0 x Î R (e ∗ x) log(e ∗ x) , (p ∗ x) log(p ∗ x)
<
Þ (cos x + sin x) < a x Î R (p ∗ x)(e ∗ x){log (e ∗ x)}2
As the max. value of (cos x + sin x) is 2 In [0, ¥), denominator > 0 and numerator <
The above is possible when a ³ 2 0, since, e + x < p + x. Hence, f(x) is decreasing
in [0, ¥).
dy æ dy ö 42. (b) Given equation : y = 2x3 – 3x2 – 12x + 5 ...(i)
33. (b) y = sin x Þ = cos x Þ ç ÷ = –1
dx è dx ø ( p, 0) Differentiate equ. (i),
dy
Therefore the equation of tangent at ( p, 0) = 6x 2 – 6x – 12 ...(ii)
is given by dx
y – 0 = –1 (x – p) Þ x + y = p æ dy ö
34. (a) We have, f(x) = cosx For max. value, put ç ÷ = 0
è dx ø
f ¢(x) = – sinx 0 = 6x2 –6x – 12 Þ x2 – x – 2 = 0
(a) Since, for each x Î (0, p), sinx > 0 we have (x – 2 ) (x + 1) = 0 Þ x = – 1, 2
f ¢(x) < 0 and so f is strictly decreasing in (0, p). Again differentiating equ. (ii), we get
(b) Since, for each x Î (p, 2p), sinx < 0 we
have f ¢(x) > 0 and so f is strictly decreasing d2 y
= 12x – 6
in (0, 2p). dx 2
(c) Clearly, by (a) and (b) above, f is neither
increasing nor decreasing in (0, 2p). d2 y
at x = – 1 Þ = 12( -1) – 6
35. (b) At x = 0: f (x) = cos 0 = 1 ® maximum value dx 2
1 x2 d2 y d2 y
36. (c) f ¢(x) = - +1
2f ¢ (x) = Þ f ¢ (x) ³ 0 Þ = -12 – 6 Þ = –18(–ve)
1+ x 1+ x2 dx 2 dx 2
Þ Always increasing
37. (d) Here, R(x) = 13x2 + 26x + 15 Therefore, y is max at x = –1.
Þ R¢(x) = 26x + 26 So, R¢(15) = 416 43. (a) f (x) is monotonic decreasing when
38. (a) f ' (x) < 0 x Þ sinx – 2l < 0
Þ 2l > – sin x Þ 2l > 1 Þ l > 1/2
EBD_7167
http://t.me/iitjeehelps

M – 74 BITSAT Topicwise Solved Papers

Chapter
Integrals
22
cos x - 1 10
x10
ò sin x + 1 e
x
dx is equal to :
1. If [2017, 2006] 6.
ò (10 - x)10 + x10 dx is equal to [2014]
e x cos x e x sin x
0
(a) +C (b) C - 1
1 + sin x 1 + sin x (a) 10 (b) 5 (c) 2 (d)
2
ex e x cos x (x 2 - 1)
(c) C- (d) C -
1 + sin x 1 + sin x 7. ò dx is equal to [2013, 2008]
(x 2 + 1) x 4 + 1
If ò x log æç1+ ö÷ dx = f (x) log(x +1) + g(x)x 2 + Lx + c,
1
2.
è xø æ x 2 + 1ö 1 æ x2 +1ö
then [2017] (a) sec -1 ç ÷ + c (b) sec-1 ç ÷ +c
è 2x ø 2 è 2x ø
1 2
(a) f (x) = x (b) g(x) = log x 1 æ x2 +1ö
2 (c) sec-1 ç ÷ + c (d) None of these
(c) L = 1 (d) None of these 2 è 2 ø
2x 2 p /2
ò0 xe x dx sin x
3. lim 2
equals [2015, 2013] 8. Evaluate ò 1 + cos2 x
dx [2013]
x ®¥ 4x 0
e (a) p/2 (b) p/4 (c) p/3 (d) p
1 1
(a) 0 (b) ¥ (c) 2 (d)
2 9. ò dx = [2012]
4. Let f(x) be a polynomial of degree three satisfying
x x2 - 1
(a) cos–1 x + c (b) sec–1 x + c
f(0) = – 1 and f(1) = 0. Also, 0 is a stationary point
(c) cot–1 x + c (d) tan–1 x + c
of f(x). If f(x) does not have an extremum at x = 0,
f (x) x
then the value of ò 3 dx is [2015] 10. Evaluate: ò 4 - x3
dx [2012]
2 x -1
x
(a) +C (b) x + C 2 -1 æ x 3/ 2 ö
2 (a)
3
sin ç
è 2 ø
÷ +c (b)
3
sin x(
2 -1 3 / 2
+c )
x3
(c) +C (d) None of these
6 æ x3 / 2 ö 1 -1 æ x3 / 2 ö
(c) 2sin -1 ç ÷ + c (d) sin ç ÷ +c
1 è 2 ø 3 è 2 ø
5. Evaluate: ò 1 + 3sin 2 x + 8 cos 2 x dx [2014] p /2
2sin x
11. ò 2sin x + 2cos x dx equals [2012]
1 0
(a) tan -1 ( 2 tan x ) + C (a) 2 (b) p (c) p/4 (d) p/2
6
x2
(b) tan -1 ( 2 tan x ) + C 12. Evaluate ò x 2 - 1 dx [2011]
1 æ 2 tan x ö
(c) tan -1 ç ÷+C 1 æ x - 1ö
x - log ç
1 æ x + 1ö
+ c (b) x + log ç
6 è 3 ø (a)
è x + 1÷ø ÷ +c
2 2 è x -1ø
(d) None of these 1 æ x - 1ö
(c) x + log ç + c (d) None of these
2 è x + 1÷ø
http://t.me/iitjeehelps

Integrals M – 75

4p 1 2 1
(c) (x - 1) tan -1 x - x + c
13. Find the value of ò | sin x | dx [2011] 2 2
0 (d) None of these
(a) 8 (b) 6 (c) 4 (d) 2 1
dx
2
1 1
2 19. Evaluate : ò . [2009]
2 - x2
14. Let I1 = ò dx and I2 = ò dx , then 0

1+ x2 1
x (a) p/4 (b) p (c) p/2 (d) p/3
1

z
[2011] n

(a) I1 > I2 (b) I2 > I1 20. If [ x] dx = 66, then n = [2009]


(c) I1= I2 (d) None of these 0

(a) 24 (b) 9 (c) 12 (d) 7


æ pö æ 5p ö
15. ò 4 cos çè x + 6 ÷ø cos 2x.cos çè 6 + x ÷ø dx [2010] x + sin x
21. ò 1 + cos x dx = [2008]
æ sin 4x sin 2x ö
x x
(a) -çx+ + ÷ +c
è 4 2 ø (a) cot +c (b) x tan +c
2 2
æ sin 4x sin 2x ö x
(b) - çx + - ÷ +c (c) log(1+cos x) + c (d) x tan + tan x + c
è 4 2 ø 2
æ sin 4x sin 2x ö
¥
(c) -çx- + ÷ +c
è 4 2 ø dx
æ sin 4x cos 2x ö 22. ò 1 + ex equals [2008]
(d) - çx - + ÷+c 0
è 4 2 ø
(a) log 2 – 1 (b) log 2
e (c) log 4 – 1 (d) – log 2
16. If Im = ò (ln x)m dx , where m Î N, then
1 2 1 3 2 2
1 x x x
23. If I1 = ò 2 dx , I 2 = ò 2 dx , I 3 = ò 2 dx
I10 + 10 I9 is equal to - [2010] 0 0 1
2 3
e10 x
and I 4 = ò 2 dx then [2007]
(a) e10 (b) (c) e (d) e – 1 1
10
(a) I 2 > I1 (b) I1 > I 2
2 cos x - sin x + l
17. If ò cos x + sin x - 2 dx (c) I3 = I4 (d) I 3 > I 4
dx
= A ln | cos x + sin x - 2 | + Bx + C . 24. ò 1 - sin x = [2007]
Then the ordered triplet A, B, l is – [2009] (a) x + cos x + C (b) 1 + sin x + C
æ1 3 ö æ3 1 ö (c) sec x – tan x + C (d) sec x + tan x + C
(a) çè , , -1÷ø (b) çè , , -1÷ø
2 2 2 2
¥
dx
æ1 3ö
çè , -1, ÷ø (d)
æ3 1ö
çè , -1, ÷ø
25. ò is equal to [2007]
(a )
(c) 3
2
2 2 2 2 a + x2

( 3p - 8)
ò x tan 3p
-1
18. Evaluate: x dx [2009]
(a) (b)
32a 5
16a 5
1 2 1
(a) (x + 1) tan -1 x - x + c 3p 3p- 4
2 2 (c) 5 (d)
32a 16a5
1 2 1
(b) (x + 1) tan -1 x + x + c
2 2
EBD_7167
http://t.me/iitjeehelps

M – 76 BITSAT Topicwise Solved Papers

1 p/2
26. ò dx = [2006]
ò f (x)dx is equal to [2005]
(x + 1) x 2 - 1
0
1 1 1
x +1 x2 - 1 (a) (b) – (c) (d) 1
(a) +C (b) +C 4 3 2
x -1 x +1
1
x -1 x -1 29. The value of ò sin 2 x cos2 x dx is – [2005]
(c) +C (d) +C
x +1 2
x +1 (a) tan x – cot x + C
27. The value of integral (b) tan x + cot x + C
(c) – tan x – cot x + C
p /2
f(x) (d) None of these
ò f(x) + f(p / 2 - x)
dx is [2006] 2
0 x
(a) p/4 (b) p/2 30. Evaluate ò 3- x + x
dx [2005]
1
(c) p (d) None of these
(a) 1 (b) 0 (c) 1/2 (d) 2
cos x 1 0
28. If f ( x ) = 1 2 cos x 1 , then
0 1 2 cos x

Hints & Solutions


cos x - 1 x2
1. (a) Let, I =
ò sin x + 1 ex dx =
2
æ x + 1 ö 1 x + 1 -1
log ç ÷+ ò
è x ø 2 x +1
dx

cos x 1
= ò 1 + sin x ex dx – ò sin x + 1 ex dx =
x2 æ x +1 ö 1
logç
1
÷ + x - log( x + 1) + c
2 è x ø 2 2
e x .cosx -(1 + sin x) sin x - cos 2 x
=
1 + sin x
- ò
(1 + sin x)2
æ x 2 -1 ö

2
÷ log( x + 1) - x log x + 1 x + c
ç 2 ÷ 2 2
è ø
ex
exdx–
ò sin x + 1
dx 2x
ò0
2
xe x dx
3. (d) Consider lim 2
e x cos x ex e x dx x ®¥
e4x
= + ò
dx - ò
1 + sin x
e x cos x
1 + sin x 1 + sin x

= lim

0
2x 2
xe x dx
= lim

2x x 2
0
e d x2 ( )
= +C x ®¥
2
4x 2
1 + sin x 2 e 4x x ®¥ 2e
æ 1ö 2
ò
2
2. (d) x log ç1 + ÷dx [e x ]02x e 4x - 1
è xø = lim 2
= lim 2
x ®¥ e 4x x ®¥ 2e 4x
2 2
æ 1ö x x æ 1 ö x
= logç1 + ÷.
è xø 2
- ×ç- ÷×
x + 1 çè x 2 ÷ø 2
dx ò æ1
= lim ç -
1 ö 1
÷=
x ®¥ çè 2 2e4x ÷ 2
2
ø
http://t.me/iitjeehelps

Integrals M – 77

4. (b) Let f(x) = ax3 + bx2 + cx + d


Put x = 0 and x = 1 æ 1ö
x 2 ç1 - 2 ÷ dx
Then, we get f(0) = –1 and f(1) = 0 è x ø
7. (b) I= ò
Þ d = – 1 and a + b + c + d = 0 æ 1ö æ 1ö
1/2
Þ a+ b+ c= 1 ...(i) x2 ç x + ÷ ç x2 + 2 ÷
è x ø è x ø
It is given that x = 0 is a stationary point of f(x),
but it is not a point of extremum.
1 æ 1ö
Therefore, f¢(0) = 0 = f ¢¢ ( 0 ) and f ¢¢¢ ( 0 ) = 0 Let x + = p Þ ç1 - 2 ÷ dx = dp
x è x ø
Now, f(x) = ax3 + bx2 + cx + d
dp 1 p
Þ f¢(x) = 3ax2 + 2bx + c, f ¢¢ ( x ) = 6ax + 2b and I= ò = sec-1
p p2 - 2 2 2
f ¢¢¢ ( x ) = 6a f¢ = 0, f ¢¢ ( 0 ) = 0 and f ¢¢¢ ( 0 ) = 0 ¹ 0
Þ c = 0, b = 0 and a ¹ 0 1 æ x 2 + 1ö
= sec -1 ç ÷ +c
From Eqs. (i) and (ii), we get 2 è 2x ø
a = 1, b = c = 0 and d = – 1
Put these values in f(x); we get f(x) = x3 – 1 p /2
sin x
Hence, 8. (b) Let I = ò 1 + cos2 x
dx
0
f (x) x3 -1
ò x3 - 1 dx = ò x 3 - 1 dx = ò 1dx = x + C Let cos x = t and – sin x dx = dt.
Now, x = 0 Þ t = cos 0 = 1 and
1
5. (c) I = ò 1 + 3sin 2 x + 8cos 2 x dx x=
p p
Þ t = cos = 0
2 2
Dividing the numerator and denominator by
cos2x, we get 0
sin x æ -dt ö dt
0

\ I= ò 2ç
è sin x
÷
ø = -ò 2
sec 2 x sec2 x 1 1+ t 1 1+ t
I=ò dx = ò dx
sec 2 x + 3 tan 2 x + 8 4tan 2 x +9
-1
0 é pù p
Putting tan x = t Þ sec2x dx = dt, we get = éë tan t ùû = - ê0 - ú =
1 ë 4û 4
dt 1 dt 1 1 æ t ö
I =ò =ò = ´ tan -1ç ÷+ C
2 2
4t + 9 4 t + (3/2) 4 3/ 2
2 è 3/ 2 ø 1
9. (b) ò dx = sec-1 x + c
2
x x -1
1 æ 2t ö 1 -1 æ 2tan x ö
Þ I = tan-1 ç ÷ + C = tan çè ÷ +C
6 è 3ø 6 3 ø x x dx
10. (a) I = ò dx = ò
10 4 - x3 4 - x3
x10
6. (b) Let I = ò (10 – x)10 + x10 dx ...(1)
2 3/ 2
0 Here integral of x= x and
3
10 4 – x3 = 4 – (x3/2)2
(10 – x)10
I= ò (10 – x)10 + x10 dx ...(2)
2
0 Put x3/2 = t Þ x dx = dt
3
Adding (1) and (2), we get
10 2 dt 2 -1 æ x 3/ 2 ö
3 ò 4 - t2
So I = = sin ç ÷ +c
2I = ò dx Þ 2 I = 10 Þ I = 5 3 è 2 ø
0
EBD_7167
http://t.me/iitjeehelps

M – 78 BITSAT Topicwise Solved Papers

p /2 e e
2sin x 1
11. (c) I = ò dx -ò 10(ln x)9 . .x dx = e - 0 - 10ò (ln x) 9 dx
0 2sin x + 2cos x 1
x 1

p /2
= e – 10I9 + 10I9= e
2sin( p / 2- x)
I= ò 2sin( p /2 - x) + 2cos( p / 2 - x)
dx 17. (b)
d
(A ln | cos x + sin x - 2 | + Bx + C)
0 dx

2cos x cos x - sin x


p/ 2 p p = A +B
= ò 2cos x + 2sin x dx Þ 21 = ò
0
dx =
2
Þ1=
4
cos x + sin x - 2
A cos x - A sin x + Bcos x + Bsin x - 2B
æ 1 ö =
12. (c) Given integral I = ò ç1 + 2 ÷ dx cos x + sin x - 2
è x - 1ø \ 2 = A + B or –1 = – A + B; l = – 2B
\ A = 3/2, B = 1/2, l = – 1
dx
= ò dx + ò
(x - 1)(x + 1) -1 x2 1 x2
18. (a) ò x tan x dx = tan -1 x - ò dx
2 2 1 + x2
1 æ 1 1 ö 1 æ x - 1ö
ò
= x+
çè -
2 x -1 x + 1
÷ø dx = x + 2 log çè x + 1÷ø + c
=
x2 1 é
tan -1 x - ò ê1 -
1 ù
dx
13. (a) We know that |sinx| is a periodic function of p 2 2 ë 1 + x 2 ûú
4p p p 1 2 1
= (x + 1) tan -1 x - x + c
Hence ò | sin x |dx = 4ò | sin x |dx = 4ò sin xdx 2 2
0 0 0
dx -1 x
= 4 [ - cos x ] = 8
p 19. (a) ò 2
= sin
2
+c
0 2-x
2 2 1
dx dx dx x 1
14. (b) If I1 = ò 2 , I2 = ò x
So ò = sin
-1
|
1 1+ x 1 0 2 - x2 20

æ 1 ö p p
æ 2+ 5ö = sin -1 ç -1
÷ + c - sin (0) – c = 4 - 0 = 4
I1 = ln ç ÷ , I 2 = ln2 Þ I1 < I2 è 2ø
è 1+ 2 ø

z z z
n 1 2
æ pö æ 5p ö
15. (a) ò 4 cos çè x + 6 ÷ø cos 2x.cos çè 6 + x ÷ø dx 20. (c) [ x] dx = 0 dx + 1 dx
0 0 1
æ 2p ö
= 2ò çè cos ( 2x + p) cos ÷ø cos 2x dx
z z
3 n
3 + 2 dx + ...... + ( n - 1) dx
2 n -1
æ 1ö
= 2 ò çè - cos 2x - ÷ø cos 2x dx ( n - 1) n
2 = 1 + 2 + 3 +.....+ (n – 1) = = 66
2
= ò (-2 cos 2 2x - cos 2x) dx Þ n (n – 1) = 132 Þ n = 12
x x
= -ò (1 + cos 4x + cos 2x) dx x + sin x
x + 2sin cos
2 2 dx
sin 4x sin 2x
21. (b) ò 1 + cos x dx = ò 2 x
= -x - - +c 2 cos
x 2 2
e æ1 2 x xö x
I10 = ò 1.(ln x)10 dx = éë(ln x)10 x ùû
e
= ò çè x sec + tan ÷ dx = x tan + c
16. (c)
1
2 2 2ø 2
1
http://t.me/iitjeehelps

Integrals M – 79

¥
e- x dt
dx = -[log(e - x + 1)]0
¥
\ I = -ò = -ò (1 - 2t)-1/ 2 dt
22. (b) I = ò e- x + 1 1 - 2t
0
= – [log 1 – log 2] = log 2 (1 - 2t)1/2
=- + C = 1 - 2t + C
1 1 (-2) (1/ 2)
2 3
23. (b) ò
I1 = 2 x dx , I 2 = 2 x dx , ò = 1-
2
+C=
x -1
+C
0 0
x +1 x +1
2 2 p /2
x2 3 f(x)
ò ò
I3 = 2 dx, I 4 = 2 x dx " 0 < x < 1, x 2 > x3 27. (a) Let I = ò æp ö
0 f(x) + f ç - x ÷
dx then
1 1 è2 ø
æp ö
1 1 p /2 f çè 2 - x ÷ø
2 3
Þ ò ò
2 x dx > 2 x dx Þ I1 > I 2 . Also " 1 < x < 2 I= ò
æp ö
0 f ç - x ÷ + f(x)
dx
0 0
è2 ø
2 2 p /2

ò 1.dx = [ x ]0
2 3 p /2
ò
x 2 < x 3 Þ 2 x dx < 2 x dx Þ I3 < I 4 ò Adding, 2I =
0
= p/2 Þ I = p/4
1 1

dx (1 + sin x) 28. (b) f ( x ) = 4 cos 3 x - cos x - 2 cos x = cos 3x


24. (d) ò 1 - sin x = ò 1 - sin 2 x dx p/2 p/2
sin 3x ù 1
= ò sec
2
x dx + ò tan x.sec dx = tan x + sec x + c \ ò f ( x )dx =
3 úû 0
=-
3
0
¥ p/2 2
dx a sec qdq 1
25. (a) ò = ò 6 6 on putting 29. (a) ò sin 2 x cos2 x dx
(a ) a sec q
3
2 2
a +x p/4
sin 2 x + cos 2 x 1 1
x = a tan q =ò dx = ò 2
+ dx
p/ 2
2
sin x cos x 2
cos x sin 2 x
p/2
1 1 æ 1 + cos 2q ö
= 5 ò cos 4 qdq =
5 çò 2 ÷ dq = ò sec 2 x dx + ò cos ec 2 x dx = tan x - cot x +C
p/4è ø
a p/ 4 a
2
p/2 x
1 æ 1 + 4cos 4q ö 30. (c) Let I = ò dx ...(i)
=
4a 5 ò çè1 + cos 2q + 2 ÷ø d q 1 3- x + x
p/4
2
p/ 2 3- x
=
1 é3 sin 8q ù
ê 2 q + sin 2q + 8 ú =
3p - 8 Then, I = ò 3 - (3 - x) + 3 - x
dx
4a 5 ë ûp / 4 32a 5 1
2
1 3- x
26. (c) Let I = ò dx Þ I= ò dx .... (ii)
(x + 1) x - 1 2
1 x + 3- x
1 Adding (i) and (ii), we get
1
Put x + 1 = so that dx = - 2 dt 2
t t 2I = ò 1.dx = [ x ]1 = 2 - 1 = 1 Þ I = 1/ 2
2

1
EBD_7167
http://t.me/iitjeehelps

M – 80 BITSAT Topicwise Solved Papers

Chapter
Application of Integrals
23
1
1. The area under the curve y = | cos x – sin x |, (a) zero (b)
3
p 2
0£ x£ , and above x-axis is : [2017] (c) (d) 1
2 3
(a) 2 2 (b) 2 2 - 2 (c) 2 2 + 2 (d) 0 8. Area of the triangle formed by the line x + y = 3
and angle bisectors of the pair of straight lines
2. The area of the region R = {(x, y):|x| £ |y| and x2
x2 – y2 + 2y =1 is [2009]
+ y2 £ 1} is [2015]
3p 5p (a) 2 sq. units (b) 4 sq. units
(a) sq. units (b) sq. units (c) 6 sq. units (d) 8 sq. units
8 8
p p 9. The area between the parabola y = x 2 and the
(c) sq. units (d) sq. unit
2 8 line y = x is: [2008]
3. The area bounded by the x-axis, the curve 1 1
y = f(x) and the lines x =1, x =b, is equal to (a) sq. units (b) sq. units
6 3
b 2 + 1 - 2 for all b > 1, then f(x) is [2014] 1
(c) sq. units (d) None of these
(a) x -1 (b) x +1 2
x 10. The area between the curve y = 1 – |x| and the
(c) x2 +1 (d) x – axis is equal to [2007]
1+ x 2
4. Area intercepted by the curves y = cos x,
1
x Î[0, p] and y = cos 2x, x Î[0, p] , is [2013] (a) 1 sq. unit (b) sq. unit
2
3p 3 3 3p 3 3 1
(a) (b) (c) (d) (c) sq. unit (d) 2 sq. units
2 2 4 4 3
5. The area bounded by the curve y = sinx, x-axis 11. The area of the figure bounded by y = ex, y = ex
and the ordinates x = 0 and x = p/2 is [2012] and x = 1 is [2006]
(a) p (b) p/2 (c) 1 (d) 2 1
(a) 2(e – 1) (b) e + – 2
6. What is the area bounded by y = tan x , y = 0 and e
p 1 1
x= ? [2011] (c) e – +2 (d) e +
4 e e
12. The area of the region bounded by y = | x – 1 |
ln 2
(a) ln 2 sq. units (b) sq. units and y = 1 is [2005]
2
(a) 2 (b) 1 (c) 1/2 (d) 1/4
(c) 2 (ln 2) sq. units (d) None of these
7. The area of the region bounded by the curve
y = x | x |, x-axis and the ordinates x = 1, x = -1 is
given by : [2010]
http://t.me/iitjeehelps

Application of Integrals M – 81

Hints & Solutions

1. (b) y = | cos x – sin x |


1/ 2
éx 1 x2 ù
Y = 4 ê 1 - x 2 + sin -1 x - ú
f(x) = cos x
ëê 2 2 2 ûú
0
g(x) = sin x
é 1 1 1 p 1ù
= 4ê ´ + ´ - ú
ë2 2 2 2 4 4û

X é 1 p 1 ù 4p p
= 4ê + - ú = = sq units
O p/4 p/2 ë4 8 4û 8 2
b

p /4
3. (d) Given ò f (x)dx = b2 +1 - 2
1
Required area = 2 ò (cos x - sin x ) dx
Differentiate with respect to b
0
p/4
= 2 [ sin x + cos x ]0 f (b) =
b
Þ f ( x) =
x

é 2 ù b2 + 1 x2 +1
=2 ê - 1ú = (2 2 - 2) sq. units
ë 2 û 4. (d)
2. (c)
Y

y=–x P(x1, y1)


y=x
æ 1 1 ö
ç- , ÷ æ 1 1 ö
è 2 2ø ç , ÷
è 2 2ø
2 p /3
Q(x2,y2) 3 3
X Area = ò (cos x - cos 2x)dx =
4
0
p /2 p /2
æ 1 -1 ö
, 5. (c) Area= ò y dx = ò sin x dx = [- cos x]p0 / 2 =1
ç ÷
è 2 2ø 0 0
p
æ 1 1 ö
ç-
è 2
,- ÷

6. (b) Required area =
p
ò0
4 tan x dx
ln 2
= = ln 2 =
ln sec1 04
Required area = 4 (Area of the shaded region 2
7. (c) The area of the region bounded by the curve
in first quadrant) y = f (x) and the ordinates x = a, x = b is
given by

( y1 - y2 ) dx = 4 0 æç 1 - x 2 - x ö÷ dx
1 2 1 2 b
= 4ò ò
0 è ø
Area = òa y dx
EBD_7167
http://t.me/iitjeehelps

M – 82 BITSAT Topicwise Solved Papers


According to the question, \ Required area (shaded region)
1 1
ìï x 2 , x ³ 0 é x 2 x3 ù
y=x x =í
2
ò
= ( x - x 2 )dx = ê
êë 2
- ú
3 úû
ïî - x , x < 0 0 0

Y éæ 1 1 ö ù 1
y = x2 = êç - ÷ - 0ú = sq. units
ëè 2 3 ø û 6
B 10. (a) The curves are y = 1 – x, y = 1 + x, y = 0
y
C(1, 0) x=1
X
x = –1 O A(1, 0) (0, 1)
D

y = –x 2 y=1+x y = 1 –x
Required area x
= area of region OAB + area of region OCD (–1, 0)O (1, 0)
= 2 × Area of region OAB
1 2 1
= 2 ò x 2 dx = sq. units Area = ´ Base ´ Height
0 3 2
8. (a) x2 – y2 + 2y = 1 Þ x = ±( y - 1) 1
Base = 2, Height = 1 Þ Area = ´ 2 ´ 1 = 1 sq. unit
Bisectors of above line are x = 0 & y = 1 2
11. (b)
D
(1, 0)

y=x+1 x+y=1 (0,1)A


So area between x = 0, y = 1 & x + y = 3 is C
shaded Region shown in figure. O B
1

ò (e ) dx
x -x
3 Required area = -e
(2, 1) 0
1 -x 1 1
= éë e + e ùû = (e1 + e–1) – (1 + 1)= e + –2
x

3 0 e
Area = ½ × 2 × 2 = 2 sq. units 12. (b) The given region is represented by the
9. (a) For points of intersection of the equation equations
of parabola y = x2 ... (i) y = 1 – x, x £ 1 = x – 1 , x ³ 1
and equation of straight line y = x ... (ii) and y = 1; C = (2, 1) and B = (0, 1)
We have
x2 – x = 0 Þ x(x – 1) = 0
\ x = 0 or x = 1Þ y = 0 or y = 1
Hence, the coordinates of their points of
intersection are O (0, 0) and P (1, 1).

2
y=x P (1, 1)

\ the shaded area in the figure


O (0, 0)
1 1
x = BC . AC = 2 . 1 = 1.
y= 2 2
http://t.me/iitjeehelps

Differential Equations M – 83

Chapter
Differential Equations
24
1. If f (x) is a differential function, then the solution 7. The degree of the differential equation [2011]
of the differential equation dy + {y f¢(x) – f (x) 2/3
æ d3y ö d2 y dy
f¢(x)}dx = 0, is [2015] + 4-3 2 +5 = 0 is
ç 3÷
-f( x ) è dx ø dx
{( ) }
(a) y = f x - 1 + Ce dx
(a) 1 (b) 2 (c) 3 (d) None of these
yf ( x ) = {f ( x )} + C
2
(b)
dy
+ 2 y = 1 satisfying
ye ( ) = f ( x ) e ( ) + C
(c) f x f x 8. What is the solution of
dx
-f( x ) y(0) = 0 ? [2010]
(d) y - f ( x ) = f ( x ) e
1 - e -2 x 1 + e -2 x
2. The degree of the differential equation satisfying (a) y = (b) y =
2 2
1 - x 2 + 1 + y 2 = a (x – y) is [2015] x 1+ ex
(c) y = 1 + e (d) y=
(a) 1 (b) 2 (c) 3 (d) 4 2
9. The solution of differential equation
3. Solution of differential equation [2014] dy
-2 -3 2x – y = 3 represents a family of [2010]
æxö
2
æ dy ö æ x ö æ dy ö
3 dx
-1 ç ÷ ç ÷ ç ÷ ç ÷ (a) circles (b) straight lines
æ x ö dy è y ø è dx ø è y ø è dx ø
2
x =1+ ç ÷ + + + ... is (c) ellipses (d) parabola
è y ø dx 2! 3! 10. Solution of the differential equation [2009]
(a) y2 = x2 (ln x2 – 1) + C (b) y = x2 (ln x – 1) + C dy y
2 x2 + = sin x is
(c) y2 = x (ln x – 1) + C (d) y = x e + C dx x
4. The general solution of the differential equation (a) x (y + cosx) = cos x + C
(b) x (y – cosx) = sinx + C
dy (c) x (y + cosx) = sinx + C
+ sin( x + y ) = sin( x - y ) is [2013]
dx (d) None of these
(a) log tany + sin x = C dy ax + h
y 11. The solution of = represents a
(b) log tan + 2sin x = C dx by + k
2 parabola when [2008]
y (a) a = 0, b = 0 (b) a = 1, b = 2
(c) tan + log sin x = C
2 (c) a = 0, b ¹ 0 (d) a = 2, b = 1
(d) None of these dy
5. The solution to the differential equation 12. The solution of the equation = cos (x – y) is
dx
dy yf ' (x ) - y 2 [2008]
= where f (x) is a given function æ x - yö
dx f (x ) (a) y + cot çè ÷ =C
is [2013] 2 ø
(a) f ( x ) = y( x + c) (b) f ( x) = cxy æ x - yö
(b) x + cot çè ÷ =C
(c) f ( x ) = c(x + y) (d) yf (x ) = cx 2 ø
6. The differential equation whose solution is
æ x - yö
Ax 2 + By 2 = 1 where A and B are arbitrary (c) x + tan ç =C
è 2 ÷ø
constants is of [2012]
(a) second order and second degree (d) None of these
(b) first order and second degree 2 dy
(c) first order and first degree 13. The general solution of x = 2 is – [2007]
dx
(d) second order and first degree
EBD_7167
http://t.me/iitjeehelps

M – 84 BITSAT Topicwise Solved Papers


2 2 (d) None of these
(a) y =c+ (b) y = c -
x x dy
3 16. The solution of x 3 + 4 x 2 tan y = e x sec y
(c) y = 2cx (d) y = c - dx
x3 satisfying y (1) = 0 is : [2005]
14. Integrating factor of differential equation x
dy (a) tan y = ( x - 2) e log x
cos x + y sin x = 1 is: [2007] x -4
dx (b) sin y = e ( x - 1) x
(a) sec x (b) sin x (c) cos x (d) tan x (c) tan y = ( x - 1)e x x -3
15. The solution of differential equation x -3
dy (d) sin y = e ( x - 1) x
– y tan x = – y2 sec x is [2006] 17. The degree of the differential equation of all
dx –1
(a) y sec x = cot x + c tangent lines to the parabola y2 = 4ax is [2005]
(b) y–1 cosx = tan x + c (a) 1 (b) 2 (c) 3 (d) 4
(c) y–1 sec x = tanx + c

Hints & Solutions


1. (a) Given differential equation is y dy
or ò x ln x 2 dx = ò y dy
2
dy + { yf ' ( x ) - f ( x ) f ' ( x )} dx = 0 Þ ln x =
x dx
dy
Þ + f¢ ( x ) y = f ( x ) f ' ( x ) 1 y2
dx Put x2 = t Þ 2x dx = dt \
2 ò ln t dt =
2
which is a linear differential equation with C + t ln t – t = y2 or y2 = x2 (ln x2 – 1) + C
P = f ' ( x ) , Q = f ( x ) . f ' ( x ) and 4. (b) The equation is,
dy
I.F = e ò ( ) = e ( )
f ' x dx f x = sin( x - y ) - sin( x + y ) = 2cos x sin (- y )
dx
\ Solution is y.e ( ) = ò f ( x ).f ' ( x ) e ( ) dx + C
f x f x
dy
Þ + 2 cos xdx = 0
Þ y.e ( ) = ò f ( x ).e ( ) f ' ( x ) dx + C
f x fx sin y
Þ ò cos ec y dy + 2ò cos x dx = C
I II
y
Þ y.ef( x ) = f ( x ) ef( x ) - ò f¢ ( x ) ef( x ) dx + C
Þ log tan + 2sin x = C
2
y2
Þ y.ef( x ) = f ( x ) ef( x ) - ef( x ) + C
dy f ' ( x )
5. (a) We have = y-
dx f ( x ) f (x )
Þ y = éë f ( x ) - 1ùû + Ce-f( x ) Þ
dy f ' ( x)
- y=-
y2
2. (a) Put x = sin q and y = sin f dx f (x ) f (x)
Þ cos q + cos f = a (sin q – sin f) dy f ' (x ) 1
Divide by y2: y -2 - y -1 =-
q+f q-f q+f q-f dx f (x) f ( x)
Þ 2 cos cos = 2a cos sin
2 2 2 2
-1 -2 dy dz
q-f Put y = z Þ - y =
Þ cot = a Þ q – f = 2 cot–1 a dx dx
2 dz f ' ( x ) 1
Þ sin–1 x – sin–1 y = 2 cot–1 a - - ( z) = -
1 1 dy dx f (x ) f (x)
Differentiate - =0
1- x 2
1- y 2 dx dz f ' ( x ) 1
Þ + z=
So the degree is one. dx f (x ) f (x)
æ xö
-1 dy æ y ö æ dy ö f '( x )
æ ö dx
ç ÷ç ÷ ò
çè y ÷ø èç dx ø÷ è øè ø
3. 2
(a) x = e Þ x 2 = e x dx I.F. = e f (x)
= e log f ( x ) = f ( x)
http://t.me/iitjeehelps

Differential Equations M – 85

\ The solution is a
1 Þ by2 + ky = x2 + hx + C
z ( f ( x )) = ò ( f ( x)) dx + c 2
f ( x) For this to represent a parabola, one of the
Þ y -1 (f ( x ) ) = x + c Þ f (x ) = y (x + c) two terms x2 or y2 is zero. Therefore, either
2 2 a = 0, b ¹ 0 or a ¹ 0, b= 0
6. (d) Ax + By = 1 .........(1) du dy
dy 12. (b) Put u = x – y, then = 1-
Ax + By =0 ...(2) dx dx
dx du du
Þò
1 - cos u ò
2 Þ 1 – cos u = = dx
d2y æ dy ö dx
A + By + Bç ÷ =0 ...(3)
2 è dx ø 1 2 æ uö
dx Þ ò cos ec çè ÷ø du = ò dx
From (2) and (3) 2 2
Þ x + cot (u/2) = constant
ïì æ dy ö ïü
2
d2y dy æ x - yö
x í - By 2 - B ç ÷ ý + By =0 Þ x + cot çè ÷ =C
dx è dx ø ï dx 2 ø
îï þ dy 2 2
Dividing both sides by –B, we get 13. (b) = 2 Þ dy = 2 dx
dx x x
2 2 2
d y æ dy ö dy
Now integrate it . We get y = - + c
xy + xç ÷ - y =0
dx è dx ø
2 dx x
14. (a) The given differential equation is
Which is a DE of order 2 and degree 1 dy
2/3 cos x + y sin x = 1.
æ d3y ö d2 y dy dx
7. (b) ç 3 ÷ + 4-3 +5 =0 The linear differential eq. is :
è dx ø dx 2 dx
dy
2 3 + y tan x = sec x (dividing by cos x)
æ d3 y ö é d2 y dy ù dx
Þ ç 3÷ = ê 3 2 - 5 - 4ú So, I.F. = eòPdx, where P = tan x
è dx ø êë dx dx úû = eòtan x dx = elog (sec x) = sec x.
It is a differential equation of degree 2..
1 dy 1
dy dy 15. (c) 2 - tan x = - sec x ,
8. (a) + 2y = 1 Þ = 1 – 2y y dx y
dx dx
1 -1 dy dv -dv
dy 1 = v; 2 = \ – v tanx = – secx
ò 1 - 2y = ò dx - 2 log |1 - 2y | = x + C y
dv
y dx dx dx
+ v tan x = secx, Here P = tan x, Q = sec x
1 dx
at x = 0, y = 0 ; - log1 = 0 + C Þ C = 0
2 I.F. = e ò tan x dx = sec x , v sec x = ò sec 2 x dx + c
1 - e -2x
1 – 2y = e–2x Þ y = Hence the solution is y–1 secx = tanx + c
2
2 dx 16. (b) Rewriting the given equation in the form
dy
9. (d) We have, 2x = y+ 3 Þ dy = dy
dx y+3 x x 4 cos y + 4 x 3 sin y = xe x
integrating, 2 ln ( y + 3) = ln x + ln c = ln cx dx
Þ ln ( y + 3)2 = ln cx Þ (y + 3)2 = cx d 4
Þ ( x sin y ) = xe x
which is a family of parabolas dx
dy 1 é dy ù Þ x 4 sin y = ò xe x dx + c = ( x - 1) e x + c
10. (c) + . y = sinx ê Type + Py = Q ú
dx x ë dx û Since, y (1) = 0 so,c = 0.
1
ò dx -4 x
e ò Pdx = = elog x = x
e x Hence, sin y = x ( x - 1) e
\ Sol. is y x = ò x sin x dx + C 17. (b) Equation of the tangent y = mx + a/m
where m is arbitrary constant \ order = 1
= x (–cosx) – ò 1.(- cos x)dx + C dy a
dy
= – xcosx + sinx + C Þ x (y + cosx) = sinx + C = m.1 + 0 = m \ y = x+
dx dx ( dy / dx )
dy ax + h 2
11. (c) = Þ (by + k) dy = (ax + h) dx æ dy ö dy
dx by + k x ç ÷ - y + a = 0 \ degree = 2
è dx ø dx
EBD_7167
http://t.me/iitjeehelps

M – 86 BITSAT Topicwise Solved Papers

Chapter
Vector Algebra
ur ur ur
25
r r r
1. Let a, b & c be non-coplanar unit vectors 7. If a , b , c are three non-coplanar vectors, then
equally inclined to one another at an acute angle q. ur ur ur ur ur ur
rrr a .(b ´ c) b .(a ´ c)
the value of ur ur ur + ur ur ur is : [2013]
Then | [a b c] | in terms of q is equal to [2017] (c ´ a).b c .(a ´ b)
(a) (1 + cos q) cos 2q (a) 0 (b) 2 (c) 1 (d) None of these
8. If vectors 2i – j + k, i + 2j – 3k and 3i + aj + 5k are
(b) (1 + cos q) 1 - 2cos 2q coplanar, then the value of a is [2013]
(c) (1 - cos q) 1 + 2cos 2q (a) 2 (b) –2 (c) –1 (d) –4
9. The unit vector perpendicular to the vectors
(d) None of these
2. The dot product of a vector with the vectors 6iˆ + 2jˆ + 3kˆ and 3iˆ - 6jˆ - 2kˆ is – [2012]
ˆi + ˆj - 3kˆ , ˆi + 3jˆ - 2kˆ and 2iˆ + ˆj + 4kˆ are 0, 5 2iˆ - 3jˆ + 6kˆ 2iˆ - 3jˆ - 6kˆ
(a) (b)
and 8 respectively. The vector is [2017] 7 7
ˆ ˆ ˆ
(a) i + 2 j + k (b) - i + 3j - 2kˆ
ˆ ˆ
2i + 3jˆ - 6kˆ
ˆ 2i + 3jˆ + 6kˆ
ˆ
(c) (d)
(c) ˆi + 2ˆj + 3kˆ (d) ˆi - 3jˆ - 3kˆ 7 7
3. Let a, b and c be three vectors satisfying a × b = 10. If a.b = a.c and a × b = a × c, then correct statement
is [2012]
(a ×c), |a| = |c| = 1, |b| = 4 and |b × c| = 15 .
(a) a || (b – c) (b) a ^ (b - c)
If b – 2c = la, then l equals [2015]
(a) 1 (b) – 1 (c) 2 (d) – 4 (c) a = 0 or b = c (d) None of these
r r r r
4. If the middle points of sides BC, CA & AB of 11. Two vectors A and B are such that | A + B | =
triangle ABC are respectively D, E, F then r r
position vector of centre of triangle DEF, when | A– B | . The angle between the two vectors will
position vector of A, B, C are respectively be– [2011]
iˆ + ˆj, ˆj + kˆ, kˆ + iˆ is [2014] (a) 60° (b) 90° (c) 180° (d) 0°
r r rr r r
12. If (a ´ b )2 + (a.b ) 2 = 676 and | b |= 2 then | a |
(a)
3
(
1 ˆ ˆ ˆ
i + j +k ) (b) ( iˆ + ˆj + kˆ ) is equal to [2010, 2007]
(a) 13 (b) 26 (c) 39 (d) None of these
(c) (
2 iˆ + ˆj + kˆ )
2 ˆ ˆ ˆ
3
i + j +k
(d) ( ) 13. Which one of the following is the unit vector
r
perpendicular to both a = -ˆi + ˆj + kˆ and
5. The angle between any two diagonal of a cube is r
[2014] b = ˆi - ˆj + kˆ ? [2010]
(a) 45° (b) 60° (c) 30° (d) tan -1 (2 2 ) ˆi + ˆj ˆj + kˆ ˆi - ˆj
uur uur uur (a) (b) k̂ (c) (d)
6. If a , b , c are three unit vectors such that 2 2 2
uur uur uur uur ur 14. With respect to a rectangular cartesian coordi-
a + b + c = 0 , where 0 is null vector, then nate system, three vectors are expressed as :
uur uur uur uur uur uur r r r
a .b + b . c + c . a is : [2013] a = 4iˆ - ˆj, b = -3iˆ + 2jˆ an d c = - kˆ where

3 ˆi, ˆj, kˆ are unit vectors, along the X, Y and Z-


(a) – 3 (b) – 2 (c) - (d) 0
2 axis respectively. The unit vector r̂ along the
http://t.me/iitjeehelps

Vector Algebra M – 87

direction of sum of these vector is – [2010] 19. Find the angle between th e vectors
r r
1 ˆ ˆ ˆ 1 ˆ ˆ ˆ A = ˆi + ˆj - 2kˆ and B = -ˆi + 2jˆ - kˆ . [2007]
(a) r̂ = (i + j - k) (b) r̂ =
(i + j - k)
3 2 (a) 15° (b) 45° (c) 35° (d) 60°
r r r
1 1 ˆ ˆ ˆ 20. If a, b , c are non-coplanar vectors and l is a
(c) r̂ = (iˆ - ˆj + k)
ˆ (d) r̂ = (i + j + k)
3 2 real number, then the vectors a + 2b + 3c ,
15. If the middle points of sides BC, CA & AB of
triangle ABC are respectively D, E, F then lb + 4c and (2l - 1)c are non coplanar for
position vector of centre of triangle DEF, when [2006]
position vector of A, B, C are respectively i + j, (a) no value of l
j + k, k + i is – [2010] (b) all except one value of l
(a) (1/3) (i + j + k) (b) (i + j + k) (c) all except two values of l
(c) 2 (i + j + k) (d) (2/3) (i + j + k) (d) all values of l
16. If the position vectors of the vertices A, B, C of 21. A vector of magnitude 5 and perpendicular to

a triangle ABC are 7 ˆj + 10kˆ , -iˆ + 6 ˆj + 6kˆ and (iˆ - 2 ˆj + kˆ) and (2iˆ + ˆj - 3kˆ) is : [2006]

-4iˆ + 9 ˆj + 6kˆ respectively, the triangle is :[2008] (a)


5 3 ˆ ˆ ˆ
(i + j + k ) (b)
5 3 ˆ ˆ ˆ
(i + j - k )
(a) equilateral 3 3
(b) isosceles
(c) scalene 5 3 ˆ ˆ ˆ 5 3 ˆ ˆ ˆ
(c) (i - j + k ) (d) (-i + j + k )
(d) right angled and isosceles also 3 3
17. For unit vectors b and c and any non-zero vector 22. i × ( j × k ) + j × ( k × i ) + k ( i × j ) equals [2005]
a, the value of {(a + b) × (a + c)} × (b + c)}. (a) i (b) j (c) k (d) 0
(b + c) is [2008] 23. What is the vector joining the points (3, 1, 14)
(a) | a |2 (b) 2 | a |2 and (–2, –1, –6) ? [2005]
(c) 3 | a |2 (d) None of these (a) -i + 2j +12k
ˆ ˆ ˆ (b) -i - 2 j +12kˆ
ˆ ˆ
18. If q be the angle between vectors a = i + 2j + 3k
and b = 3i + 2j + k, then cos q equals [2007] (c) -ˆi - 2jˆ -12kˆ (d) 5iˆ + 2jˆ + 20kˆ
(a) 5/7 (b) 6/7 (c) 4/7 (d) 1/2

Hints & Solutions


uuur r uuur r uuur r
1. (c) OA = a, OB = b & OC = c are unit AB = OA 2 + OB2 - 2OA.OB.cos q
vectors and equally inclined to each other at an
= 2 - 2 cos q = 2 1 - cos q
acute angle q.
O 3 3
\ Area of D ABC = AB2 = (1- cos q)
4 2
If G is the centroid of the D ABC, then
B
C 1 r r r
G OG = |a +b +c |
3
A
1 2 2 2 rr rr rr
\ ABC is an equilateral triangle and = a + b + c + 2a.b + 2b .c + 2c .a
3
EBD_7167
http://t.me/iitjeehelps

M – 88 BITSAT Topicwise Solved Papers

1 Now given, b - 2c = la Þ b - 2c 2 = la 2
= 1 + 2 cos q
3
r r r Þ b
2
+ 4 c - 4 ( b.c ) = l 2 a
2
\ [a b c ] = Volume of parallelopiped
= OG × 2 ar (D ABC) Þ 16 + 4 - 4 b c cos q = l2 Þ 20 - 16cos q = l2
1 3
= 2. 1 + 2 cos q ´ (1 - cos q) Þ 20 - 4 = l 2 Þ l 2 = 16 Þ l = ± 4
3 2
4. (d) The position vector of points D, E, F are
respectively
= (1 - cos q) 1 + 2cos q
2. (a) Let the required vector be iˆ + ˆj ˆ ˆ kˆ + ˆj iˆ + kˆ ˆ
+ k, i + and +j
r 2 2 2
v = xiˆ + yjˆ + zkˆ , then
r D
( )
v. ˆi + ˆj - 3kˆ = 0 Þ x + y - 3z = 0 ...(i)
C

r
v. ( iˆ + 3jˆ - 2kˆ ) = 5 Þ x + 3y - 2z = 5 ...(ii) 1 ^ – ^j)
(k
r 2 1 ^ ^
and v. ( 2iˆ + ˆj + 4kˆ ) = 8 Þ 2x + y + 4z = 8
(i –j)
...(iii) 2
Subtracting (ii) from (i),we have
– 2y – z = –5 Þ 2y + z = 5 ..(iv) A 1 B
^ –i)
(k ^
Multiply (ii) by 2 and subtracting (iii) from it, we 2
obtain : 5y – 8z = 2 ...(v)
So, position vector of centre of DDEF
Multiply (iv) by 8 and adding (v) to it, we have
21y = 42 Þ y = 2 ...(v) 1 é iˆ + ˆj ˆ ˆ kˆ + ˆj iˆ + kˆ ù
ˆj ú
= ê +k +i + +
Substituting y = 2 in(iv), we get 3ë 2 2 2 û
2× 2+ z= 5Þz=5–4 =1
2 ˆ ˆ ˆ
= éi + j + k ù
Substituting these values in (i),we get 3ë û
x+ 2–3= 0Þx=3–2=1 5. (d) for a unit cube unit vector
Hence, the required vector is along the diagonal z
C
r OP =
1
(î + ĵ + k̂ )
v = xiˆ + yjˆ + zkˆ = ˆi + 2jˆ + kˆ 3 P
y
3. (d) Let q be the angle between b and c. unit vector along the
O B
diagonal
x A D
Q b ´ c = 15 Þ b c sin q = 15
1
CD = (î + ĵ - k̂ )
15 15 15 3
Þ sin q = Þ sin q = =
b c 4 ´1 4 1 1
\ cos q = (1 + 1 - 1) = \ tan q = 2 2
3 3
15 1 1
\cos q = 1 - = =
16 16 4
http://t.me/iitjeehelps

Vector Algebra M – 89

uur uur uur uur r r r r


6. (c) We have a + b + c = 0 11. (b) A + B | = |A – B |
uur uur uur uur uur uur
\ | a + b + c |= 0 Þ| a + b + c |2 = 0 Þ A 2 + B 2 + 2AB cos q = A 2 + B 2 - 2AB cos q
uur uur uur Squaring both the sides, we get
Þ | a |2 + | b |2 + | c |2 r r rr r r rr
A 2 + B2 + 2AB cos q = A 2 + B2 - 2AB cos q
uur uur uur uur uur uur rr
+2( a . b + b . c + c . a ) = 0 or 4ABcosq = 0 or cos q = 0 (since the scalar
r r r r r r or dot product is zero) . Therefore angle between
Þ 1 + 1 + 1 + 2 (a . b + b . c + c . a ) = 0 r r
A to B is 90°
uur uur uur uur uur uur 3 r r r r
Þ a .b + b.c + c.a = - 12. (a) (a ´ b ) 2 + (a × b )2 = 676
2
7. (a) By definition of scalar triple product r r r r
(| a | . | b | sin qnˆ ) 2 + (| a | × | b | cos q) 2 = 676
r r r rrr
a × (b ´ c ) can be written as [a b c] 2
Þ a 2 b 2 sin 2 q + a 2 b 2 cos 2 q = 676 [(nˆ ) = 1]
r r r r r r rrr rrr
a.(b ´ c ) b.(a ´ c ) [a b c ] [b a c ]
r r r+r r r = rrr + rrr 2 676 676
a2b2 (sin2 q + cos2 q) = 676 Þ a = =
(c ´ a ).b c.(a ´ b ) [c a b ] [c a b ] b 2 4
rrr rrr
[a b c ] [a b c ] r 676 r 26 r
= r r r - r r r = 1-1 = 0 | a |= Þ | a |= Þ | a |= 13
[a b c ] [a b c ] 4 2
rrr rrr rrr 13. (a) According to question a = -iˆ + ˆj + kˆ and
Q [a b c ] = [b c a ] = [c a b ]
rrr rrr b = iˆ - ˆj + kˆ
but [b a c ] = – éëa b c ùû
8. (d) If given vectors are coplanar, then there ex- iˆ ˆj kˆ
ists two scalar quantities x and y such that Then, a × b = -1 1 1
2i – j + k = x (i + 2j – 3k) + y (3i + aj + 5 k) .....(1) 1 -1 1
Comparing coefficient of i, j and k on both sides of (1)
we get x + 3y = 2 , 2x + ay = –1 , –3x + 5y = 1 ...(2)
Solving first and third equations, we get = iˆ [1 + 1] - ˆj [-1 - 1] + kˆ[1 - 1] = 2(iˆ + ˆj)
x = 1/2, y = 1/2 and |a × b| = 4+4 = 2 2
Since the vectors are coplanar, therefore these
values of x and y will satisfy the equation 2(iˆ + ˆj) iˆ + ˆj
\ Required unit vector = ± =±
2x + ay = – 1 2 2 2
\ 2 (1/2) + a (1/2) = – 1 Þ a = –4 r r r r ˆ ˆ ˆ ˆ ˆ ˆ ˆ ˆ
9. (c) Unit vector perpendicular to both the given 14. (a) r = a + b + c = 4i - j - 3i + 2j - k = i + j - k
vectors is, r
r ˆi + ˆj - kˆ ˆi + ˆj - kˆ
(6iˆ + 2ˆj + 3k)
ˆ ´ (3iˆ - 6ˆj - 2k)
ˆ 2iˆ + 3jˆ - 6kˆ r̂ = = =
= |r| 12 + 12 + (-1)2 3
ˆ ˆ ˆ ˆ ˆ ˆ
| (6i + 2 j + 3k) ´ (3i - 6 j - 2k) | 7
10. (c) a. b. = a.c Þ a.(b – c) = 0 15. (d) The position vector of points D, E, F are
Þ a = 0 or b – c = 0 or a ^ (b – c) respectively
Þ a = 0 or b = c or a ^ (b – c) ...(1) i+ j k+ j i+k
Also a x b = a x c Þ a × (b – c) = 0 +k , i+ and +j
2 2 2
Þ a = 0 or b – c = 0 or a || (b – c) So, position vector of centre of DDEF
Þ a = 0 or b = c or a || (b – c) ...(2)
Observing to (1) and (2) we find that 1 éi + j k+ j i+k ù 2
= +k+i + + jú = [i + j + k]
a = 0 or b = c 3 êë 2 2 2 û 3
EBD_7167
http://t.me/iitjeehelps

M – 90 BITSAT Topicwise Solved Papers


uuur r r r r r
16. (d) Given, OA = 7 ˆj + 10kˆ , 20. (c) Vectors a + 2b + 3c , lb + 4c , and
uuur uuur
OB = -iˆ + 6 ˆj + 6kˆ , OC = -4iˆ + 9 ˆj + 6kˆ 1 2 3
uuur uuur uuur r
(2l - 1)c are coplanar if 0 l 4 =0
Þ AB = OB - OA = -iˆ - ˆj - 4kˆ
0 0 2l - 1
uuur uuur uuur
BC = OC - OB = -3iˆ + 3 ˆj
uuur uuur uuur 1
CA = OA - OC = 4iˆ - 2 ˆj + 4kˆ Þ l(2l - 1) = 0 Þ l = 0 or
2
uuur \ Forces are noncoplanar for all l, except
Þ | AB |= 12 + 12 + 42 = 18 = 3 2
1
uuur l = 0,
2
BC = 32 + 32 = 18 = 3 2

uuur 21. (a) Let A = ( î - 2 ˆj + k̂ ) and B = ( 2 î + ˆj - 3 k̂ )


| CA |= 42 + 22 + 42 = 36 = 6
ˆi ˆj kˆ
ur uur
3 2 , 3 2 & 6 are sides of a right angled D. Now A ´ B = 1 -2 1
2 1 -3
Q (3 2 ) 2 + (3 2 ) 2 = 36 = 6 2
Hence, the DABC is a right -angled and isosceles also.
= iˆ(6 - 1) - ˆj( -3 - 2) + kˆ(1 + 4) = 5 iˆ + 5 ˆj + 5 kˆ
17. (d) The given expression
= {{a × c + b × a + b × c} × (b × c)}. (b + c)
and A ´ B = 25 + 25 + 25 = 5 3
= {(a × c) × (b × c) + (b × a) × (b × c)}. (b + c)
= [(a.(b × c))c – (c.(b × c))a + (b.(b × c))a r
– (a.(b × c)b)].(b+ c) r a
We know a unit vector along a is given as r .
= [(a. (b × c))(c - b). (b + c)] = (a. (b × c)) a
[|c|2–|b|2] = 0 r r
[ |b| = |c| = 1]. A´B
\ unit vector along A ´ B is = r r
A´B
a.b 3+ 4+ 3 5
18. (a) cosq = = =
| a || b | 14 14 7
r r 5(iˆ + ˆj + kˆ) æ iˆ + ˆj + kˆ ö
19. (d) Here A = ˆi + ˆj - 2kˆ , B = -ˆi + 2jˆ - kˆ
=ç ÷
r r 5 3 è 3 ø
r r A.B
We know that A.B = AB cos q or cos q = Thus required vector of magnitude 5 is
AB
5 3 ˆ ˆ ˆ
Now, A = 12 + 12 + (-2) 2 = 6 , (i + j + k )
3
22. (d) i × ( j × k) + j × ( k × i) + k × ( i × j )
B = (-1)2 + 22 + (-1) 2 = 6
=i× i+j× j+k×k=0+0+0=0
23. (d) If P and be the points represented by the
r r
A.B = (iˆ + ˆj - 2k).(
ˆ -ˆi + 2jˆ - k)
ˆ = -1 + 2 + 3 = 3 coordinates (3, 1, 14) and (–2, –1, –6) respectively
uuur
then, PQ = p.v. of Q – p.v. of P
3 3 1
cos q = = = or q = 60°
6´ 6 6 2 = (-2iˆ - ˆj - 6k)
ˆ - (3iˆ + ˆj + 14k)
ˆ = -5iˆ - 2jˆ - 20kˆ
uuur uuur
and OQ = - PQ = 5iˆ + 2jˆ + 20kˆ
http://t.me/iitjeehelps

Three Dimensional Geometry M – 91

Chapter
Three Dimensional Geometry
26
1. The ratio in which the join of ( 2, 1, 5) and (3, 4, 3) 8. Find the angle between the two planes 2x + y –
1 2z = 5 and 3x – 6y – 2z = 7. [2013]
is divided by the plane (x + y – z) = is: [2017]
2 (a) cos–1 (4/21) (b) cos–1 (2/21)
(a) 3 : 5 (b) 5 : 7 (c) 1 : 3 (d) 4 : 5 (c) cos–1 (1/21) (d) cos–1 (5/21)
2. If the line through the points A (k, 1, –1) and B 9. What is the value of n so that the angle between
(2k, 0, 2) is perpendicular to the line through the the lines having direction ratios (1, 1, 1) and
points B and C (2 + 2k , k, 1), then what is the (1, –1, n) is 60°? [2012]
value of k? [2017]
(a) 3 (b) 6
(a) –1 (b) 1 (c) –3 (d) 3
(c) 3 (d) None of these
y z 10. The foot of the perpendicular from the point
3. Two lines L1 : x = 5, = ,
3 - a -2 (7, 14, 5) to the plane 2x + 4y – z = 2 are [2012]
y z (a) (1, 2, 8) (b) (3, 2, 8)
L2 : x = a, = are coplanar. Then, a (c) (5, 10, 6) (d) (9, 18, 4)
-1 2 - a
can take value (s) [2015] 11. Find the coordinates of the point where the line
(a) 1, 4, 5 (b) 1, 2, 5 (c) 3, 4, 5 (d) 2, 4, 5 joining the points (2, –3, 1) and (3, – 4, – 5) cuts
4. A line makes the same angle q with each of the the plane [2012]
X and Z-axes. If the angle b, which it makes 2x + y + z = 7.
with Y-axis, is such that sin 2 b = 3sin2 q, then (a) (1, 2, – 7) (b) (1, – 2, 7)
cos2 q equals [2015] (c) (–1, – 2, 7) (d) (1, 2, 7)
(a) 2/5 (b) 1/5 (c) 3/5 (d) 2/3
x -1 y +1 z - 3
x +1 y z - 3 12. Gives the line L : = = and the
5. Find the angle between the line = = 3 2 -1
2 3 6 plane p : x – 2y – z = 0. Of the following asser-
and the plane 10x + 2y – 11z = 3. [2014] tions, the only one that is always true is [2011]
æ 8ö æ 5ö (a) L is ^ to p (b) L lies in p
(a) sin -1 ç ÷ (b) sin -1 ç ÷
è 21ø è 21ø (c) L is not parallel to p (d) None of these
æ 7ö æ 1ö 13. The perpendicular distance of P(1, 2, 3) from the
(c) sin -1 ç ÷ (d) sin -1 ç ÷
è 21ø è 21ø x -6 y-7 z-7
line = = is [2010]
6. The equation of the right bisector plane of the 3 2 -2
segment joining (2, 3, 4) and (6, 7, 8) is [2014] (a) 7 (b) 5 (c) 0 (d) 6
(a) x + y + z + 15 = 0 (b) x + y + z – 15 = 0 14. The equation of the plane containing the line
(c) x – y + z – 15 = 0 (d) None of these
7. The coordinates of the point where the line x - x1 y - y1 z - z1
= = is [2010]
through the points A (3, 4, 1) and B (5, 1, 6) crosses l m n
the XY - plane are [2013, 2010] a (x – x1) + b (y – y1) + c (z – z1) = 0.
æ 13 23 ö æ 13 23 ö Correct option is
(a) ç , ,0÷ (b) ç - , , 0 ÷ (a) ax1 + by1 + cz1 = 0 (b) al + bm + cn = 0
è5 5 ø è 5 5 ø
æ 13 23 ö æ 13 -23 ö a b c
(c) ç , - ,0÷ (d) ç - , ,0÷ (c) = = (d) lx1 + my1 + nz1 = 0
è5 5 ø è 5 5 ø l m n
EBD_7167
http://t.me/iitjeehelps

M – 92 BITSAT Topicwise Solved Papers


20. Find the direction cosines of the line joining the
x-4 y-2 z-k
15. If the line = = lies in the plane points A(6, –7, –1) and B (2, –3, 1). [2006]
1 1 2
2x – 4y + z = 7, then the value of k is [2009] æ 2 2 1ö æ 2 1 1ö
(a) çè ± , ± , ± ÷ø (b) çè ± , ± , ± ÷ø
(a) 4 (b) – 7 3 3 3 3 3 3
(c) 7 (d) No real value æ 1 2 2ö æ 1 1 2ö
16. A line segment has length 63 and direction ra- (c) çè ± , ± , ± ÷ø (d) çè ± , ± , ± ÷ø
3 3 3 3 3 3
tios are 3, –2, 6. If the line makes an obtuse
21. Find the equation of the line [2006]
angle with x–axis, the components of the line
3x – 6y – 2z – 15 = 2x + y – 2z + 5= 0 in parametric
vector are [2009]
form.
(a) 27, – 18, 54 (b) – 27, 18, 54
(c) – 27, 18, –54 (d) 27, – 18, – 54 x + 3 y -1 z x - 3 y -1 z
(a) = = (b) = =
x - 2 y +1 z -1 14 2 15 4 2 15
17. The line, = = intersects the
3 2 -1 x-2 y-3 z x - 3 y +1 z
(c) = = (d) = =
curve xy = c2, z = 0 if c is equal to [2008] 4 2 15 14 2 15
1 22. The point of intersection of the lines
(a) ± 1 (b) ±
3 x +1 y + 3 z + 5
= = and
(c) ± 5 (d) None of these 3 5 7
18. The angle between two planes is equal to x-2 y-4 z-6
= = is [2005]
[2008] 1 3 5
(a) The angle between the tangents to them æ1 1 1ö æ1 1 3ö
from any point (a) ç , , ÷ (b) ç , - , - ÷
è 2 2 2 ø è 2 2 2ø
(b) The angle between the normals to them
from any point æ1 1 2ö
(c) ç ,- , - ÷ (d) None of these
(c) The angle between the lines parallel to the è 3 3 3ø
planes from any point 23. The equation of line of intersection of planes
(d) None of these 4x + 4y – 5z = 12, 8x + 12y – 13z = 32 can be
19. What is the angle between the lines written as [2005]
x -1 y + 2 z x -1 y - 2 z
x - 2 y +1 z + 2 x - 1 2y + 3 z + 5 (a) = = (b) = =
= = and = = ? 2 –3 4 2 3 4
1 -2 1 1 3 2
[2007] x y +1 z - 2 x y z-2
(c) = = (d) = =
2 3 4 2 3 4
p p
(a) (b)
2 3
p
(c) (d) None of these
6

Hints & Solutions


1 1 B
1. (b) As given plane x + y – z = divides the line k
2 (3, 4, 3)
joining the points A (2, 1, 5) and B (3, 4, 3) at a A C
point C in the (2, 1, 5)
ratio k : 1. Then coordinates of C
http://t.me/iitjeehelps

Three Dimensional Geometry M – 93

æ 3k + 2 4k + 1 3k + 5 ö y z
ç , , ÷ L2 : x - a = =
è k + 1 k + 1 k +1 ø -1 2 - a
Point C lies on the plane, Since, these lines are coplanar.
Þ Coordinates of C must satisfy the equation 5-a 0-0 0-0
of plane. 3 - a -2 = 0
Therefore, 0
3k + 2 ö æ 4k + 1 ö æ 3k + 5 ö 1
So, çæ ÷+ç ÷-ç ÷=
0 -1 2 - a
è k + 1 ø è k +1 ø è k + 1 ø 2 Þ (5 – a) (3 – a) (2 – a) – 2 = 0
1 Þ (5 – a) (6 – 3a – 2a + a2 – 2] = 0
Þ 3k + 2 + 4k + 1 – 3k – 5 = (k+ 1) Þ (5 – a) (a2 – 5a + 4] = 0
2
Þ (5 – a) (a – 1)(a – 4) = 0 Þ a = 1, 4, 5
1
Þ 4k – 2 = (k + 1) Þ 8k – 4 = k + 1 4. (c) Let l, m and n be the direction cosines.
2 Then, l = cos q, m = cos b, n = cos q
5 we have l2 + m2 + n2 = 1
Þ 7k = 5 Þ k = Ratio is 5 : 7. Þ cos2 q + cos2 b + cos2 q = 1
7
2. (d) Given points are A (k, 1, –1), B (2k, 0, 2) Þ 2cos2 q + 1 – sin2 b = 1
and C (2 + 2k, k, 1); Let r1 = length of line Þ 2cos2 q – sin2 b = 0
Þ 2cos2 q – 3sin2 b = 0 Þ tan2 q = 2/3
( 2k - k ) + ( 0 - 1) + ( 2 + 1)
2 2 2
AB = 2 1 1 3
\ cos q = = =
1 + tan q 1 + 2 / 3 5
2
= k 2 + 10 5. (a) Let q be the angle between the line and the
and r2 = length of line normal to the plane. Converting the given equa-
BC = ( 2)
2
+ k 2 + ( -1) = k 2 + 5
2 tions into vector form, we have
r
r = (-ˆi + 3k)
ˆ + l (2iˆ + 3jˆ + 6k)
ˆ and
Now, let l1 , m1 , n1 be direction-cosines of line AB
r ˆ ˆ ˆ =3
and l 2 , m2 , n2 be the direction cosines of BC. r.(10i + 2j - 11k)
r r
Since AB is perpendicular to BC Here, b = 2iˆ + 3jˆ + 6kˆ and n = 10iˆ + 2ˆj - 11kˆ
\ l1 l 2 + m1 m2 + n1 n2 = 0
(2iˆ + 3jˆ + 6k).(10i
ˆ ˆ + 2jˆ - 11k)
ˆ
k -1 sin f =
Now, l1 = , m1 = , 22 + 32 + 62 10 2 + 22 + 112
k 2 + 10 k 2 + 10
3 2 -40 -8 8 æ 8ö
n1 = and l 2 = , = = = or f = sin -1 ç ÷
7 ´ 15 21 21 è 21ø
k 2 + 10 k2 + 5
6. (b) If the given points be A (2, 3, 4) and B
k -1
m2 = , n2 = (6, 7, 8), then their mid-point N(4, 5, 6) must lie on
k2 + 5 k2 +5 the plane. The direction ratios of AB are 4, 4, 4, i.e.
So, l1 l 2 + m1 m2 + n1 n2 = 0 1, 1, 1.
A (2, 3, 4)
2k k
Þ -
k 2 + 10 k 2 + 5 k 2 + 10 k 2 + 5
3 N
- =0
k + 10 k 2 + 5
2

Þ 2k - k - 3 = 0 Þ k = 3
For k = 3, AB is perpendicular to BC. B (6, 7, 8)
3. (a) The equations of given lines can be written as \ The required plane passes through N (4, 5, 6)
y z and is normal to AB. Thus its equation is
L1 : x - 5 = = 1(x - 4) + 1( y - 5) + 1(z - 6) = 0 Þ x + y + z = 15
3 - a -2
EBD_7167
http://t.me/iitjeehelps

M – 94 BITSAT Topicwise Solved Papers


7. (a) Equation of the line through the given a - x1 b - y1 g - z1
x - 3 y - 4 z -1 = =
points is = = a b c
5 - 3 1 - 4 6 -1
x -3 y - 4 z -1 æ ax1 + by1 + cz1 + d ö
Þ = = = - çè ÷ ...(1)
2 -3 5 a 2 + b2 + c 2 ø
Any point on this line can be taken as (3 + 2l, 4
In the given ques,, x1 = 7, y1 = 14, z1 = 5,
– 3l, 1 + 5l)
If this point lies on XY-plane then the z- a = 2 b = 4, c = -1, d = -2
coordinate is zero By putting these values in (1), we get
1 a - 7 b - 14 g - 5 63
Þ 1+5l=0Þ l=– = = =-
5
2 4 -1 21
Thus the required coordinates of the point are
Þ a = 1, b = 2 and g = 8
æ 2 æ 1 ö ö æ 13 23 ö
ç 3 - , 4 - 3ç - ÷ , 0 ÷ º ç , , 0 ÷ Hence, foot of ^ is (1, 2, 8)
è 5 è 5ø ø è 5 5 ø 11. (b) The direction ratios of the line are
8. (a) The angle between two planes is the angle 3 – 2, – 4 – (–3), – 5 –1 i.e. 1, –1, – 6
between their normals. From the equation of the Hence equation of the line joining the
r
planes, the normal vectors are N1 = 2iˆ + ˆj - 2kˆ x - 2 y + 3 z -1
r given points is = = = r(say)
and N2 = 3iˆ - 6ˆj - 2kˆ 1 -1 -6
r r Coordinates of any point on this line are
N1.N 2 (r + 2, – r – 3, – 6r + 1)
Therefore, cos q = r r
| N1 | | N 2 | If this point lies on the given plane 2x + y + z = 7,
then 2(r + 2) + (– r – 3) + (– 6r + 1) = 7 Þ r = – 1
(2iˆ + ˆj - 2k).(3i
ˆ ˆ - 6ˆj - 2k)
ˆ æ 4ö Coordinates of any point on this line are
= =ç ÷ (– 1 + 2, – (– 1) – 3, – 6 (–1) + 1) i.e. (1, – 2, 7)
4 + 1 + 4 9 + 36 + 4 è 21ø
12. (b) Since 3(1) + 2(– 2) + (–1) (–1) = 3 – 4 + 1 = 0
\ Given line is ^ to the normal to the planei.e.,
Hence, q = cos-1 æç ö÷
4
è 21ø given line is parallel to the given plane.
9. (b) If (l1, m1, n1) and (l2, m2, n2) are the direction Also, (1, –1, 3) lies on the plane x – 2y – z = 0 if
ratios then angle between the lines is 1 – 2 (–1) – 3 = 0 i.e., 1 + 2 – 3 = 0
which is true \ L lies in plane p.
l1l2 + m1m2 + n1n2 13. (a) The point A (6, 7, 7) is on the line . Let the
cos q =
l1 + m12 + n12 l22 + m22 + n22
2 perpendicular from P meet the line in L. Then
Here l1 = 1, m1 = 1, n1 = 1 and AP 2 = (6 - 1) 2 + (7 - 2) 2 + (7 - 3) 2 = 66
l2 = 1, m2 = – 1, n2 = n and q = 60º P
1´ 1 + 1 ´ (–1) + 1´ n Also AL = projection of AP on line
\ cos 60º =
12 + 12 + 12 ´ 12 + 12 + n 2 æ 3 2 -2 ö
çç actual d.c.' s , , ÷÷ L
1 n è 17 17 17 ø
Þ 2= Þ3(2 + n 2 ) = 4 n 2 A ( 6, 7, 7)
3 2 + n2 3 2 -2
Þ (6 - 1). + (7 - 2). + (7 - 3)
Þ n2 = 6 Þ n = ± 6 17 17 17
10. (a) We know that the length of the
= 17
perpendicular from the point ( x1 , y1 , z1 ) to the
\ ^ distance d of P from the line is given by
plane ax + by + cz + d = 0 is
ax1 + by1 + cz1 + d d 2 = AP 2 - AL2 = 66 - 17 = 49 so that d = 7
14. (b) If the given plane contains the given line
a 2 + b2 + c2 then the normal to the plane, must be
and the co-ordinate (a, b, g ) of the foot of the perpendicular to the line and the condition for
^ are given by the same is al + bm + cn = 0
http://t.me/iitjeehelps

Three Dimensional Geometry M – 95

15. (c) The point (4, 2, k) on the line also lies on


(3iˆ - 6jˆ - 2k)
ˆ ´ (2iˆ + ˆj -= ˆ 14iˆ + 2ˆj + 15kˆ is
2k)
the plane 2x – 4y + z = 7.
So, 8 – 8 + k = 7 Þ k = 7 parallel to the required line i.e., ratios of the
16. (c) Let the components of the line vector be a, b, c. required line are 14, 2 and 15.
Then a2 + b2 + c2 = (63)2 ... (i) Now for a point on the required line, we find the
a b c point of intersection of the required line with the
Also = = = l (say), then a = 3l , xy plane (having equation z = 0). The x and y
3 -2 6 coordinates of the point on the plane z = 0 where
b = –2l and c = 6l and from (i) we have the required line intersects it is the solution of
9l 2 + 4l 2 + 36l 2 = (63) 2 Þ 4 9l 2 = (63) 2 3x – 6y = 15 and 2x + y = 5 Þ x = 3, y = 1.
63 Hence required point is (3, –1, 0).
Þ l=± = ±9 x - 3 y +1 z
7 The required equation is = =
Since a = 3l < 0 as the line makes an obtuse 14 2 15
angle with x-axis, l = –9 and the required compo- x +1 y + 3 z + 5
nents are –27, 18, –54. 22. (b) Let = = =l
3 5 7
17. (c) We have, z = 0 for the point where the line
intersects the curve. Therefore, So that a general point on it is
x - 2 y + 1 0 -1 P(3l - 1, 5l - 3, 7l - 5) .
= =
3 2 -1 x -2 y -4 z -6
Let = = =m
x-2 y +1 1 3 5
Þ = 1 and = 1 Þ x = 5 and y = 1
3 2 So that a general point on it is
Put these value in xy = c2, we get, 5 = c2 Q(m + 2, 3m + 4, 5m + 6) .
Þc=± 5 The two lines intersect if P and Q coincide
18. (b) It is a fundamental concept. \ 3l - 1 = m + 2, 5l - 3 = 3m + 4, 7l - 5 = 5m + 6
19. (a) The given lines are:- 1 3
Solving first two eqs., we get l = , m = -
x - 2 y - ( -1) z - ( -2 ) 2 2
= = and
1 -2 1 These values satisfy the third equation.
Hence, co-ordinates of point of intersection are
æ 3ö
y-ç- ÷
x -1
= è 2 ø = z - ( -5 ) æ1 1
ç , - , - ÷.

1 3 2 è 2 2 2ø
2 23. (b) We have the equations of lines of planes
dr’s of Ist line are:- a1 = 1, b1 = –2, c1 = 1 are
dr’s of IInd line are:- a2 = 2, b2 = 3, c2 = 4 4x + 4y – 5z = 12; 8x + 12y – 13z = 32
Let ‘q’ be the angle b/w two lines, then, If d’rs of the line are (x, y, z) º (l, m, n) then
equations are
a1 a 2 + b1 b2 + c1 c2 4l + 4m – 5n = 0 ..... (i)
cos q =
8l + 12m –13n = 0 ..... (ii)
a12 + b12 + c12 . a 22 + b22 + c 22
From (i) and (ii)
cos q = 0
l m n l m n
p = = ; = =
Þ q= 8 12 16 2 3 4
2
At z = 0 planes are
20. (a) Direction ratios of AB are (4, – 4, –2)
= (2, –2, –1) a2 + b2 + c2 = 9 4x + 4y = 12 Þ x + y = 3 ..... (iii)
8x + 12y = 32 Þ 2x + 3y = 8 ..... (iv)
æ 2 2 1ö Put value of x from (iii) in (iv), we get
Direction cosines are çè ± , ± , ± ÷ø .
3 3 3 2 (3 – y) + 3y = 8 Þ 6 – 2y + 3y = 8
Þ y = 2 and x = 3 – 2 = 1 Þ x = 1, z = 0
21. (d) 3iˆ - 2ˆj - 2kˆ and 2iˆ - ˆj - 2kˆ are the vectors
perpendicular to the given planes and hence to x -1 y - 2 z
\ The equation of line = =
the required lin e. Hence 2 3 4
EBD_7167
http://t.me/iitjeehelps

M – 96 BITSAT Topicwise Solved Papers

Chapter
Probability
27
1. One mapping is selected at random from all whereas the other coins are fair. One coin is
mappings of the set S = {1, 2, 3, ......n} into itself. selected at random and tossed. If the probability
3 7
The probability that it is one-one is . Then the that toss results in heads is , then the value
32 12
value of n is [2017] of n is. [2014]
(a) 3 (b) 4 (c) 5 (d) 6 (a) 3 (b) 4
2. A random variable X has the probability (c) 5 (d) None of these
distribution 8. A coin is tossed 7 times. Each time a man calls
head. Find the probability that he wins the toss
X 1 2 3 4 5 6 7 8
on more occasions. [2014]
p(X) 0.15 0.23 0.12 0.10 0.20 0.08 0.07 0.05
2 1 3 1
For the events E = {X is a prime number} and (a) (b) (c) (d)
3 2 4 3
F = {X < 4}, then P(E È F) is [2017] 9. For k = 1, 2, 3 the box Bk contains k red balls and
1 1
(a) 0.50 (b) 0.77 (c) 0.35 (d) 0.87 (k + 1) white balls. Let P(B1 ) = , P(B2 ) =
3. A man takes a step forward with probability 0.4 2 3
and backward with probability 0.6. The 1
and P(B3 )= . A box is selected at random and
probability that at the end of eleven steps he is 6
one step away from the starting point is [2016] a ball is drawn from it. If a red ball is drawn, then
the probability that it has come from box B2, is
2 5.35 æ 6 ö
5 [2013, 2011]
(a) (b) 462 ´ ç ÷ 35 14 10 12
510 è 25 ø (a) (b) (c) (d)
78 39 13 13
35 10. The probability of India winning a test match
(c) 231´ (d) None of these
510 against West Indies is 1/2. Assuming indepen-
4. A bag contains (2n + 1) coins. It is known that n dence from match to match, the probability that
of these coins have a head on both sides, in a 5 match series India’s second win occurs at
whereas the remaining (n + 1) coins are fair. A the third test, is – [2013]
coin is picked up at random from the bag and (a) 2/3 (b) 1/2 (c) 1/4 (d) 1/8
tossed. If the probability that the toss results in 11. A boy is throwing stones at a target. The
a head is 31/42, then n is equal to [2015] 1
(a) 10 (b) 11 (c) 12 (d) 13 probability of hitting the target at any trial is .
2
5. A bag contains 3 red and 3 white balls. Two balls The probability of hitting the target 5th time at
are drawn one by one. The probability that they the 10th throw is : [2012]
are of different colours is. ` [2015] 10
5 63 C5
(a) 3/10 (b) 2/5 (a) (b) (c) (d) None
10 9 10
(c) 3/5 (d) None of these 2 2 2
12. Two dice are thrown together 4 times. The
16
6. If in a binomial distribution n = 4, P(X = 0) = , probability that both dice will show same
81 numbers twice is - [2012]
then P(X = 4) equals [2015]
1 25
1 1 1 1 (a) (b)
(a) (b) (c) (d) 3 36
16 81 27 8
7. A bag contains n + 1 coins. It is known that one 25
(c) (d) None of these
of these coins shows heads on both sides, 216
http://t.me/iitjeehelps

Probability M – 97

13. If mean of a poisson distribution of a random (c) P(AB) = P(A) + P(B) – P (A È B)


variable X is 2, then the value of P(X > 1.5) is (d) If A and B are independent events then
[2010] P(AB) = 0
3 3 18. A fair coin is tossed 99 times. If X is the number of
(a) (b) times head occurs, P(X = r) is maximum when r is
e2 e
3 (a) 49 or 50 (b) 50 or 51 [2007]
3
(c) 1 – (d) 1 – 2 (c) 51 (d) None of these
e e 19. Bag A contains 4 green and 3 red balls and Bag B
2 1 1 contains 4 red and 3 green balls. One bag is taken
14. If P(A È B) = , P(A Ç B) = and P(A) = at random and a ball is noted it is green. The prob-
3 6 3
then [2010] ability that it comes from Bag B is [2007]
(a) A and B are independent events 2 2 3 1
(a) (b) (c) (d)
(b) A and B are disjoint events 7 3 7 3
(c) A and B are dependent events 20. A problem in mathematics is given to three
(d) None of these students A, B, C and their respective probability
15. It is given that the events A and B are such that 1 1 1
of solving the problem is , and .
1 1 2 2 3 4
P ( A) = , P ( A | B ) = an d P ( B | A) = .
4 2 3 Probability that the problem is solved is [2006]
Then P(B) is [2009, 2006] 3 1 2 1
1 1 2 1 (a) (b) (c) (d)
4 2 3 3
(a) (b) (c) (d) 21. A man is known to speak the truth 3 out of 4
6 3 3 2
16. The random variable X has the following times. He throws a die and reports that it is a six.
probability distribution The probability that it is actually a six is [2005]
3 1 3 1
x 0 1 2 3 4 (a) (b) (c) (d)
8 5 4 2
P(X = x) k 3k 5k 2k k 22. Let A and B be two events such that [2005]
Then the value of P(X ³ 2) is [2009] 2 7 1
1 2 3 1 P(A) = , P(A È B) = and P(B / A) = ,
(a) (b) (c) (d) 5 10 2
3 3 4 4 then P (B) =
17. If A and B are two events, then which is not (a) 1/2 (b) 23/100
correct - [2008] (c) 2/5 (d) None of these
(a) P(ABC) = P (A) – P (AB)
(b) P(ABC) + P(ACB) = P (A È B) – P (AB)

Hints & Solutions


n! 3 n! 8´3 4! be a success and a step backward be a failure.
1. (b) = Þ n = = Then, the probability of success in one step
n n 32 n 8 ´ 32 4 4 2
\ n=4 = p = 0.4 =
5
2. (b) P(E) = P ( 2 or 3 or 5 or 7) The probability of failure in one step
= 0.23 + 0.12 + 0.20 + 0.07 = 0.62 3
P ( F ) = P (1 or 2 or 3) = q = 0.6 = .
5
= 0.15 + 0.23 + 0.12 = 0.50 In 11 steps he will be one step away from
P ( E Ç F ) = P(2 or 3) = 0.23 + 0.12 = 0.35 the starting point if the numbers of
\ P( EUF ) = P( E ) + P ( F ) - P ( E Ç F ) successes and failures differ by 1.
So, the no. of successes = 6
= 0.62 + 0.50 - 0.35 = 0.77 The no. of failures = 5 or the no. of successes = 5,
3. (b) As 0.4 + 0.6 = 1, the man either takes a step The no. of failures = 6
forward or a step backward. Let a step forward
EBD_7167
http://t.me/iitjeehelps

M – 98 BITSAT Topicwise Solved Papers


4
\ the required probability æ1ö 1
\ P(X = 4) = 4C4p4q0 = p4 = ç ÷ =
= 11C6p6q5 + 11C5p5q6 3
è ø 81
6 5 5 6 7. (c) Let E1 denote the event "a coin with head
11 æ 2 ö æ 3 ö 11 æ 2 ö æ 3 ö on both sides is selected" and E2 denotes
= C 6 ç ÷ .ç ÷ + C 5 ç ÷ .ç ÷
è5ø è5ø è5ø è5ø the event " a fair coin is selected". Let A
5 5
11! æ 2 ö æ 3 ö ì 2 3 ü be the event " he toss, results in heads".
= .ç ÷ .ç ÷ .í + ý 1 n
6 !5 ! è 5 ø è 5 ø î 5 5 þ \ P(E1 ) = , P(E 2 ) = and
5 n +1 n +1
11.10.9.8.7 2 5.35 æ 6 ö
= . = 462 ´ ç ÷
120 510 è 25 ø æAö æ A ö 1
P ç ÷ =1 , P ç ÷=
4. (a) Total number of coins= 2n+1 è E1 ø è E2 ø 2
Consider the following events: æAö æ A ö
E1 = Getting a coin having head on both \ P(A) = P(E1 )P ç ÷ + P(E 2 )P ç ÷
sides from the bag. E
è 1ø è E2 ø
E2 = Getting a fair coin from the bag 7 1 n 1
Þ = ´1+ ´
A = Toss results in a head 12 n + 1 n +1 2
31 n Þ 14n + 14 = 24 + 12 n Þ n = 5
Given: P ( A ) = , P ( E1 ) =
8. (b) The man has to win at least 4 times.
42 2n + 1
n +1 \ Reqd. probability =
and P ( E 2 ) = 4 3 5 2
2n + 1 7 æ 1ö æ 1ö æ 1ö æ 1ö
Then, C4 ç ÷ . ç ÷ + 7 C 5 ç ÷ .ç ÷
è 2ø è 2ø è 2ø è 2ø
( ) (
P ( A ) = P ( E1 ) P A E1 + P ( E 2 ) P A E 2 ) 6 7
æ 1ö 1 æ 1ö
31 n n +1 1 + 7 C6 ç ÷ . + 7 C7 ç ÷
Þ = ´1 + ´ è 2ø 2 è 2ø
42 2n + 1 2n + 1 2
1 64 1
31 n n +1 = (7C4 + 7C5 + 7C7 + 7C7) . = =
= + 27 27 2
42 2n + 1 2 ( 2n + 1) 9. (b) In a box, B1 = 1R, 2W; B2 = 2R, 3W and
31 3n + 1 B3 = 3R, 4W
31 3n + 1
Þ 42 = 2 ( 2n + 1) Þ = Þ n = 10 1
Also, given that, P(B1)= , P(B2 ) =
1
21 2n + 1 2 3
5. (c) Let A º event that drawn ball is red 1
and P(B3 )=
B º event that drawn ball is white 6 æ R ö
P(B2 )P ç ÷
Then AB and BA are two disjoint cases of æB ö
Pç 2 ÷= è B2 ø
\ è R ø æRö æ R ö æ R ö
the given event. P(B1)P ç ÷ +P(B2 )P ç ÷ +P(B3 )P ç B ÷
è B1 ø è B2 ø è 3ø
\ P (AB + BA) = P(AB) + P (BA)
1 2 2
æ Bö æ Aö 3 3 3 3 3 ´
3 5 15 14
= P(A) P ç ÷ + P(B) P ç ÷ = . + . = = = = .
è Aø è Bø 6 5 6 5 5 1 1 1 2 1 3 1 2 1 39
´ + ´ + ´ + +
16 2 3 3 5 6 7 6 15 14
6. (b) Given n = 4 and P(X = 0) = 10. (c) The sample space is [LWW, WLW]
81
\ P (LWW) + P (WLW)
Let p be the probability of success and q = Probability that in 5 match series, it is India’s
that of failure in a trial. second win
16 = P (L) P (W) P (W) + P (W) P (L) P (W) =
Then, P(X = 0) = 4C0p0q4 =
81 1 1 2 1
+ = =
2
4
1 8 8 8 4
4 æ2ö
Þ q =ç ÷ ,Þ q= Þp= 11. (b) The probability of hitting the target 5th time
è3ø 3 3 at the 10th throw = P(the probability of hitting
the target 4 times in the first 9 throws) × (the
http://t.me/iitjeehelps

Probability M – 99

probability of hitting the target at the 10th throw) 1


(100) ( ) = 50 so that the maximum value of
é 4 5ù 10 2
9 æ ö æ ö
1 1 æ1ö 9! æ 1 ö 63 P(X = r) occurs at r = (n + 1) p = 50 and at r = (n + 1)
= ê C 4 ç ÷ ç ÷ úç ÷ = ´ç ÷ =
êë è2ø è2ø úûè 2 ø 4!5! è 2 ø 29 p–1 = 49
19. (c) P æç
12. (c) The probability of showing same number by Bag B ö
=
both dice p =
6 1
= è Green ÷ø
36 6 æ green ö
1 P (Bag B) ´ P ç
In binomial distribution here n = 4, r = 2, p = ,
6 è Bag B ÷ø
5 æ green ö æ green ö
q= P (Bag B) ´ P ç + P (Bag A) ´ P ç
6 è Bag B ÷ø è Bag A ÷ø
\ req. probability = n C r q n–r p r = 4 C 2
2 2 1 3
æ 5ö æ 1 ö ´
çè ÷ø çè ÷ø 2 7 3
6 6 = =
1 3 1 4 7
æ 25 ö æ 1 ö 25 ´ + ´
= 6 çè ÷ø çè ÷ø = 2 7 2 7
36 36 216
1 1 1
e– l lr 20. (a) P(E1) = , P (E2) = and P (E3) =
2 3 _ 4 _
;
13. (d) Since, P (X = r) = (where l = mean) _
r! P ( E1UE2UE3 ) = 1 - P ( E1 ) P ( E 2 ) P ( E 3 )
\ P(X = r > 1.5) = P (2) + P (3) + ... ¥
= 1 – P (X = r £ 1) = 1 – P (0) – P(1) æ 1ö æ 1ö æ 1ö
= 1 - ç1 - ÷ ç 1 - ÷ ç1 - ÷
æ è 2 ø è 3ø è 4ø
e –2 ´ 2 ö 3
= 1 – ç e –2 + ÷ =1 – 2 1 2 3 3
è 1! ø e =1 - ´ ´ =
2 3 4 4
14. (a) P(A È B) = P(A) + P(B) – P(AB) 1
21. (a) P(Six occurs) = P ( S1 ) =
2 1 1 1 6
Þ = + P(B) – Þ P(B) = 5
3 3 6 2 P(Six does not occur) = P(S2) =
Now, P(AB) = P(A) P(B), A and B are 6
independent events 3
P ( E | S1 ) = P(man speaks truth) =
1 4
15. (b) P(A) = 1/4, P(A/B) = , P(B/A) = 2/3
2 P ( E | S2 ) = P(man does not speak the truth)
By conditional probability, 1
=
P(A Ç B) = P(A) P(B/A) = P(B)P(A/B) 4
1 2 1
Þ ´ = P ( B ) ´ Þ P( B ) =
1 \ P ( S1 | E ) = P (he reports that six has
4 3 2 3 occured is actually a six)
16. (b) Since, å Pi ( X = x ) = 1 1 3
P ( S1 ) P ( E | S1 ) ´
3
\ k + 3k + 5k + 2k + k = 1\12k = 1 \ k =
1 = = 1 36 54 1 =
P (S1 ) P ( E | S1 ) + P ( S2 ) P ( E | S2 ) ´ + ´
8
12
Now, P(X ³ 2) = P(X = 2) + P(X = 3) + P(X = 4) 6 4 6 4
7
æ 1ö 2 22. (a) = P(A È B) = P(A) + P(B) - P(A Ç B)
= 5k + 2k + k = 8k = 8 ç ÷ = 10
è 12 ø 3 2
17. (d) If A and B are mutually exclusive events then =+ P(B) - P(A).P(B / A)
5
P(AB) = 0 and when A and B are independent 3 1 1
3 2 1
events then P(AB) = P(A). P(B) \ = P(B) - ´ \ P(B) = + =
10 5 2 10 5 2
1
18. (a) Putting n = 99 and p = ,we have (n + 1)p =
2
EBD_7167
http://t.me/iitjeehelps

M – 100 BITSAT Topicwise Solved Papers

Chapter
Properties of Triangles
28
1. An observer on the top of a tree, finds the angle (a) sin (A + B) (b) sin (A – B)
of depression of a car moving towards the tree
to be 30°. After 3 minutes this angle becomes æA-Bö
(c) cos (A + B) (d) sin ç ÷
60°. After how much more time, the car will reach è 2 ø
the tree? [2017] 9. A person standing on the bank of a river
(a) 4 min. (b) 4.5 m (c) 1.5 min(d) 2 min. observes that the angle subtended by a tree on
2. In a DABC, if cos A = cos B = cos C , and the the opposite bank is 60º. when he retreats 20
a b c feet from the bank, he finds the angle to be 30º.
side a = 2, then area of the triangle is [2016] The breadth of the river in feet is : [2009, 2006]

3 (a) 15 (b) 15 3 (c) 10 3 (d) 10


(a) 1 (b) 2 (c) (d) 3
2 10. In a DABC , if angle C is obtuse, then [2008]
3. In a DABC, the lengths of the two larger sides (a) tan A tan B < 1 (b) tan A tan B £ 1
are 10 and 9 units, respectively. If the angles are (c) tan A tan B > 1 (d) None of these
in AP, then the length of the third side can be 11. The top of a hill observed from the top and
[2015] bottom of a building of height h is at angles of
(a) 5 ± 6 (b) 3 3 elevation p and q respectively. The height of
(c) 5 (d) None of these the hill is [2008]
4. In a triangle ABC, if a = 2, B = 60° and C = 75°, h cos q h cot p
then b equals [2012] (a) (b)
cot q - cot p cot p - cot q
(a) 3 (b) 6 (c) 9 (d) 1+ 2
5. A ladder rests against a wall so that its top h tan p
(c) (d) none of these
touches the roof of the house. If the ladder makes tan p - tan q
an angle of 60° with the horizontal and height of 12. The horizontal distance between two towers is 60
the house be 6 3 meters, then the length of metres and the angular depression of the top of
the ladder in meters is [2011] the first tower as seen from the top of the second.
(a) 12 3 (b) 12 is 30°. If the height of the second tower be 150
metres, then the height of the first tower is [2007]
(c) 12 / 3 (d) None of these (a) 150 - 60 3m (b) 90 m
6. In an equilateral triangle, the in radius, circum-
radius and one of the ex-radii are in the ratio (c) 150 - 20 3m (d) None of these
[2011] 13. If the elevation of the sun is 30° then the length
(a) 2 : 3 : 5 (b) 1 : 2 : 3 (c) 3 : 7 : 9 (d) 3 : 7 : 9 of the shadow cast by a tower of 150 metres
7. If a flagstaff of 6 metres high placed on the top height is [2006]
of a tower throws a shadow of 2 3 metres along (a) 75 3 m (b) 200 3 m
the ground, then the angle (in degrees) that the (c) 150 3 m (d) None of these
sun makes with the ground is [2010]
(a) 60° (b) 80° 14. An aeroplane flying horizontally 1 km above the
(c) 75° (d) None of these ground is observed at an elevation of 60° and
after 10 s the elevation is observed to be 30°.
a 2 - b2 The uniform speed of the aeroplane in kilometre
8. In a triangle ABC, ÐC = 90°, then is per hour is [2005]
a 2 + b2
equal to : [2009] (a) 60 3 (b) 240 (c) 240 3 (d) 480
http://t.me/iitjeehelps

Properties of Triangles M – 101

15. If A = 75°, B = 45°, then b + c 2 equals (a) a (b) a + b + c


(c) 2a (d) (1/2) (a + b + c)
[2005]

Hints & Solutions


1. (c) d = h cot 30° – h cot 60° and time = 3 min. 3a
3 2
h (cot 30° - cot 60°) 6. (b) We have D = a ,s=
\ Speed = per minute 4 2
3 D a abc a3 a
\ r= = , R= = =
It will travel distance s 2 3 4D 3a 2
3
2
h cot 60° in h D 3 / 4a 3
and r1 = = = a
h cot 60° ´ 3 s-a a/2 2
60° 30°
h (cot 30° - cot 60°) d a a 3
Hence, r : R : r 1 = : : a =1:2:3
= 1.5minute 2 3 3 2
cos A cos B cos C 7. (a) Accordingly,
2. (d) = =
a b c h h+6 6
cos A cos B cos C
tan q = = = Þ q = 60°
Þ = = x x+2 3 2 3
2R sin A 2R sin B 2R sin C [Since the triangles AEC and BDC are similar]
A
Þ cot A = cot B = cot C
Þ A = B = C = 60° Þ DABC is equilateral h cot a = (h – 100) cot b
6m
100 cot b
3 2 \h = B
Hence, area of D = a = 3. cot b - cot a
4 h
3. (a) Let A, B and C be the three angles of DABC E
q q
and let a = 10 and b = 9 2 3mD h C
It is given that the angles are in AP. 8. (b) A + B = 180° – C = 90°
\ 2B = A + C on adding B both the sides, a = 2R sin A, b = 2R sin B, c = 2R sin C
we get 3B = A + B + C Þ 3B = 180° Þ B = 60°
a 2 - b 2 sin 2 A - sin 2 B
2
a +c -b 2 2 \ =
Now, we know cos B = a 2 + b 2 sin 2 A + sin 2 B
2ac sin( A + B) sin( A - B)
=
10 2 + c2 - 92 sin 2 A + sin 2 (90° - A )
Þ cos 60° =
2 ´ 10 ´ c [Q A + B = 90°]
1 100 + c2 - 81 sin 90° sin( A - B)
Þ = = = sin( A - B)
2 20c sin 2 A + cos 2 A
9. (d) Let h be the height of tree PQ and breadth
Þ c2 – 10c + 19 = 0 Þ c = 5 ± 6 of river PS be x ft. Angle of elevation
4. (b) A = 180° – 60° – 75° = 180° – 135° = 45° subtended by a tree is 60°. Also, when he
a b retreats 20 feet, the angle becomes 30°.
Now, =
sin A sin B Also, in DPQS, Q

2 b 2.( 3 / 2) h
Þ = Þ b= = 6 tan 60° =
sin 45° sin 60° 1/ 2 x h
6 3
5. (b) Length of ladder = = 12m Þ h = 3x
sin 60° 30° 60°
and in DPQR, R 20 ft. S x P
EBD_7167
http://t.me/iitjeehelps

M – 102 BITSAT Topicwise Solved Papers

1 h h cot p
tan 30° =
h Þ = =h+x =
3 x + 20 cot p - cot q
x + 20
12. (c) (150 – h) cot 30° = 60
B
Þ x + 20 = 3h Þ h = 150 – 20 3
Þ x + 20 = 3 x Þ 2 x = 20 Þ x = 10

150m
Hence breadth of river is 10 ft.
10. (a) Since A + B + C = p A 30°
h
\ A + B = p - C Þ tan( A + B) = tan( p - C) O 60 m O
tan A + tan B 13. (c) Height of the tower = AB = 150 m
Þ = - tan C Angle of elevation of the sun = 30°
1 - tan A tan B

Þ tan A + tan B Let AC be the length


B
> 0 [Q angle C is obtuse
1 - tan A tan B of the shadow cast
\ tan C < 0] by the tower,

150 m
But C is obtuse angle , so A and B will both Now
p 30°
be less than AC
2 = cot 30° = 3 A C
\ Both tan A and tan B are positive. 150
Hence from (1),
\ AC = 150 3 m
1 - tan A tan B > 0 Þ tan A tan B < 1
11. (b) Let DE be the building of height h and d
14. (c)
AC be the hill. Then In D ADC
h+x
tan q = .... (i)
y H 1 km
x 60°
and in D AEB tan p = .... (ii)
y A 30°

From (i) and (ii), x d = H cot 30° - H cot 60°


h+x p Time taken = 10 second
tan q = E B
x cot p y \ speed
Þ x cot p = (h + x) cot q h h
q cot 30° - cot 60°
h cot q D C = ´ 60 ´ 60 = 240 3
x= 10
cot p - cot q 15. (c) C = 180° – 120° = 60°
h cot q
h+x=h+ a b c
cot p - cot q Use sine rule = = =k
h cot p - h cot q + h cot q sin 75° sin 45° sin 60°
=
cot p - cot q Þ (b + c 2 ) = k (sin 45° + 2 sin 60°)
h cot p
Þ h+x=
cot p - cot q 3 +1 3 +1
=k = 2k = 2k sin 75° = 2k sin
2 2 2
Hence, height of the hill AC
A = 2a
http://t.me/iitjeehelps

Linear Programming M – 103

Chapter
Linear Programming
29
1. The maximum value of z = 3x + 2y subject to x y
x + 2y ³ 2, x + 2y £ 8, x, y ³ 0 is : [2017] (a) 0.08 x + 0.10 y (b) +
(a) 32 (b) 24 2000 2500
(c) 40 (d) None of these x y
n m (c) 2000x + 2500 y (d) +
8 10
2. Minimise Z = åå cij xij 6. For the constraints of a L.P. Problem given by
j=1 i =1 x1 + 2x2 £ 2000, x1 + x2 £ 1500 and x 2 £ 600 and
m x1, x2 ³ 0, which one of the following points does
Subject to å x ij = b j , j = 1, 2,..., n not lie in the positive bounded region [2011]
i =1 (a) (1000, 0) (b) (0, 500)
n (c) (2, 0) (d) (2000, 0)
å x ij = b j , i = 1, 2,..., m is a LPP with number
7. A wholesale merchant wants to start the
business of cereal with ` 24000. Wheat is ` 400
j=1
of constraints [2015] per quintal and rice is ` 600 per quintal. He has
capacity to store 200 quintal cereal. He earns the
m profit ` 25 per quintal on wheat and ` 40 per
(a) m – n (b) mn (c) m + n (d)
n quintal on rice. If he store x quintal rice and y
quintal wheat, then for maximum profit the
x y x y objective function is [2010]
3. Consider + ³ 1 and + £ 1, x, y ³ 0.
2 4 3 2 (a) 25 x + 40 y (b) 40x + 25 y
Then number of possible solutions are :
[2014, 2013] 400 600
(c) 400x + 600y (d) x+ y
(a) Zero (b) Unique 40 25
(c) Infinite (d) None of these 8. The minimum value of the function z = 4x + 3y
4. A shopkeeper wants to purchase two article A subject to the constraints 3x + 2y ³ 160, 5x + 2y
and B of cost price ` 4 and 3 respectively. He ³ 200, x + 2y ³ 80, x ³ 0, y ³ 0 is [2009]
thought that he may earn 30 paise by selling (a) 320 (b) 300 (c) 220 (d) 200
article A and 10 paise by selling article B. He has 9. The constraints –x1 + x2 £ 1, –x1 +3x2 £ 9,
not to purchase total article worth more than x1,x2 ³ 0 define on [2008]
` 24. If he purchases the number of articles of A (a) Bounded feasible space.
and B, x and y respectively, then linear (b) Unbounded feasible space.
constraints are [2013] (c) Both bounded and unbounded feasible
(a) x ³ 0, y ³ 0, 4x +3 y £ 24 space.
(b) x ³ 0, y ³ 0, 30x + 10 y £ 24 (d) None of these.
(c) x ³ 0, y ³ 0, 4x +3 y ³ 24 10. The maximum value of z = 3x + 4y subject to the
(d) x ³ 0, y ³ 0, 30x +40 y ³ 24 constraints x + y £ 40, x + 2y £ 60, x ³ 0, y ³ 0 is
5. Prabhat wants to invest the total amount of [2007]
` 15,000 in saving certificates and national saving (a) 120 (b) 140 (c) 100 (d) 160
bonds. According to rules, he has to invest at 11. The maximum value of z = 3x + 4y subject to the
least ` 2000 in saving certificates and ` 2500 in condition x + y £ 40 , x + 2y £ 60, x,y ³ 0 is [2007]
national saving bonds. The interest rate is 8% (a) 130 (b) 120 (c) 40 (d) 140
on saving certificate and 10% on national saving 12. The point at which the maximum value of ( 3x +
bonds per annum. He invest ` x in saving 2y) subject to the constraints x + y £ 2, x ³ 0,
certificate and ` y in national saving bonds. Then the y ³ 0 is obtained, is [2006]
objective function for this problem is [2012] (a) (0, 0) (b) (1.5, 1.5) (c) (2, 0) (d) (0, 2)
EBD_7167
http://t.me/iitjeehelps

M – 104 BITSAT Topicwise Solved Papers


13. The solution set of constraints x + 2y ³ 11, (a) (2, 3) (b) (3, 2)
3x + 4y £ 30, 2x + 5y £ 30 and x ³ 0 , y ³ 0 , (c) (3, 4) (d) (4, 3)
includes the point [2005]

Hints & Solutions


1. (b) Given : x + 2y ³ 2 .....(i) From graph region is finite but numbers of
x + 2y £ 8 .....(ii) possible solutions are infinite because for
different values of x and y we have different or
and x, y ³ 0
infinite no. of solutions.
Y
4. (a) x, y ³ 0 and 4x + 3y £ 24.
(0, 4)
x+
2y <
8
5. (a) The function is given by profit function
(0, 1) (8, 0)
X 8 10
(2, 0) = x. + y´ = 0.08 + 0.10y.
x+ 100 100
2y ³
2
6. (d) Clearly point (2000, 0) is outside.
x y
For equation (i) + = 1 7. (b) For maximum profit, z = 40x + 25y.
2 1
x y 8. (c)
and for equation (ii) + = 1 9. (b) It is clear from the graph, the constraints
8 4 define the unbounded feasible space.
Given : z = 3x + 2y X2
At point (2, 0); z = 3 × 2 + 0 = 6
At point (0, 1); z = 3 × 0 + 2 × 1 = 2
At point (8, 0); z = 3 × 8 + 2 × 0 = 24 (0,3)
At point (0, 4); z = 3 × 0 +2 × 4 = 8
\ maximum value of z is 24 at point (8, 0). (0,1)
(–9,0)
X1
2. (c) Constraints will be (–1,0) O
x11 + x21 + .... + xm1 = b1 ; x12 + x22 + ..... xm2 = b2
x1n + x2n + .... + xmn = bn ; x11 + x12 + ... + x1n = b1 10. (b)
x21 + x22 + ...+ x2n = b2 ; xm1 + xm2 + ... +xmn = bn 11. (d) Obviously, z = 3x + 4y will be maximum at
So, total number of constraints = m + n (20, 20)
\ Max z = 3 × 20 + 4 × 20 = 60 + 80 = 140
x y x y
3. (c) Consider + ³ 1, + £ 1,
2 4 3 2 12. (c) Hence maximum z is at ( 2, 0).
x, y ³ 0 convert them into equation and solve Y
them and draw the graph of these equations
we get y = 1 and x = 3/2 (0,2)

Y
x+y=2
(0, 4)

(0, 2) (3/2, 1)
X (2,0)
(2, 0) (3, 0) X
x
y
O
3 + £1
2
13. (c) Obviously, solution set of constraints
x + 4
2

y ³

included the point (3, 4).


1
http://t.me/iitjeehelps

SECTION IV - ENGLISH PROFICIENCY& LOGICAL REASONING

UNIT -1 ENGLISH PROFICIENCY

Chapter
Vocabulary & Grammar
1
DIRECTIONS (Qs. 1 - 3): In the following question, 11. EMBEZZLE
out of the four alternatives, choose the one which best (a) Misappropriate (b) Balance
expresses the meaning of the given word. [2017] (c) Remunerate (d) Clear
1. LOQUACIOUS DIRECTIONS (Qs. 12 - 13) : Choose the word which
(a) Talkative (b) Slow is the exact OPPOSITE of the given words. [2016]
(c) Content (d) Unclear 12. ARROGANT
2. EMBEZZLE (a) Humble (b) Cowardly
(a) Misappropriate (b) Balance
(c) Egotistic (d) Gentlemanly
(c) Remunerate (d) Clear
3. INDICT 13. EXODUS
(a) Condemn (b) Reprimand (a) Influx (b) Home-coming
(c) Accuse (d) Allege (c) Return (d) Restoration
DIRECTIONS (Qs. 4 - 6): Choose the word opposite DIRECTIONS (Qs. 14 - 16) : Read the each sentence to
in meaning to the given word. [2017] find out whether there is any grammatical error in it. The
4. METICULOUS error, if any will be in one part of the sentence. The letter of
(a) Forgetful (b) Destructive that part is the answer. If there is no error, the answer is 'd'.
(c) Careless (d) Flagrant (Ignore the errors of punctuation, if any). [2016]
5. EXODUS
14. (a) We discussed about the problem so
(a) Influx (b) Home-coming
(c) Return (d) Restoration thoroughly
6. FLAGITIOUS (b) on the eve of the examination
(a) Innocent (b) Vapid (c) that I found it very easy to work it out.
(c) Ignorant (d) Frivolous (d) No error.
DIRECTIONS (Qs. 7 - 9) : In questions, some parts of 15. (a) An Indian ship
the sentences have errors and some are correct. Find (b) laden with merchandise
out which part of a sentence has an error. If a sentence (c) got drowned in the Pacific Ocean.
is free from error, mark (d) in your answer. [2017] (d) No error.
7. My father gave me (a) / a pair of binocular (b) / 16. (a) I could not put up in a hotel
on my birthday. (c) / No error. (d) (b) because the boarding and lodging charges
8. The teacher as well as his students, (a) / all left
(c) were exorbitant.
(b) / for the trip. (c) / No error. (d)
9. (a) I could not put up in a hotel/ (b) because the (d) No error.
boarding and lodgin g char ges/ (c) were DIRECTIONS (Qs. 17 - 19): Choose the word which
exorbitant./ (d) No error. best expreses the meaning of the underlined word in
DIRECTIONS (Qs. 10 - 11) : Choose the word which the sentence. [2015]
best expresses the meaning of the given word. [2016]
17. Decay is an immutable factor of human life.
10. CORPULENT (a) important (b) unique
(a) Lean (b) Gaunt (c) unchangeable (d) awful
(c) Emaciated (d) Obese
EBD_7167
http://t.me/iitjeehelps

EL – 2 BITSAT Topicwise Solved Papers


18. It was an ignominious defect for the team. 28. AUXILIARY
(a) shameful (d) admirable (a) Chief (d) Supplemental
(c) unaccountable (d) worthy (c) Negligible (d) Separate
19. The attitude of western countries towards the third DIRECTIONS (Qs. 29 - 31): Choose the word
world countries is rather callous to say the least. opposite is meaning to the given word. [2014]
(a) cursed (d) unkend 29. AUSPICIOUS
(c) unfeeling (d) passive (a) Prosperous (b) Unfavourable
(c) Improper (d) New
DIRECTIONS (Qs. 20 - 22): Choose the word which
30. RECOMPENSE
is closest to the opposite in meaning of the underlined
word in the sentence. [2015] (a) Emolument (d) Reward
(c) Payment (d) Penalty
20. Hydra is biologically believed to be immortal. 31. IMPEDE
(a) undying (b) perishable
(a) Block (b) Delay
(c) ancient (d) eternal
21. The Gupta rulers patronised all cultural activities (c) Push (d) Freeze
and thus Gupta period was called the golden DIRECTIONS (Qs. 32 - 34): A part of sentence is
era in Indian History. underlined. Belence are given alternatives to the
(a) criticised (b) rejected underlined part a, b, c and d which may improve the
(c) opposed (d) spurned sentence. Choose the correct alternative. [2014]
22. The General Manager is quete tactful and 32. They requested me to follow them.
handles the workers union very effectively. (a) ordered (b) urged
(a) incautious (b) discreet (c) asked (d) No improvement
(c) strict (d) disciplned 33. She did not believed me.
DIRECTIONS (Qs. 23 - 25): In each ot the following (a) believing (b) believe to
questions, out of the four alternatives, choose the one (c) believe (d) No improvement
which can be substituted for the given words/sentence. 34. I am fine, what about you?
(a) your (b) your’s
[2015] (c) yours (d) No improvement
23. A person who does not believe in any religion.
(a) Philatelist (b) Rationalist DIRECTIONS (Qs. 35 - 37): Out of the four
(c) Atheist (d) Pagan alternatives, choose the one which express the correct
24. A person who believes that pleasure is the chief meaning of the word. [2013]
good. 35. SAGACIOUS
(a) Stoic (d) Hedonist (a) Shameless (b) Wise
(c) Epicure (d) Sensual (c) Powerless (d) Foolish
25. A person who is incharge of museum. 36. REMEDIAL
(a) caretaker (b) warden (a) Corrective (d) Proficient
(c) supervisor (d) curator (c) Damaging (d) Optional
37. RETICENT
DIRECTIONS (Qs. 26 - 28): Out of the four (a) Confident (d) Sad
alternatives, choose the one which express the correct (c) Truthful (d) Secretive
meaning of the word. [2014]
DIRECTIONS (Qs. 38 - 40): Choose the word
26. AUGMENT opposite is meaning to the given word. [2013]
(a) Increase (b) Decrease 38. FIDELITY
(c) Save (d) Mention (a) Faith (b) Devotedness
27. CONSOLATION (c) Allegiance (d) Treachery
(a) Comfort (d) Problem 39. INFRANGIBLE
(c) Sadness (d) Solution (a) Complecated (d) Breakable
(c) Weird (d) Software
http://t.me/iitjeehelps

Vacabulary & Grammar EL – 3


40. PROGENY 50. Power got with money is the most craved for
(a) Kid (b) Parent today.
(c) Friend (d) Enemy (a) sought after (b) wished for
(c) welcomed for (d) No improvement
DIRECTIONS (Qs. 41 - 43): A part of sentence is 51. You are asked to copy this letter word by word.
underlined. Below are given alternatives to the (a) word for word (b) word with word
underlined part (a), (b), (c) and (d) which may improve (c) word to word (d) No improvement
the sentence. Choose the correct alternative. [2013]
DIRECTIONS (52 - 53): In the following questions,
41. It was not possible to drag any conclusion so two sentences are given. There may be an error in the
he left the case. sentence(s). Mark as your correct answer. [2011]
(a) Fetch (b) Find
(c) Draw (d) No improvement 52. I. Although he was innocent, baseless
42. I am looking after my pen which is missing. accusations were leveled at him.
(a) Looking for (b) Looking in II. Despite of repeated representations from the
(c) Looking back (d) No improvement people, the authorities have failed to take
43. “Mind your language !” he shouted. any action.
(a) change (b) inspect (a) if there is an error only in the first sentence;
(b) if there is an error only in the second
(c) hold (d) No improvement
sentence;
DIRECTIONS (Qs. 44 - 46): In the following questions (c) if there are errors in both sentences; and
choose the word opposite in meaning to the given (d) if there is no error in either of the sentences.
word. [2012] 53. I. I deem it as a privilege to address the
gathering.
44. Florid II. Perfection can be achieved with practice.
(a) Weak (b) Pale (a) if there is an error only in the first sentence;
(c) Monotonous (d) Ugly (b) if there is an error only in the second
45. Verity sentence;
(a) Sanctity (b) Reverence (c) if there are errors in both sentences; and
(c) Falsehood (d) Rarity (d) if there is no error in either of the sentences.
46. Perspicuity DIRECTIONS (Qs. 54 - 56): For each of the following
(a) Vagueness (b) Dullness questions, select the option which is CLOSEST in
(c) Unfairness (d) Unwillingness meaning to the capitalized word. [2011]
DIRECTIONS (Qs. 47 - 49): In question out of the 54. TURBULENCE
four alternative, choose the one which best expresses (a) Treachery (b) Triumph
the meaning of the given word. [2012] (c) Commotion (d) Overflow
55. DEFER
47. Disgrace (a) Discourage (b) Minimize
(a) Disrespect (b) Jealousy (c) Postpone (d) Estimate
(c) Disregard (d) Shame 56. ADAGE
48. Striking (a) Proverb (b) Youth
(a) Attractive (b) Violent (c) Supplement (d) Hardness
(c) Funny (d) Hateful
49. Fiasco DIRECTIONS (Qs. 57 - 59): Choose the word, which
(a) Festival (b) Failure is most OPPOSITE in meaning as the word given in
(c) Fortune (d) Feast bold. [2011]
DIRECTIONS (Qs. 50 - 51): In the following questions 57. FRAGRANCE
a part of the sentence is bold. Below are given (a) Aroma (b) Perfume
alternatives to the bold part at (a), (b) and (c) which (c) Smell (d) Stink
may improve the sentence. Choose the correct 58. PECULIAR
alternative. In case no improvement is needed, your (a) Characteristic (b) Special
answer is (d). [2012] (c) Specific (d) Universal
EBD_7167
http://t.me/iitjeehelps

EL – 4 BITSAT Topicwise Solved Papers


59. ETERNAL DIRECTIONS (Qs. 68 - 70): In each of the following
(a) Momentary (b) Continual questions, choose the alternatives which can be
(c) Everlasting (d) Endless
substituted for the given word. [2009]
DIRECTIONS (Qs. 60 - 62): In the following 68. Agnostic
questions, choose the alternative which can most (a) One who is not sure about God's existence.
appropriately replace the group of words italicised in (b) One who believes in God's existence.
the sentenc. [2010] (c) One having different style of living.
60. Despite being in the career of singing for the (d) None of above.
last 10 yr, he has not been able to earn fame on 69. Bohemian
account of his practice of borrowing ideas and (a) waves in the sea.
words from others and using them as his own.
(a) adaptation (b) pantomime (b) fresh mood.
(c) imitation (d) plagiarism (c) irritatation.
61. Every person is not allowed to enter the place (d) an unconventional style of living.
where public, government or historical records 70. Cacographist
are kept (a) One who is having ego.
(a) scullery (b) pantry
(c) archives (d) coffer (b) One who has unique style.
62. The advertisement assured the public that the (c) One who is bad in spelling.
medicine would give back to the users, their (d) One who is good in spelling
youthful vigour and appearance.
(a) rejuvenate (b) restore DIRECTIONS (Qs. 71 - 73): Which one of the
(c) replenish (d) render following word is correctly spelt? [2009]
71. Spelling test - find correct spelling :
DIRECTIONS (Qs. 63 - 65) : Choose the alternative
which is most similar in meaning to the word given in (a) Vetarinary (b) Veterinary
capital letters. [2010] (c) Veteninary (d) Vetinary
72. Spelling test - find correct spelling:
63. PARAMOUR
(a) Rigerous (b) Rigorous
(a) Lover (b) Companion
(c) Friend (d) Rival (c) Regerous (d) Rigourous
64. REFECTORY 73. Spelling test - find correct spelling :
(a) Dining Room (b) Parlour (a) Itinerary (b) Itinarary
(c) Living Room (d) Restaurant (c) Itnerary (d) Itinerory
65. ASSENT
(a) Compromise (b) Judgement DIRECTIONS (Qs. 74 - 76) : In each of the following
(c) Agreement (d) Expression questions, choose the alternative which is opposite in
meaning to the word given in capital letters. [2009]
DIRECTIONS (Qs. 66 - 67) : Choose the alternative
which best expresses the meaning of the given idiom/ 74. REPRIMAND
phrase. [2010] (a) Reward (b) Appreciate
66. To show one’s teeth (c) Encourage (d) Praise
(a) To ridicule 75. IMPERTINENT
(b) To face difficulties (a) Polite (b) Indifferent
(c) To adopt a threatening attitude (c) Unpleasant (d) Stubborn
(d) To be humble 76. EQUIVOCAL
67. To pour oil in troubled water
(a) Mistaken (b) Quaint
(a) To foment trouble
(b) To add to the trouble (c) Clear (d) Universal
(c) To instigate DIRECTIONS (Qs. 77 - 78): Read each sentence to
(d) To calm a quarrel with soothing words find out whether there is any grammatical error in it.
http://t.me/iitjeehelps

Vacabulary & Grammar EL – 5


The error, if any, will be one part of the sentence. The 86. The doctor advised us to give him wholesome
number of that part is the answer. If there is no error, nutrition.
the answer is (d). [2008] (a) sickly (b) stupendous
77. (a) The aim of the recently held/ (b) rose show (c) depressing (d) fragmentary
was to instil a sense of / (c) competition between the 87. He is good fellow; but what I dislike is his reckless
rose growers. / (d) No error. handling of things.
78. (a) In our country Agriculture / (b) must keeps (a) intelligent (b) cautious
pace with / (c) industrial development. / (d) No (c) soft (d) brilliant
error.
DIRECTIONS (Qs. 88 - 89) : In each of the following
DIRECTIONS (Qs. 79 - 80) : In each of the following questions, choose the alternative which best expresses
questions, choose the alternative which is most nearly the meaning of the italicised word. [2007]
the same in meaning to the word given in capital
88. The one who is rich possesses many superfluous
letters. [2008]
things.
79. EPHEMERAL (a) needless (b) superior
(a) Uneral (b) Mythical (c) essential (d) expensive
(c) Short-living (d) Artificial 89. Many of his acquaintances avoid him because
80. STUBBORN he is so garrulous.
(a) Easy (b) Obstinate (a) proud (b) unreasonable
(c) Willing (d) Pliable (c) talkative (d) quarrelsome
DIRECTIONS (Qs. 81 - 82) : In each of the following DIRECTIONS (Qs. 90 - 92) : In question some parts of
questions, choose the alternative which is opposite in the sentences have errors and some have none. Find
meaning to the word given in capital letters. [2008] out which part of a sentence has an error corresponding
81. GATHER to the appropriate letter (a, b, c). If a sentence is free from
(a) Separate (b) Suspend error, mark (d) as your answer. [2007]
(c) Scatter (d) Spend 90. I whistled thrice with full might and raise my arms
82. EXALT (a) (b)
(a) Depreciate (b) Ennoble towards the sky. No error.
(c) Glorify (d) Simplify (c) (d)
91. Science and religion
DIRECTIONS (Qs. 83 - 85) : In each of the following
(a)
questions, choose the alternative which can be
are both necessary for man and for their
substituted for the given words/sentence. [2008]
(b)
83. Elderly woman in charge of a girl on social outer and inner self respectively. No error.
occasions (c) (d)
(a) Spinster (b) Matron 92. At certain seasons, some areas on Mars
(c) Chaperon (d) Chandler (a) (b)
84. Land so surrounded by water as to be almost an is subject to strong winds. No error.
island
(c) (d)
(a) Archipelago (b) Isthmus
(c) Peninusula (d) Lagoon DIRECTIONS (Qs. 93 - 98) : Each of the idioms or
85. A place adjoining kitchen, for washing dishes phrases is followed by four meanings out of which only
etc. one is correct. Pick out the correct meaning. [2006]
(a) Cellar (b) Wardrobe
(c) Scullery (d) Pantry 93. A hard nut to crack :
(a) Difficult things require extra effort
DIRECTIONS (Qs. 86 - 87) : In each of the following (b) A difficult problem to solve
questions, choose the alternative which is closest to (c) A difficult problem to solved effortlessly
the opposite in meaning of the italicised word. [2007] (d) Costly things need careful handling
EBD_7167
http://t.me/iitjeehelps

EL – 6 BITSAT Topicwise Solved Papers


94. To beat about the bush: DIRECTIONS (Qs. 103 - 105) : In the following
(a) Not to come to the point questions out of the four alternatives, choose the one
(b) Vigorous search for the culprit which best expresses the meaning of the given word.
(c) Easily achieved success without much [2005]
effort 103. Vociferous
(d) Working hard to achieve the goal (a) Violent (b) Loud
95. Which among the following is the SYNONYMS (c) Secret (d) True
of to the word "amateurish"? 104. Fictional
(a) degenerate (b) ruin (a) Genuine (b) Authentic
(c) restore (d) gauche (c) Fanciful (d) Real
96. What is the synonyms of prodigal ? 105. Trivial
(a) wasteful (b) excessive (a) Crucial (b) Significant
(c) creature (d) leninent (c) Vital (d) Ordinary
97. What is the synonyms "moderate" of ? DIRECTIONS (Qs. 106 - 108) : Choose the word
(a) check opposite in meaning to the given word. [2005]
(b) bridle
106. Fabricate
(c) mediocre
(a) Unearth (b) Construct
(d) release
(c) Demolish (d) Renovate
98. Find the correctly spelt word.
107. Gregarious
(a) Adulation (b) Adlation
(a) Sociable (b) Societal
(c) Aduletion (d) Addulation
(c) Unsociable (d) Solitary
DIRECTIONS (Qs. 99 - 100): Choose the correctly 108. Pragmatic
spelt. [2006] (a) Indefinite (b) Vague
99. (a) Adventitious (c) Optimistic (d) Idealistic
(b) Adventitous DIRECTIONS (Qs. 109 - 111) : In the following
(c) Adventitus questions, one part of the sentence may have an error.
(d) Adventituous Find out which part of the sentence has an error. If
100. (a) Satelite (b) Sattelite the sentence is free from error, mark "No error" option.
(c) Satellite (d) Sattellite
[2005]
DIRECTIONS (Qs. 101 - 102): Read each sentence 109. Too great a (a)/ variety of studies (b)/ distract
to find out whether there is any grammatical error in the mind (c)/ No Error (d)
it. The error, if any, will be one part of the sentence. (a) a (b) b (c) c (d) d
The number of that part is the answer. if there is no 110. The severe cyclonic storm (a)/ has left behind
error, the answer is (d). [2006] (b)/ a trial of misery (c)/ No Error (d)
101. (a) This Medicine should / (b) only be taken / (c) (a) a (b) b (c) c (d) d
by the supervision of a doctor. / (d) No error. 111. My sister has been (a)/ interested in medicine
102. (a) I shall go to America / (b) for higher studies / (b)/ ever since she was a child (c)/ No Error (d)
(c) either you like it or not. /(d) No error. (a) a (b) b (c) c (d) d
http://t.me/iitjeehelps

Vacabulary & Grammar EL – 7

Hints & Solutions


1. (a) The word Loquacious (Adjective) means : 25. (d)
talking a lot; talkative. Option (a) is the right 26. (a) Augment means make greater, so increase
synonym while others have different is the correct option.
meanings. 27. (a) Consolation means ‘comfort received by a
2. (a) Embezzle means steal money, often from person after a loss or disappointment’, so
employer. Misappropriate is the best comfort is correct option.
expresses the meaning of embezzle. 28. (b) Auxiliary means ‘providing additional
3. (c) Indict means formally accuse of a crime help’, so supplemental is correct option.
(Law). Accuse is the best expresses the 29. (b) Auspicious means ‘favourable’, so
meaning of Indict. ‘unfavourable’ is best opposite word
4. (c) The word Meticulous (Adjective) means : for it.
paying careful attention to every detail; 30. (d) Recompense means ‘reward given for loss,
fastidious; thorough. so ‘penalty’ is the correct opposite word
Careless in option (c) is the correct for it.
antonomy. 31. (c) Impede means ‘hinder’ or ‘obstruct’, so
5. (a) 6. (a) ‘push’ is correct opposite word for it.
7. (b) Delete 'pair of' before binocular because 32. (b) Here a sense of command is depicted in
the word 'binocular' itself suggests a pair. sentence, so we should use ‘ordered’ for
8. (b) Delete 'all' before 'left'. Here the usage of proper meaning of sentence.
'all' is superfluous as 'the teacher as well as 33. (c) Sentence is in past tense and V1 is used in
his students' itself signifies everyone. those sentence which contain ‘did’, so
9. (a) 'I could not put up at a hotel'. option (c) is correct.
10. (d) 11. (a) 12. (a) 13. (a) 34. (d) No improvement is needed as sentence is
14. (a) 15. (c) 16. (a) right.
17. (c) ‘Immutable’ means ‘unchangeable’. So, 35. (b) Sagacious means ‘judicious’, so ‘wise’ is
option (c) is correct choice. correct answer.
18. (a) ‘ignominious’ means ‘shameful’. So, option 36. (a) Remedial means ‘reformative’, so
(a) is correct choice. ‘corrective’ is correct answer.
19. (c) ‘callous’ means ‘showing or having an 37. (d) Reticent means ‘quiet’ so ‘secretive’ is
insensitive and cruel disregard for others’. correct answer.
So, option (c) is correct choice. 38. (d) Fidelity means ‘faithfulness in relations’,
20. (b) Immortal means living forever, never dying so ‘treachery’ is correct antonym.
or decaying. So, perishable is the correct 39. (b) Infrangible means ‘strong’, so ‘breakable’
opposite to it. is correct antonym.
21. (c) Opposed is the correct answer of this. To 40. (b) Progeny means ‘child’, so ‘parent’ is correct
patronise means favour or pat on the back.
antonym.
22. (a) Tactful means having or showing skill and
41. (c) Use of ‘draw’ is more suitable for using
senstivity in dealing with others or with
before word ‘conclusion’, so option (c) is
defficult essues.
correct.
So, in cautious is the correct opposite of
factful. 42. (a) Use of ‘looking for’ is proper because look
23. (c) Atheist is the best alternative. for means ‘to search for something’ which
24. (c) ‘Epicure’ is the best alternative. suits here.
EBD_7167
http://t.me/iitjeehelps

EL – 8 BITSAT Topicwise Solved Papers


43. (d) ‘Mind your language’ is proper to use here 72. (b) Rigorous
because it gives proper sense of sentence. 73. (a) 74. (b) 75. (a) 76. (c)
44. (b) The word Florid (Adjective) means : rosy; 77. (c) Replace "Between" with "Among"
gaudy; ornated; red; having too much 78. (b) 'keep'
decoration or detail. 79. (c) 80. (b) 81. (c) 82. (a)
The word Pale (Adjective) means : light in 83. (c) 84. (c) 85. (c) 86. (a)
colour; not strong or bright; having skin 87. (b) 88. (a) 89 (c)
that is almost white because of illness. 90. (b) The sentence is in past simple. Hence, with full
Hence, the words florid and pale are might and raised my arms .... should be used.
antonymous. 91. (b) Use both are
45. (c) The word Verity (Noun) means : a belief or 92. (c) Here, subject (some areas) is in plural
principle about life that is accepted as true; number. Hence, plural verb i.e. are subject
truth. to strong winds ......... should be used.
Hence, the words verity and falsehood are 93. (b) 94. (a)
antonymous. 95. (d) gauche - unsophisticated and socially
46. (a) The word Perspicuity (Noun) means : awkward.
clarity. 96. (a) Wasteful - (of a person, action, or process)
The word vagueness (Noun) means : no using or expending something of value
clarity in a person’s mind. carelessly, extravagantly, or to no purpose.
Hence, the words perspicuity and 97. (c) mediocre - of only average quality; not very
Vagueness are antonymous. good.
47. (d) Disgrace means a state of shame. 98. (a) 99. (a) 100. (c)
48. (a) Striking means extraordinary, attractive. 101. (c) under the supervision of a doctor
49. (b) Fiasco means a complete failure. 102. (c) whether you like it or not
103. (b) The meaning of word Vociferous (Adjective)
50. (d)
is :
51. (a) word for word means : in exactly the same
words or when translated exactly equivalent outspoken, blunt. Its synonym should be :
Loud.
words.
52. (c) Sentence I, accusations were leveled 104. (c) The meaning of word Fictional (Adjective)
against him not at him. Sentence II, despite is : Imaginary, unreal, fabricated, mythical
is not followed by of. Its synonym should be : fanciful
53. (c) Sentence I: I deem it a privilege not as a 105. (d) The meaning of word Trivial (Adjective) is :
privilege. not important.
Sentence II: …… achieved through practice Its synonym should be : ordinary.
not with practice. 106. (c) Fabricate means construct something
54. (c) Commotion means an disorderly outburst artificial or untrue while demolish means
or tumult. Its most close to tubulence which destroy completely which is just opposite.
means unstable flow of a liquid or gas. 107. (c) Gregarious denotes tending to associate
Turbulence also refers to a state of with others of the same species and
disturbance. unsociable is the opposite.
55. (c) Other synonyms are prorogue, put off, set 108. (d) Pragmatic means concerned with practical
back, shelve. matters while 'idealistic' means that very
56. (a) An adage is a proverb or byword. good things can be achieved, often when
57. (d) 58. (d) 59. (a) 60. (d) this does not seem likely to others.
61. (c) 62. (a) 63. (a) 64. (a) 109. (c) distracts the mind
65. (c) 66 (c) 67. (d) 68. (a) 110. (c) trail of misery
69. (d) 70. (c) 71. (b) 111. (d)
http://t.me/iitjeehelps

Chapter
Composition &
Comprehension 2
DIRECTIONS (Qs. 1 - 4): Read the passage carefully 3. According to the passage, writing style can be
and choose the best answer to each question out of improved by
the four alternatives. [2017] (a) thinking logically
(b) writing clearly
To write well you have to be able to write clearly and (c) undergoing training
logically, and you cannot do this unless you can think (d) reading widely
clearly and logically. If you cannot do this yet you 4. Famous writers have achieved success by
should train yourself to do it by taking particular (a) using their linguistic resources properly
problems and following them through, point by point, (b) disciplining their skill
to a solution, without leaving anything out and (c) following only one idea
without avoiding any difficulties that you meet. (d) waiting for inspiration
At first you find clear, step-by-step thought very DIRECTIONS (Qs. 5&6) : In questions below,
difficult. You may find that your mind is not able to sentences are given with blanks to be filled in with an
concentrate. Several unconnected ideas may occur appropriate word (s). Four alternatives are suggested
together. But practice will improve your ability to for each question. Choose the correct alternative out
concentrate on a single idea and think about it clearly of the four. [2017]
and logically. In order to increase your vocabulary 5. China is a big country, in area it is bigger than
and to improve your style, you should read widely any other country _________ Russia.
and use a good dictionary to help you find the exact (a) accept (b) except
meanings and correct usages of words. (c) expect (d) access
Always remember that regular and frequent 6. The treasure was hidden ______ a big shore.
practice is necessary if you want to learn to write (a) on (b) underneath
well. It is no good waiting until you have an inspiration (c) toward (d) off
before you write. Even with the most famous writers, DIRECTIONS (Qs. 7 - 10) : Read the following passage
inspiration is rare. Someone said that writing is ninety- and answer the questions that follows. [2016]
nine percent hard work and one percent inspiration, At this stage of civilisation, when many nations are
so the sooner you get into the habit of disciplining brought in to close and vital contact for good and evil,
your-self to write, the better. it is essential, as never before, that their gross
1. To write well, a person must train himself in ignorance of one another should be diminished, that
they should begin to understand a little of one another's
(a) dealing with a difficult problem
historical experience and resulting mentality. It is the
(b) not leaving anything out
fault of the English to expect the people of other
(c) thinking clearly and logically countries to react as they do, to political and
(d) following a step-by-step approach international situations. Our genuine goodwill and good
2. Initially it is difficult to write because intentions are often brought to nothing, because we
(a) a good dictionary is not used expect other people to be like us. This would be corrected
(b) ideas occur without any sequence if we knew the history, not necessarily in detail but in
(c) aids to correct writing are not known broad outlines, of the social and political conditions
(d) exact usages of words are not known which have given to each nation its present character.
EBD_7167
http://t.me/iitjeehelps

EL-10 BITSAT Topicwise Solved Papers


7. According to the author of 'Mentality' of a 12. S1: Calcutta unlike other cities kepts its trams.
nation is mainly product of its P: As a result there horrendous congestion.
(a) History Q: It was going to be the first in South Asia.
(b) International position R: They run down the centre of the road
(c) Politics S: To ease in the city decided to build an
(d) Present character underground railway line.
8. The need for a greater understanding between S6: The foundation stone was laid in 1972.
nations The Proper sequence should be: [2016]
(a) was always there (a) PRSQ (b) PSQR
(b) is no longer there (c) SQRP (d) RPSQ
(c) is more today than ever before DIRECTIONS (Qs. 13 & 14) : Pick out the most effective
(d) will always be there word from the given words to fill in the blank to make
9. The character of a nation is the result of its the sentence meaningfully complete. [2016]
(a) mentality
13. The miser gazed ...... at the pile of gold coins in
(b) cultural heritage
front of him.
(c) gross ignorance
(a) Avidly (b) Admiringly
(d) socio-political conditions
(c) Thoughtfully (d) Earnestly
10. According to the author his countrymen should 14. I saw a ...... of cows in the field.
(a) read the story of other nations (a) Group (b) Herd
(b) have a better understanding of other nations (c) Swarm (d) Flock
(c) not react to other actions
DIRECTIONS (Qs. 15 - 17): Fill in the blank. [2015]
(d) have vital contacts with other nations
15. Freedom and equality are the ______ rights of
DIRECTIONS (Qs. 11 & 12) :In questions below, every human.
each passage consist of six sentences. The first and (a) inalienable (b) inscrutable
sixth sentence are given in the begining. The middle (c) incalculable (d) institutional
four sentences in each have been removed and jumbled 16. The team was well trained and strong, but some
up. These are labelled as P, Q, R and S. Find out the how their ______ was low.
proper order for the four sentences. [2016] (a) morale (d) moral
(c) feeling (d) consciousness
11. S1 : A force of exists between everybody in the 17. His speech was disappointing: it ______ all the
universe. major issues.
P : Normally it is very small but when the one (a) projected (b) revealed
of the bodies is a planet, like earth, the force (c) skirted (d) analysed
is considerable.
Q : It has been investigated by many scientists DIRECTIONS (Qs. 18 - 20): Choose the order of the
sentences marked A, B, C, D and E to form a logical
including Galileo and Newton.
paragraph. [2015]
R : Everything on or near the surface of the
earth is attracted by the mass of earth. 18. (A) Tasty and healthy food can help you bring
S : This gravitational force depends on the mass out their best.
of the bodies involved. (B) One minute they are toddlers and next you
see them in their next adventure.
S6 : The greater the mass, the greater is the
(C) Your young ones seem to be growing so
earth's force of attraction on it. We can call
fast.
this force of attraction gravity.
(D) Being their loving custodians, you always
The Proper sequence should be:
want to see them doing well.
(a) PRQS (b) PRSQ (E) Their eye sparkle with curiosity and endless
(c) QSRP (d) QSPR questions on their tongues.
http://t.me/iitjeehelps

Composition & Comprehension EL-11

Codes DIRECTIONS (Qs. 24 - 26): Arrange the following


(a) DBCEA (b) CADEB sentences in correct pattern and mark at the correct
(c) CBEDA (d) ECABD combination. [2014]
19. (A) It is hoping that overseas friends will bring
in big money and lift the morale of the 24. 1. Today we live in modern technology era.
people. P. We have a log of problems now.
(B) But a lot needs to be done to kick start Q. We want to get everything in one day.
industrial revival. R. Ancient time was quite pleasant.
(C) People had big hopes from the new S. We has no problems then.
government. 6. Perhaps greed is the main cause for this.
(D) So far government has only given an (a) PQRS (b) PRSQ
incremental push to existing policies and (c) SRQP (d) RPQS
programmes. 25. 1. He is a common man.
(E) Government is to go for big time reforms, P. Yesterday our city saw a brutal crime.
which it promised. Q. Police is trying to arrest innocent persons.
Codes R. The criminals are well known.
(a) BCDAE (b) EADCB S. Police as well as whole system in corrupt.
(c) DABCE (d) CDEAB 6. Police will arrest him as he is an easy target
20. (a) Forecasting the weather has always been a because of being a common man.
defficult business. (a) PRSQ (b) PQSR
(B) During a period of drought, steams and (c) PQRS (d) PSQR
rivers dried up, the cattle died from thirst 26. 1. I want to change the room.
and were ruined. P. Last month I got a job.
(C) Many different things affect the weather and Q. I had been living there for six months.
we have to study them carefully to make R. The office is far from the room.
accurate forecast. S. I want to cut expenses of travelling.
(D) Ancient egyptians had no need of weather 6. Hopefully I will do this next week.
in the Nille valley hardly ever changes. (a) PQRS (b) PRSQ
(E) In ear ly times, when there were no (c) QPRS (d) PQSR
instruments, such as their mometer or the DIRECTIONS (Qs. 27 - 29): Fill in the blanks. [2013]
barometer, a man looked for tell tale signs in 27. I ..... to go there when I was student.
the sky. (a) liked (b) used
(a) ABDCE (b) EDCBA (c) prefer (d) denied
(c) ACBDE (d) BDCAE 28. She was angry ...... me.
DIRECTIONS (Qs. 21 - 23): Fill in the blanks. [2014] (a) at (d) about
(c) with (d) in
21. They were afraid ........... the lion, so they dropped
the idea of hunting in jungle. 29. You should not laugh ...... the poor.
(a) on (b) at
(a) in (b) to
(c) with (d) over
(c) from (d) of
22. Our company signed a profitable ...... last month. DIRECTIONS (Qs. 30 - 32): In the questions below,
(a) issue (d) agenda each passage consists of six sentences. The first and
(c) deal (d) paper sixth sentence are given in the beginning. The middle
23. What is your ......... for tonight? four sentences in each have been removed and jumbled
(a) Principle (b) Motto up. These are labeled as P, Q, R and S. Find out people
(c) Plan (d) Objective order for the four sentences. [2013]
EBD_7167
http://t.me/iitjeehelps

EL-12 BITSAT Topicwise Solved Papers


30. 1. He is a famous doctor. 35. 1. The most vulnerable section of the society
P. Once I had to consult with him. are the students.
Q. I never believed him. P. Revolutionary and new fledged ideas have
R. He suggested me a proper remedy. a great appeal to them.
S. I become completely fine. Q. Agitations may be non-violent methods of
6. Now I also admit this fact. protest.
(a) PQ RS (b) Q P S R R. They cannot resist the charm of persuasion.
(c) QP RS (d) R Q S P S. They are to be taught that without discipline
31. 1. We don’t know the plan of Ram. they cannot get proper education.
P. He cares for his friends. 6. However if these become violent, the
Q. He is a complete person. antisocial elements get encouraged and they
R. We want some help and advice. put all proper working out of gear.
S. As we are in a trouble. (a) PRSQ (b) RSQP
6. We hope he will do his best for us. (c) SRPQ (d) RPQS
(a) PRSQ (b) Q P R S 36. 1. Venice is a strange city.
(c) PQ RS (d) P S R Q P. There are about 400 odd bridges connecting
32. 1. It is not my problem. the islands of Venice.
P. All residents of this society are careless. Q. There are no motor cars, no horses and no
Q. I am unable to convince anyone buses there.
R. They don’t want to do some good. R. These small islands are close to one another.
S. Every one seems to be unwise here. S. It is not one island but a hundred islands.
6. We all have to suffer one day. 6. This is because Venice has no streets.
(a) PRSQ (b) P R Q S (a) SRPQ (b) PSRQ
(c) PQ RS (d) P S R Q (c) RQPS (d) QSRP
DIRECTIONS (Qs. 33-34) : Sentences are given with DIRECTIONS (QS. 37-39) : In question number 37 to 39,
blanks to be filled in with an appropriate word(s). Four you have two passages with 5 questions in each passage.
alternatives are suggested for each question. Choose Read the passages carefully and choose the best answer
the correct alternative out of the four: [2012] to each question out of the four alternatives. [2012]
33. Let us quickly __________. The World health Organisation is briefly called W.H.O.
(a) muddle (b) huddle It is a specialised agency of the United Nations and was
(c) hurdle (d) puddle established in 1948.
34. Rajesh’s car wasn’t __________ Ramesh’s, so
International health workers can be seen working
we were too exhausted by the time we reached
in all kinds of surroundings in deserts, jungles,
home.
mountains, coconut groves, and rice fields. They help
(a) such comfortable
the sick to attain health and the healthy to maintain
(b) as comfortable as
their health.
(c) comfortable enough
This global health team assists the local health
(d) so comfortable that
workers in stopping the spread of what are called
DIRECTIONS (Qs. 35-36) : In the following questions, communicable diseases, like cholera. These diseases
the 1st and the last sentences of the passage are can spread from one country to another and so can be
numbered 1 and 6. The rest of the passage is split into a threat to world health.
four parts and named P, Q, R and S. These four parts W.H.O. assists different national health
are not given in their proper order. Read the sentence authorities not only in controlling diseases but also in
and find out which of the four combinations is correct. preventing them altogether. Total prevention of
Then find the correct answer. [2012] diseases is possible in a number so ways. Everyone
http://t.me/iitjeehelps

Composition & Comprehension EL-13

knows how people, particularly children, are vaccinated 42. 1. making ourselves
against one disease or another. Similarly, most people P. our language
are familiar with the spraying of houses with poisonous Q. part of growing into
substances which kill disease-carrying insects. R. Masters of
37. "It is a specialised agency of the United Nations S. is an important
and was established in 1948". Here specialised 6. full manhood or womanhood
means : (a) PSRQ (b) SQPR
(a) made suitable for a particular purpose (c) RPSQ (d) PRSQ
(b) expert 43. 1. The very first battle they fought
(c) extraordinary P. and they had to fall back
(d) uncommon Q. cross the border
38. "International health workers can be seen R. was lost
working in all kinds of surroundings: in deserts, S. letting the enemy
jungles, mountains, coconout groves, and rice 6. an enter the country
fields". Here International means: (a) RQSP (b) RPSQ
(a) belonging to the whole world (c) QRPS (d) QPRS
(b) drawn from all countries of the world 44. 1. A nation
(c) believing in cooperation among nations P. the material assets it possesses
(d) belonging to an organisation which has Q. is not made by
something to do with different nations. R. and collective determination
39. They help the sick to attain health and the healthy S. but by the will
to maintain their health. here they stands for: 6. of the people
(a) deserts (a) PQRS (b) QPSR
(b) rice fields (c) RSPQ (d) SRPQ
(c) international health workers 45. 1. When the Governor
(d) jungles P. the bell had rung
Q. justice should be immediately
DIRECTIONS (Qs. 40 & 41): Pick out the most R. he ordered that
effective word from the given words to fill in the S. found out why
blanks to make the sentence meaningfully complete 6. done to the horse
in the contest of the sentence. [2011] (a) RSPQ (b) PQSR
40. ____________to popular belief that red meat (c) SPRQ (d) SQRP
makes human aggressive, scientist have found 46. 1. When you ponder over
that it actually has a calming effect. P. that the only hope
(a) Sticking (b) Similarly Q. you will realize
(c) Opposite (d) Contrary R. of world peace lies
41. From its_____________opening sequence, it S. the question deeply
is clear that we are in the grip of a delicious new 6. in the United Nations
voice, a voice of breathtaking. (a) QRSP (b) SPQR
(a) Imagination (b) Evocative (c) SQPR (d) RSPQ
(c) Mesmerizing (d) Resonance DIRECTIONS (Qs. 47-51): Rearrange the following
DIRECTIONS (Qs. 42-46): In the following passages, six sentences (A), (B), (C), (D), (E), (F) in the proper
the first and the last parts of the sentence are sequence to form a meaningful paragraph; then answer
numbered 1 and 6. The rest of the sentence is split the questions given below them. [2010]
into four parts and named, P, Q, R and S. These four (A) This is being done to ensure that when the cess
parts are not given in their proper order. Read the is removed or merged ,the flexibility to impose
parts and find out which of the four combinations is higher rate on luxury goods is not taken away ,a
correct. Then find the correct answer. [2011] senior finance ministry official told ET.
EBD_7167
http://t.me/iitjeehelps

EL-14 BITSAT Topicwise Solved Papers


(B) This is only an enabling provision and the 53. Information technology, and the hardware and
highest rate levied on goods will still be 28% software______________with the IT industry.
(14% central GST and 14% state GST ).The (a) amalgamation (b) associated
demerit and luxury goods will attract higher 28% (c) regulated (d) use
rate plus cess.
DIRECTIONS (Q. 54-56): In each of the following
(C) India has decided to peg the peak goods and
questions, choose the most appropriate altarnative
services tax (GST) rate at40% in the legislation
to fill in the blank. [2009]
instead of 28%, giving it the flexibility to raise
rates without having to reach out to parliament. 54. It is difficult to believe what he tells us because
(D) "Some members of the council felt such an his account of any event is always full of ............
enabling provision was needed." the official of all sorts.
privy to the development said. (a) discrepancies (b) differences
(E) The GST council has proposed to peg the peak (c) discretions (d) distinctions
tax rate at 40% (20% central GST and 20% state 55. The bank clerk tried to ............... money from his
GST) in the model GST law to preclude the friend’s account.
requirement of approaching parliament or state (a) empower (b) embellish
assemblies for any change in future. (c) embroil (d) embezzle
(F) This enabling provision will also allow the 56. Eight scientists have ............. the national awards
government to remove the cess at some stage for outstanding contribution and dedication to
and instead have a higher GST rate only, which the profession.
will make for a neater GST. (a) bestowed (b) picked
47. Which sentence should come second in the (c) bagged (d) conferred
paragraph?
DIRECTIONS (Q. 57-59): In the following questions,
(a) B (b) A
some parts have been jumbled up. You are required to
(c) F (d) E
rearrange these parts, which are labelled P, Q, R and S
48. Which sentence shouldcome before the last ?
to produce the correct sentence. [2009]
(a) F (b) E
(c) D (d) A 57. Freedom, is the restricted kind in the sense/(P),
49. Which sentence will come complete the passage ? the rich and poor woman/(Q), that a wide gulf
(a) C (b) A separates/(R), which a modern woman enjoys
(c) D (d) B (S)
50. Which sentence will be third after th e (a) P S R Q (b) S R Q P
rearrangement? (c) R Q P S (d) S P R Q
(a) F (b) E 58. In life, some rules are/(P), as in business/(Q),
(c) B (d) D they seem almost instinctive/(R), learnt so early
51. Which sentence will start the passage? that/(S)
(a) E (b) C (a) R S P Q (b) Q P S R
(c) E (d) B (c) R P S Q (d) Q S P R
DIRECTIONS (Qs. 52 & 53): Pick out the most 59. Kapil, left in an aeroplane/(P), after reading a
effective word from the given words to fill in the sailing magazine/(Q), had decided/(R), to build
blanks to make the sentence meaningfully complete his own boat nine years earlier/(S)
in the contest of the sentence. [2010] (a) P R Q S (b) R S Q P
(c) R Q P S (d) P S R Q
52. A novel of real_____________ must invent its
own language, and this one does, DIRECTIONS (Qs. 60-62 ): In each of the following
(a) impulsive (b) ambition questions, put the parts P, Q, R and S in their proper
(c) intricate (d) abstruse order to produce the correct sentence. [2008]
http://t.me/iitjeehelps

Composition & Comprehension EL-15

60. The Bible, (c) The suspect is claimed to have been seen
(P) has in many respects in several cities
(Q) the sacred book of all Christians (d) The suspect was seen by people in several
(R) among all the books of the world cities.
(S) a unique character and position 66. The teacher punished the boys who had not
(a) QPSR (b) QRPS done their homework.
(c) RPQS (d) RQPS (a) The boys who had not done their homework
61. The ultimate hope had been punished by their teacher
(P) will foce the nations (b) The boys were punished by their teacher
(Q) that the destructive nature of weapons who had not done their homework
(R) to give up war (c) The boys who had not done their homework
(S) has not been fulfilled were punished bby the teacher
(a) PQRS (b) PRQS (d) The boys who had not done their homework
(c) QPRS (d) RSQP were being punished by the teacher
62. It was DIRECTIONS (Qs. 67 & 68): In each of the following
(P) in keeping with my mood questions, choose the best alternative to fill in the
(Q) a soft summer evening blank. [2007]
(R) as I walked sedately 67. Mr. Shyam Lal has gone to his native village
(S) in the direction of the new house [2008] with the ........ or starting an adult school.
(a) QPRS (b) QRPS (a) suggestion (b) presumption
(c) SQPR (d) SRPQ (c) opinion (d) intention
DIRECTIONS (Qs. 63& 64): Fill in the Blanks to make 68. The twins are so alike that I cannot ....... one
the sentence meaningfully complete. [2008] from the other.
63. The metallic letters at the entrance of the house (a) discern (b) tell
read 'Swati Mukundam' ans as one steps inside (c) say (d) notice
the hall, one can see a series of paintings DIRECTIONS (Qs. 69-72): Read the passage
____________the walls. carefully and choose the best answer. [2007]
(a) lying (b) sticking The crowd surged forward through the narrow streets
(c) adorning (d) creating of Paris. There was a clatter of shutters being closed
64. With the _______________ of Cloud hastily by trembling hands the citizens of Paris knew
Computing, Microsoft has a real opportunity in that once the fury of the people was excited there was
India, the company's India-born CEO Satya no telling what they might do. They came to an old
Nadella has said. house which had a workshop on the ground floor. A
(a) come (b) outbreak head popped out of the door to see what it was all
(c) break (d) emergence about "Get him! Get Thimonier! Smash his devilish
DIRECTIONS (Qs. 65-66): In each of the following machines!" yelled the crowd.
questions, a sentence has been given in Active/ They found the workshop without its owner. M.
Passive voice. Out of the four alternatives, select the Thimonier had escaped by the back door. Now the
one which best expresses the same sentence in fury of the demonstrators turned against the machines
that were standing in the shop, ready to be delivered
Passive/Active voice. [2007]
to buyers. They were systematically broken up and
65. People claim to have seen the suspect in several destroyed – dozens of them. Only when the last wheel
cities. and spindle had "been trampled under foot did the
(a) The suspect is being seen in several cities infuriated crowd recover their senses.
(b) The suspect has been the people in several
"That is the end of M'Sieur Thimonier and his
cities
sewing machines," they said to one another and went
EBD_7167
http://t.me/iitjeehelps

EL-16 BITSAT Topicwise Solved Papers


home satisfied. Perhaps now they would find work, could curb the indomitable urge for freedom of the
for they were all unemployed tailors and seamstresses people who bid defiance to time. They fought and
who believed that their livelihood was threatened by fought heroically; they never submitted. Their struggle
that new invention. for independence is an inspiring and exhilarating story.
69. The passage throws light on It is a story not only of firm determination and will but
(a) why inventions should be avoided. also of sacrifice and suffering, a story of heroism and
(b) how a well meant invention can be courage that happens in all revolutionaries.
misunderstood 73. Why did the Aryans settle permanently in India?
(c) what mischief an inventor can do to ordinary (a) Because here food and fodder were available
people. in plenty
(d) how dangerous an invention can be. (b) Because they invaded India
70. The crowd was protesting against (c) Because here food was available in plenty
(a) the closings of workshops. (d) Because here fodder was available in plenty
(b) the misdoings of Thimonier. 74. British came to India:
(c) the newly invented sewing machine (a) to exploit her rich resources
(d) Thimonier for keeping the invention a secret (b) to become her master
71. The aim of the crowd was to (c) to civilize her
(a) kill Thimonier (d) None of the above
(b) drive Thimonier away 75. The Indians indomitable urge for freedom could
(c) humiliate Thimonier be curbed neither :
(d) destroy the sewing machines (a) by the nerve-racking exploitation of the
72. The people thought that British
(a) their lives were in danger. (b) by the treachery of some of her sons
(b) Thimonier was mad. (c) by the imperialist might
(c) the sewing machine was dangerous. (d) all of the above
(d) they would be deprived of their livelihood.
DIRECTIONS (Qs. 73-75): Read the following DIRECTIONS (Q. 76): Choose the suitable
passage carefully and answer the questions given preposition from the given alternatives to fill in the
below it. [2006] blanks in the following sentences. [2006]
PASSAGE 76. So many servants attended ......... him during his
illness.
India is a country which has been subjected to foreign
(a) with (b) on
invasions since the down of Indian history. The fertile
(c) for (d) to
plains of India have been attracting avaricious tribals
from all over the world. Long back the Aryans from DIRECTIONS (Q. 77): Each of the idioms of phrases
Central Asia invaded India and settled down is follwed by four meanings out of which only one is
permanently in this beautiful land where food and correct. Pick out the correct meaning. [2006]
fodder were available in plenty. After a chain of 77. To beat about the bush:
invasions from the bordering countries through land (a) Not to come to the point
routes, the European nations, including the British, (b) Vigorous search for the culprit
finally came to India to exploit her rich resources. The (c) Easily achieved sucess without much effort
Englishmen came to India as traders but stealthily (d) Working hard to achieve the goal
became her masters. India became the 'brightest Jewel'
DIRECTIONS (Qs. 78-79) : Sentences are given with
in the British diadem. They proclaimed to civilize her
blanks to be filled in with an appropriate word (s). Four
and started to exploit her. Neither the imperialist might,
alternatives are suggested for each question. Choose
not the treachery of some of her sons, nor the treachery
the correct alternative out of the four. [2005]
of some of her sons, nor the nerve-racking exploitation
http://t.me/iitjeehelps

Composition & Comprehension EL-17

78. He will dispense ______ your services. strangers he appears either proud or ill at ease. At any
(a) of (b) off rate the postmaster had but little company, nor had he
(c) on (d) with much to do.
79. I have given her ______ . At times he tried his hand at writing a verse or
(a) a work (b) some works two. That the movement of the leaves and clouds of
(c) a piece of work (d) many works the sky were enough to fill life with joy - such were
DIRECTIONS (Qs. 80-82) : Read the passage the sentiments to which he sought to give expression.
carefully and choose the best answer to each question But God knows that the poor fellow would have felt it
out of the four. [2005] as the gift of a new life, if some genie of the Arabian
The postmaster first took up his duties in the village Nights had in one night swept away the trees, leaves
of Ulapur. Though the village was a small one, there and all, and replaced them with a macadamised road,
was an indigo factory nearby and the proprietor, an hiding the clouds from view with rows of tall houses.
Englishman, had managed to get a post office 80. What does the phrase 'ill at ease' in the passage
established. mean?
Our postmaster belonged to Calcutta. He felt like (a) Forward (b) Disease
a fish out of water in this remote village. His office (c) Comfortable (d) Uneasy
and living-room were in a dark thatched shed, not far 81. What does the phrase 'little company' in the
from a green, silmy pond, surrounded on all sides by passage mean?
a dense growth. (a) Bad friendship (b) Hardly any friends
The men employed in the indigo factory had no (c) Small business (d) Business-like
leisure, moreover they were hardly desirable 82. At times, the postmaster wrote
companions for decent folk. Nor is a Calcutta boy an (a) poems (b) novels
adept in the art of associating with others. Among (c) short stories (d) dramas
EBD_7167
http://t.me/iitjeehelps

EL-18 BITSAT Topicwise Solved Papers

Hints & Solutions


1. (c) 2. (b) 3. (d) 4. (b) 40. (d) Sticking-extending out above a surface or
5. (b)` China is a big country. In area it is bigger boundary contrary-very opposite in nature
than any other country except Russia. or character or purpose.
[except means other than, accept means 41. (c) Evocative-recreate strong feelings,
consent, expect means to anticipate and memory etc.
access means entrance]. Mesmerizing-attract strongly.
6. (d) The treasure was hidden off the shore. 42. (c) 42. (b) 44. (b) 45. (c)
When something is hidden “off the shore”, 46. (c) 47. (a) 48. (d) 49. (c)
If just means that it’s hidden some where 50. (a) 51. (b)
near it. 52. (b) Ambition-a cherished desire
7. (a) 8. (c) 9. (d) 10. (b) Abstruse-hard to understand intricate-
11. (d) 12. (d) 13. (a) 14. (b) complex
15. (a) Option (d) institutional as the word means 53. (b) Amalgamation-joining of two organization
relating to principles esp. of law, so legally to form one large organization
also every human has rights of freedom Regulated means controlled.
and equality. 54. (a) 55. (d) 56. (c) 57. (d)
16. (a) 17. (c) 58. (b) 59. (b) 60. (a) 61. (c)
18. (c) CBEDA 62. (a)
19. (d) CDEAB 63. (c) 1st 2 options need prepositions here, as
20. (c) 'lying on' ….'sticking to' …paintings dont
21. (d) Afraid agrees with preposition ‘of’, so create the walls. 3Rd one is correct
option (d) is correct. 64. (d) emergence
22. (c) Normally, company signs a contract or Confusion may arises among option 2nd
deal, so use of ‘deal’ is proper here. and 4th only, then outbreak, we tend to
23. (c) The question gives a sense of query about use for negative happenings, emergence
normal routine of some special/specific day, fits here.
so use of ‘plan’ is more proper here. 65. (c) 66. (c) 67. (d) 68. (b)
24. (b) 25. (a) 26. (c) 69. (b) The passage throws light on how a well-
27. (b) ‘Use to’ is used when any habit is to be means invention can be misunderstood.
shown, so use of option (b) is proper. 70. (c) The crowd was protesting against the
28. (c) ‘Angry’ agrees with preposition ‘with’, so newly invented sewing machine.
use of option (c) is correct here. 71. (d) The aim of the crowd was to destroy the
29. (b) Laugh agrees with preposition ‘at’, so use sewing machines.
of option (b) is correct here. 72. (d) People thought they would be deprived of
30. (c) 31. (b) 32. (a) their livelihood.
33. (b) Huddle : come close in a group 73. (a) 74. (a) 75. (d) 76. (d)
34. (b) Right use of as - as comparison 77. (a)
35. (a) 36. (a) 37. (a) 38. (a) 78. (d) dispens with is the correct phrasal verb
39. (c) 79. (c) 80. (b) 81. (b)
82. (c)
http://t.me/iitjeehelps

UNIT -2 LOGICAL REASONING


Chapter
Verbal Reasoning
3
1. Look at this series: (c) Both I and II are implicit
J14, l16, __, P20, R22, ... What number should fill (d) Both I and II are not implicit
the blank? [2017] DIRECTION (Q. 6) : Choose the correct alternative
(a) S24 (b) N18 (c) M18 (d) T24 that will continue the same pattern and replace the
2. If A × B means A is to the south of B; A + B question mark in the given series.
means A is to the north of B; A % B means A is to
6. 3, 4, 7, 7, 13, 13, 21, 22, 31, 34, ? [2016]
the east of B; A - B means A is to the west of B;
(a) 42 (b) 43 (c) 51 (d) 52
then in P % Q + R - S, S is in which direction with
7. Introducing a boy, a girl said, "He is the son of
respect to Q? [2017]
the daughter of the father of my uncle." How is
(a) South-West (b) South-East the boy related to the girl? [2016]
(c) North-East (d) North-West (a) Brother (b) Nephew
3. When Rahul was born, his father was 32 years (c) Uncle (d) Son-in-law
older than his brother and his mother was 25
years older than his sister. If Rahul's brother is 6 DIRECTIONS (Qs. 8 - 9) : In these series, you will be
years older than him and his mother is 3 years looking at both the letter pattern and the number
younger than his father, how old was Rahul's pattern. Fill the blank in the middle of the series or
sister when he was born? [2017] end of the series.
(a) 7 years (b) 10 years 8. QAR, RAS, SAT, TAU, _____ [2016]
(c) 14 years (d) 19 years (a) UAV (b) UAT (c) TAS (d) TAT
4. Deepak said to Nitin, "That boy playing with 9. DEF, DEF2, DE2F2, _____, D2E2F3
the football is the younger of the two brothers (a) DEF3 (b) D3EF3 (c) D2E3F (d) D2E2F2
of the daughter of my father's wife." How is the DIRECTIONS (Qs. 10 - 11) : In each question below
boy playing football related to Deepak? are given two statements followed by two conclusions
[2017] numbered I and II. You have to take the given two
(a) Son (b) Brother statements to be true even if they seem to be at
(c) Cousin (d) Brother-in-law variance from commonly known facts. Read the
conclusion and then decide which of the given
DIRECTIONS (Q. 5) : In the following question, one conclusions logically follows from the two given
statement is given followed by two assumptions I statements, disregarding commonly known facts.
and II. You have to consider the statement to be true
Give answer: [2016]
even if it seems to be at variance from commonly
(a) If only conclusion I follows
known facts. You have to decide which of the given
(b) If only conclusion II follows
assumptions, if any, follow from the given statement.
(c) If neither I nor II follows and
5. Statements : Politicians become rich by the votes (d) If both I and II follow.
of the people. [2017] 10. Statements : Raman is always successful. No
Assumptions : fool is always successful.
I. People vote to make politicians rich. Conclusions :
II. Politicians become rich by their virtue. I. Raman is a fool.
(a) Only I is implicit II. Raman is not a fool.
(b) Only II is implicit
EBD_7167
http://t.me/iitjeehelps

EL-20 BITSAT Topicwise Solved Papers


11. Statements : Some desks are caps. No cap is and so on, then which of the following would be
red. seventh letter from the right. [2014]
Conclusions : (a) O (b) P (c) R (d) S
I. Some caps are desks. 20. Today is Thrusday. The day after 59 days will be
II. No desk is red. (a) Sunday (b) Monday [2014]
12. Complete the series by replaing ‘? mark (c) Tuesday (d) Wednesday
G4T, J9R, M20P, P43N, S90L,? [2015] 21. Which of the following represents coal mines,
(a) S90L (b) V185J factories and fields? [2014]
(c) M20P (d) P43N
13. Neeraj starts walking towards South. After
walking 15 m, he turns towards North. After
walking 20 m, he turns towards East and walks (a) (b)
10 m. He then turns towards South and walks 5
m. How far is he from his original position and in
which direction? [2015]
(a) 10 m, East (b) 10 m, South-East (c) (d)
(c) 10 m, West (d) 10 m, North-East
14. The average age of 8 men is increased by 2 yr 22. Find out the missing term in the series. [2014]
when one of them whose age is 20 yr is replaced 1, 8, 27, ? , 125, 216
by a new man. What is the age of the new man (a) 52 (b) 58 (c) 64 (d) 65
[2015] 23. If ‘+’ means ‘×’, ‘–’ means ‘+’, ‘×’ means ‘÷’ and
(a) 28 yr (b) 36 yr (c) 34 yr (d) 35 yr ‘÷’ means ‘–’, then 6 – 9 + 8 × 3 ÷ 20 = ? [2014]
15. Shikha is mother-in-law of Ekta who is sister-in- (a) – 2 (b) 6 (c) 10 (d) 12
law of Ankit. Pankaj is father of Sanjay, the only 24. Here are some words translated from an artificial
brother of Ankit. How is Shikha related to Ankit? language.
(a) Mother-in-law (b) Aunt [2015] mallon piml means blue light
(c) Wife (d) Mother mallon tifl means blue berry
16. In a queue of children, Arun is fifth from the left arpan tifl means rasp berry
and Suresh is sixth from the right. When they Which word could means ‘light house’?
interchange their places among themselves, Arun (a) tiflmallon (b) pimlarpan [2014]
becomes thirteenth from the left. Then, what will (c) mallonarpan (d) pimldoken
be Suresh's position from the right? [2015] 25. In a certain code language ‘DOME’ is written as
(a) 8th (b) 14th (c) 15th (d) 16th ‘8943’ and ‘MEAL’ is written as ‘4321’. What
17. In a certain code language, ‘SAFER’ is written group of letters can be formed for the code
as ‘5@3#2’ and ‘RIDE’ is written as ‘2©%#’, how ‘38249’? [2013]
would ‘FEDS’ be written in that code? [2014] (a) EOADM (b) MEDOA
(a) 3#©5 (b) 3@%5 (c) EMDAO (d) EDAMO
(c) 3#%5 (d) 3#%2 26. Find the missing number from the given
18. Find the missing number from the given response. [2013]
response. [2014]
93 27 79 38 67 16
4 2 9 4 6 20

72 720 ?
63 37 42
3 3 10 2 1 6

(a) 72 (b) 720 (c) 7200 (d) 38


3 4 ?
19. If the first and second letters in the word
DEPRESSION were interchanged, also the third Fig. 1 Fig. 2 Fig. 3
and fourth letters, the fifth and the sixth letters (a) 5 (b) 6 (c) 8 (d) 9
http://t.me/iitjeehelps

Verbal Reasoning EL-21

27. Which of the following correctly represents the 33. In a code language, if SUMMER is coded as
relationship among illiterates, poor people and SDNLVR, then the word WINTER will be coded
unemployed? [2013] as: [2012]
(a) SDUMJV (b) SDMUJV
(c) SUUMVJ (d) VJMUDS
(a) (b)
DIRECTION (Q. 34): In question number, select the
missing number from the given responses.
34.
(c) (d) 2 3 9
1 41 5 4 159 6 4 ? 8
28. Sushma walks 20m towards north. Then she 3 2 3
turns right and walks 30m. Now, she turns right
and walks 35m. Now turning left, she walks 15m. [2012]
Again, she turns left and moves 15m. Finally, (a) 888 (b) 788 (c) 848 (d) 842
she turns left and walks 15m. In which direction 35. Today is Monday. After 61 days, it will be:
and how far is she from her original position. [2012]
[2013] (a) Wednesday (b) Saturday
(a) 15m East (b) 30 m East (c) Tuesday (d) Thursday
(c) 15m West (d) 45 m West 36. Rahul and Nitesh are standing in a row of
29. In a classroom, there are 5 rows and 5 children persons. Rahul is 12th from left side and Nitesh
A, B, C, D and E are seated one behind the other is 18th from the right side of the row. If they
in 5 separate rows as follows. [2013] interchanged their positions Rahul becomes 25th
• A is sitting behind C but in front of B. from left. Find the new position of Nitesh from
• C is sitting behind E and D is sitting in front right side? [2012]
of E. (a) 38 (b) 32 (c) 42 (d) 31
• The order in which they are sitting from the
37. One of the numbers does not fit into the series.
first row to the last is [2013]
Find the wrong number.
(a) D E C A B (b) B A C E D
(c) A C B D E (d) A B E D C 52, 152, 414, 1312, 5348, 26840 [2012]
30. Which of the following will fill the series? (a) 152 (b) 414 (c) 1312 (d) 5348
2, 9, 28, ? , 126 [2013] 38. In the following question and D stands for any
(a) 64 (b) 65 (c) 72 (d) 56 of Mathematical signs at different places, which
31. Two signs in the equations have been are given as choices under each question. Select
interchanged, find out the two signs to make the choice with the correct sequence of signs
equation correct. [2013] which when substituted makes the question as
3 ÷ 5 × 8 + 2 – 10 = 13 correct equation?
(a) + and – (b) × and ÷
24 D 4 D 5 D 4 [2012]
(c) ÷ and – (d) ÷ and +
32. Assertion:[A] = India is a democratic country. (a) × + = (b) = × +
Reason: [R] = India has a constitution of its own (c) + × = (d) = + ×
Choose the correct alternative from the given 39. Which represents carrot, food, vegetable?
options. [2013] [2012]
(a) Both (A) and (R) are true and (R) is correct
explanation of (A). (a) (b)
(b) Both (A) and (R) are true but (R) is not the
correct explanation of (A)
(c) (A) is true (R) is false. (c) (d)
(d) (A) is false (R) is true.
EBD_7167
http://t.me/iitjeehelps

EL-22 BITSAT Topicwise Solved Papers


40. "All the members of the Tennis club are members 48. Which of the given Venn diagrams out of (a),
of the badminton club too". No woman plays (b), (c) or (d) correctly represents the relationship
badminton? [2012] among the following classes?
(a) Some women play Tennis Rose, Flower, Lotus [2011]
(b) No member of Tennis club plays badminton
(c) Some women are members of the Tennis club
(d) No woman is a member of Tennis club (a) (b)
DIRECTION (Q. 41&42): In the following question,
a series is given with one term missing. Choose the
correct alternative from the given ones that will
complete the series: (c) (d)
41. One of the, numbers does not fit into the series.
Find the wrong number. [2011] DIRECTION (Q. 49): In the following question, select
15, 20, 45, 145, 565, 2830 the related letter/word/number from the given
(a) 20 (b) 45 (c) 145 (d) 565 alternatives.
42. VWX, BCD, HIJ, ? [2011] 49. EFJK : LMQR :: LMQR : ? [2010]
(a) NOQ (b) NOP (a) EFJK (b) STXY
(c) MNO (d) OPQ (c) KJFE (d) YXTS
43. In a code language, if TARGET is coded as
201187520, then the word WILLIUM will be coded DIRECTION (Q. 50): In the following question, select
as [2011] the related letter/word/number from the given
(a) 239121292113 (b) 239121291213 alternatives.
(c) 239122191213 (d) 239121292213 50. Mahatma Gandhi : Porbandar : : Pt. Jawaharlal
44. Sanjay is taller than Suresh but shorter than Nehru : ? [2010]
Rakesh. Rakesh is taller than Harish but shorter (a) Allahabad (b) Calcutta
than Binit. Who among is the tallest? [2011] (c) New Delhi (d) Mumbai
(a) Suresh (b) Sanjay DIRECTION (Q. 51): In the question below is given
(c) Binit (d) Rakesh a statement followed by three assumptions numbered
45. In a row of 62 persons. Rahul is 36th from left I, II and III. You have to consider the statement and
side of the row and Nitesh is 29th form the right the following assumptions, decide which of the
side of the row. Find out the number of persons assumptions is implicit in the statement and choose
sitting between them? [2011]
your answer accordingly.
(a) 1 (b) 2 (c) 3 (d) 4
46. The missing number in the given figure is 51. Statement: The education of a student at
[2011] collegiate level, not taking into account
maintenance expenses, costs four hundred
rupees a year. Collegiate education is thus
25 17 38 18 89 16 drawing heavily upon the national resources of
an impoverished community. So college
6 8 ? education should be restricted to a brilliant few.
Assumptions:
(a) 13 (b) 15 (c) 17 (d) 19 I. Our resources are very limited.
47. Select the combination of number so that the II. Only a few students should be admitted to
letters arranged will from a meaningful word. the colleges. [2010]
H N R C A B
(a) Only assumption I is implicit
1 2 3 4 5 6 [2011]
(b) Only assumption II is implicit
(a) 2, 5, 3, 4, 1, 6 (b) 3, 5, 6, 4, 1, 2
(c) Neither I nor II is implicit
(c) 4, 1, 5, 6, 2, 3 (d) 6, 3, 5, 2, 4, 1
(d) Both I and II are implicit
http://t.me/iitjeehelps

Verbal Reasoning EL-23

52. In a code language, if BANGED is coded as (b) Only II and III are implicit
JJKQCC, then the word STRAY will be coded as (c) Only III is implicit
[2010] (d) None of these
(a) DEUTV (b) DEUVT DIRECTION (Q. 58): In each of the following
(c) EFVWT (d) EFVVS question, select the related letter/word/number from
DIRECTION (Q. 53): In the following question, a the given alternatives.
series is given with one terms missing. Choose the 58. FLMO : ? : : BFEN : ARSO [2009]
correct alternative from the given ones that will (a) BZYS (b) CZYS (c) SZYB (d) YZBC
complete the series: 59. If A denotes '+'
B denotes '-'
53. 2, 3, 7, 22, 155, ? [2010]
C denotes '×'
(a) 1706 (b) 1550 (c) 3411 (d) 3100 Then what is the value of (10 C 4) A (4 C 4) B 6?
54. Which one of the following diagram represents [2009]
the correct relationship among (a) 60 (b) 50 (c) 56 (d) 46
Colour, Black and White. [2010] 60. In the question, three words are given. They are
followed by four words one of which stands for
the class to which these three words belong.
(a) (b)
Identify that word.
Newspaper, Hoarding. Television [2008]
(c) (d) (a) Press (b) Media
(c) Broadcast (d) Rumour
DIRECTION (Qs. 55) : In each of the following DIRECTION (Q. 61) : In the following questions, a
question, select the related letter/word/number from series is given with one term missing. Choose the
the given alternatives. correct alternative from the given ones that will
55. Distance : Odometer :: ? : Barometer [2009] complete the series:
(a) Humidity (b) Pressure 61. 6, 9, 15, 24, 39, ? [2008]
(c) Thickness (d) Wind (a) 60 (b) 63 (c) 66 (d) 62
56. One of the, numbers does not fit into the series.
Find the wrong number DIRECTION (Q. 62): In each question below is given
13, 16, 38, 124, 504, 2535 a statement followed by two conclusions numbered I
(a) 16 (b) 38 (c) 124 (d) 504 and II. You have to assume everything in the statement
to be true, then consider the two conclusions together
DIRECTION (Q. 57): In each question below is given
a statement followed by three assumptions numbered and decide which of them logically follows beyond a
I, II and III. You have to consider the statement and reasonable doubt from the information given in the
the following assumptions, decide which of the statement.
assumptions is implicit in the statement and choose 62. Statement: The standard of education in private
your answer accordingly. schools is much better than Municipal and Zila
57. Statement: In order to reduce the gap between Parishad-run schools.
income and expenditure, the company has Conclusions:
decided to increase the price of its product from I. The Municipal and Zila Parishad should make
next month. serious efforts to improve standard of their
Assumptions: schools.
I. The rate will remain more or less same after the II. All Municipal and Zila Parishad schools should
increase. be closed immediately. [2008]
II. The expenditure will more or less remain the same
(a) if only conclusion I follows
in near future.
(b) if only conclusion II follows
III. The rival companies will also increase the price
of the similar product. [2009] (c) If either I or II follows
(a) Only I and II are implicit (d) if both I and II follow
EBD_7167
http://t.me/iitjeehelps

EL-24 BITSAT Topicwise Solved Papers

63. In the given question, four numbers/number- DIRECTION (Q. 71): Find the missing character from
pairs are given. Select the one which is different among the given alternatives.
from the other three. [2008]
(a) 1234 (b) 2345 (c) 4567 (d) 7896 71. 3 15 [2006]
2 25 441 6
DIRECTION (Q. 64): Find the group of letters from
the four alternative which is obtained by applying the 289 ?
10 13
same rule to this given word to the right of the sign: 7 12
64. WUS : QOM : : ? : FPB (a) 625 (b) 25 (c) 125 (d) 156
(a) LIG (b) MBH (c) LJH (d) LKH 72. Find the wrong term in the letter-number series
given below: [2006]
65. Coconut is related to Shell in the same way as G4T, J10R, M20P, P43N, S90L
Letter is related to __________: [2007] (a) G4T (b) J10R (c) M20P (d) P43N
(a) Envelope (b) Post 73. Victory is related to Joy in the same way as Sorrow
(c) Postal Stamp (d) Letter Box is related to: [2005]
(a) Defeat (b) Depression
66. In the following question, four groups of letters
(c) Melancholy (d) Cry
are given. Three of them are alike in a certain 74. In the following question, four groups of letters
way while one is different. Select the one which are given. Three of them are alike in a certain
is different. [2007] way while one is different. Select the one which
(a) 6859 (b) 4923 (c) 5106 (d) 1738 is different. [2005]
67. Which number should replace the question mark (a) MONJK (b) ACAZV
(c) GMRGS (d) METEI
in third figure? [2007]
75. In the following question, a statement/ group of
statements is given follwed by some conclusions.
11 14 15 17 19 26 Choose the conclusion which logically follows
92 73 76 from the given statement.
15 21 ? Statement:
1. Only students can participate in the race.
(a) 21 (b) 31 (c) 41 (d) 51 2. Some participants in the race are females.
3. All female participants in the race are invited
68. In the following question, one number is missing for coaching. [2005]
in the series. You have to understand the pattern Conclusions:
of the series and insert the number. (a) All participants in the race are invited for
coaching.
0, 2, 6, 12, ? , 30, 42 [2007] (b) All participants in the race are males.
(a) 24 (b) 20 (c) 21 (d) 22 (c) All students are invited for coaching.
(d) All participants in the race are students.
69. Grain is related to Warehouse in the same way 76. What term will fill the blank space? [2005]
as Water is related to _________: [2006] ELFA, GLHA, ILJA, ________, MLNA
(a) Drink (b) Dam (c) Canal (d) River (a) OLPA (b) KLMA
(c) LLMA (d) KLLA
70. In the following question, four groups of letters 77. Find the missing terms.
are given. Three of them are alike in a certain
4 7 6 11 2 [2005]
way while one is different. Select the one which 5
23 54 40
is different. [2006] 3 1 3 4 ? 9
(a) BCE (b) PQT (c) GHJ (d) VWY
(a) 04 (b) 01 (c) 02 (d) 03
http://t.me/iitjeehelps

Verbal Reasoning EL-25

Hints & Solutions


1. (b) In this series, the letters progress by 2, and Total age = 8(x + 2) yr
the numbers increase by 2. Difference of ages = 8(x + 2) – 8x
2. (b) According to P % Q + R - S = 8x + 16 – 8x = 16 yr
Q P Age of new man = 20 + 16 = 36 yr
So, the new man is 16yr older to the man by
whom the new man is replaced.
15. (d) The relation is as following:
R S Couple
S is in the South-East of Q. Shikha
aw (–) Pankaj
n-l
3. (b) When Rahul was born, his brother's age = 6 e r-i (+)
years; oth Mother father
M Couple
his father's age = (6 + 32) years = 38 years, Ekta Sanjay Brother Ankit
his mother's age = (38 - 3) years = 35 years; (–) (+) (+)
his sister's age = (35 - 25) years = 10 years. Sister-in-law
4. (b)
5. (d) It is clearly shown that Shikha is the mother
6. (b) of Ankit.
7. (a) The father of the boy's uncle is the grandfa- 16. (b) Since Arun and Suresh interchange places,
so Arun's new position (13th from left) is
ther of the boy and daughter of the grand- the same as Suresh's earlier position (6th
father is sister of father. from right).
8. (a) In this series, the third letter is repeated as So, number of children in the queue = (12 +
the first letter of the next segment. The middle 1 + 5) = 18.
letter, A, remains static. The third letters are Now, Suresh's new position is the same as
in alphabetical order, beginning with R. Arun's earlier position fifth from left.
9. (d) In this series, the letters remain the same: Therefore Suresh's position from the right
DEF. = (18 – 4) = 14th.
The subscript numbers follow this series: 17. (c)
111, 112, 122, 222, 223, 233, 333, ... 18. (b) From the given responses,
10. (b) Since both the premises are universal and 4 × 2 × 3 × 3 = 72
one premise is negative, the conclusion 9 × 4 × 2 × 10 = 720
must be universal negative and should not Similarly, 6 × 20 × 1 × 6 = 720
contain the middle term. So, only II follows.
11. (a) Since one premise is particular and the other 19. (b) Since, consecutive two letters are
premise is negative, the conclusion must interchanged. Therefore,
be particular negative and should not con- DE PR ES SI ON
tain the middle term. So, it follows that 'Some
desks are not red'. However, I is the con- → → → → →
verse of the first premise and thus it holds.
12. (b) ED R P SE IS NO
+23 +47 +95 7th from Right.
+5 +11 20. (a) Every day of week repeats after seven days.
G4 T J 9R M20P P 43 N S 90 L V 185J
–2 –2 –2 –2 Hence, 59 = 7 × 8 + 3 = 56 + 3
[ It will be Thursday after 56 days.
–2
+3 +3 +3 +3 +3
13. (a) [ 57th day = Thursday Þ 58th day = Friday
14. (b) Let the average age of 8 men = x yr 59th day = Saturday Þ 60th day = Sunday
Total age of 8 men = 8x yr [ It will be Sunday after 59 days.
Now, new average age = (x + 2)yr
EBD_7167
http://t.me/iitjeehelps

EL-26 BITSAT Topicwise Solved Papers

Field 53 + 1 = 125 + 1 = 126


21. (b) 31. (d) Interchanging symbols ÷ and + as given in
option (d), we get
3 + 5 × 8 ÷ 2 – 10
8
Coal mines = 3 ∗ 5≥ – 10 = 3 + 20 – 10 = 13
Factories 2
Both coal mines and factories are located 32. (b) Both Assertion and Reason are correct but
in the fields. India is a democratic country because the
22. (c) From the given series, government is elected by its citizens and
not because India has its own constitution.
13 ↑ 1 33. (a)
34. (a) (1 × 2 × 3 × 5) + (1 + 2 + 3 + 5) = 41
23 ↑ 8
(3 × 4 × 2 × 6) + (3 + 4 + 2 + 6) = 159
33 ↑ 27 (9 × 8 × 3 × 4) + (9 + 8 + 3 + 4) = 888
43 ↑ 64 35. (b) Each day of the week is repeated after 7
days.
53 ↑ 125
So, after 63 days, it will be Monday.
63 ↑ 216 After 61 days, it will be Saturday.
Therefore, 64 will come in place of 36. (d)
questions mark. 37. (b) The number should be 404.
23. (c) Interchanging the symbols as given in the × 1 + 100, × 2 + 100, × 3 + 100……
above question, th e above equation 38. (b) After putting sign
becomes.
24 = 4 × 5 + 4
8
6 + 9 × 8 ÷ 3 – 20 = 6 ∗ 9 ≥ – 20 24 = 24
3 Hence, (b) is correct choice.
= 6 + 24 – 20 = 10 39. (a) 40. (d)
24. (d) 25. (d) 41. (c) The number should be 140.
26. (d) From fig. 1, 93 – (27 + 63) = 93 – 90 = 3 × 1 + 5, × 2 + 5, × 3 + 5……
From fig. 2, 79 – (38 + 37) = 79 – 75 = 4 42. (b) The pattern is:
From fig. 3, 67 – (16 +42) = 67 – 58 = 9 V WX B C D H I J N O P
Hence, option (d) is correct.
27. (b) Illiterate Poor people +6 +6 +6
+6 +6 +6
Unemployed +6 +6 +6
Hence, option (b) is correct. 43. (a)
28. (b) T A R G E T
29. (a) From the information given in the question
the arrangement of students is 20 1 18 7 5 20
1st D
2nd E Similarly, W I L L I U M
3rd C
4th A 23 9 12 12 9 21 13
5th B 44. (c) From the question we get,
30. (b) The given series follows the pattern Rakesh > Sanjay > Suresh
13 + 1 = 2 Binit > Rakesh > Harish
23 + 1 = 8 + 1 = 9 So, Binit is the tallest among them.
33 + 1 = 27 + 1 = 28 45. (a) No. of Persons between Rahul and Nitesh
43 + 1 = 64 + 1 = = (36 + 29) - 62-2 = 65 - 62-2=1
65
http://t.me/iitjeehelps

Verbal Reasoning EL-27

46. (a) 47. (d) BRANCN 48. (b) Similarly,


49. (b) L J H F DB

As, EFJK LMQR +6


+7 +6
+6
Similarly, LMQR STXY 65. (a) As: 'Coconut' is inside the 'Shell', similarly,
+7 'Letter ' is inside the "Envelope'.
50. (a) As, the birth place of Mahatma Gandhi was 66. (a) Except (a), all other numbers are non-perfect
Porbandar. Similarly, the birth place of Pt. cubes.
Jawaharlal Nehru was Allahabad.
51. (d) The use of the words 'impoverished 67. (b) (11)2 + (14)2 – (15)2 = 92
community' in the statement makes I implicit (15)2 + (17)2 – (21)2 = 73
while the phrase 'college education should (19)2 + (26)2 – (31)2 = 76
be restricted to a brilliant few' makes II 68. (b) Here the respective difference between the
implicit.
terms is as follows-
52. (b)
53. (c) 2 , 4 , 6 , 8 , ……….
Therefore, 12 + 8 = 20
2 3 7 22 155 3411
Which is the required term.
69. (b) Grain is stored in warehouse. Similarly, water
2×1+1 2×3+1 3×7+1 7×22+1 22×155+1 is stored by constructing dam.
54. (b)
55. (b) Distance is measured by Odometer. 70. (b) Except (b), each group has a gap of zero
Similarly, Pressure is measured by Barometer. letters between first two letters and a gap of
56. (c) The number should be 123. 1 letter between last two letters.
×1+3, ×2+6, ×3+9…… 71. (a) Clearly, (3 + 2)2 = 25
57. (d) Clearly, the company intends to reduce the (15 + 6)2 = 441
gap between income and expenditure by (10 + 7)2 = 289
increasing the price of its product i.e. by So, missing number = (12 + 13)2 = 625
keeping the expenditure unaltered and 72. (b) The first letter of each term is moved three
increasing the income only. So, II is implicit steps forward and the last letter is moved
while I is not. However, the rival companies two steps backward to obtain th e
may or may not follow the same pursuit. So, corresponding letters of the next term. The
III is not implicit. number follows the sequence x2+1, x2+2,
58. (a) x2+3, x2+4.
59. (b) Using correct symbols we have: So, 10 is wrong and must be replaced by
(10 × 4) + (4 × 4) – 6 = 40 + 16 – 6 = 50 (4 × 2 + 1) i.e. 9.
60. (b) 73. (a) Victory is joyful. Similarly, defeat is
61. (b) The pattern is: sorrowful.
6 9 15 24 39 63 74. (a) Expect in MONJK in all others at least one
letter is repeated.
75. (d)
+3 +6 +9 +15 +24
76. (d) The second and forth letters in the series, L
62. (a) Conclusion I follows as it is directly implied
and A, are static. The first and third letters
by the given statement.
consist of an alphabetical order beginning
Conclusion II does not follow because it is
with the letter E.
too severe.
63. (d) 77. (c) (5 × 4) + (3 × 1) = 23
(7 × 6) + (3 × 4) = 54
64. (c) W U S Q OM
(11 × 2) + (? × 9) = 40
+6 Or, ? × 9 = 40 – 22 = 18
+6 ? = 18/9 = 2
+6
EBD_7167
http://t.me/iitjeehelps

EL-28 BITSAT Topicwise Solved Papers

Chapter
Non Verbal Reasoning
4
1. Which answer figure complete the form in
question figure ? [2017]
Question figure :

? (a) (b) (c) (d)


4. From the four positions of a dice given below,
find the color which is opposite to yellow?
Answer figures : [2017]

Yellow Orange Blue Red

Yellow

Yellow
Blue

Blue
Orange Violet Red Rose
(a) (b) (c) (d)
2. Which answer figure includes all the
components given in the question figure? (a) Violet (b) Red
Question Figure : [2017] (c) Rose (d) Blue
5. Choose the set of figure which follows the given
rule.
Rule: Closed figures gradually become open
and open figures gradually become closed.

Answer Figures : (1)

(2)

(a) (b) (c) (d)


DIRECTION (Qs. 3): Select a suitable figure from the
four alternatives that would complete the figure matrix. (3)

3. [2017]

(4)

[2017]
(a) (1) (b) (2)
? (c) (3) (d) (4)
http://t.me/iitjeehelps

Non Verbal Reasoning EL-29

6. Select a suitable figure from the four alternatives 10. Choose the correct answer figure which will make
that would complete the figure matrix. [2016] a complete square on joining with the problem
figure [2015]
Problem figure

(1) (2) (3) (4)


(a) 1 (b) 2 (c) 3 (d) 4 Answer Figures
7. Select a suitable figure from the four alternatives
that would complete the figure matrix. [2016]

(a) (b)

(1) (2) (3) (4) (c) (d)


(a) 1 (b) 2 (c) 3 (d) 4
8. Select a suitable figure from the four alternatives
11. In the following question, five figures are given.
that would complete the figure matrix. [2016]
Out of them, find the three figures that can be
joined to form square. [2015]

A B C D E
(a) ABC (b) BCD
(c) ACE (d) CDE
12. Choose the answer figure which completes the
(1) (2) (3) (4) problem figure matrix. [2015]
(a) 1 (b) 2 (c) 3 (d) 4 Problem Figures
9. Choose the set of figures which follows the given
rule.
Rule : Closed figures losing their sides and
open figures gaining their sides. [2016]
(a)

(b)
?
Answer Figures
(c)

(d) (a) (b) (c) (d) (e)


EBD_7167
http://t.me/iitjeehelps

EL-30 BITSAT Topicwise Solved Papers


13. What is the opposite of 3, if four different
positions of dice are as shown below: [2015]

2 5 6 3
3 6 2 4 3 1 5 1 (a) (b) (c) (d)
How will it appear when unfolded? [2014]
(i) (ii) (iii) (iv)
(a) 6 (b) 4
(c) 1 (d) 2
14. In the following questions, one or more dots (a) (b)
are placed in the figure marked as (A). The figure
is followed by four alternatives marked as (a),
(b), (c) and (d). One out of these four options (c) (d)
contains region(s) common to the circle,
square, triangle, similar to that marked by the 17. Which one of the following figures completes
dot in figure (A). [2015] the original figure? [2013]
Problem Figure

(A) ?
(a) (b) (a) (b)

(c) (d) (c) (d)

15. What is the water image of below figure? 18. How many squares are there in the following
[2014] figure ? [2013]

(a) 24 (b) 25 (c) 26 (d) 27


19. [2012]
(a) (b)

(a) (b)
(c) (d)

16. A piece of paper is folded and penched as shown (c) (d)


in the figure below
http://t.me/iitjeehelps

Non Verbal Reasoning EL-31

20. Which answer figure is the exact mirror image of 23. Find out the alternative figure which contains
the given figure when the mirror held form the figure (X) as its part. [2010]
right at PQ? [2012]
P

(X)
Q

(a) (b)
(a) (b)

(c) (d)
21. A piece of paper is folded and a cut is made as
sh own below. Fr om the given responses (c) (d)
indicate how it will appear when opened?
[2011]
24. A piece of paper is folded and cut. From the
figures given, indicate how it will appear when
opened?
Question figure: [2010]

(a) (b)

Answer figure

(c) (d)
(a) (b)
22. Which answer figure will complete the question
figure?
Question figure [2011]
(c) (d)

25. Identify the figure that completes the pattern.


? [2010]

?
(a) (b) (X)

(a) (b)

(c) (d)
(c) (d)
EBD_7167
http://t.me/iitjeehelps

EL-32 BITSAT Topicwise Solved Papers


26. Find out which of the figures (a), (b), (c) and (d) 29. Figure (x) is embedded in anyone of the four
can be formed from the pieces given in figure alternative figures.
(X). Choose the alternative which contains figure (x).
[2010] [2009]

(X)
(X)

(a) (b) (a) (b)

(c) (d)
(c) (d)
DIRECTION (Q. 27): In this questions, two figure/
words are given to the left of the sign and one the 30. Which symbol will appear on the opposite
sign:: with four alternatives under it out. of which one surface to the symbol x? [2009]
of the alternatives has the same relationship with the
figures/words to the right of the sign:: as between the ¸ D
two figures/words to the left of the sign (::). Find the D = O
×
correct alternative.
27. (i) (ii)
? (a) ‘¸’ (b) ‘×’ (c) ‘+’ (d) ‘–’
31. The three figures marked X, Y, Z show the manner
1 2 3 4 5 in which a paper is folded step by step and then
[2009] cut. From the answer figures (a), (b), (c), (d),
select the one, showing the unfolded position
(a) (b) of the Paper after the cut. [2009]

(c) (d)

28. Identify the missing part of the figure and select P X Y Z


it from the given alternatives. [2009]

? (a) (b)

(c) (d)
(X)

DIRECTION (Q. 32) : In this question, two figure/


(a) (b) words are given to the left of the sign:: and one figures
word to the right of the sign:: with four alternatives
under it out of which one of the alternatives has the
same relationship with the figures/words to the right of
(c) (d) the sign:: as between the two figures/words to the left
of the sign (::). Find the correct alternative.
http://t.me/iitjeehelps

Non Verbal Reasoning EL-33

32. [2008] DIRECTION (Q. 35) : In the following question a


: :: :? piece of paper is folded, cut and unfolded. One of the
four figures given below is exactly like this unfolded
paper. Find this out.

35. [2008]
(a) (b)

(c) (d)

33. Select the series which obeys the given rule: (a) (b)
Any figure can be traced by a single unbroken
line without retracing [2008]

(a) (c) (d)

DIRECTIONS (Q. 36) : In the question a figure is


given, its components are given in one of the four
(b) alternative figures. Find this one
36. [2008]

(c)

(a) (b)
(d)

34. Select from amongst the four alternative figures, (c) (d)
the one which complete the pattern in the
problem figure. [2008] 37. In the following question, a set of three figures
X, Y and Z showing a sequence in which a paper
is folded and finally cut from a particular
section. Below these figures a set of answer
figures marked (a, b, c and d) showing the design
which the paper actually acquires when it is
unfolded. You have to select the answer figure
which most closely resembles the unfolded
piece of paper. [2008]
(a) (b)

(c) (d) X Y Z
EBD_7167
http://t.me/iitjeehelps

EL-34 BITSAT Topicwise Solved Papers

[2007]
(a) (b)

(c) (d) ?

DIRECTION (Q. 38) : In the following question, choose


the set of figures which follows the given rule. (a) (b)
38. Rule: Closed figures become more and more
open and open figures become more and more
closed. [2007] (c) (d)

DIRECTION (Q. 41): In the following question, find


(a)
out which of the figure (a), (b), (c) and (d) can be
formed from the pieces given in (x)

(b) 41. [2007]

(c)

(a) (b)
(d)

DIRECTIONS (Q. 39) : In the following question,


find out which of the figures (a), (b), (c) and (d) can be (c) (d)
formed from the pieces given in (x).
39. In (X) [2007]
42. Select a figure from the four alternatives, which
when placed in the blank of Fig. (X) would
complete the pattern. [2006]

(x)
?

(a) (b) (a) (b)

(c) (d)
(c) (d)
Statement the data given by the V.S.
40. Select one alternative figure out of (a), (b), (c) DIRECTION (Q. 43): In the following question, choose
and (d) which completes the given matrix. the set of figures which follows the given rule,
http://t.me/iitjeehelps

Non Verbal Reasoning EL-35

43. Closed figure becomes more and more open. Question Figure:
[2006]

(a)

(b) (a) (b)

(c)
(c) (d)

(d) 47. Which one of the answer figures shall complete


the given question figure? [2005]
DIRECTION (Q. 44): In the following question, find Question Figure:
out which of the figure (a), (b), (c) and (d) can be
formed from the pieces given in (x).

44. [2006]

(a) (b)
(a) (b)

(c) (d)
(c) (d)
48. Consider th e figure X and Y showing a
45. Select one alternative figure out of (a), (b), (c) rectangular sheet of paper folded in Fig. X and
and (d), which completes the given matrix punched in Fig. Y. From amongst the answer
[2006] figures a, b, c, and d, select the figure, which will
most closely resemble the unfolded position of
Fig. Y. [2005]

? (X) (A)

(a) (b)
(a) (b)
(c) (d)
(c) (d) 49. Which one of the given sets of figures follows
the following rule.
46. A piece of paper is folded and punched as shown Rule: “Sectors get converted to triangles one
below. From the given responses indicate how it by one”. [2005]
will appear when opened. [2006]
EBD_7167
http://t.me/iitjeehelps

EL-36 BITSAT Topicwise Solved Papers

(a) (c) (d)

(b) DIRECTION (Q. 51): In the following question,


choose the set of figures which follows the given
rule.
(c) 51. Rule: Closed figures become more and more
open and open figures become more and more
closed. [2005]
(d) (a)

(b)
DIRECTION (Qs. 50): In the following questionfind
out which of the figures (a), (b), (c) and (d) can be
formed from the pieces given in (X)
(c)

50. (X) [2005]


(d)
(a) (b)

Hints & Solutions


1. (b) 2. (a) contents of the first two figures. Lines are
3. (b) In each row, the second figure is obtained carried forward from the first two figures
from the first figure by adding two mutually to the third one, except where two lines
perpendicular line segments at the centre appear in the same position, in which they
and the third figure is obtained from the are cancelled out.
first figure by adding four circles outside 13. (b) From figure, (i), (iii) and (iv), we have
the main figure. concluded that 2, 6, 1 and 5 appear adjacent
4. (a) The colours adjacent to yellow are orange, so 3. clearly, 4 will appear opposite to 3.
blue, red and rose. Hence violet will be 14. (c) In figure (A), the dot is placed in the region
opposite to yellow. which is common to the circle and triangle.
5. (c) Now, we have to find similar common region
6. (a) In each row, the third figure comprises of a in all the four options. Only in figure (c),
black circle and only those line segments we find such a region which is common to
which are not common to the first and the the circle and triangle.
second figures. 15. (b) 16. (b) 17. (b) 18. (c) 19. (d)
7. (d) In each row (as well as each column), the
third figure is a combination of all the, 20. (c) 21. (b) 22. (c) 23. (d) 24. (a)
elements of the first and the second figures. 25. (d) 26. (c) 27. (d) 28. (b) 29. (b)
8. (c) In each row, the third figure is a collection 30. (d) 31. (b) 32. (a) 33. (b) 34. (b)
of the common elements (line segments) of 35. (a) 36. (c) 37. (c) 38. (b) 39. (b)
the first and the second figures. 40. (b) 41. (b) 42. (d) 43. (a) 44. (a)
9. (c) 10. (c) 11. (c) 45. (d) 46. (b) 47. (c) 48. (d) 49. (b)
12. (b) The contents of the third figure in each 50. (a) 51. (a)
row (and column) are determined by the

You might also like